You are on page 1of 424

This page intentionally left blank

RADIOLOGY
CASE REVIEW SERIES | Breast Imaging
Notice
Medicine is an ever-changing science. As new research and clinical experience broaden our knowledge, changes
in treatment and drug therapy are required. The authors and the publisher of this work have checked with sources
believed to be reliable in their e orts to provide information that is complete and generally in accord with the
standards accepted at the time of publication. However, in view of the possibility of human error or changes in
medical sciences, neither the authors nor the publisher nor any other party who has been involved in the preparation
or publication of this work warrants that the information contained herein is in every respect accurate or complete,
and they disclaim all responsibility for any errors or omissions or for the results obtained from use of the information
contained in this work. Readers are encouraged to con rm the information contained herein with other sources. For
example and in particular, readers are advised to check the product information sheet included in the package of each
drug they plan to administer to be certain that the information contained in this work is accurate and that changes
have not been made in the recommended dose or in the contraindications for administration. This recommendation
is of particular importance in connection with new or infrequently used drugs.
RADIOLOGY
CASE REVIEW SERIES | Breast Imaging

Ch ris I. Flowers, MD, FRCR, FSBI


Director of Breast Imaging and Research
Department of Diagnostic Imaging
H. Lee Mo tt Cancer Center & Research Institute
Associate Professor of Radiology
Department of Oncological Sciences
College of Medicine
University of South Florida
Tampa, Florida

Ma rkus K. Holzh a uer, MD


Windsong Radiology Group
Williamsville, New York

S ERIES ED ITO R

Rola n d Ta la now, MD, Ph D


President
Department of Radiology Education
Radiolopolis, a subdivision of InnoMed, LLC
Stateline, Nevada

New York Chicago San Francisco Athens London


Madrid Mexico City Milan New Delhi Singapore
Sydney Toronto
Copyright © 2014 by McGraw-Hill Education. All rights reserved. Except as permitted under the United States Copyright Act of 1976, no part of this
publication may be reproduced or distributed in any form or by any means, or stored in a database or retrieval system, without the prior written permission of
the publisher.

ISBN: 978-0-07-178763-5

MHID: 0-07-178763-1

The material in this eBook also appears in the print version of this title: ISBN: 978-0-07-178719-2,
MHID: 0-07-178719-4.

eBook conversion by codeMantra


Version 1.0

All trademarks are trademarks of their respective owners. Rather than put a trademark symbol after every occurrence of a trademarked name, we use names in
an editorial fashion only, and to the bene t of the trademark owner, with no intention of infringement of the trademark. Where such designations appear in this
book, they have been printed with initial caps.

McGraw-Hill Education eBooks are available at special quantity discounts to use as premiums and sales promotions or for use in corporate training programs. To
contact a representative, please visit the Contact Us page at www.mhprofessional.com.

TERMS OF USE

This is a copyrighted work and McGraw-Hill Education and its licensors reserve all rights in and to the work. Use of this work is subject to these terms.
Except as permitted under the Copyright Act of 1976 and the right to store and retrieve one copy of the work, you may not decompile, disassemble, reverse
engineer, reproduce, modify, create derivative works based upon, transmit, distribute, disseminate, sell, publish or sublicense the work or any part of it without
McGraw-Hill Education’s prior consent. You may use the work for your own noncommercial and personal use; any other use of the work is strictly prohibited.
Your right to use the work may be terminated if you fail to comply with these terms.

THE WORK IS PROVIDED “AS IS.” McGRAW-HILL EDUCATION AND ITS LICENSORS MAKE NO GUARANTEES OR WARRANTIES AS
TO THE ACCURACY, ADEQUACY OR COMPLETENESS OF OR RESULTS TO BE OBTAINED FROM USING THE WORK, INCLUDING ANY
INFORMATION THAT CAN BE ACCESSED THROUGH THE WORK VIA HYPERLINK OR OTHERWISE, AND EXPRESSLY DISCLAIM ANY
WARRANTY, EXPRESS OR IMPLIED, INCLUDING BUT NOT LIMITED TO IMPLIED WARRANTIES OF MERCHANTABILITY OR FITNESS FOR
A PARTICULAR PURPOSE. McGraw-Hill Education and its licensors do not warrant or guarantee that the functions contained in the work will meet your
requirements or that its operation will be uninterrupted or error free. Neither McGraw-Hill Education nor its licensors shall be liable to you or anyone else for
any inaccuracy, error or omission, regardless of cause, in the work or for any damages resulting therefrom. McGraw-Hill Education has no responsibility for
the content of any information accessed through the work. Under no circumstances shall McGraw-Hill Education and/or its licensors be liable for any indirect,
incidental, special, punitive, consequential or similar damages that result from the use of or inability to use the work, even if any of them has been advised of
the possibility of such damages. This limitation of liability shall apply to any claim or cause whatsoever whether such claim or cause arises in contract, tort or
otherwise.
To our wives and families who have had to put up
with endless hours of poring over cases and our computers,
when we could have been taking the family to the Mall or
o on road trips. Thank you for your patience,
and here is the reward.
This page intentionally left blank
Contents

Series Preface ix
Preface xi

Easy Cases 1

Moderately Di cult Cases 173

Most Di cult Cases 353

Subject Index 407


Di culty Level Index 409
Author Index 410

vii
This page intentionally left blank
Series Preface

M
aybe I have an obsession or cases, but when I was with images and questions and page 2 with the answers and
a radiology resident I loved to learn especially rom explanations. This approach avoids unintentional peeking at
cases, not only because they are short, exciting, and the answers be ore deciding on the correct answers yoursel .
un—similar to a detective story in which the aim is to get to We keep it strict: one case per page! This way it remains up
“the bottom” o the case—but also because, in the end, that’s to your own knowledge to f gure out the right answer.
what radiologists are aced with during their daily work. Another example that residents (including me) did miss
Since medical school, I have been ascinated with learning, in traditional case review books is that these books did not
not only or my own benef t but also or the sake o teaching highlight the pertinent f ndings on the images: sometimes,
others, and I have enjoyed combining my IT skills with my even looking at the images as a group o residents, we could
growing knowledge to develop programs that help others in not f nd the abnormality. This is not only rustrating but also
their learning process. Later, during my radiology residency, time consuming. When you prepare or the boards, you want
my passion or case-based learning grew to a level where the to use your time as e iciently as possible. Why not show
idea was born to create a case-based journal: integrating new annotated images? We tackled that challenge by provid-
concepts and technologies that aid in the traditional learn- ing, on the second page o each case, the same images with
ing process. Only a ew years later, the Journal of Radiology annotations or additional images that highlight the f ndings.
Case Reports became an internationally popular and PubMed When you are preparing or the boards and managing
indexed radiology journal—popular not only because o your clinical duties, time is a luxury that becomes even more
the interactive eatures but also because o the case-based precious. Does the resident preparing or the boards truly
approach. This led me to the next step: why not tackle some- need lengthy discussions as in a typical textbook? Or does the
thing that I especially admired during my residency but that resident rather want a “rapid ire” mode in which he or she
could be improved—creating a new interactive case-based can “ ly” through as many cases as possible in the shortest
review series. I imagined a book series that would take into possible time? This is the reality when you start your work
account new developments in teaching and technology and a ter the boards! Part o our concept with the new series is
changes in the examination process. providing short “pearls” instead o lengthy discussions. The
As did most other radiology residents, I loved the tradi- reader can easily read and memorize these “pearls.”
tional case review books, especially or preparation or the Another challenge in traditional books is that questions
boards. These books are quick and un to read and ocus in a are asked on the f rst page and no direct answer is provided,
condensed way on material that will be examined in the f nal only a lengthy block o discussion. Again, this might become
boards. However, nothing is per ect and these traditional case time consuming to ind the right spot where the answer
review books had their own intrinsic aws. The authors and is located i you have doubts about one o several answer
I have tried to learn rom our experience by putting the good choices. Remember: time is money—and li e! There ore,
things into this new book series but omitting the bad parts we decided to provide explanations to each individual ques-
and exchanging them with innovative eatures. tion, so that the reader knows exactly where to f nd the right
What are the eatures that distinguish this series rom answer to the right question. Questions are phrased in an
traditional series o review books? intuitive way so that they it not only the print version but
To save space, traditional review books provide two also the multiple-choice questions or that particular case in
cases on one page. This requires the reader to turn the page our online version. This system enables you to move back
to read the answer or the irst case but could lead to unin- and orth between the print version and the online version.
tentional “cheating” by seeing also the answer o the second In addition, we have provided up to 3 re erences or
case. Doesn’t this de eat the purpose o a review book? From each case. This case review is not intended to replace tra-
my own authoring experience on the USMLE Help book ditional textbooks. Instead, it is intended to reiterate and
series, it was well appreciated that we avoided such acciden- strengthen your already existing knowledge ( rom your train-
tal cheating by separating one case rom the other. Taking the ing) and to f ll potential gaps in your knowledge.
positive experience rom that book series, we decided that However, in a collaborative e ort with the Journal of
each case in this series should consist o two pages: page 1 Radiology Case Reports and the international radiology

ix
x Series Preface

community Radiolopolis, we have developed an online this series is structured on di f culty levels so that the series
repository with more comprehensive in ormation or each also becomes use ul to an audience with limited experience
case, such as demographics, discussions, more image exam- in radiology (nonradiologist physicians or medical students)
ples, interactive image stacks with scroll, a window/level up to subspecialty-trained radiologists who are preparing or
eature, and other interactive eatures that almost resemble their CAQs or who just want to re resh their knowledge and
a workstation. In addition, we are planning ahead toward use this series as a re erence.
the new Radiology Boards ormat and are providing rapid I am delighted to have such an excellent team o US and
ire online sessions and mock examinations that use the international educators as authors on this innovative book
cases in the print version. Each case in the print version is series. These authors have been thoroughly evaluated and
crosslinked to the online version using a case ID. The case selected based on their excellent contributions to the Journal
ID number appears to the right o the diagnosis heading at of Radiology Case Reports, the Radiolopolis community, and
the top o the second page o each case. Each case can be other academic and scientif c accomplishments.
accessed using the case ID number at the ollowing web It brings especially personal satis action to me that this
site: www.radiologycasereviews.com/case/ID, in which project has enabled each author to be involved in the over-
“ID” represents the case ID number. I you have any ques- all decision-making process and improvements regarding
tions regarding this web site, please e-mail the series editor the print and online content. This makes each participant not
directly at roland@talanow.in o. only an author but also part o a great radiology product that
I am particularly proud o such a symbiotic endeavor o will appeal to many readers.
print and interactive online education and I am grate ul to Finally, I hope you will experience this case review book
McGraw-Hill or giving me and the authors the opportunity as it is intended to be: a quick, pertinent, “get to the point”
to provide such a unique and innovative method o radiology radiology case review that provides essential in ormation or
education, which, in my opinion, may be a trendsetter. the radiology boards in the shortest time available, which, in
The primary audience o this book series is the radiol- the end, is crucial or preparation or the boards.
ogy resident, particularly the resident in the inal year who
is preparing or the radiology boards. However, each book in Roland Talanow, MD, PhD
Preface

F
or physicians working in the breast imaging f eld, there With this background, the authors, who both are o
have been more challenges than in most disciplines in European origin and work in the United States, one in aca-
radiology. Breast cancer diagnosis and treatment have demic practice and one in community (private) practice,
changed signif cantly over the last decade or two, along with have produced a case review book that consists o images,
the technologies that have been used and the planes in which test questions, and easy-to-read answers. For those readers
we have visualized the breast. Along with the changes in tech- preparing to take their f nal board examinations, the cases,
niques and advances in knowledge have come the requirements discussions, and pearls should cover most o the topics
o regulatory and quality control, in both the United States and that may appear on the examination. For those who seek
Europe. In the United States, the US Food and Drug Admin- more extensive learning, the book is supplemented with an
istration regulates the ield according to the Mammography interactive online component that includes high-resolution
Standards Quality Act (MSQA), with minimum requirements images and extra teaching points. We hope that you enjoy
or physicians practicing in this f eld. At the same time, the US reading through this textbook and that you benef t greatly
board examinations have changed to include a high standard o rom using it.
knowledge in a ew subjects in the f nal examination.

xi
This page intentionally left blank
Any abnormality in the left breast?

1. I there is a one-view-only asymmetry, what


could be the next step?

2. I a patient was in the o f ce at the time o the


exam, what could be the next step?

3. What is a technical repeat in general?

4. What is the most likely malignancy explaining


a one-view-only asymmetry?

5. What are the actors that make the judgment


di f cult, i this is a real f nding?

1
Normal screening mammogram 1626
Case ranking/dif culty: Category: Screening

Electronically magni ed image of the left MLO screening view. Repeat left MLO view with di erent angle demonstrating resolution
of the questionable asymmetry.

Answers
1. I there is an asymmetric density seen in one view only, 5. One way to eliminate the call back o “one-view-only
decision has to be made f rst whether this is real or it is a f ndings” is to correlate the images with prior studies.
result o projection and overlying tissue. I the f nding is Also help ul is i the breast is not very dense which
believed to be real, the patient has to be recalled, and i makes correlation with the other plane easier. In case
the f nding is believed to be the result o projection, then o nipple discharge or palpable abnormality, the patient
the patient can be classif ed as BI-RADS 1 negative. should be diagnosed in the f rst place.

2. I the patient is still in o f ce, a repeat MLO view with


di erent angle could be done and the patient could still
be handled as a screening patient. This is not a common Pearls
environment, since batch reading is per ormed in the • The overwhelming numbers o one-view-only densities
vast majority o places in the United States. I a spot are the result o superimposed breast tissue.
compression view is per ormed, the patient had to be • According to the study o Sickles, 82.7% o these cases
handled as “recall” and the exam should be labeled as could be classif ed as superimposed tissue based on the
diagnostic mammogram. standard views or through additional workup.
3. I there is an indeterminate f nding that needs workup, • O the remainder o the cases, only a very small
it is called a diagnostic mammogram. I there is limited raction turned out to be malignant (less than 2%). O
exam because o motion, incomplete inclusion o tissue, these cases, most were lobular invasive carcinomas.
or bad compression, repeat standard views or XCCL
view can be added as a “technical repeat.”
I this mammogram is obtained with the patient still Suggested Readings
available in the o f ce, an additional image can be Pearson KL, Sickles EA, Frankel SD, Leung JW. E f cacy
added with a di erent angle and the exam could still be o step-oblique mammography or conf rmation and
classif ed as a screening exam. However, this scenario localization o densities seen on only one standard
is not common, since most screening exams are read mammographic view. AJR Am J Roentgenol.
without the patient being present (batch reading). 2000;174(3):745-752.
4. Lobular invasive carcinoma is most likely the type Sickles EA. Findings at mammographic screening on only
o malignancy that can mani est in the orm o an one standard projection: outcomes analysis. Radiology.
asymmetry seen on one view only. 1998;208(2):471-475.

2
Screening mammogram

1. I you see this density on the CC view on


screening mammogram, what is the next step?

2. What other options might help i there is


clinical concern?

3. How requent is that f nding?

4. What else could be help ul to determine


i density in the medial posterior breast
is benign?

5. I this was a new suspicious f nding, where


could it likely hide on the MLO view?

3
Sternalis muscle 730
Case ranking/dif culty: Category: Screening

Screening mammogram, right MLO view. Screening mammogram, right CC view.

Answers
1. This is a typical appearance o a sternalis muscle; in this Pearls
particular case, bilaterally—it is considered a congenital • The sternalis muscle is an uncommon anatomic variant
variant—patient can return a ter 1 year or screening. o the chest wall musculature, which is present in
about 8% o both males and emales, based on cadaver
2. I clinical concern, urther workup with modif ed CC
studies.
views and possible ultrasound might be considered. I
• The sternalis muscle is more requently unilateral than
this is a sternalis muscle, ultrasound will be normal.
bilateral; it is longitudinal in extent and parasternal
As an alternative, correlation with old chest CT could
in location and it is more superf cial than the rectus
be help ul.
abdominis muscle.
3. It is present in about 8% o the population based on • The correct diagnosis on a mammogram can be
cadaver studies—in about 30% o these, it would show achieved by recognizing the typical location and
up on the mammogram. conf guration.
• I there is remaining concern, correlation with old CT
4. I seen bilaterally and i it is stable since prior studies,
o the chest or urther evaluation with chest CT could
there is no doubt this is a presentation o sternalis muscle
be help ul.
and benign. I it is seen on one view only, and does not
show the typical orm and location or the presence o
sternalis muscle, it is more o a concern—it might not
be covered on the MLO view and workup might be Suggested Reading
necessary. Bradley FM, Hoover HC, Hulka CA, et al. The sternalis
5. Finding such as seen above in the right CC view o the muscle: an unusual normal f nding seen on mammography.
screening mammogram could hide in the in ramammary AJR Am J Roentgenol. 1996;166(1):33-36.
old. However, in this case, it is benign f nding,
consistent with sternalis muscle, given the typical shape,
orm, and location.

4
Prior lumpectomy

1. What BI-RADS classif cation is used or this


entity?

2. What is the one most likely cause or these


f ndings?

3. What is the next best imaging test?

4. What type o biopsy should be per ormed as


one o these is palpable?

5. What type o surgical suture is more prone to


calci y on mammograms?

5
Calci ed foreign bodies 1572
Case ranking/dif culty: Category: Diagnostic

4. The most prudent thing would be to leave them alone.


Catgut sutures are supposed to be reabsorbed over time,
but they now have at necrosis around them. Sometimes,
the patient would pre er to see a surgeon to discuss
removal. Removal o oreign bodies (eg, biopsy clips)
can be done using vacuum-assisted biopsy, so in theory,
it could be used here.
5. Catgut usually gets reabsorbed over a period o months
or years. However, the sutures can potentially give rise to
a severe orm o at necrosis due to the oreign proteins
in the catgut. For this reason, many man-made sutures
have been used over the years, some o which are still
broken down by the human body, and others that have
more permanent properties.

Pearls
Close-up image shows one of the sutures has a tie. • Common in older patients, as this was more commonly
ound with older catgut suture. A variant o at necrosis
with calcif cations around the body o the suture, which
Answers then does not get resorbed.
1. I you do not describe the abnormality, and as there is
nothing else in the breast, you could use BI-RADS 1.
However, it is di f cult to get away without describing Suggested Readings
the f ndings here, and so a more appropriate impression
would be BI-RADS 2: benign. Libshitz HI, Montague ED, Paulus DD. Calcif cations
and the therapeutically irradiated breast. AJR Am J
2. Postradiation changes may cause calcif cations in Roentgenol. 1977;128(6):1021-1025.
vessels and coarse ductal calcif cation as part o induced Stacey-Clear A, McCarthy KA, Hall DA, et al. Calcif ed
apoptosis. Calcif cations postimplant removal are more suture material in the breast a ter radiation therapy.
typically at the posterior aspect o the breast disc, and Radiology. 1992;183(1):207-208.
there may be associated silicone granulomas. Calcif ed
guinea worm is sometimes ound in women rom an area
where the worm is prevalent. Ruptured oil cysts show
discontinuous calcif cations.
3. No urther imaging tests are required or this calcif ed
oreign body. The imaging eatures are diagnostic o
calcif ed sutures.

6
Prior benign surgical biopsy of the left breast

1. What BI-RADS classif cation should be used


here?

2. What is the one most likely cause or these


f ndings?

3. What produce calcif ed lesions that can be


identif ed on mammography?

4. What is the best view to identi y the track o a


VP shunt?

5. What is the risk o malignant trans ormation


around a VP shunt?

7
Ventriculoperitoneal shunt 1642
Case ranking/dif culty: Category: Diagnostic

4. In general, the MLO view shows the tube entering rom


the superior breast and exiting through the in erior part.
There are variable appearances on the CC, depending on
its track. XCCL, cleavage views, and laterals sometimes
help i you cannot see the tube adequately, but they are
not usually required or diagnosis.
5. There is a published case report o a multi ocal tumor
around a VP shunt, but this is likely to be a very rare
f nding.

Pearls
• It is a calcif ed artif cial structure, so what kind o tube
is it?
• What is the direction ( rom/to)?
Bilateral MLO exam showing another case of a calci ed ventricle- • The answers you come up with will lead you to the
peritoneal shunt that had been in situ for the life of the patient. correct f nding.

Answers
Suggested Readings
1. In general, i you describe a f nding, then BI-RADS 2
Ioannis K, Ioannis K, Angelos L. Routine mammographic
should be used. However, a BI-RADS 1 could equally
imaging: it was only a needle. Breast J. 2006;12(5):493.
be used as the f nding is not within the breast itsel .
Lee D, Cutler B, Roberts S, Manghisi S, Ma AM. Multi-
2. This is typical o a ventriculoperitoneal shunt, as centric breast cancer involving a ventriculoperitoneal
treatment or hydrocephalus. shunt. Breast J. 2010;16(6):653-655.
Vimalachandran D, Martin L, Laf M, Ap-Thomas A.
3. Dracunculiasis is a guinea worm. When the parasite
Cerebrospinal uid pseudocyst o the breast. Breast.
dies, it calcif es and appears as a loosely coiled tubular
2003;12(3):215-216.
structure. Sutures may calci y, particularly i the
patient has had radiation treatment. Surgical clips are
inert and do not typically calci y. Pacemaker wires
have occasionally been reported as calcif ed in the
subdermal portion o its track. VP shunts calci y in the
two examples shown here, as they are in the body or a
very long time.

8
Screening—asymptomatic

1. What BI-RADS classif cation should be used


here?

2. What should be the next diagnostic imaging


exam?

3. What type o surgery may this patient have


had?

4. What type o biopsy should be per ormed?

5. What type o ollow-up surveillance would you


recommend?

9
Breast reduction scars 1753
Case ranking/dif culty: Category: Screening

Pearls
• Common, benign appearance. Watch or swirling lines
that do not correspond to normal anatomy, particularly
in the lower hal o the breast.

Suggested Readings
Beer GM, Kompatscher P, Hergan K. Diagnosis o breast
tumors a ter breast reduction. Aesthetic Plast Surg.
1997;20(5):391-397.
Muir TM, Tresham J, Fritschi L, Wylie E. Screening or
breast cancer post reduction mammoplasty. Clin Radiol.
2010;65(3):198-205.
Rubin JP, Coon D, Zuley M, et al. Mammographic changes
a ter at trans er to the breast compared with changes a ter
breast reduction: a blinded study. Plast Reconstr Surg.
2012;129(5):1029-1038.

Another case of breast reduction showing a transverse line across


the posterior part of the breast disc. Lines occurring that are not
expected.

Answers
1. The f ndings o the scars are characteristic. You can
ignore the scars and give a negative or malignancy
BI-RADS 1 assessment, or describe the f nding and
give it a BI-RADS 2, benign.
2. No urther workup is required, as the f nding is normal
postsurgical appearances.
3. These are the scars rom a mastopexy (otherwise known
as a breast reduction). TRAM reconstruction has its own
characteristic imaging f ndings. Bilateral lumpectomy
scars or multiple benign surgical biopsies could in theory
give these appearances.
4. No biopsy is required as this is an “Aunt Minnie”
appearance o postreduction scars. Biopsy may be
required o palpable areas o at necrosis occurring
ollowing this type o surgery.
5. The patient can be ollowed with routine screening,
unless the operation was relatively recent, in which case
annual diagnostic mammography is recommended.

10
Di erent breast compositions—what does that mean?

1. What are the di erent choices in BI-RADS


lexicon or relatively dense breasts?

2. What are the two choices in BI-RADS lexicon


or relatively less dense breasts?

3. What is the consequence o very dense


f broglandular tissue?

4. Why is mammography not worthless in very


dense patients?

5. What other breast screening exams exist in the


United States?

11
Importance of breast composition 1298
Case ranking/dif culty: Category: Screening

Screening mammogram, right Screening mammogram, Screening mammogram, left Screening mammogram,
MLO view demonstrating right CC view MLO view demonstrating “almost left CC view demonstrating
“scattered broglandular demonstrating “scattered entirely fatty replaced” tissue. “almost entirely fatty
tissue.” broglandular tissue.” replaced” tissue.

Answers physicians (gray zone) with the indication such as


1. There are our categories to describe the composition o “f brocystic changes.”
the breast: The two categories describing denser breasts
are (1) “extremely dense” (>75% glandular tissue) and
(2) “heterogeneously dense” (51–75% glandular tissue). Pearls
• The amount o f broglandular tissue in general is more
2. There are two categories describing less dense breast
prominent in younger patients in the reproductive age
compositions: (1) “scattered f broglandular tissue”
and decreases over time.
(25–50% glandular tissue) and (2) “almost entirely atty
• The amount o f broglandular tissue also depends on
replaced” (<25% glandular tissue).
the hormonal status, including intake o estrogens, and
3. The sensitivity o mammography is signif cantly reduced it is related to congenital di erences.
and this results in less value o the mammogram. • Dense f broglandular tissue is considered risk actor or
However, it is still valuable and does not eliminate breast cancer in several regards: (1) It does limit the value
the need or screening. To obtain old images is even o mammography and its sensitivity to detect cancer.
more important. Additional exams, such as MRI and (2) Because o higher quantity o tissue, the likelihood to
ultrasound, might be help ul. In the United States, develop malignancy is higher. (3) There is correlation to
only screening MRI is o f cially accepted as additional some proli erative orms o aging o the parenchyma that
screening exam. Ultrasound is more controversial. is o ten times considered as risk actor or malignancy.
4. Mammography is still very help ul. It can still show
calcif cations, and might be able to show distortion or Suggested Readings
masses. Mammography will be limited in value because
Boyd NF, Martin LJ, Bronskill M, Ya e MJ, Duric N, Minkin S.
o its reduced ability to detect developing densities.
Breast tissue composition and susceptibility to breast
5. Only MRI is accepted by insurance companies or cancer. J Natl Cancer Inst. 2010;102(16):1224-1237.
screening, but only in high-risk patients (li etime risk Boyd NF, Martin LJ, Ya e MJ, Minkin S. Mammographic
o more than 20–25%). Ultrasound is not accepted as density and breast cancer risk: current understanding and
a screening exam but sometimes ordered by re erring uture prospects. Breast Cancer Res. 2011;13(6):223.
12
Screening—asymptomatic

1. What BI-RADS classif cation should be used


here?

2. What type o tissue is this lesion composed?

3. What is the next best imaging test?

4. What type o biopsy would you recommend?

5. What are the likely clinical f ndings on


palpation?

13
Lipoma in dense breasts 583
Case ranking/dif culty: Category: Diagnostic

Answers
1. This lesion is characteristically benign, BI-RADS 2. Pearls
• Aunt Minnie type o case.
2. This lesion is composed o at (adipose). The lesion
• Harmless atty density mass.
would have glandular density and there ore invisible i
it were composed o normal f broglandular elements.
Cooper’s ligaments are the small curvilinear lines
attaching the glandular disc to the skin. Suggested Readings
3. This f nding is characteristic enough to recommend Kapila K, Pathan SK, Al-Mosawy FA, George SS, Haji BE,
return to routine screening. It should be visible on Al-Ayadhy B. Fine needle aspiration cytology o breast
prior examinations. Lipomas can be surprisingly hard masses in children and adolescents: experience with
to demonstrate on ultrasound. Non- at sat T1 MRI can 1404 aspirates. Acta Cytol. 2009;52(6):681-686.
conf rm that the lesion contains at. Tomosynthesis Kirova YM, Feuilhade F, Le Bourgeois JP. Breast lipoma.
should be able to demonstrate the f ndings clearly, Breast J. 2002;8(2):117-118.
compared with conventional 2D mammograms. Lanng C, Eriksen BØ, Ho mann J. Lipoma o the breast: a
diagnostic dilemma. Breast. 2004;13(5):408-411.
4. Clearly, i this lesion is diagnostic o a lipoma, then
biopsy is not indicated.
5. The palpation f ndings o a lipoma are usually either
nothing or a so t lump. Sometimes, the margins may not
be easily elt, and then described as a vague so t lump.
In rare instance that a lipoma gets in ected, the f ndings
may be o a hard lump, but there are clearly other signs
o in ection.

14
Screening mammography 2009 (left) and 2011 (right)

1. Can you see any abnormality on the new


screening mammogram?

2. Which o the mammogram is digital?

3. What is the advantage o digital mammogram


over f lm mammography?

4. What is the disadvantage o digital


mammography in comparison with f lm
mammography?

5. What are the practical consequences reading


digital mammograms?

15
Digital versus lm mammography 306
Case ranking/dif culty: Category: Screening

Screening lm mammogram, right CC view 2006. Screening digital mammography, right CC view 2011.

Answers
1. The exam rom 2011 does not demonstrate any Pearls
abnormality in comparison with the exam rom 2009. • Digital mammography has superior contrast resolution,
whereas f lm mammography has an advantage o
2. The 2011 exam is digital, whereas the 2009 exam
spatial resolution.
is screen f lm mammography. The digital study
• Digital mammography has better sensitivity to detect
demonstrates better contrast resolution.
developing “asymmetries” in dense breast tissue.
3. Digital mammography has the advantage o better • Digital mammography also has higher sensitivity or
contrast resolution, which is help ul to detect developing detection o calcif cations.
malignancy in dense breast tissue.
4. Digital mammography has less spatial resolution then
f lm mammography—this was or a long time the limiting Suggested Readings
actor in establishing digital mammography. The advantage Karssemeijer N, Bluekens AM, Beijerinck D, et al. Breast
o better contrast resolution, however, has been shown to cancer screening results 5 years a ter introduction o
outweigh the disadvantage o less spatial resolution. digital mammography in a population-based screening
5. Switching rom reading screening f lm mammograms program. Radiology. 2009;253(2):353-358.
to reading screening digital mammograms requires to Lewin JM, Hendrick RE, D’Orsi CJ, et al. Comparison
adjust the threshold to recall patient or densities as a o ull-f eld digital mammography with screen-f lm
result o the increased contrast resolution and to adjust mammography or cancer detection: results o 4,945
the threshold to recall patient or calcif cations as well. paired examinations. Radiology. 2001;218(3):873-880.

16
Well woman screening exam

1. What BI-RADS classif cation should be used


here?

2. In which groups o women did the DMIST


study show benef t?

3. What is the dose o a digital mammogram


relative to analog?

4. Which o the ollowing are some o the


benef ts o digital versus analog?

5. What o the ollowing f ndings or structures are


better seen with digital mammography?

17
Analog versus digital comparisons 1728
Case ranking/dif culty: Category: Screening

Answers Philips MicroDose, which has approximately 50%


1. Normal dense breast tissue, better seen on digital. normal radiation dose. Digital allows you to perceive
The appropriate BI-RADS classif cation is there ore calcif cations much easier. A small digital reimbursement
1: negative or malignancy. The f rst time a digital supplement that is likely to disappear now as analog
mammogram is per ormed on the woman, there is a is virtually extinct. CAD has been available or analog
higher probability o f nding calcif cations that you are systems, but it is more e f cient on digital systems.
not sure could be seen on the analog mammogram. In 5. Most o the above are correct, and some may argue
that instance, you need to give a BI-RADS 0 and recall that all are correct, as the dynamic range o digital and
or workup. the contrast resolution o digital mammography makes
2. The DMIST study was per ormed to answer the question everything easier to see.
as to whether digital mammography was better than
analog mammography, but only really had to show
equivalence to gain acceptance. There were three groups Pearls
o women where some benef t was shown: those with
dense breasts, women younger than 50 years, and • Digital mammograms show normal breast tissue
perimenopausal women. There is some evidence that clearly, especially in patients with denser breast tissue.
digital mammography is less e ective than screen-f lm • Microcalcif cations are also much easier to identi y and
mammography in women with atty breasts. However, to characterize.
this is being addressed by some manu acturers, changing
their anodes to tungsten rom molybdenum.
Suggested Readings
3. It can vary by tissue type, but there has generally been
a drop in mean glandular radiation dose during the Kopans DB, Pisano ED, Acharyya S, et al. DMIST
switch to digital. Further reductions have also occurred results: technologic or observer variability? Radiology.
using photon counting techniques, where radiation 2008;248(2):703; author reply 703.
doses are approximately 50% lower than regular Pisano ED, Hendrick RE, Ya e MJ, et al. Diagnostic
mammograms. Tomosynthesis as a new technology accuracy o digital versus f lm mammography:
started at approximately three times the regular dose exploratory analysis o selected population subgroups
o mammography as the FDA required a regular in DMIST. Radiology. 2008;246(2):376-383.
mammogram in addition to the 3D exam. Measures Zuley M. How to transition to digital mammography. J Am
have been taken, which has reduced to approximately Coll Radiol. 2007;4(3):178-183.
1.6 times, and still within the 2 mGy FDA requirement.
4. The e ects on the environment and concerns over the
disposal o silver used to be a big concern. There is
little di erence in radiation dose compared with analog
mammography, except or a ew systems, or example,

18
Patient with screening exam

1. What is the workup or “round” mass seen on


f rst mammogram?

2. What is the next step i at cannot be


visualized?

3. What is the characteristic f nding o an


intramammary lymph node?

4. Where is the location o the mass?

5. What would you do i the mammogram does


not show at but ultrasound demonstrates a
large atty hilum?

19
Lymph node on rst screening 394
Case ranking/dif culty: Category: Screening

Diagnostic mammogram, left CC view demonstrates benign-


appearing mass with fat.

Pearls
Diagnostic mammogram, left MLO view demonstrates benign-
• Typical location or intramammary lymph node is the
appearing mass with fat.
upper outer quadrant; however, they can exist anywhere
in the breast.
Answers • I at can be identif ed on screening mammogram in
a well-circumscribed mass, it is in general consistent
1. Next step in general is workup with spot compression
with lymph node and mammogram can be classif ed
views.
as BI-RADS 2, benign.
2. Ultrasound can be used or urther characterization.
I that does not show any abnormality, the f nding is
probably benign and 6-month ollow-up mammography
Suggested Reading
is recommended. I ultrasound demonstrates the mass, it
can be urther characterized on ultrasound. I ultrasound Meyer JE, Ferraro FA, Frenna TH, et al. Mammographic
demonstrates cysts in internal echoes or thin membranes appearance o normal intramammary lymph nodes in
(“complicated cyst”), cyst aspiration or as an alternative an atypical location. AJR Am J Roentgenol. 1993;161:
6-month ollow-up and ultrasound surveillance over 779-780.
2 years is recommended.
3. It is generally located in the upper outer quadrant. I at is
seen on the mammogram, it is diagnostic or lymph node.
Lymph nodes are also sometimes located in other parts o
the breast. They are in general well circumscribed.
4. It is located slightly superior on MLO and very lateral on
CC view.
5. In this case, the ultrasound f nding likely does not
correlate to the mammogram. Mammogram should show
at as well. The ultrasound f nding is benign and does
not need ollow-up. The mammogram f nding does need
ollow-up i it is not new but seen on f rst mammogram
and does not contain def nitely at. Bottom line: i
ultrasound f nding does not correlate, 6-month ollow-up
mammogram is recommended or a well-circumscribed
mass seen on f rst mammogram.

20
Prior chemotherapy— nding of palpable right axilla

1. What BI-RADS classif cation should be used


here?

2. What is the next best imaging test?

3. Which o the ollowing are known causes


o bilateral axillary adenopathy?

4. From which cell line does Non-Hodgkin


lymphoma (NHL) arise?

5. How does primary breast lymphoma present?

21
Lymphoma patient 1646
Case ranking/dif culty: Category: Diagnostic

Axilla ultrasound shows one oval solid node with loss of the
normal hilum, and a second with di use thickening of the cortex
and e acement of the hilum.

Answers Note multiple enlarged lymph nodes in both axillae.


1. These lymph nodes are abnormal and do not look like
lymphatic spread rom a breast cancer. I the patient
is with a known lymphoma and under treatment, you Pearls
could use BI-RADS 2: benign or the breasts, but make
a note o the axillary f ndings. I not known about, then • Bilateral nodes seen in axilla on mammography are not
a BI-RADS 0 could be used to get the patient seen, and necessarily benign.
ultrasound o the axilla per ormed and possibly biopsied • Systemic disease can mani est itsel on mammograms.
so that they can be passed on to the hematologists. • Do not orget lymphatic disorders as a cause o axillary
adenopathy on mammograms.
2. In the setting o likely lymphoma, ollowing a needle
biopsy, a PET/CT may give the most staging in ormation
be ore a decision is made.
Suggested Readings
3. HIV-related lymphadenopathy, systemic in ammatory
Gorkem SB, O’Connell AM. Abnormal axillary lymph
condition, and lymphoma/chronic lymphocytic
nodes on negative mammograms: causes other than breast
leukemias can present with nodes. In ammatory breast
cancer. Diagn Interv Radiol. 2012;18(5):473-479.
cancer usually presents with unilateral lymphadenopathy.
Valente SA, Levine MD, Silverstein MD, et al. Accuracy
Breast abscesses can also present as unilateral
o predicting axillary lymph node positivity by
lymphadenopathy. Pelvic abscess does not usually
physical examination, mammography, ultrasonography,
present with axillary nodes.
and magnetic resonance imaging. Ann Surg Oncol.
4. NHL can arise rom any o the T or B line white cells, 2012;19(6):1825-1830.
progenitors, or more mature cells; 10% to 35% o Walsh R, Kornguth PJ, Soo MS, Bentley R, DeLong DM.
patients have extranodal primary lymphoma at the time Axillary lymph nodes: mammographic, pathologic,
o presentation. and clinical correlation. AJR Am J Roentgenol.
1997;168(1):33-38.
5. Very uncommon as breast primary, but breast commonly
involved when known systemic lymphoma. Painless
breast mass usually a ecting the right breast; 30%
to 40% have ipsilateral axillary adenopathy; average
age 55 to 60 years, and right breast more common
than le t.

22
Palpable abnormality

1. What is the f nding on the diagnostic


mammogram?

2. What is the next step in the workup o the


patient?

3. I ultrasound does not show any abnormality,


what is the next step?

4. What would be the next step i ultrasound


demonstrates hypoechoic nodule with internal
echoes?

5. What is the def nition o a simple cyst—i that


is the ultrasound f nding what is the next step?

23
Simple cyst 1307
Case ranking/dif culty: Category: Diagnostic

Gray-scale ultrasound demonstrating “anechoic,”“well- Gray-scale ultrasound demonstrating simple cyst.


circumscribed” mass with “posterior acoustic enhancement.”

Answers
1. Mammogram o right breast, including spot compression Pearls
views, does not show any def nite abnormality. • I ultrasound can prove that simple cyst explains lump
elt by the patient, the f nal assessment is “benign”-
2. Next step in workup is targeted ultrasound directed to
BI-RADS 2 and patient can return in 1 year or next
the right upper outer quadrant.
screening mammogram.
3. I ultrasound does not show abnormality, the f nal • I there is any doubt that this a simple cyst, as a
assessment is BI-RADS 1 negative, and a sentence result o the internal echoes or debris, it should be
should be added that “ urther assessment o the palpable called “complicated cyst” and cyst aspiration can
abnormality should be based on clinical grounds.” be per ormed.
I patient is high risk or i the clinical f ndings are • Alternative management can be to ollow “complicated
overwhelming and the breast parenchyma is dense and cyst” over 2 years with ultrasound and call it “probably
might obscure abnormality, MRI might be an option as benign,” in particular in case o more than one
problem-solving tool. “complicated cysts.”
• I there are scattered cysts bilaterally o which some
4. In case o corresponding hypoechoic mass with
are “complicated,” they can be called “benign” and
internal echoes but without ow on duplex, this is
no ollow-up is necessary.
called “complicated cyst” and cyst aspiration should be
• I there are mural nodules at the wall, or i there
per ormed. Since the mass was not seen on mammogram,
was thickening o the wall or the presence o thick
there is no need to repeat mammogram. Alternative
membrane, f nding is called “complex mass” and core
could be to call “complicated cyst” “probably benign”
biopsy should be per ormed.
and per orm ollow-up ultrasound in 6 months, another
a ter 6 months, and then a ter 1 year.
5. Simple cyst is def ned as round and oval “well-
Suggested Readings
circumscribed,” “anechoic” mass with “posterior
acoustic enhancement.” This is the description o the Berg WA, Campassi CI, Io e OB. Cystic lesions o the
f nding seen on this particular patient. Assessment is breast: sonographic-pathologic correlation. Radiology.
BI-RADS 2, benign, and patient can return to normal 2003;227(1):183-191.
screening exam. Dennis MA, Parker SH, Klaus AJ, Stavros AT, Kaske TI,
Clark SB. Breast biopsy avoidance: the value o normal
mammograms and normal sonograms in the setting o a
palpable lump. Radiology. 2001;219(1):186-191.
Rinaldi P, Ierardi C, Costantini M, et al. Cystic breast
lesions: sonographic f ndings and clinical management.
J Ultrasound Med. 2010;29(11):1617-1626.

24
Screening—asymptomatic

1. What BI-RADS classif cation should be used


here?

2. What are the potential pathologies based on the


imaging?

3. What should be the next imaging investigation?

4. How should the lesion be managed i it turns


out to be a cystic lesion?

5. I you aspirate a cyst, should you send the uid


or cytology?

25
Multiple cysts 582
Case ranking/dif culty: Category: Screening

Answers
1. This is a characteristic f nding, where using the Pearls
‘multiple masses’ f nding note allows you to give this • Circumscribed mass in young woman likely to be
a BI-RADS 2. I you are uncertain about any o the either a cyst or f broadenoma.
masses, with indistinct margins, or the mass is partially • PROVISO: triple negative breast cancer can present as
obscured, you should give the patient a BI-RADS 0 and circumscribed masses in young women, although rare
per orm a diagnostic workup to include spot views and in everyday practice.
ultrasound scanning. • Ultrasound is the quickest, easiest, and non ionizing
test to rule out a solid mass, and conf rm a cyst.
2. The most common cause o these f ndings are cysts and
f broadenomas, sometimes a mix o both. Metastases
have circumscribed margins and should be considered
in the presence o a known OTHER cancer, which could Suggested Readings
potentially metastasize to the breast. Rarely a triple Lister D, Evans AJ, Burrell HC, et al. The accuracy o breast
negative ductal cancer may present as a circumscribed ultrasound in the evaluation o clinically benign discrete,
mass, but does not tend to have the appearances as o symptomatic breast lumps. Clin Radiol. 1998;53(7):
this exam. 490-492.
3. I you have not used the multiple masses, and given a BI- Shetty MK, Shah YP. Sonographic f ndings in ocal
RADS 2, then the next test should be a routine ollow-up f brocystic changes o the breast. Ultrasound Q.
mammogram (1 year in the United States). I the patient 2002;18(1):35-40.
has a palpable lump being worked up, then an ultrasound
may be the best f rst-line investigation, as we need to
conf rm whether the lump is cystic or solid. For margins,
a single tomosynthesis projection is showing promise in
the workup o women with masses.
4. Simple cysts can come and go rapidly, changing even
day to day. Some cysts remain over a long period,
developing thick proteinaceous debris, which may show
as a snowstorm appearance on ultrasound. Sometimes,
this debris is adherent to a cyst wall, and prompt short-
term surveillance or even biopsy. Historically, cysts were
sometimes surgically excised.
5. Cyst aspirations are not routinely per ormed anymore. It
may help i the uid is bloody, but this is usually due to a
traumatic tear o a small vessel around the cyst wall, and
not related to the cyst at all (a bloody tap—especially at
the end o aspiration).

26
Screening—asymptomatic

1. What BI-RADS classif cation should be used


here?

2. What is the most likely pathology based on the


imaging?

3. What is the next best imaging test?

4. What type o intervention would you


recommend?

5. What is the risk o breast cancer in patients


with multiple cysts?

27
Multiple masses—cysts or broadenomas 586
Case ranking/dif culty: Category: Screening

Pearls
• Multiple noncalcif ed masses in the breast are a benign
f nding, and are described in the BI-RADS manual as a
special case.
• To meet the criteria, the masses have to be
circumscribed, not calcif ed, and to have at least two
on one side and one on the contralateral breast.

Suggested Readings
Berg WA, Sechtin AG, Marques H, Zhang Z. Cystic breast
masses and the ACRIN 6666 experience. Radiol Clin
North Am. 2010;48(5):931-987.
Multiple simple cysts in both breasts con rmed on ultrasound
exam.
Chang YW, Kwon KH, Goo DE, Choi DL, Lee HK,
Yang SB. Sonographic di erentiation o benign and
malignant cystic lesions o the breast. J Ultrasound Med.
Answers 2007;26(1):47-53.
1. The multiple, noncalcif ed, circumscribed, bilateral Heinig J, Witteler R, Schmitz R, Kiesel L, Steinhard J.
masses are likely due to f broadenomas or cysts. Accuracy o classif cation o breast ultrasound f ndings
A BI-RADS 2, benign, note is appropriate. based on criteria used or BI-RADS. Ultrasound Obstet
Gynecol. 2008;32(4):573-578.
2. The f ndings are most likely cysts or f broadenomas,
although rarely you can get multiple phyllodes tumors
in both breasts; the key here is that those lesions are not
stable and tend to be rapidly growing. Metastases to
the breast can look identical to this, but is in the setting
o a known “other” cancer that has the potential to
metastasize to the breast.
3. I you wish to work this up, then ultrasound on its own
can distinguish between solid and cystic masses. Some
claim that tomosynthesis has a role here, but the data
are currently lacking. As this is a benign f nding, routine
mammograms are indicated.
4. No intervention is required or multiple benign lesions.
5. There was a reported statistical relationship between
simple cysts and subsequent breast cancer, likely related
to the sensitivity o breast tissue to circulating estrogens,
but this has not been validated, and does not reach the
risk levels associated with hyperplastic or borderline
neoplastic lesions o the breast.

28
Screening—asymptomatic

1. What BI-RADS classif cation should be used


here?

2. What is the inheritance pattern o


neurof bromatosis?

3. Which chromosome is a ected by the NF-1


mutation?

4. What type o breast biopsy should be


per ormed to conf rm the diagnosis?

5. What are the other supportive eatures or the


diagnosis o NF-1?

29
Von Recklinghausen disease —type 1 neuro bromatosis 1864
Case ranking/dif culty: Category: Screening

Answers
1. This is a benign f nding, and there ore a BI-RADS 2
assessment would be appropriate.
2. The de ect on chromosome 17 arises by autosomal
dominant genetics, but 50% occur by spontaneous
mutation.
3. Chromosome 17 has the locus or the mutated gene
a ected in NF-1.
4. No biopsy o the breast needs to be per ormed to
conf rm the diagnosis. The diagnosis is usually obvious
on physical examination. Also, they may be under
preexisting care or known neurof bromatosis.
5. The NF-1 gene is on chromosome 17 and a ects cell
signaling. As a result, there is overgrowth causing
benign tumors and also scoliosis (in 20%) or other limb
de ormities. Epilepsy is observed in approximately 7% o
patients. Learning di f culties or other psychological issues
are common. The gastrointestinal system is generally not
involved with tumors in NF-1. Tumors are more common
in the nervous system with plexi orm neurof bromas, and
schwannomas. Pheochromocytoma is a complication.

Pearls
Close-up view of irregular, lobulated, overgrowth of nipple, and • Cutaneous masses. Usually, this is obvious when you
two circumscribed super cial masses. look at the patient.
• A de ect in the NF-1 gene on chromosome 17 (type 1
neurof bromatosis).
• Autosomal dominant pattern o inheritance, but up
to 50% o NF-1 cases arise because o spontaneous
mutation.
• 1:3500 live births

Suggested Readings
Cao MM, Hoyt AC, Bassett LW. Mammographic signs o
systemic disease. Radiographics. 2012;31(4):1085-1100.
Goksugur N, Gurel S. Neurof bromatosis o nipple-areola
complex. Breast J. 2012;17(4):424.
Sherman JE, Smith JW. Neurof bromas o the breast and
nipple-areolar area. Ann Plast Surg. 1981;7(4):302-307.

30
Screening mammogram—status post–right lumpectomy many years ago

1. What is the signif cance o the lesion in the


right breast?

2. What is the diagnosis?

3. How can at necrosis appear on mammogram?

4. I there is concern or recurrent malignancy,


what would be the next step?

5. I there is remaining concern or recurrent


malignancy, what would be the next step?

31
Fat necrosis 737
Case ranking/dif culty: Category: Screening

Current mammogram, right spot compression CC view


demonstrating retraction of the scar and development of “coarse,
heterogenous” calci cations.

postoperative enhancement due to granulation tissue


rom enhancement as a result o recurrent tumor.

Old mammogram, right spot compression CC view demonstrating


fat necrosis. Pearls
• Fat necrosis is a benign in ammatory process, mostly
related to prior surgery or trauma.
Answers • Mammographic eatures o at necrosis include the
1. Focal area does contain at and is a benign f nding. This presence o lipid cysts, microcalcif cations, coarse
is a benign lesion BI-RADS 2. Correlate with history— calcif cations, and sometimes spiculated areas o
patient had lumpectomy. increased density.
• Lipid cysts are round and oval lucent masses with
2. This is typical appearance o scar a ter lumpectomy with
thin rim that may or may not be calcif ed and are
at necrosis.
unequivocally benign.
3. Fat necrosis can present a wide variety o morphology, • Fat necrosis can also present in the orm o
o which some are more pathognomonic and some are microcalcif cations that might be even “pleomorphic”
more di f cult to distinguish rom possible malignancy. in shape and cannot be distinguished rom malignancy
I there is at within the ocal f nding or the abnormality and biopsy is warranted.
contains “coarse and heterogeneous” calcif cations, • I f brosis is the dominant eature o at necrosis, it can
it is relatively specif c or at necrosis. However, i appear spiculated in shape.
the microcalcif cation or the mass is spiculated, it is • “Heterogenous and coarse” calcif cations are common
unspecif c and sometimes biopsy is warranted. eature o at necrosis but o ten times require additional
monitoring and might be called “probably benign,”
4. Diagnostic mammogram with spot compression MLO
depending on the morphology.
and CC views or, i calcif cations are the concern, workup
with magnif cation ML and CC views. In some breast
centers, all patients a ter lumpectomy receive diagnostic
mammograms in the f rst place, which o ten times includes Suggested Readings
spot compression or magnif cation views in the f rst place. Hogge JP, Robinson RE, Magnant CM, Zuurbier RA. The
5. The best test or questionable recurrent malignancy mammographic spectrum o at necrosis o the breast.
would be to per orm breast MRI—however, there Radiographics. 1995;15(6):1347-1356.
should be at least 6-month time interval between Taboada JL, Stephens TW, Krishnamurthy S, Brandt KR,
surgery and MRI; otherwise, it is di f cult to distinguish Whitman GJ. The many aces o at necrosis in the breast.
AJR Am J Roentgenol. 2009;192(3):815-825.
32
Palpable lump in the left breast

Right MLO. Left CC.

Right CC. Left MLO.

1. What is the BI-RADS category or this 4. Which o the new technologies in breast
diagnostic exam? imaging is likely to help our diagnosis o
f broadenoma?
2. Which o the ollowing are the Stavros benign
criteria? 5. What types o biopsies can make the diagnosis
o f broadenoma?
3. What is the risk o malignancy in a
circumscribed mass in a woman younger
than 25 years?

33
Fibroadenoma in young women 602
Case ranking/dif culty: Category: Diagnostic

5. I you have a trained breast cytologist (more common


in Europe than in the United States), then a diagnosis
o f broadenoma can be made. All other types o biopsy
can assist making the diagnosis o f broadenoma, except
perhaps an incisional biopsy, which takes superf cial
tissue (eg, in patients with in ammatory breast cancer
with no known primary, or in suspect Pagets disease o
the nipple).

Ultrasound shows an “oval mass”with a “narrow zone of transition” Pearls


(circumscribed). The echo texture is predominantly homogeneously • Oval circumscribed mass in less than 25 years age
hypoechoic, with some smaller cystic spaces within. group may be diagnosed on ultrasound alone. Biopsy
is not required.
Answers • Over 25 years, you can choose surveillance to
determine stability, or i there are any suspicious
1. There are various approaches to a circumscribed benign- eatures, proceed to ultrasound-guided core biopsy
looking mass on both mammography and ultrasound. or diagnostic vacuum-assisted excision.
The patient is very young and the likely scenario is a
f broadenoma. Malignancy is very unusual in this age
group, and some groups advocate using ultrasound only.
According to BI-RADS, i this is the f rst visit, and the Suggested Readings
eatures are entirely benign ( ulf ll the Stavros criteria), Hamilton L, Evans A, Corn ord E, James J, Burrell H.
then the patient may be given a BI-RADS 3 and 6-month Ultrasound diagnosis o f broadenoma—is biopsy always
ultrasound ollow-up per ormed or stability. Some necessary? Clin Radiol. 2008;63(9):1070-1071.
groups advocate or biopsy every solid mass at any age, Stavros AT, Thickman D, Rapp CL, Dennis MA, Parker
and there ore would use the BI-RADS 4 category. SH, Sisney GA. Solid breast nodules: use o sonography
2. Stavros criteria or a benign lesion: (1) up to two or three to distinguish between benign and malignant lesions.
gentle lobulations; (2) thin echogenic pseudo capsule; Radiology. 1995;196:123-134.
(3) intense hyperechogenicity; (4) ellipsoid shape; Tagaya N, Nakagawa A, Ishikawa Y, Oyama T, Kubota K.
(5) the absence o malignant eatures. Experience with ultrasonographically guided vacuum-
assisted resection o benign breast tumors. Clin Radiol.
3. The pretest probability o cancer in an under 25 years 2008;63(4):396-400.
old is very low. A benign-looking mass that ulf lls
the criteria or a f broadenoma on ultrasound can be
sa ely ollowed with physical examination, and does
not need a biopsy, i remains stable. Over the age o
25, the malignancy rate climbs to between 1% and
2% (BI-RADS 3) and there ore these lesions may be
biopsied or ollowed with short-term ollow-up.
4. Many o the new technologies are being investigated
as an adjunct in characterization o lesions. There
is requently an overlap between f ndings in benign
and malignant lesions with the same morphologic
characteristics; there ore, none o these are currently
used in the place o biopsy. Power Doppler helps
to identi y the blood vessels, which are said to be
characteristically at the periphery. In young women,
however, the f broadenoma may be actively growing
and contain large vascular channels.

34
21-year-old patient with palpable lump

1. What is the reason to biopsy this mass that is


palpable and circumscribed?

2. I this mass was an incidental f nding on an


ultrasound, what would be the next step?

3. What di erentiates a phyllodes tumor rom


a f broadenoma?

4. What would be the management i there are


two masses like this—one palpable and one
was an incidental f nding?

5. What would be the management i there were


multiple palpable benign-appearing masses?

35
Phyllodes tumor 379
Case ranking/dif culty: Category: Diagnostic

Ultrasound-guided core biopsy with 12-gauge needle.


Mammogram of left ML view Mammogram of left CC view
with mass and clip after with mass and clip after biopsy.
Answers biopsy.
1. Any new palpable mass can be considered suspicious
due to the act that it can be assumed that it has grown
in size, since it was not palpable be ore. However, there
Pearls
are some radiologists who argue that a benign-appearing
mass in a young patient is most likely a f broadenoma, • The histological appearance o phyllodes tumors may
even i it is palpable and could be ollowed in 6 months. be the same as o large f broadenomas—although they
However, during the time o monitoring, any signif cant have greater cellularity and cell activity.
increase in size would then trigger biopsy. • Malignant behavior o phyllodes tumors, which can
include metastasis to the lungs, is extremely rare. Most
2. It would be BI-RADS 3, “probably benign,” and patient malignant phyllodes tumors reported in the literature
would be advised to return or ultrasound ollow-up in 6 had on histology an obvious sarcomatous element.
months, then, i f nding is unchanged, 6 months later again, • Incomplete excision o phyllodes tumors has been
and then i stable, 1 year later. This results in time period stressed as a major determinant or local recurrence.
o observation o 2 years. During that time, the f nding
remains “probably benign”—a ter 2 years, i the f nding
is unchanged, it can be classif ed as “benign” BI-RADS 2
and there is no need or urther monitoring. Suggested Readings

3. It shows in general histology with higher cellular activity Barsky S, Gradishar W, Recht A, et al. The Breast. 4th ed.
and cellularity. Local recurrence rate is higher and in rare Saunders Elsevier USA; 2009.
cases, there can be metastasis, or example, to the chest. Buchberger W, Strasser K, Heim K, et al. Phyllodes
tumor f ndings on mammography, sonography and
4. In this case, it would be reasonable to biopsy the aspiration cytology in 10 cases. AJR Am J Roentgenol.
palpable mass—again under the assumption that it 1991;157(4):715-719.
has grown (was not palpable be ore) and ollow the Guillot E, Couturaud B, Reyal F, et al. Management o
incidental, benign-appearing solid mass in 6 months. phyllodes breast tumors. Breast J. 2011;17(2):129-137.
5. It would not be unreasonable is in a young patient, where
these well-circumscribed masses are most likely all
f broadenomas, to biopsy the largest o the f ndings and
ollow the rest in 6 months. MRI is not the f rst choice—
unless there is a strong amily history, then it might be
considered as an additional “problem solving” modality
in that particular case.

36
Palpable lump in the right breast

1. Why should a technical repeat be considered


appropriate in this case?

2. What should be the next diagnostic imaging


exam?

3. What is the likely pathology o a circumscribed


mass?

4. Which o the ollowing BI-RADS descriptors


is supportive o a benign diagnosis?

5. Which f ndings avors phyllodes tumor over


f broadenoma?

37
Fibroadenoma versus phyllodes tumor 1870
Case ranking/dif culty: Category: Diagnostic

Targeted ultrasound shows a predominantly circumscribed


mass with one edge that shows a lobulated area. Another view
showed that this was artifactual, and the lesion was completely
circumscribed.

Right ML spot lms show that this mass is circumscribed


with gentle lobulations. but also can occasionally be ound in malignant lesions.
A “narrow zone o transition” is the opposite o a
hyperechoic rim you see around some cancers.
Answers
5. A phyllodes tumor may be identical to a large
1. This is a diagnostic workup and not a screening due f broadenoma, and have large vascular channels through
to the age o the patient, and the palpable mass. A it. Also, it may have wide channels where the ducts are
BI-RADS 0 is appropriate at this stage until urther not as distorted as they are in f broadenomas, giving the
workup is per ormed, even though in reality the patient lea -like architecture seen on gross pathology.
has not completed the mammographic workup.
2. As this patient is young, and there is a mass, you could
go direct to ultrasound. I you wish to see the margins
Pearls
clearly to ully characterize the mass be ore ultrasound,
then tomosynthesis or diagnostic mammograms with • “Circumscribed noncalcif ed mass.”
spot (+/− magnif cation) views are recommended. MRI • Likely cyst or f broadenoma—conf rm with diagnostic
will give a lot o in ormation, but at this stage, it is not workup.
necessary. Some groups may consider PEM i the mass
was not visualized, but there is a hard mass present. The
downsides are the radiation dose rom the isotope injection. Suggested Readings
3. The key here is the “most likely” pathology. Cysts and Chao TC, Lo YF, Chen SC, Chen MF. Sonographic eatures
f broadenomas are common at this age. All the other o phyllodes tumors o the breast. Ultrasound Obstet
diagnoses listed are in the di erential diagnosis or Gynecol. 2002;20(1):64-71.
this f nding, but urther workup is required. I this were Jacobs TW, Chen YY, Guinee DG, et al. Fibroepithelial
a large and solid circumscribed mass, then phyllodes lesions with cellular stroma on breast core needle biopsy:
tumor is much higher in the list. Ductal carcinoma in situ are there predictors o outcome on surgical excision? Am
(DCIS) can cause a noncalcif ed mass and can simulate a J Clin Pathol. 2005;124(3):342-354.
f broadenoma, but it is rare in a radiologist’s practice. Veneti S, Manek S. Benign phyllodes tumour vs
4. “Hypoechoic” is commonly used but does not help f broadenoma: FNA cytological di erentiation.
as a descriptor to di erentiate benign rom malignant Cytopathology. 2001;12(5):321-328.
morphology. “Acoustic enhancement” is more
commonly ound in benign solid masses and with cysts,

38
High-risk screening—family history. No change from prior lms

1. What is the BI-RADS category or this


screening exam?

2. I no prior f lms are available, what BI-RADS


category should you give?

3. On prior workup, an ultrasound shows a


circumscribed oval mass with homogenous
echo pattern. What is the diagnosis?

4. I this is a new mass on mammography, what


would be your recommendations?

5. Core biopsy is reported as a f broepithelial


lesion. What is your recommendation?

39
Stable broadenoma mammo 610
Case ranking/dif culty: Category: Screening

Ultrasound shows an oval circumscribed mass, which is parallel to


the skin. There is an apparent notch on this single image, but on
LCC spot magni cation con rms that the lesion has a clean sharp
others, it was truly oval. If you see a notch, then you can turn on
margin. The overall descriptors are therefore “oval circumscribed
power Doppler and see a vessel going into the fatty hilum, which
mass.”
would con rm your suspicions that it was a lymph node.

Answers 5. A f broepithelial lesion is a high-risk lesion and may


1. This lesion was present on multiple prior examinations. prove to be either f broadenoma or phyllodes tumor at
It was stable in size and shape. The appearances are surgical excision. Age may be a actor in this patient being
consistent with a cyst or f broadenoma. I it has been young, but there is a signif cant overlap between the risk
present or over 3 years and stable, some readers would o f broadenoma and phyllodes based on age range.
call this BI-RADS 1: negative.
2. This lesion has a low risk o malignancy, but i urther
examination conf rms the presence o a cyst, then Pearls
nothing urther needs to be done. Some malignancies can • Fibroadenomas are the most common f nding in
present as a “benign” f nding; there ore i a f rst visit, adolescent girls and young women.
you need to recommend urther workup. I it is solid and • I not excised, they atrophy postmenopause, when they
meets the Stavros criteria or a benign lump, then you tend to calci y with characteristic dystrophic “popcorn”
can give it BI-RADS 3 and use short-term ollow-up or calcif cations.
a couple o years or stability. • I greater than 3 cm, o ten excised to exclude phyllodes
3. The patient has no eatures o pregnancy or lactation- tumor.
related change, and there ore the f rst two answers are • Very rare risk o malignant phyllodes trans ormation.
unlikely. Phyllodes tumor is usually diagnosed in a
growing “f broadenoma” (ie, documented evidence o
interval change). Mucinous carcinoma typically has Suggested Readings
slightly ill-def ned margins on ultrasound, and may have
Harvey JA, Nicholson BT, Lorusso AP, Cohen MA, Bovbjerg
a very heterogeneous echo pattern.
VE. Short-term ollow-up o palpable breast lesions with
4. I you have tomosynthesis, then this may be the only tool benign imaging eatures: evaluation o 375 lesions in 320
you need to achieve analysis o the margins o the mass. women. AJR Am J Roentgenol. 2009;193(6):1723-1730.
I not, then spot (magnif cation) views to evaluate the Nishimura R, Taira N, Sugata S, Takabatake D, Ohsumi S,
mass margins should be per ormed. MRI is unlikely to Takashima S. Suspicious calcif cations in benign breast
add anything to the diagnosis at this point. Ultrasound- lesions: a radio-pathologic correlation. Breast Cancer.
guided core biopsy is going to be required, as a 2011;18(1):33-36.
developing mass has a much higher pretest probability Sabate JM, Clotet M, Torrubia S, et al. Radiologic evaluation
or malignancy. o breast disorders related to pregnancy and lactation.
Radiographics. 2007;27(Suppl 1):S101-S124.

40
Patient with history of multiple moles— rst screening mammogram

1. Why should a technical repeat be considered


appropriate in this case?

2. What is the appropriate workup o the mass in


the upper outer quadrant, i it is not a mole?

3. What are typical BI-RADS 3 lesions?

4. What is the management o a probably benign


lesion?

5. What are other typical BI-RADS 3 lesions


a ter diagnostic workup?

41
Multiple masses on rst screening mammogram—most are moles 305
Case ranking/dif culty: Category: Screening

Screening mammogram of left Screening mammogram of left Screening mammogram of Screening mammogram of
MLO view of patient with moles. CC view of patient with history of left MLO view with marker on left CC view with marker on
Noted are several benign- moles. Noted are several benign- multiple moles. However, one multiple moles. One mass is not
appearing masses. Nipple not appearing scattered masses. mass is not a mole. a mole. Nipple now is in pro le.
in pro le (arrow). Nipple not in pro le (arrow).

Answers 5. The three classical mammogram BI-RADS 3 lesions


1. Because the nipple is not in prof le in both views. are as ollows: (i) a round and oval mass (as in our case)
on a baseline mammogram a ter diagnostic workup,
2. The f rst step is a diagnostic mammogram that includes a (ii) group o oval and round calcif cations on a baseline
spot compression MLO and CC view to assess the margin mammogram a ter diagnostic workup, and (iii) ocal
o the lesion. asymmetry on baseline mammogram a ter diagnostic
3. There is no BI-RADS 3 on any screening exam! workup without suspicious f nding on ultrasound.
Screening exams should only result in BI-RADS 0 All o these lesions should be biopsied i new or i on
(recall)—BI-RADS 1 (normal) or BI-RADS 2 (benign). urther ollow-up exams show any signif cant change.
I there are masses like in this patient on a baseline exam In addition, “complicated cyst” can also be ollowed
and additional workup does not show any corresponding as BI-RADS 3, or it can be aspirated.
suspicious eatures and ultrasound is normal and does
not show a corresponding f nding, it is a classic example
o a probably benign f nding (BI-RADS 3). Pearls
4. The purpose o the “probably benign” category is • Mole markers are help ul to di erentiate real mass
to prove stability o a lesion def ned as having a rom mole(s).
probability o malignancy o less than 2%. For that • I there is mass on f rst mammogram and f nding has
purpose, the most decisive images should be chosen “circumscribed margins” and is “round and oval”
as the ollow up image. The ollow-up should include and ultrasound is unremarkable, f nding is “probably
magnif cation views, in case o calcif cations, and benign,” BI-RADS 3.
spot compression views, in case o “mass” or “ ocal • According to the BI-RADS lexicon, probably benign
asymmetry.” Follow-up should cover 2 years. The f nding will be monitored over 2 years or even 3 years
best approach is a ter the initial workup and next with spot compression views to prove stability—a ter
unilateral diagnostic mammogram in 6 months, then 2/3 years o being stable, it can be classif ed as “benign”
next bilateral diagnostic mammogram another 6 months BI-RADS 2 and patient can return to screening.
later (12 months a ter the initial workup), and then the
last ollow-up diagnostic workup another 12 months
later (24 months a ter initial workup). Each diagnostic Suggested Reading
workup needs to include the additional magnif cation or Sickles EA. Probably benign breast lesions: when should
spot compression views. ollow-up be recommended and what is the optimal
ollow-up protocol? Radiology. 1999;213(1):11-14.
42
Prior breast cancer surgery—now palpable nodule near scar

1. What BI-RADS classif cation should be used


here?

2. What are causes o oil cysts in the breasts?

3. What is the next best imaging test?

4. What BI-RADS descriptors would you apply


to this lesion?

5. What are the long-term sequelae o oil cysts?

43
Palpable oil cyst 1610
Case ranking/dif culty: Category: Diagnostic

Left CC spot magni cation view with BB marker. Palpable marker


con rmed to lie immediately over the oil cyst. The imaging
Left lateral spot magni cation view with BB marker overlying the features are diagnostic.
calci ed oil cyst.

Answers scars have been associated with trauma, and it is not


1. Classical f ndings o a calcif ed oil cyst. A benign known i this is another post trauma variant process that
f nding. causes both conditions.

2. Well, we had to slip in an easier question. Oil cysts


in the breast are very common. Just read a day o
screening mammograms and you will observe your air Pearls
share. Minimal trauma, through seat belt injury and • Calcif ed oil cyst is diagnostic on mammography.
miscellaneous others. • No urther tests required.
3. No urther workup is warranted when the f ndings like • I you per orm ultrasound, you get a hard shadowing
this are characteristic. mass, which may make you call this harmless
abnormality suspicious.
4. This is a type o at necrosis and typically we talk about
egg-shell calcif cations, which BI-RADS re ers to as
“curvilinear.” Linear calcif cations are used in suspicious
Suggested Readings
areas o calcif cations. The actual calcif c particles
within the wall o the oil cyst can appear amorphous and Bilgen IG, Ustun EE, Memis A. Fat necrosis o the breast:
pleomorphic, but the best f t to a diagnosis o oil cyst is clinical, mammographic and sonographic eatures. Eur J
“curvilinear.” Radiol. 2001;39(2):92-99.
Harvey JA, Moran RE, Maurer EJ, DeAngelis GA.
5. There is no increased risk o breast cancer in a
Sonographic eatures o mammary oil cysts. J Ultrasound
patient with underlying oil cysts. In most cases, oil
Med. 1997;16(11):719-724.
cysts in screening populations are noted to resolve
Mendelson EB. Evaluation o the postoperative breast.
spontaneously. Rarely, they may go on to an established
Radiol Clin North Am. 1992;30(1):107-138.
orm o at necrosis. Sarcomatous change does not occur.
This is more likely in metaplastic carcinoma. Radial

44
Palpated lump in the left breast

1. What BI-RADS classif cation should be used


here?

2. What is the most likely pathology based on the


imaging appearances?

3. What is the next best imaging test?

4. What type o biopsy should be per ormed?

5. Which radiological eature would lead you to


suspect sarcomatous change in a atty lump?

45
Epidermoid cyst 1796
Case ranking/dif culty: Category: Diagnostic

Spot left XCCL shows the palpable Left MLO close-up—as of XCCL. Ultrasound exam con rms an “isoechoic
nding to be a “fatty density” lesion mass” consistent with a fatty mass—lipoma
with a “thin sharply marginated” or epidermoid cyst.
cortex.

4. This lesion is entirely harmless, and unless the patient


requests that it be removed, the patient can sa ely
be returned to routine screening. No biopsy need be
per ormed.
5. Sarcomatous change in this type o lesion is very rare.
The only thing you have to be aware o is an increase
in solid component o the atty mass, or example, i a
lipoma that was completely lucent becomes more like a
hamartoma, with both solid and lucent areas.

Pearls
Ultrasound exam with power Doppler shows no evidence of • Classical appearances o lipoma or subdermal cyst.
abnormal ow. • Ultrasound is not REQUIRED or the diagnosis,
as this is typical enough on mammography.
Answers
1. This is a benign f nding; there ore, a BI-RADS 2
assessment is appropriate. I the f nding has been stable Suggested Readings
or more than 3 years, some would ignore the f nding Adibelli ZH, Oztekin O, Gunhan-Bilgen I, Postaci H, Uslu
and give a BI-RADS 1 negative assessment. A, Ilhan E. Imaging characteristics o male breast disease.
2. The imaging appearances are similar in men and women, Breast J. 2010;16(5):510-518.
and in both entities. Sometimes, a lipoma can be elt as Herreros-Villaraviz M, Mallo-Alonso R, Santiago-Freijanes
a so t lesion, whereas inclusion cysts are usually under P, Díaz-Veiga MJ. Epidermal inclusion cysts o the breast.
high tension and are hard. Breast J. 2009;14(6):599-600.
Lam SY, Kasthoori JJ, Mun KS, Rahmat K. Epidermal
3. The images on mammography are characteristic enough inclusion cyst o the breast: a rare benign entity.
to call this benign and leave alone. In this case, the Singapore Med J. 2010;51(12):e191-194.
lesion was palpable, and so completion o a diagnostic
workup was per ormed, and also an ultrasound scan to
conf rm with the patient that what we were seeing was
what was being elt. (The BB marker already told us this,
but sometimes the patient asks or urther conf rmation
that they can see or themselves.)

46
40-year-old female — rst screening mammogram

1. What is the f nding on this f rst screening


exam?

2. When can a global asymmetry be called


benign?

3. What makes an asymmetry more concerning?

4. When patient returns or diagnostic workup,


what is the f rst step?

5. What is the next step a ter ultrasound does not


show any suspicious f nding?

47
Focal asymmetry 340
Case ranking/dif culty: Category: Diagnostic

Ultrasound, left upper outer quadrant


demonstrating incidental nding of simple
cysts.

Diagnostic mammogram, spot compression Diagnostic mammogram, spot


left MLO view con rms the presence of focal compression left CC view con rms the
asymmetry. presence of focal asymmetry.

Answers
1. Mammogram demonstrates the le t breast “ ocal Pearls
asymmetry” in the upper outer quadrant. “Global • I ocal asymmetry does not show any underlying
asymmetry” would cover more than one quadrant. distortion or mass on spot compression views nor
The small mass le t superior breast on the MLO is small any abnormality on ultrasound and i patient does not
intramammary lymph node. eel lump in that area, f nding is most likely normal
f broglandular tissue and can be classif ed as BI-RADS
2. A global asymmetry is most likely normal f broglandular
3 and can be ollowed over a time period o 2 years.
tissue i on additional spot compression views there is
• A ter monitoring or a period o 2 years, it can be
no underlying distortion, calcif cations, or mass and
called benign, BI-RADS 2.
ultrasound is negative and there is no palpable mass
• I the patient would eel a lump in that area, in general,
associated with it. It can then be called BI-RADS 2
the presence o corresponding ocal asymmetry is more
“benign” or i it remains still questionable, it could be
concerning and stereotactic biopsy is required.
called “probably benign” BI-RADS 3 and ollowed in
6 months and monitored over a time period o 2 years.
3. Any increasing asymmetry, any palpable abnormality,
Suggested Readings
and any other morphological suspicious eatures are
concerning and require biopsy. In case o a palpable Leung JW, Sickles EA. Developing asymmetry identif ed
abnormality in that area that correlates to the ocal on mammography: correlation with imaging outcome
asymmetry, despite normal ultrasound, patient should and pathologic f ndings. AJR Am J Roentgenol.
in general receive stereotactic biopsy. 2007;188(3):667-675.
Youk JH, Kim EK, Ko KH, Kim MJ. Asymmetric
4. The f rst step is to per orm le t spot compression CC mammographic f ndings based on the ourth edition
and MLO views. I there is no suspicious underlying o BI-RADS: types, evaluation, and management.
distortion or other abnormality, patient needs additional Radiographics. 2009;29(1):e33.
ultrasound or urther workup.
5. In case o negative ultrasound and negative diagnostic
mammogram, this is a classical BI-RADS 3 lesion and
patient needs to return in 6 months or le t MLO and CC
view and spot compression CC and MLO views to prove
stability o this “most likely benign” f nding. Small benign
simple cysts as seen on ultrasound in this case do not
change the approach—they are incidental benign f ndings.
48
Palpable nding in the right breast

1. What BI-RADS classif cation should be used


here?

2. What should be the next step i you perceive


the mammogram as showing no signif cant
f nding?

3. The image rom 1 year later can help you


determine that abnormality on the original
mammogram. What type o biopsy would you
recommend?

4. The core biopsy comes back as f brosis and


plasma cells. What do you do next?

5. A developing ocal asymmetry has what risk o


malignancy associated with it?

49
“Developing”“focal asymmetry” with palpable nding 1574
Case ranking/dif culty: Category: Screening

Spot magni cation of the “focal asymmetry” reveals its true Targeted ultrasound shows an “irregular mass” with “angular
suspicious character. margins” and dense “acoustic shadowing.”

Answers adequately, then repeating the biopsy may help to get


1. This is a diagnostic exam. There is a palpable marker accurate and representative tissue rom the mass. I you
over the area in the right upper outer quadrant at the used a 14-gauge core biopsy, it is time to bring out the “big
site o patient complaint. I you do not see something guns” and go or vacuum-assisted biopsy. I the biopsy was
underlying a palpable marker, then the next step should be already vacuum assisted, then surgical excision may be
to per orm a targeted ultrasound to determine i there is a appropriate. A problem-solving MRI is unlikely to help, as
f nding. The mammographic f nding in this case is a subtle your f ndings are already very suspicious.
ocal asymmetry with possible distortion. Spot views 5. Depending on the associated eatures, the average risk o
could have been per ormed to determine i there was malignancy o a developing ocal asymmetry is around
any distortion, which would have conf rmed a suspicious 20%. I there is associated segmental microcalcif cations
f nding, prompting urther investigation. A BI-RADS 4 with a developing ocal asymmetry, then this risk is more
assessment is the most appropriate in this situation. than 50%.
2. The next best examination with a palpable f nding is a
targeted ultrasound, combined with care ul palpation to
determine i the palpation f ndings are a correlate to the
Pearls
ultrasound f ndings. I all examinations are normal, but
you still are concerned, then a troubleshooting MRI may • Always work up a “developing ocal asymmetry”
help. You have already determined that the mammograms as high risk o malignancy compared with other
are negative, so you would not do more mammograms. mammographic f ndings.
3. You should be able to see the mass on ultrasound,
especially as there is a palpable f nding, and a BB or
Sharpie marker may be on the patient’s skin. Ultrasound Suggested Readings
remains the astest, cheapest, and best type o biopsy or Leung JW, Sickles EA. Developing asymmetry identif ed
the patient to have in most settings bar calcif cations or on mammography: correlation with imaging outcome
occult lesions. Ultrasound FNA may have a place i you and pathologic f ndings. AJR Am J Roentgenol.
have a breast cytopathologist, but this is not adequate or 2007;188(3):667-675.
preoperative tissue markers that are needed. Sickles EA. The spectrum o breast asymmetries: imaging
4. The f ndings are not concordant, especially as this is a eatures, work-up, management. Radiol Clin North Am.
developing lesion and f brosis does not explain the imaging 2007;45(5):765-771, v.
f ndings. You are responsible or ensuring that this case Venkatesan A, Chu P, Kerlikowske K, Sickles EA, Smith-
is managed appropriately. In a quest or a preoperative Bindman R. Positive predictive value o specif c
diagnosis, and the original biopsy was not sampled mammographic f ndings according to reader and patient
variables. Radiology. 2009;250(3):648-657.

50
Screening mammogram—any abnormality?

1. What is a diagnostic recall? Where is the


abnormality?

2. What is the expected recall rate?

3. What is the target cancer detection rate in the


United States in a screened population?

4. What will in uence the cancer detection rate?

5. What are strategies to lower recall rate?

51
Focal density not well covered, consistent with benign lymph node 169
Case ranking/dif culty: Category: Screening

Screening mammogram, left CC view demonstrates uncertain Screening mammogram, left XCCL view shows that the density of
density of lateral posterior breast. left lateral breast is consistent with benign lymph node.

Answers
Pearls
1. To work up an indeterminate abnormality seen on a good
quality screening mammogram in general includes spot • “Focal densities” seen only on one view o concern,
compression and/or magnif cation views. I screening as they might be outside the f eld in the corresponding
mammogram is o not appropriate quality, additional second view.
views can be added as technical repeat and the patient • In this particular case, a technical repeat with XCCL
can remain screening patient. In this particular case, i view, to include more tissue, did solve the issue.
the patient has le t the acility a ter screening, the exam • In the United States, the recall rate is supposed to be
can be called BI-RADS 0 and patient will be recalled or 10%. However, be ore a radiologist should adjust the
technical repeat and XCCL view will be obtained. There recall rate, f rst it is crucial to make sure that the cancer
is a ocal asymmetry seen on the le t CC view, lateral detection rate is in the expected range.
posterior breast. • The recall rate depends on many actors, including
screening penetration o the population, the presence
2. I a radiologist reads 100 screening mammograms and o prior images, and reading setup (immediate vs. batch
calls 10 patients back or workup o an abnormality, the reading).
radiologist has a recall rate o 10%.
3. The cancer detection rate in a screened population like
in the United States is 3 to 5 cancers per 1000 screening Suggested Readings
mammograms. Carney PA, Sickles EA, Monsees BS, et al. Identi ying
4. The cancer detection rate depends on the skill o the minimally acceptable interpretive per ormance criteria or
radiologist, as well as the quantity o exams read per screening mammography. Radiology. 2010;255(2):354-361.
year, which may be re ected in the recall rate. Ghate SV, Soo MS, Baker JA, Walsh R, Gimenez EI, Rosen
EL. Comparison o recall and cancer detection rates
5. There are multiple e orts that can reduce recall rate. or immediate versus batch interpretation o screening
Besides training to recognize calcif cations that can be mammograms. Radiology. 2005;235(1):31-35.
called benign on standard views (oil cysts, dystrophic
calcif cations, etc.), it is extremely help ul to maximize
the availability o old mammograms or comparison.
Also help ul is to train to recognize densities that are
caused by superimposed tissue.
52
Screening—asymptomatic

1. What is the BI-RADS category or this


screening exam?

2. I this was a diagnostic exam, what descriptors


would you use to describe the f ndings?

3. Based on the initial mammograms, where in


the breast do you expect to f nd the mass?

4. What is the likely f nal pathology in this


patient?

5. What additional mammographic views would


you order?

53
Small focal asymmetry in fatty breasts 680
Case ranking/dif culty: Category: Screening

Close-up of the mass shows the “indistinct margins.” Ultrasound shows small mass as a correlate to the mammographic
ndings.

Answers 5. The lateral exam can be per ormed in two directions.


1. There is a f nding that requires urther workup, and It is best to choose the one that puts the lesion closest
there ore this should be a BI-RADS 0. Although your to the bucky to reduce geometric unsharpness. Hence,
suspicion level may be high that this is a cancer, based an LM versus an ML exam. Spot f lms should be
on prior negative exams, it is possible that this might per ormed. These can be magnif ed to increase the
disappear on compression and urther workup; there ore, resolution, and the choice is dependent on the center you
to give it a BI-RADS 4 or 5 would commit the patient to practice in. My approach is to ask that any spot f lms
having an unnecessary biopsy. It is always best to wait are always magnif ed, to get the best resolution possible
to get more images and complete the mammographic rom the exam, and to characterize mass margins and
workup BEFORE you commit yoursel to the f nal calcif cations better.
BI-RADS assessment.
2. The abnormality is seen on two views and is there ore
a “ ocal asymmetry” (compared with a “one view Pearls
asymmetry”). I you are sure that the lesion is space • Easy to spot mass in atty breasts.
occupying, then you can use the term “mass.” In this • The borders are not seen clearly enough to be given a
case, the margins are indistinct. negative or benign screening result.
• Diagnostic workup conf rms that this is not a benign
3. Remember that the images shown are not orthogonal,
lesion.
and obtaining a lateral f lm will assist you in estimating
the position o the lesion or ultrasound exam. In the
MLO projection, a medial lesion o ten moves up on the
lateral, so although it currently appears at 3 o’clock, the Suggested Reading
lesion was actually seen at 2 o’clock on ultrasound. Venkatesan A, Chu P, Kerlikowske K, Sickles EA, Smith-
4. It is in the correct position or a sebaceous cyst, but Bindman R. Positive predictive value o specif c
malignancy has to be excluded. Fibroadenomas generally mammographic f ndings according to reader and patient
do not appear as masses in older women, but usually as variables. Radiology. 2009;250(3):648-657.
coarse dystrophic calcif cation, which in the early stages
can be developing coarse heterogeneous and there ore
prompt biopsy.

54
Screening—asymptomatic. Current and 1 year prior

1. What is the BI-RADS category or this


screening exam?

2. What is the next best examination you


recommend?

3. What is the risk o malignancy in a regional


asymmetry?

4. What type o biopsy should be per ormed?

5. When pregnancy has been completed, what


happens to the asymmetry?

55
Developing focal asymmetry secondary to pregnancy 678
Case ranking/dif culty: Category: Screening
Answers
1. Findings were similar on the MLO f lms, with an Pearls
asymmetric increase in breast density in the le t upper • Normal physiological changes in pregnancy can give
hal . No explanation is given or the recent increase in asymmetries (usually at least a regional asymmetry, to
asymmetric breast density. The patient does not report a global asymmetry).
any symptoms on her questionnaire. This is there ore • These are rapidly reversible ollowing childbirth and
a BI-RADS 0, needs urther workup. I you think this cessation o breast eeding.
is normal and had been stable, you could give it a • In postmenopausal women, you can get similar changes
BI-RADS 1, but we have evidence o developing change. with hormone replacement therapy.
2. It is important to understand the reason or the increase
in breast density in this young woman, and the best
way to do this is a complete history and physical exam Suggested Readings
be ore giving her any more radiation. An ultrasound
Canoy JM, Mitchell GS, Unold D, Miller V. A radiologic
examination was per ormed f rst, as we believed her
review o common breast disorders in pregnancy
to be an unreliable historian, and wanted to rule out
and the perinatal period. Semin Ultrasound CT MR.
pregnancy f rst. Ultrasound showed normal glandular
2012;33(1):78-85.
breast tissue.
Kizer NT, Powell MA. Surgery in the pregnant patient.
3. The risk o malignancy is very low with a regional Clin Obstet Gynecol. 2011;54(4):633-641.
asymmetry. In this case, however, it is a new f nding,
and would be better described as a developing regional
asymmetry, which is slightly more suspicious. The
risk is nowhere near as great as in developing ocal
asymmetries, which have a high PPV or malignancy.
4. No treatment is required or physiological breast changes
with pregnancy. I the area is palpable, some surgeons
wish to ollow up with interval physical exam and
ultrasound.
5. The postpregnancy response is either none, staying
stable, or a mild-to-marked reduction in f broglandular
volume as the physiological changes accompanying
pregnancy diminish. This response can be quite dramatic
in many women.

56
Screening—asymptomatic

1. What BI-RADS classif cation should be used


here?

2. What descriptors would you use or the


calcif cations present with the asymmetry?

3. What is the next best imaging test?

4. What type o biopsy should be per ormed?

5. I you see suspicious calcif cations with an


associated mass, what is the likely pathology?

57
Focal asymmetry with calci cations 1863
Case ranking/dif culty: Category: Screening

Right CC spot magni cation views. Right ML spot magni cation views Targeted ultrasound shows a hypoechoic “mass”
show an “ill-de ned mass” with with “acoustic attenuation (shadowing).” The
some possible “spiculation.” There is lesion is “parallel” and “sharply marginated,”
also some associated “pleomorphic but those BI-RADS descriptors do not t the
calci cation” at one edge of the mass. suspicious assessment you are about to make,
and therefore should not be used.

Answers
1. This is an abnormal screening exam, and so a BI-RADS Pearls
0 is appropriate as the f nding requires urther workup. A • “Developing ocal asymmetry” is high yield or a
suspicious BI-RADS assessment should not be given as cancer.
the f nding could be due to superimposition or a benign • Full mammographic workup, ollowed by ultrasound
f nding, and a diagnostic workup is required. i conf rmed.
• You need to explain any developing ocal asymmetry.
2. At this stage, the patient is having a screening exam, and
you do not have spot magnif cation views to characterize
the calcif c particles, so strictly you do not need to
describe the calcif cations. Best guess at this stage is that Suggested Readings
they are anything rom “amorphous” through “coarse Leung JW, Sickles EA. Developing asymmetry identif ed
heterogeneous” to “f ne pleomorphic.” on mammography: correlation with imaging outcome
3. Tomosynthesis has been proven to be good at and pathologic f ndings. AJR Am J Roentgenol.
di erentiating masses and asymmetries, but its utility 2007;188(3):667-675.
in characterizing calcif cations has not been proven. Sickles EA. Mammographic eatures o 300 consecutive
There ore, although tomosynthesis could be per ormed, nonpalpable breast cancers. AJR Am J Roentgenol.
you still need to per orm spot magnif cation views to 1986;146(4):661-663.
truly characterize the calcif c particles. Sickles EA. The spectrum o breast asymmetries: imaging
eatures, work-up, management. Radiol Clin North Am.
4. The most important thing is to diagnose the invasive 2007;45(5):765-771, v.
component o any disease, so ultrasound targeting o the
mass is the way to go. You can still x-ray the specimens
to see i you have harvested any calcif cations. I you
target the calcif cations with stereotactic core biopsy,
then you may miss the mass and the invasive disease.
5. The common pathological f nding or highly suspicious
calcif cations associated with a mass is a high-grade
invasive ductal cancer with high-grade DCIS. Low-grade
DCIS is much more likely to be amorphous or like the
benign calcif cations associated with LCIS.

58
Routine screening

1. What BI-RADS classif cation should be used


here?

2. What additional views do you need on a patient


with implants?

3. What are the signs o implant rupture on


ultrasound?

4. I there has been extracapsular rupture o


the silicone implant, which o the ollowing
eatures are seen on ultrasound?

5. In intracapsular rupture o a silicone implant,


which o the ollowing are characteristic signs
on MRI?

59
Prepectoral silicone implants 615
Case ranking/dif culty: Category: Screening

MRI is the best examination to look for implant


integrity, especially in women who have large
volume implants relative to their remaining breast
Right CC implant displaced (RCCID or Left CC implant displaced (LCCID tissue. You can see the normal radial folds of the
Eklund). Virtually impossible to get or Eklund). We can see a little more implants as wavy outlines to the implants. Silicone
good views of the right breast as the of the breast tissue on this implant saturation sequences can also be performed if you
implant will not displace further. displaced view, but still not ideal. are trying to determine whether a patient has a
rupture.

Answers contains the silicone, but the implant wall has de ated
1. This is a screening exam. There is no evidence o a and allen to the most dependent part o the capsule. The
malignancy. Describe the position o the implants and second is the teardrop sign, which re ers to the presence
the lack o f ndings. As there is a f nding, the appropriate o silicone both inside and outside o a radial old,
assessment is BI-RADS 2: benign. indicative o rupture. A dark f brous capsule is a normal
f nding. Reactive uid is commonly seen around textured
2. The implant displaced view, developed by Eklund, is the implants, but is not a sign o rupture.
most common additional exam. For ultrasound readers,
this means a di erence between a diagnostic and a
screening exam, as screening should be just CC and
MLO views (with the exception o occasional XCCL Pearls
views i needed to cover the breast tissue). However, as • Silicone gel breast implants commonly used by
the patient has no other problem, some centers do all the cosmetic surgeons.
views but still charge as a screening exam. Displacement • Placement o ten prepectoral (subglandular) as easy
views or screening are not routinely used in European to place.
screening programs. • Capsule o prepectoral implant may calci y.
3. The stepladder sign re ers to multiple, discontinuous, • May also occur with saline implants.
parallel, linear echoes in the lumen. It is the most
reliable ultrasound f nding in intracapsular rupture. It is
analogous to the linguine sign at MRI. A sidewinder is Suggested Readings
a type o missile. Neither serpentine or pasta signs help
here. “Pasta” re ers to the MRI “Linguine” sign. Friedman HI, Friedman AC, Carson K. The ate o the
f brous capsule a ter saline implant removal. Ann Plast
4. Silicone disperses sound and you get a marked snowstorm, Surg. 2001;46(3):215-221.
or white noise type o image. Frequently, granulomas Peters W, Pritzker K, Smith D, et al. Capsular calcif cation
may orm that contain cyst-like uid, but with oci associated with silicone breast implants: incidence,
o hyperechoic change with loss o posterior detail. determinants, and characterization. Ann Plast Surg.
Sensitivity o ultrasound or rupture with these signs 1998;41(4):348-360.
ranges rom 47% to 74% with a specif city o 55% to 96%. Peters W, Smith D, Lugowski S, Pritzker K, Holmyard D.
5. There are two signs on MRI o intracapsular rupture. Calcif cation properties o saline-f lled breast implants.
The f rst is the Linguine sign, where the capsule still Plast Reconstr Surg. 2001;107(2):356-363.

60
Pain in the right upper outer quadrant (the rst two gures on the left:
MLO and CC views; the next two gures on the right show additional
implant replacement views)

1. What is the workup in a patient with ocal


breast pain?

2. What is the best workup in a patient with


breast implant and ocal pain?

3. When is MRI indicated to work up implants


or rupture?

4. What is the major di erence between saline


implants and silicone implants or imaging?

5. What is the likelihood o implant rupture due


to spot compression view?

61
Saline implant—diagnostic workup 343
Case ranking/dif culty: Category: Diagnostic

Ultrasound of right breast lateral outer quadrant is


Diagnostic mammogram, right spot Diagnostic mammogram, right spot unremarkable as well.
compression MLO view does not compression CC view does not show
show any abnormality. any abnormality.

Answers 5. Spot compression views ought to be per ormed with


1. In a patient with ocal breast pain (patient can direct silicone and saline implants. Likelihood o rupture is
with index f nger to a specif c area), the patient needs extremely low, and the benef ts o additional workup
diagnostic mammogram including spot compression outweigh the very minimal chance o rupture in general.
views and workup with ultrasound targeted to the However, ultrasound in some cases can be a substitute
area o concern. In a patient younger than 30 years, replacing additional spot compression views, or
ultrasound can be the f rst test, given the act that example, i there is an obvious mass.
in younger patients, radiation is more signif cant. I
ultrasound is negative, as in most cases and in the
absence o palpable lump, ollow-up with mammogram Pearls
can be an option at later time i the pain does not resolve.
• Diagnostic mammographic views should include
2. Workup is the same. Mammograms and even spot implant displaced views as spot compression views to
compression views are part o the normal workup, an area o concern, which should be marked with a BB.
even i patient has implant. Every patient with implants • In this case, there is no abnormality identif ed. The next
receives in addition to the standard MLO and CC views step in the workup is to per orm an ultrasound directed
implant replacement MLO and CC views. This is crucial to that particular area.
to expose as much tissue as possible, without overlap by • Pit all: Make sure to pay attention to the tissue behind
the implant. the implant. In case o a saline implant, a mass or even
3. In saline implants, there is no need or MRI. small calcif cations, i present, can o ten times be seen
Intracapsular rupture is not an issue. In silicone implants, behind the implant.
dependent on the level o suspicion and the clinical
situation, MRI is the best test to assess the integrity o
the implant. Suggested Reading
4. Saline implants can rupture, but then the saline will be Steinbach BG, Hardt NS, Abbitt PL, Lanier L, Ca ee HH.
absorbed by the tissue and the implant will completely Breast implants, common complications, and concurrent
collapse. Silicone implants can show intra- and breast disease. Radiographics. 1993;13(1):95-118.
extracapsular rupture. Extracapsular rupture indicates
silicone leaking into the breast tissue and is in general an
indication or replacement. It is easier to penetrate with
mammography saline implants and detect abnormalities
behind the implant.

62
Screening—asymptomatic

1. What is the BI-RADS category or this


screening exam?

2. What type o implant is present?

3. What is the position o the implant?

4. What are the signs o a potential complication


with prepectoral implants?

5. What are the radiological signs o silicone in


the breast?

63
Normal silicone implants—di erent positions 616
Case ranking/dif culty: Category: Screening

Answers
1. BI-RADS 2: benign. I you use the BI-RADS system Pearls
and describe a f nding in the breast, in this case, the • Positioning o implants is in one o two tissue
implants, then you should give it a BI-RADS 2. I you compartments, separated by the pectoral muscle.
choose to NOT give a f nding, then BI-RADS 1 negative. • Frequently called retropectoral or prepectoral,
This f ts the European normal/benign category. sometimes subglandular is used, although the term is
less specif c.
2. These are dense implants, and a valve is not seen,
consistent with silicone implants.
3. Right retropectoral, le t prepectoral.
Suggested Readings
4. Specif cally, silicone granulomas present as so t tissue Brower TD. Positioning techniques or the augmented breast.
densities with indistinct margins around the implant Radiol Technol. 1990;61(3):209-211.
capsule. They may arise rom a previous implant rupture, Glicenstein J. History o augmentation mammoplasty [in
or example, current saline implants ollowing a ruptured French]. Ann Chir Plast Esthet. 2005;50(5):337-349.
silicone implant; there ore, the images may show Tebbetts JB. Breast augmentation with ull-height anatomic
potential silicone granulomas, BUT the patient has saline saline implants: the pros and cons. Clin Plast Surg.
implants. 2001;28(3):567-577.
5. Extracapsular silicone can look very suspicious i there
is no other evidence o prior implants on an exam.
Silicone injections, common in Asia or augmentation,
can give spiculate densities, and you can sometimes see
the injection tracks. Most commonly, the ree silicone
gets walled o by in ammatory tissue, which explains
the ill-def ned margins.

64
High-risk BRCA mutation carrier—annual MRI

1. What BI-RADS classif cation should be used


here?

2. What is the name given to the radiologic sign


shown here?

3. In addition to normal sequences on MRI, what


other sequences would you ask or to diagnose
a rupture?

4. What is the risk o malignancy in patients with


ruptured implants?

5. What is the average li espan o a silicone


implant?

65
Intracapsular rupture of silicone implant—linguine sign 1741
Case ranking/dif culty: Category: Diagnostic

MRI—T1 postcontrast sagittal.

T2 sagittal silicone bright sequence. Note the coiled up inner


capsule of the implant surrounded by silicone.

5. In a historic paper out o Baylor in the 1990s, the median


li espan o a silicone gel implant was estimated to be
16.4 years. (Goodman et al. 1998).

Pearls
• Linguine sign is pathognomonic o intracapsular
implant rupture.
Subtracted axial MIP—note that the implant is subtracted out of
the image, and if you do not deliberately look for it, you may miss
the implant rupture. Suggested Readings
Goodman CM, Cohen V, Thornby J, Netscher D. The li e
Answers span o silicone gel breast implants and a comparison o
1. These f ndings are benign; there ore, a BI-RADS 2 is mammography, ultrasonography, and magnetic resonance
appropriate. imaging in detecting implant rupture: a meta-analysis.
Ann Plast Surg. 1998;41(6):577-585; discussion 85-86.
2. Linguine sign—just like the pasta. Even i you just Gorczyca DP, Gorczyca SM, Gorczyca KL. The diagnosis
remember the pasta, it helps. o silicone breast implant rupture. Plast Reconstr Surg.
3. The silicone sequences are very help ul to spot the 2007;120(7, Suppl 1):49S-61S.
intracapsular rupture o an implant. Both silicone bright Juanpere S, Perez E, Huc O, Motos N, Pont J, Pedraza S.
and silicone dark sequences may be per ormed. This Imaging o breast implants—a pictorial review. Insights
may be particularly important in a patient with simple Imaging. 2011;2(6):653-670.
cysts when you suspect extracapsular silicone. Mund DF, Farria DM, Gorczyca DP, et al. MR imaging o
the breast in patients with silicone-gel implants: spectrum
4. The same rate o malignancy is ound as the general o f ndings. AJR Am J Roentgenol. 1993;161(4):773-778.
population at approximately 5 per 1000 women
screened.

66
Prior cosmetic implants—screening

1. What BI-RADS classif cation should be used


here?

2. What is the most likely cause or these


appearances?

3. What is the next best imaging test?

4. What type o biopsy would you recommend?

5. When should the patient have her next


mammography, i o screening age?

67
Ruptured retropectoral silicone implant 581
Case ranking/dif culty: Category: Screening

Right MLO—another implant with Left MLO of explanted prosthesis with Left CC of explanted patient
calci cation of the capsule. residual calci ed capsule. showing characteristic dystrophic
calci cations.

Answers
1. These f ndings are characteristically benign. BI-RADS 2 Pearls
is the most appropriate assessment to give. • Coarse dystrophic contiguous calcif cations are
characteristic or implant capsule calcif cation.
2. There is a retropectoral silicone implant. There is also a
• “Aunt Minnie” or board exams.
circumscribed density around the implant. It is likely that
prior implant had ruptured, and a new one placed.
3. Ultrasound and MRI are also good tools, but in the
Suggested Readings
retropectoral placed implant, MRI would be the best tool
to image the f ndings. Juanpere S, Perez E, Huc O, Motos N, Pont J, Pedraza S.
Imaging o breast implants—a pictorial review. Insights
4. This is an obvious one, i you have already determined Imaging. 2011;2(6):653-670.
the diagnosis on imaging. No biopsy is required. I you Peters W, Pritzker K, Smith D, et al. Capsular calcif cation
have a potential silicone granuloma on ultrasound, then associated with silicone breast implants: incidence,
the eatures are not necessarily characteristic enough to determinants, and characterization. Ann Plast Surg.
avoid biopsy. 1998;41(4):348-360.
5. Depending on the guidelines in your country. In Peters W, Smith D. Calcif cation o breast implant capsules:
the United States, her next mammogram should be incidence, diagnosis, and contributing actors. Ann Plast
per ormed in 1 year. In Europe, in general, the screening Surg. 1995;34(1):8-11.
interval is 2 years (United Kingdom, 3 years).

68
50-year-old woman with status postlumpectomy 2006 for invasive ductal
carcinoma—spot compression views 2008 ( gure on the extreme left)
and 2010 ( gures in the middle and on the right): any change?

1. What are the f ndings on the spot compression


views?

2. What is a typical time period where dystrophic


calcif cations develop a ter lumpectomy?

3. What would be good descriptors or benign


calcif cations a ter lumpectomy?

4. I there is concern o new calcif cations in a


lumpectomy bed, what could be the next step?

5. What would be the benef t o an MRI in this


scenario?

69
Benign dystrophic calci cations 167
Case ranking/dif culty: Category: Diagnostic

Left CC, additional electronic magni cation demonstrating Left ML, additional electronic magni cation demonstrating
“coarse and heterogeneous” calci cations. “coarse and heterogeneous” calci cations.

Answers
Pearls
1. Spot compression views demonstrate architectural
distortion. Noted is a clip due to prior benign biopsy. • Typical f ndings a ter lumpectomy on mammograms
Noted is also development o dystrophic calcif cations. are skin thickening, “architectural distortion,” and
They are “coarse” and are benign in nature. benign “dystrophic” calcif cations and oil cysts.
• I there is concern that calcif cations near lumpectomy
2. Dystrophic calcif cations develop, in general, between bed are malignant, stereotactic biopsy should be
2 and 44 months a ter lumpectomy. per ormed.
3. The two types o benign calcif cations that, in general, • It is also help ul to compare the shape o the
develop a ter lumpectomy could be related to at calcif cations with the appearance o the initial
necrosis and ormation o oil cysts and have “egg shell” preoperative calcif cations that have been proven to
appearance with “lucent centers.” Other typical benign be malignant.
calcif cations would be dystrophic calcif cations that • MRI can be use ul to assess recurrent malignancy,
are mostly macrocalcif cations (larger than 1 mm) and but should be per ormed not early than 6 months a ter
coarse and plaque-like. surgery.

4. Depending on the level o suspicion, next steps could


include 6-month ollow-up, or example, to document
the maturation o calcif cations into an oil cyst, or MRI Suggested Readings
to see i there are any signs or abnormal enhancement in Krishnamurthy R, Whitman GJ, Stelling CB, Kushwaha AC.
the scar area, or to repeat stereotactic biopsy. Mammographic f ndings a ter breast conservation therapy.
5. MRI can be help ul to detect ocal recurrent Radiographics. 1999;19 (Spec no.):S53-S62; quiz
malignancy i per ormed about 6 months a ter S262-S263.
lumpectomy. I the MRI is per ormed too early, it Pinsky RW, Rebner M, Pierce LJ, et al. Recurrent cancer
is impossible to distinguish between neogenesis a ter breast-conserving surgery with radiation therapy
o blood vessels due to tumor recurrence rom or ductal carcinoma in situ: mammographic eatures,
postoperative changes. method o detection, and stage o recurrence. AJR Am J
Roentgenol. 2007;189(1):140-144.

70
87-year-old woman with screening mammogram

1. What is the appropriate descriptor or these


calcif cations?

2. What is an important di erential diagnosis and


why?

3. What is the typical appearance o calcif cations


in comedocarcinoma?

4. What is the assessment o the mammogram?

5. What is the consequence o the assessment?

71
Large “rod-like” calci cations 309
Case ranking/dif culty: Category: Screening

Diagnostic mammogram, right MLO magni cation view Right CC magni cation view demonstrates “rod-like” calci cations.
demonstrates “rod-like calci cations.”

Answers
1. These are typical “large rod-like” calcif cations Pearls
in bilateral and symmetric distribution. Secretory • “Large rod-like” calcif cations are typical benign
calcif cations are large (more than 1 mm in general), calcif cations also called secretory calcif cations.
rod-like, and bilateral and symmetric. • They a ect the larger and intermediate ducts in mostly
older and asymptomatic patients.
2. An important di erential diagnosis is “casting”
• Secretory calcif cations are most o ten bilateral,
(Tabar) calcif cations as seen in high-grade DCIS
but when present unilateral can be con used with
(comedocarcinoma).
comedocarcinoma.
3. Comedocarcinoma shows typical “casting” (Tabar) • Unlike comedocarcinoma calcif cations, secretory
calcif cations in segmental distribution that are more f ne calcif cations are solid and smoothly marginated and
and pleomorphic in orm and are in general consistent sometimes more than 1 cm in size and widely spaced
with ast progressive high-grade DCIS. and usually not branching.
4. These calcif cations are BI-RADS 2: benign.
5. These are benign calcif cations and patient can return
Suggested Readings
or next screening exam in 1 year.
Bland KI, Copeland EM. The Breast: Comprehensive
Management o Benign and Malignant Diseases. 4th ed.
Philadelphia, PA: Saunders Elsevier; 2009.
D’Orsi CJ, Bassett LW, Berg WA, et al. Breast Imaging
Reporting and Data System: ACR BI-RADS–
Mammography. 4th ed. Reston, VA: American College
o Radiology; 2003.
72
Asymptomatic patient with screening mammogram—how signi cant
are these calci cations?

1. What are BI-RADS descriptors or benign


calcif cations?

2. What is the name o these calcif cations and


the assessment?

3. What is the di erence between “milk o


calcium” and an “oil cyst”?

4. I you suspect the presence o skin


calcif cations, how can you prove it?

5. How can you obtain a tangential view?

73
Egg shell and skin (dermal) calci cations 165
Case ranking/dif culty: Category: Diagnostic

4. I there is a group o calcif cations that are likely within


the skin, tangential views are the way to prove it.
5. A tangential view is obtained by putting the patient in
a mammogram unit using a paddle with window (like
or a needle localization)—mark the calcif cations with
BB. Then, patient is put into position in which the BB
is tangential at the edge o the scan f eld (tangential
view). I the calcif cations are not seen within the
subcutaneous at, the view is either not really tangential
to the calcif cations and needs to be adjusted or the
calcif cations are not in the skin.

Pearls
• Most breast calcif cations are benign and this
Magni cation right CC view, demonstrating skin calci cations includes “lucent-centered” skin calcif cations,
(arrow) and scattered oil cysts. “coarse and popcorn” type calcif cations, “tram-like”
vascular calcif cations, and “milk o calcium” type
calcif cations.
Answers
• For diagnostic workup o calcif cations, never
1. “Amorphous” and “coarse and heterogeneous” are per orm magnif cation MLO view, but always per orm
descriptors or calcif cations o intermediate concern. magnif cation ML view to maximize chance to detect
“Eggshell or rim” calcif cations are benign calcif cations benign “milk o calcium.”
related to oil cysts. “Rod-like” calcif cations are benign • To prove location o calcif cations within the skin, it is
calcif cations related to plasma cell mastitis. “Coarse prudent to obtain tangential views.
or popcorn-like” calcif cations are benign calcif cations • “Eggshell”-type calcif cations are typical or oil cysts.
related to f broadenoma.
2. These are typical benign (BI-RADS 2) calcif cations
consistent with diagnosis o oil cysts. They are Suggested Reading
round and o ten times in multiple locations and have
D’Orsi CJ, Bassett LW, Berg WA, et al. Breast Imaging
a characteristic appearance with “eggshell or rim”
Reporting and Data System: ACR BI-RADS–
calcif cation with lucent center. There is no need or
Mammography. 4th ed. Reston, VA: American College
ollow-up or biopsy.
o Radiology; 2003.
3. The etiology o oil cysts is at necrosis. The etiology
o “milk o calcium” are proli erative changes o the
breast parenchyma with ormation o multiple small
cysts related to the lobules o the parenchyma with small
collection o liquid calcium within the cysts causing
typical layering as seen on the ML view. Both are
“benign” f nding according to the BI-RADS lexicon.

74
Call back with abnormal screening nding—asymptomatic

1. What BI-RADS classif cation should be used


here?

2. What is the most likely pathology based on the


imaging?

3. What urther imaging would you recommend?

4. In determining extent o disease, what tools


should be considered?

5. What type o biopsy would you per orm?

75
High-grade DCIS—calci cation descriptors 1765
Case ranking/dif culty: Category: Screening

Right ML spot magni cation. Note the distribution of the


calci cations, which is segmental toward the nipple.
Specimen x-ray–extensive coarse “pleomorphic” calci cations
Answers in most cores—evidence of good sampling. “Fine pleomorphic”
calci cations on mammograms often appear coarse on specimen
1. This is a diagnostic workup; there ore, BI-RADS 4 is the
x-ray, which illustrates limitations in our current technology for
best f t, as the f nding has at least a 50% risk o malignancy
analyzing calci cation morphology in situ.
with this type and distribution o calcif cations.
2. High-grade calcif cations typically present with o treatment. Adding a specimen x-ray to the procedure
these eatures. Low-grade DCIS is o ten “punctate” helps to improve the specif city o the test. Stereotactic
or “amorphous.” It may be aint and di f cult to see. core biopsy is the standard type o biopsy with this
Sometimes re erred to as “powdery” calcif cations, likely f nding. MRI biopsy should not be needed at this stage.
arising in the TDLU rather than the ducts. PEM biopsy is help ul in patients with dense breasts and
3. At this stage, the diagnostic workup needs to be occult mammographic disease, but not in DCIS.
completed and the calcif cations characterized. A lateral
view helps localization within the breast. Spot views
are not good enough on their own without the increased Pearls
resolution o magnif cation. Tomosynthesis has not
• Fine pleomorphic calcif cations are suspicious and
yet shown to have any utility in microcalcif cation
require biopsy (BI-RADS 4C according to the 5th
characterization. MRI may be used later in the workup
edition) (risk o malignancy >50%).
when proven DCIS or extent o disease.
• Stereotactic core with specimen x-ray.
4. Ultrasound may be used, especially i there is an • I visible on ultrasound, biopsy and include specimen
associated density with the calcif cations. Also, when x-ray.
the patient has dense breasts, ultrasound can more
easily visualize a mass associated with DCIS. DCIS
can be ound among di use benign calcif cations, and Suggested Readings
sometimes multiple stereotactic core biopsies need to be
Burnside ES, Ochsner JE, Fowler KJ, et al. Use o
used to determine extent. MRI has utility with high-grade
microcalcif cation descriptors in BI-RADS 4th edition to
DCIS, although some groups also say it has value in ALL
strati y risk o malignancy. Radiology. 2007;242(2):388-395.
types o DCIS. PEM and BSGI may be use ul in dense
Ho vind S, Iversen BF, Eriksen L, Styr BM, Kjellevold K,
breast tissue when there are multiple invasive cancers, but
Kurz KD. Mammographic morphology and distribution
there are no data on disease extent with DCIS.
o calcif cations in ductal carcinoma in situ diagnosed in
5. I there is an associated mass with the organized screening. Acta Radiol. 2011;52(5):481-487.
microcalcif cations, there is a 50% chance o invasive Yamada T, Mori N, Watanabe M, et al. Radiologic-pathologic
disease, and that is the area that needs targeting, as it will correlation o ductal carcinoma in situ. Radiographics.
direct the per ormance o a sentinel node biopsy as part 2010;30(5):1183-1198.
76
Screening mammogram 6 years ago (the rst two gures on the left) and
follow-up exam (the next two gures on the right)

1. What is the appropriate description o these


calcif cations?

2. What is the next step?

3. What is the description o the increasing


group o calcif cations in the new screening
mammogram?

4. What is the assessment o the second


mammogram?

5. What is the signif cance o new/increasing


calcif cations in general?

77
“Coarse or popcorn-like” benign calci cations 307
Case ranking/dif culty: Category: Screening

Right MLO view with increasing group of calci cations in Right CC view with increasing group of calci cations in lateral
posterior lateral breast. posterior breast.

Answers
Pearls
1. This is a typical case o “coarse and popcorn-like”
calcif cations that are characteristic or f broadenoma. • “Coarse and popcorn-type” calcif cations are typical or
involuting f broadenoma. Finding is benign (BI-RADS 2).
2. This is a benign mammogram and patient can return • Fibroadenoma is the most requently seen mass in
in 1 year or next mammogram. young patients and this is because o proli eration o
3. These new calcif cations are o the same characteristics. lobular, epithelial, and mesenchymal elements under
They are “coarse” and “popcorn like” as well. They estrogen stimulation.
appear to have increased in number since prior study. • Fibroadenomas develop, in general, in young patients
However, the area was not well covered on the prior and involute during older age due to withdrawal
exam. o estrogens and the process o hyalinization and
subsequent calcif cation.
4. These calcif cations are o the same characteristics as • In early stage o involution, calcif cations may not be
the other calcif cations in the anterior breast and are also easy to di erentiate rom “pleomorphic” or “casting”
consistent with benign calcif cations—despite the act (Tabar)-type calcif cations.
that they are new.
5. It depends on the morphology o the calcif cations. There
are benign calcif cations that can develop over time and Suggested Reading
that need no urther workup.
Nussbaum SA, Feig SA, Capuzzi DM. Breast imaging
case o the day. Fibroadenoma with microcalcif cation.
Radiographics. 1998;18(1):243-245.

78
Diagnostic mammogram—patient is asymptomatic

1. What is the best description o the group o


calcif cations?

2. What is the next step i the group is new?

3. What is the technique with the highest


specif city or milk o calcium?

4. What is “milk o calcium”?

5. What is the appropriate f nal assessment?

79
New group of calci cations 1311
Case ranking/dif culty: Category: Diagnostic

Diagnostic mammogram, left ML magni cation view Diagnostic mammogram, left CC magni cation view
demonstrating layering and “tea cup shape” of some of demonstrating group of “amorphous” calci cations.
the calci cations within the concerning group.

Diagnostic mammogram, right CC magni cation view


Diagnostic mammogram, right ML magni cation view demonstrating group of “amorphous” calci cations.
demonstrating group of calci cations with “milk of calcium.”

Answers
1. Given the presence o layering and tea cup ormations on Pearls
ML view, the group is consistent with milk o calcium. • This is a nice example to demonstrate that all
magnif cation views should be per ormed in
2. I there is layering seen, the group is benign and
perpendicular angle (ML and CC) to each other to
consistent with milk o calcium—patient can return
maximize the e ect o layering.
or next exam in 1 year.
• There is no need ever to per orm a MLO magnif cation
3. Perpendicular magnif cation views are the appropriate view, except there is no other way to reach most
technique to document milk o calcium. There is never posterior areas o the breast near the chest wall.
a need to per orm a MLO magnif cation view. All
magnif cation views should be per ormed in ML and CC
plane—in particular, i milk o calcium is suspected. Suggested Reading
4. Milk o calcium is a orm o proli erative breast change Imbriaco M, Riccardi A, Sodano A, et al. Milk o calcium
with accumulation o calcium containing uid in in breast microcysts with adjacent malignancy. AJR Am J
microcysts. Roentgenol. 1999;173(4):1137-1138.
5. This is a typical benign f nding: BI-RADS 2.

80
Call back with abnormal screening nding—asymptomatic

1. What BI-RADS classif cation should be used


here?

2. Which o the ollowing calcif cation


distribution modif ers are most suspicious
or malignancy?

3. What is a special type o calcif cations that you


need to avoid using suspicious descriptors or?

4. What type o intervention would you


recommend?

5. What type o pathology could amorphous


microcalcif cations represent?

81
Calci cation description modi ers 1738
Case ranking/dif culty: Category: Diagnostic

Answers 5. High-grade DCIS are o ten denser calcif cations, seen


1. The calcif cations are made up o three separate clusters, best when analyzing specimen x-rays. Tabar classif es
but orm a segmental distribution giving an approximate these as variants o crushed stone. “Amorphous”
risk o malignancy o 30%. According to the 5th edition calcif cations can be due to a spectrum o normal to
o BI-RADS, this is a BI-RADS 4B classif cation. disease, rom milk o calcium seen in the CC view,
to low/intermediate-grade DCIS. ADH and LCIS are
2. Some linear and branching calcif cations are seen requently picked up on stereotactic core biopsy. The
in classically benign f ndings such as “secretory”
calcif cations in LCIS are not related to the LCIS itsel ,
calcif cations o duct ectasia, and are eatured as a
as the calcif cations are normally benign.
special case in the BI-RADS manual. All special
cases eatured in the BI-RADS manual are typical
board questions, and you should be able to recognize
these entities and di erentiate them rom suspicious Pearls
calcif cations.
• Analysis o microcalcif cations requires the use o
3. The special type re erred to here is “secretory” spot magnif cation views or “f ne” settings on photon
calcif cations that are a result o duct ectasia counting systems.
or periductal mastitis. These calcif cations are • Determine the individual shapes o the calcif cations.
characteristic, but are otherwise LINEAR, and I they are di f cult to describe, then they can be called
may be BRANCHING. The distribution may be “amorphous.”
“SEGMENTAL.” All o these BI-RADS descriptors and • Determine the most suspicious distribution o the
their distribution modif ers are suggestive o malignancy, calcif cations—“scattered,” through “grouped” and
and there ore should not be used. A simple description o “clustered” to “segmental.”
secretory calcif cations is enough.
4. Standard mammographic workup including spot
magnif cation views is needed to characterize the calcif c Suggested Readings
particles. Critical examination o any prior exams also Bent CK, Bassett LW, D’Orsi CJ, Sayre JW. The positive
needed to determine whether these calcif cations are predictive value o BI-RADS microcalcif cation descriptors
developing. I stability is greater than 3 years, they are and f nal assessment categories. AJR Am J Roentgenol.
more likely to be either benign or possibly low-grade 2010;194(5):1378-1383.
DCIS, o no emergent need or intervention. However, Burnside ES, Ochsner JE, Fowler KJ, et al. Use o
this is controversial. I the patient has not been worked microcalcif cation descriptors in BI-RADS 4th
up be ore, and the calcif cations look suspicious, edition to strati y risk o malignancy. Radiology.
recommend biopsy, with specimen x-ray to determine 2007;242(2):388-395.
satis actory sampling.

82
Patient with rst baseline screening mammogram

1. What is the f nding on screening mammogram?

2. What is the next step?

3. What is the appropriate f rst step o workup?

4. What is the next step?

5. What would be the benef t o MRI?

83
Intermediate -grade DCIS 1302
Case ranking/dif culty: Category: Screening

Diagnostic mammogram, right ML Diagnostic mammogram, right CC MRI, MIP image, after IV contrast, demonstrates
magni cation view demonstrating magni cation view demonstrating di erent enhancement kinetic patterns,
group of “pleomorphic” calci cations, group of “pleomorphic” calci cations including washout enhancement.
in “linear” distribution. in “linear” distribution.

4. In this patient, f rst step is stereotactic biopsy to obtain


pathology, which did show intermediate-grade DCIS.
Then additional MRI was per ormed to assess or
multicentric and multi ocal disease.
5. MRI did show additional linear enhancement in lateral
right breast, which did not correlate to any calcif cations
and would have been missed. This could be conf rmed
with MRI-guided biopsy and patient was treated with
mastectomy because o the presence o multicentric
disease (disease in more than one quadrant). MRI in this
case did change surgical management.

MRI after IV contrast and subtraction demonstrating linear Pearls


enhancement and second area more laterally (arrow). • Large group o “pleomorphic and linear” calcif cations in
“segmental” distribution could be classif ed as BI-RADS
Answers 5: “highly suspicious.”
• Given the extent o more than 7 cm in antero-posterior
1. Noted are “pleomorphic” calcif cations right medial (AP) diameter and the additional MRI biopsy-
breast in “segmental” distribution. proven area o additional DCIS lateral right breast,
2. Patient needs to be called back BI-RADS 0 or mastectomy was per ormed.
additional workup with diagnostic mammogram.
3. The f rst step is magnif cation ML and CC views. There
is almost never a reason to per orm MLO magnif cation Suggested Reading
views—except where the ML projection might not Virnig BA, Tuttle Tm, Shmliyan T, et al. Ductal carcinoma in
be able to cover the posterior tissue. On ML and CC situ o the breast: a systematic review o incidence, treatment
magnif cation views, it is easier to detect milk o and outcomes. J Nat Cancer Inst. 2010;102(3):170-178.
calcium. Then ultrasound is recommended to assess
patient or solid component.

84
Diagnostic mammogram—what is the signi cance of these calci cations?

1. What it the appropriate BI-RADS descriptor


or the calcif cation?

2. This was a new group o calcif cations—what


is the next step?

3. I the calcif cations are seen on f rst screening


mammogram, what would be reasonable?

4. What is the appropriate workup o a group o


calcif cation?

5. What is the f nal assessment based on the


magnif cation views, i the calcif cations are
new?

85
Intermediate -grade DCIS 1312
Case ranking/dif culty: Category: Screening

MRI, T1-weighted sequence after IV contrast with MIP technique MRI, T1-weighted sequence after IV contrast demonstrating
demonstrates corresponding area of strong enhancement with linear area of increased enhancement in the right breast,
washout kinetics right breast. corresponding to the mammogram nding.

Answers
1. This is a group o “round and oval” calcif cations in the Pearls
right retroareolar breast. • I group o calcif cations is seen on a f rst screening
mammogram and on subsequent diagnostic
2. I this was a new group o calcif cations, patient needs
mammogram with magnif cation views and is
to be biopsied with stereotactic biopsy device. The
described as “round and oval,” it can be called
calcif cations are relatively benign, since “round and
“probably benign.”
oval”—but the act that they are new is a red ag.
• However, i the group is new, like in this case,
3. On screening exam, patient should never be categorized biopsy is mandatory.
as BI-RADS 3; it had to be BI-RADS 1 or BI-RADS • Follow-up diagnostic mammograms should include
2 or BI-RADS 0 “incomplete” like in this case. In magnif cation views, since any change, such as new
this case, the f nding needed to be worked up with amorphous calcif cations during the ollow-up time
magnif cation views. period o 2 years, would trigger biopsy.
• The BI-RADS lexicon gives the option o 2 or 3 years
4. For calcif cations, an ML and CC view should be
ollow-up; during that time period, the calcif cations
per ormed, NEVER a MLO magnif cation view. This is
remain “probably benign.”
true, since milk o calcium is much better characterized
on two perpendicular views (ML and CC) than on MLO
and CC view.
Suggested Readings
5. BI-RADS 4 i this is a new group o calcif cations and
was not seen on prior mammogram. BI-RADS 3, i it Rosen EL, Baker JA, Soo MS. Malignant lesions initially
is the f rst mammogram, would be appropriate and then subjected to short-term mammographic ollow-up.
they should be ollowed over 2 years. Follow-up should Radiology. 2002;223(1):221-228.
be per ormed with the images with being most sensitive Sickles EA. Breast calcif cations: mammographic evaluation.
and specif c to detect change, with magnif cation Radiology. 1986;160(2):289-293.
views—not with standard views. Sickles EA. Probably benign breast lesions: when should
ollow-up be recommended and what is the optimal
ollow-up protocol? Radiology. 1999;213(1):11-14.

86
Calci cations found on screening mammogram—diagnostic workup

1. What is the BI-RADS category or this


diagnostic exam?

2. What is the next best examination you


recommend?

3. What is the best description or the distribution


o calcif cations?

4. What other imaging test would you


recommend?

5. What type o biopsy would you recommend?

87
High-grade ductal carcinoma in situ (DCIS) 607
Case ranking/dif culty: Category: Diagnostic

5. Cytology no longer has a place in the diagnosis o DCIS.


Fourteen-gauge or vacuum-assisted core biopsy can
be used under ultrasound (i visible, and especially i
there is a mass). Stereotactic core biopsy i there is no
ultrasound f nding. The risk o upstaging—going rom
ADH to DCIS, or DCIS to invasive disease—is lessened
by increased numbers o cores and increased volume
o cores, so many radiologists would recommend large
gauge core biopsy (vacuum assisted) or DCIS.

Pearls
• High-grade DCIS has a good chance o being upstaged
to invasive disease.
• Diligently search or evidence o possible invasion.
Operative specimen containing calci cations and margin markers. • Take a good amount o tissue to reduce the risk o
undersampling.
• Remember that we are only seeing the calcif ed part o
Answers
the disease, and there may be more noncalcif ed disease
1. It is a diagnostic workup, so a BI-RADS 0 is not that we are not seeing.
applicable. The calcif cations have between a 50% • MRI is the best imaging modality or extent.
and 99% risk or DCIS, so it encompasses both the
BI-RADS 4C and BI-RADS 5 categories.
2. Further spot magnif cations views should be per ormed Suggested Readings
to urther characterize the calcif c particles.
Hayward L, Oeppen RS, Grima AV, Royle GT, Rubin CM,
3. This is a segment o calcif cations; although it may be Cutress RI. The in uence o clinicopathological eatures
orientated down a duct system, it is not strictly linear. on the predictive accuracy o conventional breast imaging
in determining the extent o screen-detected high-grade
4. The f rst test I would recommend i you see any density
pure ductal carcinoma in situ. Ann R Coll Surg Engl.
associated with calcif cations is a targeted ultrasound.
2011;93(5):385-390.)
First to see i there is a mass associated with the
Kropcho LC, Steen ST, Chung AP, Sim MS, Kirsch DL,
calcif cations, which gives you a likely risk o invasive
Giuliano AE. Preoperative breast MRI in the surgical
disease, and there ore metastatic potential. Some
treatment o ductal carcinoma in situ. Breast J.
surgeons do sentinel node biopsy in high-grade DCIS
2012;18(2):151-156.
just in case there is an occult ocus o invasive disease in
Rahbar H, Partridge SC, Demartini WB, et al. In vivo
the breast. MRI is best suited or determining both the
assessment o ductal carcinoma in situ grade: a model
extent o the DCIS and also i there is an associated mass
incorporating dynamic contrast-enhanced and di usion-
to indicate invasion. There is no need or ultrasound
weighted breast MR imaging parameters. Radiology.
staging o the axilla, as so ar there is no evidence o
2012;263(2):374-382.
invasive disease. Same argument or PET/CT.

88
Screening mammogram priors on the left

1. What is the f nding on the mammogram?

2. What is the assessment based on the


magnif cation views?

3. What is the pre erred next step in the workup?

4. Why can it be help ul to have ultrasound f rst?

5. What would be the next step a ter the


ultrasound-guided biopsy?

89
“Pleomorphic”
Diagnosis of thecalci
case (1626):
cationsNormal
due to high-grade
screening mammogram
DCIS 378
with invasive component Category: Screening

Case ranking/dif culty: Category: Diagnostic

Diagnostic mammogram, right magni cation ML view. Noted is Diagnostic mammogram, right magni cation CC view. Noted is
group of “pleomorphic” calci cations in “segmental” distribution. group of “pleomorphic” calci cations in “segmental” distribution.

5. Specimen x rays are always help ul to conf rm the


presence o calcif cations in the tissue. A clip should be
placed in all circumstances. MRI may better def ne the
extent o the pathology.

Pearls
• In some situations such as this, it is important to
per orm specimen mammogram o the cores obtained
under ultrasound-guided biopsy to make sure that the
Ultrasound of right breast upper outer quadrant demonstrates
calcif cations are within the specimen.
hypoechoic mass with associated calci cations.
• Abnormalities can be classif ed as BI-RADS 5—“highly
suspicious” or malignancy. The consequence is that i
Answers pathology would show benign f nding, such as “ ocal
f brosis and benign calcif cations,” this would not be
1. This is typical appearance o group o “pleomorphic”
concordant and the biopsy had to be repeated or the
calcif cations in “segmental” distribution.
patient had to go directly to surgery.
2. This is a typical appearance o a BI-RADS 5 f nding—it • I abnormality is called BI-RADS 4—“suspicious,” it
is “highly suspicious” or malignancy. is assumed that it could still be a benign underlying
pathology.
3. Ultrasound is the pre erred next step to urther assess or
possible invasive solid component. However, to per orm
stereotactic biopsy without prior ultrasound would also
be reasonable but not the pre erred next step. Suggested Reading
4. Ultrasound is help ul or urther evaluation to f nd a Soo MS, Baker JA, Rosen EL. Sonographic detection and
possible associated solid part o the malignancy, which sonographically guided biopsy o breast microcalcif cations.
would likely be the invasive component o the process. AJR Am J Roentgenol. 2003;180(4):941-948.
Also, even i there is no solid part, ultrasound might be
able to visualize the calcif cations and ultrasound-guided
biopsy might be an alternative approach to stereotactic
biopsy. Ultrasound-guided biopsy is in general more
convenient to the patient.

90
Patient with palpable abnormality— rst mammogram

1. What is the best description o the pertinent


abnormality?

2. What is the most likely f nal assessment a ter


diagnostic workup?

3. What is the di erence between BI-RADS 4


and BI-RADS 5?

4. What is the description o the abnormality on


ultrasound?

5. I there are suspicious lymph nodes, is biopsy


with FNA help ul?

91
Invasive ductal carcinoma 1303
Case ranking/dif culty: Category: Diagnostic

Diagnostic mammogram, left spot compression CC view On ultrasound with duplex, no abnormal ow is identi ed.
demonstrating “spiculated” mass.

Answers
1. This is an example o a highly suspicious f nding, Pearls
consistent with “mass” with “spiculated margin” with • According to the BI-RADS lexicon 4th edition, the
“high density.” group 4, “suspicious” can be divided into subgroups
4a, small; 4b, moderate; and 4c, substantial likelihood
2. BI-RADS 5 is the most likely assessment—however,
o malignancy.
f rst, additional diagnostic workup is required.
• Also, category BI-RADS 5 exists that indicates “highly
3. BI-RADS 4 has the meaning o abnormality being suspicious” or malignancy.
suspicious—according to BI-RADS lexicon edition 4, it • To di erentiate between BI-RADS 4 and BI-RADS 5
can be divided into BI-RADS 4a, b, and c depending on does have signif cant impact on the decision process,
the level o suspicion. However, all BI-RADS 4 lesions since BI-RADS 5 lesion does need surgical excision i
could represent benign pathology and it would still be stereotactic biopsy is technically not easible or i the
concordant. BI-RADS 5, however, indicates that this is pathology results demonstrate benign f nding.
highly suspicious, and even i pathology comes back as
benign, it is not concordant and patient needs to go to
surgery.
Suggested Readings
4. This mass can be described as “hypoechoic mass” with Lazarus E, Mainiero MB, Schepps B, Koelliker SL,
“nonparallel orientation” (taller than wide) and posterior Livingston LS. BI-RADS lexicon or US and
“acoustic shadowing” and “spiculated” margin. mammography: interobserver variability and positive
5. Ultrasound-guided FNA o morphologically suspicious predictive value. Radiology. 2006;239(2):385-391.
lymph nodes is not only help ul to provide the surgeon Stavros AT, Thickman D, Rapp CL, Dennis MA, Parker
with more in ormation be ore surgery replacing the SH, Sisney GA. Solid breast nodules: use o sonography
sentinel lymph node procedure, but it is also cost- to distinguish between benign and malignant lesions.
e ective. The likelihood o the presence o pathological Radiology. 1995;196(1):123-134.
axillary lymph nodes correlates to the size o the
malignancy.

92
Screening—asymptomatic

1. What BI-RADS classif cation should be used


here?

2. What is the most likely pathology based on the


imaging?

3. What is the next best imaging test?

4. Do these f ndings ulf ll the multiple masses


test o BI-RADS?

5. What should be considered i a patient is


insistent on breast conservation?

93
Multicentric breast cancer 1842
Case ranking/dif culty: Category: Screening

Ultrasound measurement of the distance between


Left CC spot magni cation. The three Left ML spot magni cation. two of the masses.
potential masses are now seen with
“spiculate” margins.
bilateral noncalcif ed f broadenomas or cysts. You need
to have at least two masses on one side and at least one
in the contralateral breast to use this rule o benignity.
5. MRI is a prerequisite in multi ocal disease, especially i
the patient does not have large volume breasts, as it may
be di f cult to achieve a good cosmetic result. However,
MRI may distinguish between multi ocal and multicentric
disease, determine whether the pectoral muscle is
involved, the absence o contralateral f ndings, and f nally
or surgical planning to give a roadmap o the disease.

Pearls
Targeted ultrasound.
• I you spot one suspicious lesion, suggestive o
malignancy, look or a second lesion—usually in the
Answers line o the milk duct up to the nipple.
1. This is a screening exam that has a potential abnormality
and there ore by def nition needs urther workup. The
appropriate BI-RADS assessment is there ore 0.
Suggested Readings
2. Potentially, these appearances be ore workup could Bauman L, Barth RJ, Rosenkranz KM. Breast conservation
be either invasive ductal carcinoma or at necrosis. in women with multi ocal-multicentric breast cancer: is it
DCIS usually presents with microcalcif cations, but easible? Ann Surg Oncol. 2010;17(Suppl 3):325-329.
may be seen as a circumscribed mass, mimicking a Howe HL, Weinstein R, Alvi R, Kohler B, Ellison JH.
f broadenoma in a young woman. Mucinous carcinoma Women with multiple primary breast cancers diagnosed
usually presents as a mass with indistinct margins, within a f ve year period, 1994-1998. Breast Cancer Res
sometimes di f cult to di erentiate rom a simple cyst. Treat. 2005;90(3):223-232.
3. The mammographic workup should be completed Rezo A, Dahlstrom J, Shadbolt B, et al. Tumor size and
be ore urther imaging tests. A lateral exam, with spot survival in multicentric and multi ocal breast cancer.
or spot magnif cation views, will help to characterize Breast. 2011;20(3):259-263.
the margins o the masses and characterize any calcif c
particles associated.
4. The answer is a clear NO. The “multiple masses” note
should be used to re ect your opinion o multiple

94
Palpable abnormality in the right breast

1. What is the f rst step o workup o a 40-year-


old emale with palpable abnormality?

2. In case o normal ultrasound, what is the next


step?

3. How appropriate is MRI as a problem-solving


modality to address inconclusive f ndings on
mammogram?

4. How would you describe the f nding on the


mammogram?

5. What do you expect to see on ultrasound?

95
Invasive ductal carcinoma 734
Case ranking/dif culty: Category: Diagnostic

Spot compression, right MLO view Spot compression, right CC view Ultrasound of right breast shows corresponding
with BB marker on area of palpable with BB marker on area of palpable “hypoechoic spiculated mass.”
abnormality. abnormality demonstrating “focal
asymmetry” with “spiculated”
margin.

Answers 5. The mass is hypoechoic in comparison with the at layer


1. Workup includes diagnostic mammogram including spot above; it is “irregular” in shape and has “spiculated”
compression views and ultrasound. margins. It is “wider than tall” (Stavros). It does not
show signif cant “posterior acoustic shadowing.”
2. Next step then would be to per orm stereotactic biopsy.
Another option could be to per orm MRI and call
mammogram BI-RADS 0—this might in particular
apply to an inconclusive mammogram in the situation Pearls
o a palpable abnormality. • In case o normal ultrasound, in particular, i there
3. To use breast MRI as a problem-solving tool in case is a palpable abnormality and i there is abnormal
o inconclusive mammogram f ndings is controversial. morphology seen on mammogram, stereotactic biopsy
It does not eliminate the need or a thorough workup should be per ormed.
with additional views and ultrasound. I the f nding is • I the f nding on mammography is inconclusive, breast
suspicious on mammogram and normal on ultrasound, MRI can be help ul in selected cases as problem-
MRI cannot eliminate the need or biopsy. However, solving modality to determine the level o concern,
in selected cases, MRI can help to urther make the because o extremely high negative predictive value.
case that an inconclusive mammogram f nding, which
otherwise would be called BI-RADS 3, does not need to
be biopsied—however, ollow-up with mammography Suggested Reading
in 6 months is still warranted despite negative MRI.
Moy L, Elias K, Patel V, et al. Is breast MRI help ul in the
While the negative predictive value o MRI is high, it is
evaluation o inconclusive mammographic f ndings? AJR
not 100% and, or example, low-grade DCIS can appear
Am J Roentgenol. 2009;193(4):986-993.
normal on breast MRI.
4. The best descriptor is “ ocal asymmetry” with
“spiculated” margin or “mass” with “spiculated” margin.
Asymmetry is pre erred, since it is not well seen on the
other projection and has concave borders.

96
Palpable lump in the left breast of a young woman

1. What descriptors can be applied to this


f nding?

2. What BI-RADS category would you place this


lesion in?

3. I the patient is younger than 30 years, what is


the likely pathology?

4. I the patient is younger than 25 years, do you


need to do a mammogram?

5. What pathologies cause increased vascularity


inside a mass?

97
High-grade invasive ductal cancer 595
Case ranking/dif culty: Category: Diagnostic

Because of her age, an ultrasound was XCCL performed, as mass is in outer half. Spot magni cation shows that the mass is
the rst imaging exam. Lateral, there is an ill de ned. Based on both the mammogram
irregular, partially obscured mass in the and ultrasound ndings, a biopsy is now
upper half of the breast. recommended. Final pathology: IDC Gd2
ER/PR+ HER2−.

Answers
1. I the mass is not a good geographic f t or round or oval, Pearls
the shape should be called IRREGULAR. • Regardless o age, a suspicious ultrasound should
prompt a mammogram or correlation, unless there is a
2. The f ndings are not classical o malignancy, and
classic abscess, in which case the mammogram should
extremes o age can give atypical radiological f ndings.
be delayed.
It is certainly in the 90% and above risk o invasive
• Key here is the irregular mass on ultrasound that makes
cancer, and some may give it a BI-RADS 5, as it is a
it malignant until proven otherwise.
cancer until proven otherwise.
• Associated DCIS o ten ound with the tumor, seen as
3. It is still most likely to be a cancer, with these eatures, re ective particles within the hypoechoic mass itsel , or
although at necrosis and an early abscess can all give in a dilated duct associated with the tumor.
the same appearances.
4. Regardless o age, the patient has suspicious ultrasound
f ndings and a mammogram should be per ormed. MRI Suggested Readings
is likely to have an important role, but may not visualize
Kim JH, Ko ES, Kim do Y, Han H, Sohn JH, Choe du H.
any associated DCIS.
Noncalcif ed ductal carcinoma in situ: imaging and
5. Vascularity is not conf ned to invasive cancers. Young histologic f ndings in 36 tumors. J Ultrasound Med.
women with a f broadenoma will o ten have very 2009;28(7):903-910.
large vascular channels, and only a biopsy will help in Park JS, Park YM, Kim EK, et al. Sonographic f ndings o
distinguishing between a f broadenoma and a phyllodes high-grade and non-high-grade ductal carcinoma in situ o
tumor. Vascularity is seen in developing abscesses the breast. J Ultrasound Med. 2010;29(12):1687-1697.
be ore lique action in a phlegmon. Cysts usually have Tozaki M, Fukuma E. Does power Doppler ultrasonography
peripheral vascularity i in amed. improve the BI-RADS category assessment and
diagnostic accuracy o solid breast lesions? Acta Radiol.
2011;52(7):706-710.

98
Any
61-year-old
abnormality
patient
leftwith
breast?
palpable abnormality in the left upper
outer quadrant (left two images are the 2 year prior study. Current
study on the right)

1. Why could the lesion on spot compression


views be called “mass” and not “asymmetric
density”?

2. What etiology could explain a developing


mass?

3. Why is f broadenoma not a likely di erential


diagnosis?

4. What would you do i ultrasound does not


show any abnormality?

5. What would you do i patient cannot tolerate


to lie on the stereotactic biopsy table and no
ultrasound f nding is detected?

99
Invasive ductal carcinoma 324
Case ranking/dif culty: Category: Diagnostic

Diagnostic mammogram, left Diagnostic mammogram, left spot Ultrasound demonstrates associated “hypoechoic mass”
spot compression MLO view compression CC view demonstrating with “irregular”shape and “angular”margin.
demonstrating mass with “lobular mass with “irregular shape.”
shape” and partially “obscured
margin.” 3. In a 61-year-old patient, it is very unlikely that there will
be a new f broadenoma. Fibroadenomas usually develop
in younger age group under the in uence o estrogen.
4. Any new mass is concerning, unless consistent with a
simple cyst or other clearly benign f nding as seen on
ultrasound. With the appropriate history, hematoma
could explain benign mass. I mass does not correlate to
any benign ultrasound f nding, patient needs stereotactic
biopsy. Any suspicious abnormality on imaging, in
general, should be biopsied f rst, be ore sending the
patient to breast surgeon.
5. Patient then should get surgical excision. Any lesion seen
on two planes can be localized with needle and send to
Ultrasound with duplex demonstrates some ow in the center of surgical excision.
the mass.

Pearls
Answers
• Ultrasound f nding o “hypoechoic,” solid mass ( ow
1. This is a case where the pictures would support the term
on duplex) with “angular margin” that correlates to
“mass” over “asymmetric density,” since it is seen on two
palpable abnormality is suspicious (BI-RADS 4) and
images and has convex shape on spot compression views.
ultrasound-guided biopsy should be per ormed.
2. Any new mass raises concern, in particular, i it is
associated with palpable abnormality. There are also
benign f ndings, such as cyst, f broadenoma, hematoma, Suggested Readings
or at necrosis that can explain new mass. However, Piccoli CW, Feig SA, Plazzo JP. Developing asymmetric
any developing mass needs to be biopsied, unless it breast tissue. Radiology. 1999; 211(1):111-117.
is a simple cyst or other def nitely benign etiology on Youk JH, Eun-Kyung K, Kung HK, et al. Asymmetric
ultrasound. In the appropriate clinical setting, i there mammographic f ndings based on the ourth edition
is history o trauma, hematoma could also explain the o BI-RADS: types, evaluation and management.
presence o new mass on mammography. Radiographics. 2009;29(1):e33.
100
Lump in left axillary tail

1. What BI-RADS classif cation should be used


here?

2. What is the most likely pathology based on the


imaging?

3. What is the next best imaging test?

4. What type o biopsy would you recommend?

5. I the axillary nodes are palpable on this


patient, what is the likelihood o nodal
metastases?

101
Ill-de ned mass on mammography but sharply marginated 1843
on ultrasound
Case ranking/dif culty: Category: Diagnostic

Ultrasound of mass with power Doppler shows no signi cant ow Ultrasound—in the orthogonal plane, the mass is seen to have
in an “oval” mass. angular margins and possibly some duct extension.

Answers 5. With a known primary cancer, palpable nodes are


1. A BI-RADS 4 or 5 is appropriate in this case that turned extremely likely to contain metastases UNLESS the
out to be a high-grade invasive ductal carcinoma. patient has recently had a breast biopsy. In that situation,
only ultrasound staging o the nodes is likely to conf rm
2. This is an ill-def ned mass in an older patient with no suspicious nodes. In a recent paper, patients with
history o trauma or surgery at that site; so, at necrosis palpable nodes and suspicious ultrasound exams had
and f broadenoma are not in the di erential diagnosis. a greater than 75% chance o positive nodes.
An in ected sebaceous cyst is usually around the bra
line, but in theory it can occur on the skin anywhere in
the breast. A complex sclerosing lesion usually presents
with architectural distortion, +/− a mass, and may have a Pearls
dark center. This should be regarded as a carcinoma until Di erential diagnosis:
proven otherwise. • High-grade malignancy.
3. Strictly, a ull mammographic workup needs to be • Metastases to the breast.
completed be ore proceeding to ultrasound, which could • Fibroadenoma.
be with spot views or tomosynthesis. However, i there
are no calcif cations associated with the mass, some
would go on directly to per orm targeted ultrasound Suggested Readings
scanning. Ultrasound core biopsy will be needed Bode MK, Rissanen T. Imaging f ndings and accuracy o
ollowing the diagnostic workup. MRI may be help ul i core needle biopsy in mucinous carcinoma o the breast.
the pectoral muscle is thought to be involved, or i other Acta Radiol. 2011;52(2):128-133.
oci are seen separate rom the index mass. Choi YJ, Seong MH, Choi SH, et al. Ultrasound and
4. Ultrasound is the easiest and cheapest way to per orm clinicopathological characteristics o triple receptor-negative
biopsy in this situation, under direct vision. In very breast cancers. J Breast Cancer. 2011;14(2):119-123.
dense breasts, or in circumstances where the lesion is Surov A, Fiedler E, Holzhausen HJ, Ruschke K, Schmoll
occult to conventional tests, MRI biopsy or PEM biopsy HJ, Spielmann RP. Metastases to the breast rom
(i avid) may be used. non-mammary malignancies: primary tumors, prevalence,
clinical signs, and radiological eatures. Acad Radiol.
2011;18(5):565-574.

102
Palpated lump in the left breast

1. What BI-RADS classif cation should be used


here?

2. What is the most likely pathology based on the


imaging?

3. What is the next best imaging test?

4. What type o biopsy would you recommend?

5. What is the medical di erential o unilateral


edema o the breast?

103
In ammatory breast with underlying cancer 1862
Case ranking/dif culty: Category: Diagnostic

Left CC spot magni cation. Note the “spiculate” margins and the Left LM spot magni cation view shows similar appearances.
“linear microcalci cation” associated with the tumor. Calci cations
appear to be growing down the spicules.

Answers lymphedema. As o the time o writing, there is no


1. This is a diagnostic examination as the patient has tomosynthesis-guided biopsy technique.
a palpable f nding. The f ndings are su f cient or 5. Unilateral lymphedema has been reported secondary
extremely high conf dence or malignancy, in that the to primary cardiac ailure, renal ailure, and
probability o malignancy is near 100%; there ore, brachiocephalic vein occlusion.
a BI-RADS 5 is appropriate. The only other setting
where these imaging appearances occur is post–
lumpectomy and radiation therapy, but you do not see Pearls
microcalcif cations unless there is residual disease, or
• Skin thickening is a sign o radiological “in ammatory”
developing at necrosis with “dystrophic” calcif cations.
breast cancer.
2. Yes, there is DCIS present, but this is associated with a • There may not be any associated clinical signs
spiculate mass. Masses are rarer in lobular cancer, where o in ammatory change.
the f ndings may be subtle, and present as mild distortion • Systemic disease can give similar appearances. Do not
or a shrinking breast. orget heart ailure.
• In ammatory cancer can occur in the presence or
3. The patient is likely to need chemotherapy in the
absence o an obvious primary (index) cancer.
neoadjuvant setting to make surgery possible, so accurate
staging o the disease is important. This includes staging
o the nodes in the axilla, as they are o ten the f rst to Suggested Readings
disappear with chemo.
Alunni JP. Imaging in ammatory breast cancer. Diagn Interv
4. In this case, the mass is visible, so an ultrasound biopsy Imaging. 2012;93(2):95-103.
is best. I no underlying mass is seen, then MRI needs Dilaveri CA, Mac Bride MB, Sandhu NP, Neal L, Ghosh K,
to be per ormed f rst to identi y any underlying mass, Wahner-Roedler DL. Breast mani estations o systemic
ollowed by second look ultrasound o any abnormality diseases. Int J Womens Health. 2012;4(4):35-43.
ound that might represent the primary disease. Uematsu T. MRI f ndings o in ammatory breast cancer, locally
Sometimes, incisional biopsy is required to conf rm that advanced breast cancer, and acute mastitis: T2-weighted
this is malignant inf ltration rather than simple unilateral images can increase the specif city o in ammatory breast
cancer. Breast Cancer. 2012;19(4):289-294.
104
Palpated lump in the left breast

1. What BI-RADS classif cation should be used


here?

2. What is the most likely pathology based on the


imaging?

3. What should be the next imaging investigation?

4. There is a palpable f nding in the le t axilla.


You f nd a node on ultrasound. What is the
chance that this is metastatic?

5. What type o biopsy would you recommend?

105
Synchronous bilateral breast cancer with nodal spread 1749
Case ranking/dif culty: Category: Diagnostic

Left breast ultrasound. Much more obvious on ultrasound


Right breast—“irregular mass” identi ed. There are few compared with the right breast as an “irregular mass” with
supporting signs of malignancy, with the absence of acoustic “heterogeneous echo pattern” and “complex acoustic
shadowing. attenuation.”“Duct extension” is seen, extending from the tumor.

Answers DNA subtyping arrays are requently per ormed to


1. Bilateral f ndings either a BI-RADS 4 or 5 depending in orm decision making about treatment and risk o
on how convinced you are that they are malignant. It is recurrence (recurrence index), which make tissue cores
alright to call a BI-RADS 5 on the le t and a BI-RADS 4 more appropriate. Stereotactic FNA cytology is historic,
on the right. You are allowed to give a separate BI-RADS having been used in the past, particularly in Europe, but
assessment or each side. it has no real use today.

2. The most common cancer is invasive ductal cancer. Lobular


cancer tends to present with distortion or shrinking breasts,
which can be di f cult to pick up when symmetrical. Pearls
3. The patient may have additional diagnostic f lms, • Bilateral f ndings i not circumscribed round or oval
especially spot magnif cation views i the patient has masses and without calcif cations are not obviously
calcif cation associated with either lesion, but the next other benign, and need urther workup.
workup is ultrasound to characterize both masses and stage • Symmetry is usually your riend.
the axilla. MRI may be used later, but in this situation, the • Use f rst principles and analyze the f ndings to
patient is likely to have a bilateral mastectomy; there ore, determine suspicion.
unless there is concern about involvement o the pectoral
muscle, MRI may not be required.
4. An abnormal node in the presence o a palpable lymph Suggested Readings
node f nding on clinical exam has been shown to Girardi V, Carbognin G, Camera L, et al. Multi ocal,
signif cantly correlate to involvement with metastases. multicentric and contralateral breast cancers: breast
One paper has the rate higher than 75%. There are MR imaging in the preoperative evaluation o patients
various published criteria or suspicious lymph nodes with newly diagnosed breast cancer. Radiol Med.
requiring biopsy: 2011;116(8):1226-1238.
Nichol AM, Yerushalmi R, Tyldesley S, et al. A case-match
• Cortical thickening >3 mm (4 mm i recent biopsy);
study comparing unilateral with synchronous bilateral
• Irregular thickening o cortex, or example, “hump”; and
breast cancer outcomes. J Clin Oncol. 2011;29(36):
• Length versus width ratio approaching 1:1.
4763-4768.
5. Ultrasound-guided core biopsy o the breast masses is Tonyali O, Tu an G, Benekli M, Coskun U, Buyukberber S.
important as tissue biomarkers including ER, PR, and Synchronous bilateral breast cancer in a patient with kindler
HER2 receptors need to be evaluated. These cannot syndrome. Clin Breast Cancer. 2012;12(2):145-146.
be easily done on FNA cytology. Also new multigene

106
Screening—asymptomatic

1. What is the BI-RADS category or this


screening exam?

2. What is the background breast density?

3. I you want to recall this patient, what are your


recommendations?

4. What is the risk o invasive cancer in this


patient?

5. I you were to describe the margins o the mass


on this mammogram, which terms would you
use?

107
Slow-growing tumor 673
Case ranking/dif culty: Category: Screening

Ultrasound—satellite lesion shows more malignant-looking


Ultrasound scan—“circumscribed” hypoechoic mass that would characteristics, being taller than wide (Stavros) with “irregular
appear benign in a di erent setting, although the margin is not margins” and some “acoustic shadowing.”
as sharp as you would expect for something like a broadenoma.

Answers 4. I you can determine the spiculations associated with this


1. There is clearly an abnormality in the le t breast that mass, you can give this a category 5 assessment, as it is
was perceived to be benign (BI-RADS 2) by the reading characteristic o malignancy. Many people pre er to call
radiologist, despite not having any prior exams. The this suspicious and give it a BI-RADS 4 (although in the
margins are irregular with some suspicion o distortion/ upper end o the BI-RADS 4 category).
spiculation, and the patient should have been given a 5. This is an “irregular mass” but with “indistinct margins.”
BI-RADS 0 and recalled or extra views and ultrasound Possibly even “spiculate.” Irregular describes the shape
scanning. I you believe that the f ndings are clearly o the mass, but not the margin. This was initially
suspicious, Sickles advises against calling it BI-RADS described as a round mass, but that is a benign descriptor
4 or 5 rom screening due to many patients going and does not apply here. Circumscribed is a margin
directly to surgery rather than being worked up f rst. descriptor, but does not apply to this case.
(Do not orget that BI-RADS 5 is more than 95% risk o
malignancy, and there are still up to 5% o those cases
that eventually turn out to be benign.)
Pearls
2. There is virtually no residual breast tissue in this
• Never ignore a mass in otherwise atty breasts.
screened woman, and so the accurate descriptor is
• Check prior f lms or stability.
“almost entirely at.”
• I never worked up, then do diagnostic workup.
3. There are di erent approaches to an obvious breast • I any suspicious eatures, then biopsy.
mass. Some people do direct ultrasound as the f rst test
on recall. BI-RADS, however, recommends completing
the diagnostic workup mammographically BEFORE Suggested Readings
proceeding with ultrasound. There are several reasons
or this, including doing the lateral f lm, so that you can Colleoni M, Rotmensz N, Maisonneuve MG, et al. Outcome
triangulate where the lesion will be ound on targeted o special types o luminal breast cancer. Ann Oncol.
ultrasound. The addition o spot (+/− magnif cation) 2012;23(6):1428-1436.
compression is used by many groups to Garne JP, Aspegren K, Linell F, Rank F, Ranstam J. Primary
prognostic actors in invasive breast cancer with special
(a) urther characterize the margins o the mass and re erence to ductal carcinoma and histologic malignancy
(b) to determine i there are any associated calcif cations, grade. Cancer. 1994;73(5):1438-1448.
representing DCIS either in or more importantly Lacroix-Triki M, Suarez PH, MacKay A, et al. Mucinous
outside o the index cancer, as this has management carcinoma o the breast is genomically distinct rom
implications. invasive ductal carcinomas o no special type. J Pathol.
2010;222(3):282-298.
108
Lump in the left breast

1. What BI-RADS descriptor would you use or


the shape o this mass?

2. Why is it important to identi y the associated


DCIS?

3. What diagnostic tests should you recommend?

4. What type o biopsy should you do?

5. I the biopsy comes back as at necrosis, what


is the next step?

109
High-grade IDC with calci cations 596
Case ranking/dif culty: Category: Diagnostic

Spot magni cation view (LSMCC) of spiculate mass with Targeted ultrasound examination—irregular mass containing
calci cations inside and outside the tumor. calci cation. In this image, you can identify the biopsy needle
passing through the inferior aspect of the tumor. More anterior
passes will be required, along with specimen x-rays, to determine
Answers harvesting of calci cation associated with the mass.

1. This is an irregular mass. I there are spicules arising


rom the mass that are easily visible, then a spiculate a place or rapid diagnosis o malignancy. But tissue is
mass would be applicable. required to personalize treatment, so core biopsy has
2. In dense breast tissue, the suspicious calcif cation may become the de ault method o biopsying the breast.
be the only clue that you have a tumor growing in the 5. I you gave it BI-RADS 5, then the f nding o at
breast. DCIS associated with invasive cancer is very necrosis is not concordant, and you have to recommend
commonly ound, but may be calcif ed on noncalcif ed. surgical excision. I it was a BI-RADS 4 with more
The density associated with the calcif cation on dystrophic calcif cations, you could do almost all o the
mammography or ultrasound is the trigger to think above. At the end o the day, you need more tissue in a
about associated invasive disease. Calcif cation outside lesion that has so many suspicious f ndings.
o the tumor due to DCIS may have an impact o the
management o the patient. This is called extra invasive
component and is associated with a high risk o local
recurrence, even a ter radiation treatment. Pearls
3. Spot magnif cation views should be per ormed o the • High-grade tumors in older women may present as
mass and calcium to get a better idea o the spread o pseudocircumscribed masses.
the calcif cations in the breast and to measure extent. • A combination o mammography and ultrasound may
Ultrasound will help to evaluate the presence o invasive be required to prompt biopsy.
disease. In this sort o case, MRI may give a better
estimate o extent o disease. PET/CT and PEM have no
role in the diagnostic workup o this patient, unless there Suggested Readings
is evidence o metastatic disease.
Atalay C, Irkkan C. Predictive actors or residual disease
4. Although all are theoretically correct, the best type o in re-excision specimens a ter breast-conserving surgery.
biopsy is the astest and cheapest to get a diagnosis. Breast J. 2012;18(4):339-344.
Since we require tissue typing and biomarkers, core Barbalaco Neto G, Rossetti C, Fonseca FL, Valenti VE,
biopsy should be per ormed. I you have trouble de Abreu LC. Ductal carcinoma in situ in core needle
seeing the lesion on ultrasound, then stereotactic core biopsies and its association with extensive in situ
biopsy may be used. MRI biopsy gives the highest component in the surgical specimen. Int Arch Med.
reimbursement, but it is not optimal in this patient. FNA 2012;5(1):19.
is used, particularly in Europe, and I believe it still has

110
Screening—asymptomatic

1. What BI-RADS classif cation should be used


here?

2. What is the risk o malignancy with a ocal


asymmetry in this position?

3. What urther imaging would you recommend?

4. What type o biopsy should be per ormed?

5. The patient states she has a skin lesion on the


inner part o the right breast. What do you do?

111
Cancer presenting as a focal asymmetry in a DANGER AREA 1583
Case ranking/dif culty: Category: Screening

RCC spot magni cation con rms an “ill-de ned” soft tissue “mass” Ultrasound con rms a correlate of an “irregular mass” at the site
in the inner half of the CC lm. of the mammographic abnormality.

Answers 5. Repeating the relevant right mammograms with a skin


1. This is a screening examination; there ore, a BI-RADS 0 marker may demonstrate clearly that the mammographic
assessment should be rendered with a recommendation f nding is in act in the skin and projected over the
or urther f lms. Further workup should include lateral breast. You can risk introducing in ection by doing a
and spot magnif cation views to determine i there is a punch biopsy o a sebaceous cyst.
mass. I truly a mass, then ultrasound may be required.
2. Same question but in a di erent ormat, as you do
not make the risk o malignancy until you have done Pearls
the diagnostic workup. There is a ocal asymmetry in • Small breast cancers can appear like normal breast
one o the danger areas in the medial aspect o the CC tissue.
f lm. This increases the risk o malignancy, similar to a • DANGER area such as the immediately posterior
developing ocal asymmetry. medial aspect o the CC f lms is one place, as is the
3. Diagnostic mammograms or tomosynthesis should be in erior mammary old on the MLO view, where
per ormed to complete the diagnostic workup be ore cancers occur simulating normal entities.
targeted ultrasound being per ormed. The mass is usually
visible on ultrasound.
4. Cytology is use ul or a rapid diagnosis o malignancy Suggested Readings
when you have a trained breast cytopathologist available. Leung JW, Sickles EA. Developing asymmetry identif ed
Core biopsy with placement o a marker clip is the best on mammography: correlation with imaging outcome
way to get a diagnosis and tissue biomarkers BEFORE and pathologic f ndings. AJR Am J Roentgenol.
surgery. Stereotactic core biopsy is technically di f cult 2007;188(3):667-675.
in this location. MRI biopsy not warranted. Surgical Sickles EA. The spectrum o breast asymmetries: imaging
excision should not be carried out be ore a preoperative eatures, work-up, management. Radiol Clin North Am.
diagnosis is obtained. 2007;45(5):765-771, v.
Venkatesan A, Chu P, Kerlikowske K, Sickles EA, Smith-
Bindman R. Positive predictive value o specif c
mammographic f ndings according to reader and patient
variables. Radiology. 2009;250(3):648-657.

112
Palpable lump in the left breast—images of pre - and posttreatment

1. Why is there a di erence between the sets o


images?

2. What BI-RADS category would you place this


lesion in?

3. What is the most likely pathology?

4. What tissue biomarkers are routinely measured


in a patient with breast cancer?

5. The patient has now completed treatment.


What is your role now to direct the surgeon?

113
Good response to neoadjuvant chemotherapy 675
Case ranking/dif culty: Category: Diagnostic

Ultrasound of mass posttreatment. There is a mix of solid and


uid (necrotic debris).

Ultrasound of mass pretreatment. Mainly uniphasic mass.


5. The surgeons need accurate marking o the position o the
lesion now that the mass in no longer palpable. Because
Answers you placed a marker clip be ore neoadjuvant chemotherapy,
1. Sometimes, postsurgery f lms can look like this, BUT you can now f nd the clip or the surgeon. It is the patient’s
there are no supporting signs that the patient has had decision, not yours, to decide whether, having shrunk the
surgery, or example, a scar, surgical clips, skin thickening, tumor, she will have a mastectomy or not. Injection o dye
and so on. I the patient is unf t or surgery, sometimes is done in the OR as it rapidly moves to the nodes. Injection
they are o ered neoadjuvant hormone therapy, as o ten o tracer or sentinel node can be done at the time o the
you can obtain control and avoid surgery. The usual needle localization or in the OR ready room.
reason or these appearances is due to the patient having
neoadjuvant chemotherapy (NAC) or the cancer. The
only reason or NAC in this patient was that the tumor was Pearls
stuck to the pectoral muscle, and de orming surgery would • Response to neoadjuvant chemotherapy can be marked,
have to have taken place i chemo was not considered f rst. with many patients having a pathological complete
response.
2. In this case, the patient was not a screener, and there ore
• MRI is a good tool to monitor response to chemotherapy,
not BI-RADS 0. I anything, it would be a BI-RADS 4 or
both with volume measurements and in determining the
5 depending on your suspicion o malignancy. However,
type o response (due to enhancement patterns).
in this patient, she has a known diagnosis o breast
• I MRI is not available, ultrasound has also been
cancer (BI-RADS 6), and is now presenting ollowing
success ully used or monitoring response, as well as
NAC with a lesion that has shrunk signif cantly to allow
molecular breast imaging and other new techniques.
success ul surgery.
3. Mucinous carcinomas typically appear semicystic or
with evidence o enhancement on ultrasound. Tubular Suggested Readings
cancers tend to present as spiculate masses with LONG de Bazelaire C, Calmon R, Thomassin I, et al. Accuracy
SPICULES. Lobular cancer is usually subtle and o per usion MRI with high spatial but low temporal
presents as vague distortion, or developing asymmetry. resolution to assess invasive breast cancer response to
DCIS typically presents with calcif cations. neoadjuvant chemotherapy: a retrospective study. BMC
Cancer. 2011;11(11):361.
4. The routine biomarkers used on all breast cancer patients
Lyou CY, Cho N, Kim SM, et al. Computer-aided evaluation
are ER, PR, and HER2 status. This assists in breaking down
o breast MRI or the residual tumor extent and response
appropriate treatment by identi ying the biological type
monitoring in breast cancer patients receiving neoadjuvant
o breast cancer. Ki-67 is a proli eration index that assists
chemotherapy. Korean J Radiol. 2011;12(1):34-43.
with recurrence risk, but not routinely measured. However,
Shin HJ, Baek HM, Ahn JH, et al. Prediction o pathologic
things are changing rapidly, and various gene panels such
response to neoadjuvant chemotherapy in patients with
as Oncotype Dx and MammaPrint are being used more and
breast cancer using di usion-weighted imaging and MRS.
more to subtype tumors and strati y chemotherapy.
NMR Biomed. 2012;25(12):1349-1359.
114
Outside abnormal screening mammogram—here for diagnostic workup

1. What BI-RADS classif cation should be used


here?

2. What is the requency o multiple f ndings in a


breast with a known primary breast cancer?

3. Patients with multi ocal breast cancer have an


increased risk o which o the ollowing?

4. What should be the next imaging


investigations?

5. What type o biopsy would you recommend?

115
Multifocal breast cancer 1589
Case ranking/dif culty: Category: Diagnostic

Ultrasound: Lesion 2—This shows the mass has a central re ective


area. Check back with the mammogram to determine if there is
Ultrasound: Lesion 1—“Ill-de ned”“irregular” mass that is any potential for associated DCIS.
“nonparallel” (taller than wide). It has low level echoes inside and
there is a minor degree of “acoustic attenuation.”
4. I you have tomosynthesis, then it can replace the
diagnostic workup or multiple masses. Targeted
ultrasound can then be per ormed to document the
masses and evaluate or locoregional spread. MRI will
also need to be per ormed as the best method to establish
extent o disease, especially i breast conservation is
being considered. Techniques such as breast specif c
gamma imaging (BSGI) and positron emission
mammography (PEM) may be used in dense breasts or
suspected multi ocality.
5. Ultrasound is the best biopsy tool i the masses can be
easily ound. Rarely, the masses are hard to visualize on
MRI—Subtracted axial MIP. This is often the best way of showing mammography and stereotactic core biopsy o two o the
the relationships of the various masses in 3D space, plus it allows masses, urthest apart may need to be done.
for staging and assessment of the extent of the mass and distance
from the nipple.

Pearls
Answers
• When you see one cancer, look or the second.
1. This is not a screening exam. There are multiple irregular • I you see two, look even harder or more.
masses containing microcalcif cation, suggesting DCIS, • Determine whether in same segment (multi ocal) or not.
and spot magnif cation is needed to characterize the • MRI should be per ormed or staging, especially i the
calcif c particles. patient wishes to consider breast conservation.
2. The data vary on this, with estimates o between 10%
and 20%. With the advent o regular MRI scans or
staging purposes, more second ipsilateral and also Suggested Readings
contralateral second primaries are being detected,
suggesting that the real number is yet unknown. Ustaalioglu BO, Bilici A, Ke eli U, et al. The importance o
multi ocal/multicentric tumor on the disease- ree survival
3. The larger tumor burden and likelihood o locoregional o breast cancer patients: single center experience. Am J
spread means that nodal involvement and systemic Clin Oncol. 2011;35(6):580-586.
metastases are more likely with multiple cancers. Spanu A, Chessa F, Battista Meloni G, et al.
Several authors have suggested that tumor size should Scintimammography with high resolution dedicated breast
be aggregated or the patients to receive appropriate camera and mammography in multi ocal, multicentric and
therapy. Currently, only the largest o the tumors is used bilateral breast cancer detection: a comparative study. Q J
or prognosis calculations. Nucl Med Mol Imaging. 2009;53(2):133-143.
Yang WT. Staging o breast cancer with ultrasound. Semin
Ultrasound CT MR. 2011;32(4):331-341.
116
Screening—asymptomatic

1. What BI-RADS classif cation should be used


here?

2. The prior exams are now available. Stable


since 3 years earlier. What is your BI-RADS
assessment?

3. I ound to be benign, what is the likely


pathology?

4. I ound to be malignant, what is the likely


pathology?

5. What type o biopsy would you recommend?

117
Screening cancer 622
Case ranking/dif culty: Category: Screening

Left CC: current exam close-up. This Left CC: 2009 exam close-up. On the prior Ultrasound—looked similar in 2009 to 2011, but
demonstrates an “irregular mass” exam, the radiologist decided the nding just smaller.
with “spiculate” margins. was due to an intramammary lymph node,
and because it was “stable” did not recall
it. Just because it is stable, does not mean
it is benign—use margin characteristics
to determine whether the nding needs
further workup.

Answers 5. This mass is not palpable. Ultrasound-guided FNA


1. This is an abnormal screening mammogram, and the cytology or core biopsy is appropriate, depending on the
appropriate BI-RADS category to use is there ore availability o a breast pathologist. Stereotactic biopsy
BI-RADS 0: Further workup needed. can be used when not visible on ultrasound scanning.
Surgical biopsy is historical in this situation, when a
2. There is no change since the prior exam. It is a solitary preoperative diagnosis can be made by needle biopsy.
mass seen in atty breasts. Some would give this a BI-
RADS 2, benign, as the lesion has been stable or several
years, but the lesion is too small and the margins not
well enough seen to leave alone without a ull diagnostic Pearls
workup. For that reason, a BI-RADS 0, needs urther • Small mass in atty breasts, suspicious until proven
imaging, would be appropriate. otherwise.
• Do ull diagnostic workup.
3. Isolated papillomas tend to be central. Multiple
• Have a low threshold or biopsy in this situation.
papillomas tend to be peripheral. Fibroadenoma and
simple cysts can give these appearances. I a cyst
has been present or 3 years, there is likely to be
proteinaceous secretions within and is there ore more Suggested Readings
likely to be a complicated cyst than a simple cyst. Note: Esserman LJ, Shieh Y, Rutgers EJ, et al. Impact o
The BI-RADS lexicon dropped the term “complex cyst” mammographic screening on the detection o good and
in the 4th edition (2003) as there was con usion, and the poor prognosis breast cancers. Breast Cancer Res Treat.
only term that should be used is “complicated cyst.” An 2011;130(3):725-734.
ectopic intramammary node is possible, but there is no Tamaki K, Ishida T, Miyashita M, et al. Correlation
notch seen or atty hilum to indicate this. between mammographic f ndings and corresponding
4. High- and intermediate-grade invasive ductal carcinoma histopathology: potential predictors or biological
is unlikely to have been relatively stable or 3 years. It characteristics o breast diseases. Cancer Sci.
is ar more likely to be a low-grade tumor or possibly 2011;102(12):2179-2185.
a special type o invasive ductal, such as a mucinous
cancer. Phyllodes tumors can be ound in older patients,
but one eature is rapid growth, even the benign end o
the spectrum o PT.

118
Screening—asymptomatic

1. What BI-RADS classif cation should be used


here?

2. What is the most likely pathology based on the


imaging?

3. What is the next best imaging test?

4. This is a special type o IDC called mucinous


(colloid). What are the ultrasound eatures that
you expect to see?

5. What is the most common age group to f nd


this special type o tumor?

119
Small mucinous carcinoma (special type of IDC) 696
Case ranking/dif culty: Category: Screening
4. This lesion is o ten seen as a solid mass with acoustic
enhancement on ultrasound. It has either irregular or
indistinct margins. The echogenicity is rarely anechoic,
unless your ultrasound machine settings are incorrect.
Spiculation is much rarer due to the lesion being very
slow growing and not attracting a strong desmoplastic
reaction.
5. This is more common in the elderly, with a peak
incidence in the 70s.

Pearls
• Special type o IDC.
Ultrasound con rms a “nonparallel” or taller than wide mass with • Good prognosis.
irregular margins. • Mucin containing, there ore, can have acoustic
enhancement on ultrasound.
Answers
1. This is clearly not a benign f nding within atty breasts,
Suggested Readings
and requires urther workup. There ore, BI-RADS 0.
Bode MK, Rissanen T. Imaging f ndings and accuracy o
2. The f ndings are typical or a developing invasive ductal core needle biopsy in mucinous carcinoma o the breast.
carcinoma, and sometimes a special subtype that does Acta Radiol. 2011;52(2):128-133.
not have specif c unique imaging eatures. Lacroix-Triki M, Suarez PH, MacKay A, et al. Mucinous
3. Diagnostic workup that may include spot (magnif cation) carcinoma o the breast is genomically distinct rom
views and ultrasound, but may include tomosynthesis in invasive ductal carcinomas o no special type. J Pathol.
place o regular diagnostic mammograms. Ultrasound 2010;222(3):282-298.
should be per ormed when you have completed the Lam WW, Chu WC, Tse GM, Ma TK. Sonographic
mammographic workup. appearance o mucinous carcinoma o the breast. AJR Am
J Roentgenol. 2004;182(4):1069-1074.

120
Lump for 3 years—diagnostic exam

1. What BI-RADS classif cation should be used


here?

2. What should be the next diagnostic imaging


exam?

3. What type o biopsy would you consider with


this lesion?

4. Pathology shows DCIS high grade with no


invasion. What is your recommendation?

5. The f nding shows Gd 3 IDC ER–PR–HER2–.


What is this f nding called?

121
Large tumor at presentation 617
Case ranking/dif culty: Category: Diagnostic

Ultrasound shows a predominantly “oval mass”with “circumscribed


margins” and two gentle lobulations. The mass is larger than the
width of the probe. The ultrasound appearances are therefore
LCC spot compression shows that the mass margins are benign, and are trumped by the mammographic appearances.
“microlobulated” and “ill de ned.” The mass contains a marker
clip from a biopsy.
to obtain, as we need tissue biomarkers preoperatively
in a patient who could potentially go on to have
neoadjuvant chemotherapy. Surgical excision without
preoperative biopsy is not standard care.
4. The patient has a f nding that is highly unlikely to be
DCIS; there ore, the f nding is nonconcordant. You need
to have an invasive cancer that is ER/PR negative be ore
chemotherapy can be justif ed. Treatment o DCIS is
surgical. Either you need to repeat the core biopsy and
target a margin o the lesion or get an excisional biopsy.
5. The lack o estrogen or progesterone-receptor positivity,
plus the lack o c-ERB (HER) 2 overexpression, means
Ultrasound—compound imaging in an attempt to show the size
that this is a triple negative.
of the mass.

Answers Pearls
1. This is not a screening examination as the patient has • Large tumors have increased risk o locoregional spread.
palpable f ndings. Although the lesion is suspicious • MRI should be considered or staging to visualize
o malignancy, the margins being so smooth and internal mammary and Rotter’s node involvement.
circumscribed means that I would give a BI-RADS 4
rather than a 5 in this situation.
Suggested Readings
2. Tomosynthesis, i you have it, would likely have already
been used. Diagnostic mammograms have lesser value Croshaw R, Shapiro-Wright H, Svensson E, Erb K, Julian T.
unless you can see associated microcalcif cations Accuracy o clinical examination, digital mammogram,
inside or outside o the tumor (extensive intraductal ultrasound, and MRI in determining postneoadjuvant
component), which would have an additional impact on pathologic tumor response in operable breast cancer
treatment. MRI or large tumors, especially i lobular, patients. Ann Surg Oncol. 2011;18(11):3160-3163.
or those associated with DCIS, is very help ul or Singer L, Wilmes LJ, Saritas EU, et al. High-resolution
extent o disease and surgical planning. Also good or di usion-weighted magnetic resonance imaging in
detecting involved internal mammary nodes and Rotters patients with locally advanced breast cancer. Acad Radiol.
(interpectoral) nodes. I the mass is locally advanced, 2012;19(5):526-534.
then PET/CT or staging is recommended. Uematsu T. MRI f ndings o in ammatory breast cancer, locally
advanced breast cancer, and acute mastitis: T2-weighted
3. Palpation-guided biopsy is not as accurate as ultrasound- images can increase the specif city o in ammatory breast
guided core biopsy. Core biopsy is the best type o tissue cancer. Breast Cancer. 2012;19(4):289-294.
122
Change in the left breast—diagnostic workup

1. What BI-RADS classif cation should be used


here?

2. What is the most likely pathology based on the


imaging?

3. The patient has had an ultrasound conf rming


the mass. What is the next diagnostic test?

4. Biopsy shows IDC. What is the next imaging


test you would recommend?

5. The patient has now completed treatment.


What is your role now to direct the surgeon?

123
Multifocal carcinoma with nodal metastases 620
Case ranking/dif culty: Category: Diagnostic

Irregular mass containing calci cations. MRI—Thin MIP showing mass plus intramammary and also
axillary nodes, not appreciated on mammography.

Answers
1. This is a diagnostic examination. The f ndings are 5. A repeat MRI to see the current playing f eld is an
characteristic, allowing you to give a B-RADS 5 important part o presurgical intervention a ter chemo.
assessment. Similar comments or PET/CT, although this may vary
by center. The tumor will be analyzed when excised
2. In ammatory breast cancer is usually invasive ductal surgically. Patients may go on to axillary dissection
cancer, although sometimes it is seen with a DCIS mass, regardless o you f nding a normal axilla at this stage.
and no proven invasive ocus on biopsy. Invasive lobular
usually presents late with a hard nodular and shrunken
breast. In ection should not present with this type o
appearance. An in ected sebaceous cyst will be obvious Pearls
on physical examination. I this were postlumpectomy, • Cancer that causes edema o the breast and skin
the eatures are more compatible with an in ammatory thickening is called in ammatory breast cancer, but
recurrence. this does not necessarily mean that there is clinical
in ammatory cancer.
3. When a patient with a likely malignancy is already having
• We can identi y early changes o in ammatory cancer
a diagnostic ultrasound, it is easy to per orm axillary
better on imaging than on physical exam.
staging at the same visit, to speed up the diagnostic
• Watch or additional oci o disease.
process. It has the added benef t o determining whether
• Evaluate nodes be ore neoadjuvant chemotherapy.
any nodes look abnormal, and you can then recommend
biopsy o the node(s). Ultrasound core biopsy needs
to be done to conf rm the diagnosis and also to allow
the measurement o tissue biomarkers to determine the Suggested Readings
subtype o the tumor. Ultrasound FNA cytology can be Alunni JP. Imaging in ammatory breast cancer. Diagn Interv
per ormed, but that limits the diagnosis to malignancy Imaging. 2012;93(2):95-103.
only rather than tissue required pre-neoadjuvant Boisserie-Lacroix M, Debled M, Tunon de Lara C, Hurtevent
chemotherapy. There is no place or surgical excision to G, Asad-Syed M, Ferron S. The in ammatory breast:
make a diagnosis o breast cancer in this setting. management, decision-making algorithms, therapeutic
4. The f rst imaging test should be MRI to determine the principles. Diagn Interv Imaging. 2012;93(2):126-136.
extent o the disease, to screen the contralateral breast, Uematsu T. MRI f ndings o in ammatory breast cancer, locally
and to image the locoregional lymphatic drainage. In advanced breast cancer, and acute mastitis: T2-weighted
many centers, a PET/CT is also used at this stage or images can increase the specif city o in ammatory breast
staging purposes. PEM and BSGI are sometimes use ul cancer. Breast Cancer. 2012;19(4):289-294.
to f nd other tumors in dense breasts. Surgical incisional
biopsy is not required, as lymphatic involvement can be
seen on core biopsy.
124
Hardness around the right nipple—prior left mastectomy

1. Which o the BI-RADS descriptors most


accurately represents the f ndings?

2. What is the most common type o tumors


giving these appearances?

3. What BI-RADS score is appropriate in this


case?

4. What are the likely f ndings on physical


examination?

5. What are the radiologic eatures o


“in ammatory” cancer?

125
Subareolar cancer 599
Case ranking/dif culty: Category: Diagnostic

4. Most o the above f ndings may be seen in a patient with


this type o mammogram. Occasionally, a subareolar
malignancy may not present with any physical f nding.
5. Radiological “in ammatory” carcinoma requently is
seen be ore any physical f ndings o skin redness or
“peau d’orange.” Blockage o skin lymphatics by tumor
emboli causes the radiological eatures o in ammatory
carcinoma. There is a higher likelihood o involved
axillary nodes in in ammatory carcinoma.

Pearls
• Skin thickening with increased density o supporting
structures o the breast is o ten due to in ammatory
carcinoma.
Ultrasound of subareolar region showing distortion and
shadowing.

Suggested Readings
Answers
Caumo F, Gaioni MB, Bonetti F, Man rin E, Remo A, Pattaro
1. Although a mass is present, it is di f cult to see because C. Occult in ammatory breast cancer: review o clinical,
o the surrounding dense breast tissue, the retroareolar mammographic, US and pathologic signs. Radiol Med.
nature o the lump, and the associated distortion. It 2005;109(4):308-320.
would be accurate to report distortion as the major Harrison AM, Zendejas B, Ali SM, Scow JS, Farley DR.
f nding in this case, and say a “possible” mass is present. Lessons learned rom an unusual case o in ammatory
2. Tubular carcinoma typically has LONG spicules with a breast cancer. J Surg Educ. 2012;69(3):350-354.
small central mass. Lobular carcinomas are requently Uematsu T. MRI f ndings o in ammatory breast cancer,
di f cult to see because o their growth pattern, and may locally advanced breast cancer, and acute mastitis:
present as an asymmetry or distortion. Invasive ductal T2-weighted images can increase the specif city
carcinoma (no special type) may cause distortion due to o in ammatory breast cancer. Breast Cancer.
degree o invasion. 2012;19(4):289-294.

3. This is typical o a central/subareolar cancer causing


distortion, so a BI-RADS 5 should be appropriate. I you
think there is a possibility o another cause, then give a
BI-RADS 4.

126
52-year-old patient with palpable lump in the left breast

1. What is included in the diagnostic workup o a


palpable mass?

2. What is the signif cance o the lymph nodes


seen on the mammogram?

3. How can the mass be described on the


mammogram?

4. What would be the next step a ter the spot


compression views?

5. I the mass is not seen on ultrasound, what


would be the next step?

127
Lymphadenopathy and palpable mass 381
Case ranking/dif culty: Category: Diagnostic

Gray-scale ultrasound of left breast hypoechoic mass with Gray-scale ultrasound with duplex demonstrating increased
“lobulated” shape and “angulated margin.” central ow.

“indistinct,” or “spiculated”), and by their density (“high,”


“equal,” or “low density”). This mass has “lobular” shape
and partially “obscured” margin and is o “equal” density
in comparison with the f broglandular tissue.
4. Ultrasound would be the next step to work up the mass
and the lymph nodes.
5. I the mass is not seen on ultrasound, stereotactic biopsy
is the next step to obtain tissue.

Pearls
• A ter ultrasound-guided biopsy o suspicious f nding
MRI, T1-weighted image after IV contrast, subtraction technique, in the breast, it is help ul to search or lymph nodes
demonstrating mass corresponding to index lesion (arrow) and and per orm biopsy, i suspicious lymph nodes can be
lymph nodes. detected.
• The biopsy o the lymph node can be per ormed as f ne
Answers needle aspiration or as core biopsy with a 14-gauge
needle or even with larger-core biopsy needle,
1. Workup includes standard mammogram and spot depending on the location o the lymph node.
compression views with BB marker on the area o • Pathology demonstrated in this case presents invasive
concern. In addition, ultrasound should be per ormed. ductal carcinoma and metastatic carcinoma in the
Thermography is a technique that uses in rared sensors suspicious lymph node.
to detect heat and is not recognized as being part o
evidenced-based breast imaging.
2. The lymph nodes as seen in the upper outer quadrant Suggested Readings
and axilla are relatively small, none is larger than
1.5 cm, but they are relatively dense and no atty Abe H, Schmidt RA, Kulkarni K, Sennett CA, Mueller JS,
hilum is recognized. Given the presence o a palpable Newstead GM. Axillary lymph nodes suspicious or breast
abnormality, the presence o lymph nodes makes the cancer metastasis: sampling with US-guided 14-gauge
palpable mass even more suspicious and raises concern core-needle biopsy—clinical experience in 100 patients.
or possible metastatic disease. Radiology. 2009;250(1):41-49.
Abe H, Schmidt RA, Sennett CA, Shimauchi A, Newstead
3. Masses are described by shape (“round” and “oval”— GM. US-guided core needle biopsy o axillary lymph
“lobular” or “irregular”), by the appearance o their nodes in patients with breast cancer: why and how to do
margin (“circumscribed,” “microlobulated,” “obscured,” it. Radiographics. 2007;27(Suppl 1):S91-S99.
128
Ulcer on the right breast—palpable mass

1. What BI-RADS classif cation should be used


here?

2. What should be the next diagnostic imaging


exam?

3. Core biopsy shows ER–PR–HER2+ IDC.


What is your recommendation?

4. MRI and PET/CT both show nodes to level 2


in the axilla and interpectoral space. What do
you recommend?

5. Three months posttreatment, ollow-up


imaging is now normal. What is your
BI-RADS assessment?

129
What if it is left too long? Fungating and bilateral 609
Case ranking/dif culty: Category: Diagnostic

LXCCL shows possibly two Right breast ultrasound—the tumor takes up the Left breast ultrasound con rms the presence of
separate masses in the outer whole eld. another cancer in the contralateral breast.
left breast.

Answers HOWEVER, the patient has had biopsy-proven


1. Hope ully, with a ungating mass, you would be happy malignancy and has not had surgery to excise the
to give this lesion a BI-RADS 5. Always watch or “cancer” and there ore the appropriate BI-RADS
second cancer. category is 6: known malignancy. Surgical treatment is
recommended. You could edit the report to say that it is a
2. With such a large tumor, urther diagnostic BI-RADS 6: known TREATED malignancy.
mammograms o tomosynthesis would be utile.
Ultrasound scanning to determine areas o viable tissue,
away rom large blood vessels or biopsy, is one o
the most important next tasks. While scanning, it is Pearls
worth staging the axilla or lymphadenopathy. MRI • Dense breast due to ungating locally advanced breast
will be used later or staging the disease and potential cancer.
or resectability. PEM does not add anything to the • Ultrasound and MRI have a major role in assessing the
diagnosis in this type o case. extent o the disease and per orming staging, as well as
3. Initial staging or a presumed stage 4 tumor is to per orm monitoring the response to neoadjuvant chemotherapy.
an MRI (i technically possible) or chest wall invasion
and mapping o lymph nodes. PET/CT gives additional
in ormation in the thorax, especially i the patient is Suggested Readings
unable to lay prone or the MRI. Surgery may not
Croshaw R, Shapiro-Wright H, Svensson E, Erb K, Julian T.
be possible until the lesion has been shrunken down,
Accuracy o clinical examination, digital mammogram,
and then is likely to be a “toilet” mastectomy post–
ultrasound, and MRI in determining postneoadjuvant
neoadjuvant chemotherapy.
pathologic tumor response in operable breast cancer
4. While the patient may need neoadjuvant chemotherapy, patients. Ann Surg Oncol. 2011;18(11):3160-3163.
staging o the nodes with either FNA or core biopsy Singer L, Wilmes LJ, Saritas EU, et al. High-resolution
has become important, especially as more patients are di usion-weighted magnetic resonance imaging in
getting complete pathological response to chemotherapy. patients with locally advanced breast cancer. Acad Radiol.
Evidence that at least one node was involved helps 2012;19(5):526-534.
in the determination o appropriate treatment Uematsu T. MRI f ndings o in ammatory breast cancer, locally
postchemotherapy. Some groups pre er small-gauge core advanced breast cancer, and acute mastitis: T2-weighted
biopsy to get tissue biomarkers rom the metastatic node. images can increase the specif city o in ammatory breast
cancer. Breast Cancer. 2012;19(4):289-294.
5. As there is no evidence o malignancy, you could use
BI-RADS 1, but there is likely to be a residual biopsy
clip, so BI-RADS 2 would be more appropriate.

130
60-year-old female with palpable mass in the right upper outer quadrant

1. What should be included in workup or


palpable mass?

2. What would be the next step i mammogram


and ultrasound are normal?

3. Patient had recent normal mammogram


3 months ago and is re erred or workup
o lump. What is the workup?

4. What is the abnormality in this case?

5. What is the appearance o the abnormality


on ultrasound?
131
Invasive ductal carcinoma 318
Case ranking/dif culty: Category: Diagnostic

Ultrasound B mode image demonstrates corresponding


“spiculated” mass.
Spot compression of right MLO view with BB on palpable
abnormality.
3. Given the presence o a normal mammogram
3 months ago, it is not unreasonable, a ter reviewing
the old images, to start with an ultrasound f rst.
I ultrasound does not show any abnormality—
depending on the situation, and also the density o
the recent mammogram—workup can be stopped or
repeat mammogram can be per ormed. I mammogram
is more than 6 months old, the f rst step is to repeat the
mammogram or the symptomatic side.
4. Noted is the mass right upper outer quadrant that is
better evaluated on the submitted spot compression view.
On spot compression view, the mass is irregular and
lobulated and o increased density.
Ultrasound with duplex of the palpable abnormality. 5. Ultrasound demonstrates “irregular” marginated mass
with increased ow and in part “posterior acoustic
enhancement.”
Answers
1. For patients older than 30 years, the f rst step is a
mammogram with BB marker over the area o palpable Pearls
lump. Mammogram also includes spot compression
views with BB marker. Next step—regardless o • The consequence o classi ying mass as BI-RADS 5
the outcome o the mammogram—is to per orm instead o BI-RADS 4 is that in case pathology would
an ultrasound. Ultrasound is important not only or show a benign f nding such as f brosis, the f nding
characterization o the mass i seen on mammogram but would not be concordant.
also or the choice or possible biopsy. • Any BI-RADS 5 f nding is highly suspicious, and
i core biopsy results in benign histology, surgical
2. Despite normal imaging, there is a possibility o the excision is the next step.
presence o a malignancy in the scenario o palpable • Any BI-RADS 4 f nding that has benign histology
abnormalities. Further management should be based on on core biopsy can be concordant, depending on the
clinical grounds, meaning that usually the gynecologist situation, and ollow-up might be appropriate.
or the primary care physician who re ers the patient has
to decide i the palpable f nding is worrisome enough to
send the patient to a breast surgeon. Based on the clinical Suggested Reading
evaluation o the breast surgeon, a biopsy might be Parikh JR. ACR appropriateness criteria on palpable breast
per ormed without imaging guidance. masses. J Am Coll Radiol. 2007;4(5):285-288.
132
Colon cancer staging CT scan

1. What BI-RADS classif cation should be used


here?

2. What is the next best imaging test?

3. I you know that this patient has primary


colon cancer, what would be the most likely
appearances o breast metastases?

4. Why should you per orm a mammogram i


there is a f nding on CT?

5. What CT eatures may assist in characterizing


a primary breast mass on CT?

133
Incidental mass on CT staging for colon cancer 601
Case ranking/dif culty: Category: Diagnostic

Left ML. Left CC. Ultrasound scanning shows second tumor toward nipple.

Answers Correlation with recent mammograms is important. A 2011


1. This is a trick question, as BI-RADS does not apply paper in European Journal of Radiology demonstrated
to incidental breast f ndings on CT scans. Appropriate increased enhancement in malignant masses on CT, but the
mammographic and ultrasound workup needs to be bottom line is that a f nding on CT should be worked up in
per ormed. the conventional manner.

2. This patient needs diagnostic mammograms to determine


the nature o the lesion seen on CT.
Pearls
I conf rmed, then an ultrasound scan can be per ormed.
A screening exam only allows or CC and MLO • Margins are the most important CT f nding.
views (in the United States). As a result, diagnostic • I circumscribed, likely benign, except or a patient
mammography is the best next test. with a known primary who may present with oval
circumscribed metastases.
3. Breast metastases are rare, but requently present as
circumscribed masses.
Calcif cations within breast masses, especially i Suggested Readings
dystrophic, have a benign connotation—usually due to
degenerating f broadenomas. Irregular masses can be Adejolu M, Huo L, Rohren E, Santiago L, Yang WT.
ound, as in this case. See case o melanoma metastases False-positive lesions mimicking breast cancer on
or comparison. FDG PET and PET/CT. AJR Am J Roentgenol.
2012;198(3):W304-W314.
4. Mammography is the gold standard or breast imaging, Nakamura N, Tsunoda H, Takahashi O, et al. Frequency and
and a suspicious f nding on CT (or other modality) clinical signif cance o previously undetected incidental
should trigger a diagnostic workup. Calcif cations are f ndings detected on computed tomography simulation
best seen with mammography. Mass margins can be scans or breast cancer patients. Int J Radiat Oncol Biol
characterized by the use o spot (magnif cation) f lms. Phys. 2012;84(3):602-605.
I cancer is present in one breast, you need to screen the Surov A, Fiedler E, Wienke A, Holzhausen HJ, Spielmann
contralateral breast. RP, Behrmann C. Intramammary incidental f ndings
5. Ultimately, it is not easy to characterize a mass on CT on staging computer tomography. Eur J Radiol.
f ndings only. However, dense dystrophic calcif cation 2011;81(9):2174-2178.
within f broadenomas can help in the diagnosis.

134
Palpable lump in the left breast upper outer quadrant

1. What BI-RADS score should be given in this


case?

2. What is the most likely pathology given these


appearances?

3. What is the next important investigation?

4. A solid mass is identif ed at ultrasound. What


should you do next?

5. Apart rom H&E stains o the core biopsy,


what other investigations would you order?

135
Invasive ductal carcinoma—palpable lump 588
Case ranking/dif culty: Category: Diagnostic

“Irregular mass” ultrasound correlate.

whether there are any pathological lymph nodes. These


are usually composed o irregular nodes or have ocal
cortical thickening o >3 mm. I they have had a recent
biopsy, a threshold o >4 mm will reduce alse-positive
calls. Ultrasound-guided core biopsy is indicated or a
suspicious mass seen on ultrasound.
5. ER, PR, and HER2 status are essential or the tailored
treatment o breast cancer, as treatment varies by
Prior mammogram—can you see the cancer in retrospect?
subtype. Ki-67 could be ordered, which is a proli eration
index. Other tests such as 20 or 70 gene tests are
requently per ormed on patients who have neoadjuvant
Answers chemotherapy.
1. This is not a screening examination, and so a def nitive
diagnosis can be made. The eatures include spiculation
and distortion, which should at least trigger a BI-RADS 4, Pearls
and more likely BI-RADS 5, as you would not accept
• Subtler signs o early breast cancer.
a nonmalignant biopsy as being concordant with these
• Developing ocal asymmetry.
appearances.
• Possible dense superimposition.
2. Invasive ductal carcinoma o no special type (NST) is
the most common carcinoma giving these appearances.
I the question was worded “what are the possible
Suggested Readings
pathologies giving these appearances,” then both tubular
carcinoma (which normally has long spicules) and Biganzoli L, Wildiers H, Oakman C, et al. Management
invasive lobular carcinoma (which presents mainly as o elderly patients with breast cancer: updated
distortion, and rarely as a mass) are possibilities. recommendations o the International Society o
Geriatric Oncology (SIOG) and European Society o
3. Physical exam and targeted ultrasound are the most Breast Cancer Specialists (EUSOMA). Lancet Oncol.
important next steps. Tomosynthesis could help i you 2012;13(4):e148-e160.
perceived distortion possibly due to superimposition. Zbar AP, Gravitz A, Audisio RA. Principles o
MRI is likely to be an important part o this patient’s surgical oncology in the elderly. Clin Geriatr Med.
management, but not just yet. 2012;28(1):51-71.
4. I a highly suspicious mass is seen on ultrasound, it
is worth scanning the ipsilateral axilla to determine
136
61-year-old patient comes with palpable abnormality in the right breast

1. What is the pertinent f nding?

2. What would be the next step o workup?

3. What is the signif cance o skin thickening?

4. I you have skin thickening but no other


suspicious lesion in the breast, what can be
the next step o workup?

5. What is the distribution o the calcif cations


in the right breast?

137
Invasive ductal carcinoma with lymphovascular invasion 377
and high-grade DCIS
Case ranking/dif culty: Category: Diagnostic

Ultrasound of right medial retroareolar breast demonstrates


“hypoechoic mass” with “irregular” shape and “posterior acoustic
shadowing.”
Diagnostic mammogram, right spot compression ML view,
demonstrating “pleomorphic” calci cations in “regional
distribution” (>2 cm). 4. Skin thickening can be related to mastitis and could
be treated with antibiotics. However, it is important to
ollow the patient within a short time period o a ew
weeks to make sure that the treatment did work. I skin
thickening persists and there are no other targets or
biopsy on mammogram or ultrasound, patient needs to
be trans erred to surgeon or punch biopsy.
5. Further assessment and description o calcif cations should
be based on the magnif cation views. However, here the
calcif cations have a “segmental” or “regional” distribution
and are not “scattered.” This makes them more suspicious.

Pearls
Ultrasound of right inferior retroareolar breast demonstrates mass
and also demonstrates the presence of the microcalci cations • I easible, ultrasound-guided biopsy is pre erred over
(arrow). stereotactic biopsy due to better patient com ort.
• I there is need to prove that the calcif cations
have been sampled—which is not a crucial issue
Answers here—specimen radiograph o the tissue sampled by
1. Noted are indeterminate calcif cations o the right breast ultrasound-guided biopsy can be obtained.
that, on additional magnif cation views, are consistent • Please note that in this case, there was marked thickening
with “pleomorphic” calcif cations in “regional (>2 cm o the skin in the in erior breast and periareolar breast as
area) distribution.” There is also thickening o the skin well as mild retraction o the right nipple.
and mild retraction o the nipple. • The histology did show intraductal invasive carcinoma
and multi ocal high-grade DCIS. The skin thickening
2. Next step is workup o the calcif cations with
did correlate to the presence o lymphovascular
magnif cation ML and CC view and ultrasound o the
invasion o the tumor.
retroareolar breast.
3. Skin thickening can be seen in case o invasive
lymphatic involvement o the skin, like in this case; it Suggested Reading
can re ect in ammatory component in case o mastitis Soo MS, Baker JA, Rosen EL, et al. Sonographically guided
or it can be due to prior radiation treatment; however, biopsy o suspicious microcalcif cations o the breast: a
this would likely be not as ocal as in this case. pilot study. AJR Am J Roentgenol. 2002;178(4):1007-1015.
138
Palpable lump in the right axilla

1. What BI-RADS classif cation should be used


here?

2. What is the most likely pathology based on the


imaging?

3. I there is no f nding on mammography except


lymphadenopathy, what is the imaging test that
is recommended?

4. What stage is the tumor when it has


metastasized to locoregional lymph nodes?

5. What are common sites or breast metastases?

139
Cancer presenting as large node in axilla 591
Case ranking/dif culty: Category: Diagnostic

Looking for internal mammary nodes due to large central breast Vascular axillary node. Power Doppler often helps to distinguish
mass. An enlarged node in the internal mammary chain a ects a normal hilum, or, in this instance, may help direct the biopsy
patient management when it comes to radiation treatment, as needle away from hitting one of these large vessels.
the eld will be extended 1 cm across the sternum.

Answers
1. By the time a cancer has metastasized to axillary nodes, Pearls
you should be able to give a BI-RADS 5 assessment • Unilateral lymphadenopathy in the absence o known
without hesitation. lymphoma should prompt a search or primary breast
cancer.
2. Fibroadenomas do not metastasize to axillary nodes.
• Mammography is requently normal.
Primary breast lymphoma tends to present as a
• MRI is the best imaging tool.
circumscribed mass in the breast. Hodgkin disease may
present with bilateral axillary lymphadenopathy. The
most common scenario with this type o imaging is a
regular invasive ductal carcinoma with locoregional Suggested Readings
spread into the ipsilateral axillary nodes. Ko EY, Han BK, Shin JH, Kang SS. Breast MRI or
3. Whole breast ultrasound is not yet widely available, evaluating patients with metastatic axillary lymph node
but may be help ul. MRI is the gold standard imaging and initially negative mammography and sonography.
procedure in this scenario, but is costly. PEM or BSGI Korean J Radiol. 2007;8(5):382-389.
may be o use, especially in dense breasts, and in Lanitis S, Behranwala KA, Al-Mu ti R, Hadjiminas D.
patients unable to tolerate MRI. The downsides are the Axillary metastatic disease as presentation o occult or
radiation dose. contralateral breast cancer. Breast. 2009;18(4):225-227.
Wang X, Zhao Y, Cao X. Clinical benef ts o mastectomy on
4. Based on the size o the tumor, it is either a stage 2 or 3. treatment o occult breast carcinoma presenting axillary
It has metastasized to a lymph node and is there ore N1. metastases. Breast J. 2010;16(1):32-37.
Until ormal staging is done, we do not know whether
there are any metastases.
5. Bony and pulmonary metastases are common sites or
metastases. Brain metastases may also occur requently.
Lobular cancer may metastasize to the peritoneum or a
segment o bowel (beware the short segment stricture).
Advanced disease can present with skin nodules.

140
Screening—asymptomatic

1. What is the BI-RADS category or this


screening exam?

2. What other examinations do you normally do


or patients with implants?

3. What is the most likely pathology?

4. What type and position o implant is present?

5. The mass is solid on ultrasound. What is your


recommendation?

141
Cancer in patients with implants 613
Case ranking/dif culty: Category: Screening

Left MLO spot compression. The question is:“Is this the index cancer Mass identi ed on ultrasound. Margins are “circumscribed”
or is this a metastatic lymph node, with unidenti ed primary?” in approximately 60% of its margin. The left lateral margins
(often di cult to assess because of edge artifacts) show some
“irregularity.” As a result, the most suspicious descriptor wins out
and prompts biopsy.

5. Parasitic in ections are a rare cause o mass in the breast.


You may see a calcif ed guinea worm on mammography,
but in the western world, it does not cause presumed
lymphadenopathy, or a mass in the breast. Ultrasound
FNA may be used purely to establish a malignancy, but
tissue is required or histology and or tissue biomarkers.
Core biopsy is pre erable. Surgical excisional biopsy has
been replaced by percutaneous needle biopsy. MRI will
be use ul once malignancy has been conf rmed.
Doppler ultrasound—the mass is avascular. No vessels approaching
the central part of the mass to indicate a hilum; therefore, more likely
a primary breast mass. Only histology will be able to di erentiate. Pearls
• In patients with implants, try to visualize as much
Answers glandular tissue as possible.
1. This is a screening exam with a positive f nding. Recall • Implant displaced views (EKLUND) have a major
or urther views, including ultrasound examination. role in maximizing your chances o picking up a
A physical examination by a trained pro essional malignancy in patients with implants.
could also be per ormed, especially i you have a • Masses that look like intramammary nodes should still
multidisciplinary diagnostic breast clinic available or be ully worked up, especially i the presumed node is
assessment o screen-detected abnormalities. enlarged.

2. The additional views mean that some groups recommend


known implant patients have diagnostic mammograms Suggested Readings
and get the implant displaced views routinely. MRI is Grubstein A, Cohen M, Steinmetz A, Cohen D. Siliconomas
the best test or implant complications. Ultrasound is mimicking cancer. Clin Imaging. 2011;35(3):228-231.
the cheapest and quickest test or implant integrity, and Nakaguro M, Suzuki Y, Ichihara S, Kobayashi TK, Ono
lumps associated with an implant. K. Epithelial inclusion cyst arising in an intramammary
3. I you look at the margins o the mass, it is lobulated, but lymph node: case report with cytologic f ndings. Diagn
some parts o the margins are not clearly seen. Even a Cytopathol. 2009;37(3):199-202.
cancer-f lled lymph node is a possibility in this site. Tang SS, Gui GP. A review o the oncologic and surgical
management o breast cancer in the augmented breast:
4. These implants are silicone and placed in a pocket diagnostic, surgical and surveillance challenges. Ann Surg
BEHIND the pectoral muscle. Oncol. 2011;18(8):2173-2181.
142
Diagnostic mammogram, prior lumpectomy

1. What BI-RADS classif cation should be used


here?

2. What should be the next diagnostic imaging


exam?

3. What type o biopsy would you consider with


this lesion?

4. What mammographic surveillance would you


recommend?

5. What signs are concerning or local recurrence


in a scar?

143
Lumpectomy scar-simulating malignancy 1744
Case ranking/dif culty: Category: Diagnostic

Right MLO of a similar case, but with more spiculation and


calci cations—which was fat necrosis but even more concerning
visually.
LM lateral view shows the relation of the soft density to the
postsurgical distortion from the scar.
mammograms so that you can go direct to spot
magnif cation plus or minus ultrasound i you detect any
Answers change. There is no evidence yet to warrant the addition
o MRI screening surveillance to mammography in
1. The postsurgical appearances can be described, and
patients with a personal history o breast cancer.
the f nding is there ore benign. I this was a screening
patient and you had no prior f lms and you may wish to 5. There are several signs on mammography o possible
do diagnostic views, you could give a BI-RADS 0 and scar recurrence. One is increasing density o the scar
then request diagnostic views. itsel , ollowing the maximal changes a ter radiation
treatment (18 months). Scars usually so ten over time.
2. I you have any doubt about the f ndings, you can
Developing microcalcif cations in the scar, which are not
per orm diagnostic views. Tomosynthesis is an
characteristically dystrophic ( at necrosis) are suspicious
alternative to diagnostic views, as it can distinguish
and o ten provoke biopsy. Distortion related to the scar can
surgical scar rom a mass. Ultrasound can be targeted
be a normal reaction postop, with some patients having at
to a palpable f nding, but scanning a scar is not
necrosis or exaggerated scarring causing these appearances.
recommended, as you virtually always have suspicious
The only proviso is i the patient has had treatment or a
f ndings, and may lock yoursel into per orming a
lobular cancer, the f nding is suspicious as lobular cancer
biopsy. The eatures are diagnostic o a postsurgical scar,
recurrence may present in this type o manner.
and so no urther workup is required. Stability when
compared with prior f lms is always use ul.
3. The f ndings are normal postlumpectomy change. Pearls
There ore, no biopsy is indicated. I the scar gets denser
over time, then that is usually a worrying sign, and • Scars can be scary.
ultrasound ollowed by core biopsy may be required. • Stability is good—look or prior f lms.
Cytology or scar recurrence is not recommended, as
tumor markers will be required or treatment. MRI may
sometimes help to distinguish a suspicious enhancing Suggested Readings
scar, rom a normal postsurgical scar.
Cox CE, Greenberg H, Fleisher D, et al. Natural history and
4. There are various approaches to the postoperative clinical evaluation o the lumpectomy scar. Am Surg.
breast. I it is the f rst 5 years postsurgery, then your 1993;59(1):55-59.
local lumpectomy protocol should be ollowed. A ter Muir TM, Tresham J, Fritschi L, Wylie E. Screening or
5 years, many groups advocate or annual screening breast cancer post reduction mammoplasty. Clin Radiol.
mammography. Others recommend diagnostic 2010;65(3):198-205.
144
Prior surgery

1. What BI-RADS should be used in this case?

2. What are the indications or a TRAM


reconstruction?

3. What are the “breast” complications o TRAM


aps?

4. What is the best way to image TRAM


reconstructions?

5. I a TRAM recurrence is suspected, what is the


best way to biopsy the lesion?

145
Transverse rectus abdominis myocutaneous 589
ap (TRAM) reconstruction
Case ranking/dif culty: Category: Diagnostic

Answers
1. This appearance is classic or a transverse rectus Pearls
abdominis myocutaneous ap (TRAM). Mammograms • TRAM reconstruction is common postmastectomy.
may be per ormed, and may have this typical appearance. • Recognize the normal TRAM mammogram.
They are a benign f nding, and there ore BI-RADS 2. • Fat necrosis with calcif cations is very common.
2. TRAM has been success ully used in patients having
mastectomy or a variety o reasons, including risk
reduction surgery or BRCA carriers. Poland syndrome Suggested Readings
a ecting the breast has been success ully treated with Glynn C, Litherland J. Imaging breast augmentation and
TRAM augmentation. Reconstruction. Br J Radiol. 2008;81(967):587-595.
Patients who are likely to need postoperative radiation Momoh AO, Colakoglu S, Westvik TS. Analysis o
have in the past had their reconstruction delayed until complications and patient satis action in pedicled
they f nished their radiation. However, this is no longer transverse rectus abdominis myocutaneous and deep
a contraindication. Patients do not need a mastectomy in erior epigastric per orator ap breast reconstruction.
or ADH. Ann Plast Surg. 2011;69(1):19-23.
Tan BK, Joethy J, Ong YS, Ho GH, Pribaz JJ. Pre erred
3. All orms o at necrosis are very common ollowing use o the ipsilateral pedicled TRAM ap or immediate
breast reconstruction, especially with TRAMs. Oil cysts breast reconstruction: an illustrated approach. Aesthetic
and dystrophic calcif cations are all part o the at necrosis Plast Surg. 2012;36(1):128-133.
spectrum. There is no increased risk o malignancy, except
or an increased risk o cancer due to having already had a
primary breast cancer. Recurrence can occur in a TRAM
ap, either in the lateral or in the medial margins.
4. MRI is the best tool in this scenario, with many papers
documenting the imaging f ndings and complications.
5. Palpation-guided biopsy is less accurate than with image
guidance, and ultrasound is usually the best method
to guide a needle. Recurrence only needs a malignant
diagnosis, and so many centers may eel that cytological
diagnosis is enough. A core biopsy may provide
additional in ormation about histological type, grade,
and receptor status.

146
Recent mastectomy for multifocal malignancy—new palpable nding

1. What is the BI-RADS category or this


diagnostic exam?

2. What should be the next diagnostic imaging


exam?

3. There is redness o the skin overlying this


mass. What should I do next?

4. Now there is no skin redness, but the mass is


visible under the skin. What urther management
recommendations do you want to make?

5. The mass has now settled. What imaging


recommendations do you want to make?

147
Mastectomy with reconstruction complication 677
Case ranking/dif culty: Category: Diagnostic

5. Strictly, there is no reason to give any recommendations


or annual mammograms, as the patient has bilateral
mammograms. MRI has not been proven or the ollow-
up or surveillance o patients with bilateral mastectomies
or o reconstructed breasts. However, in a young emale,
who already had multi ocal breast cancer, she is at high
risk o recurrence, and so some groups per orm annual
MRI in this setting. PEM and BSGI use isotopes or
imaging and do not have a screening role.

Pearls
• Complications o mastectomy and reconstruction are
A patient with an abscess for comparison. There is thickening of more common ollowing radiation treatment.
the skin as well as a track for the infection up to the skin itself. • In ection/in ammation through at necrosis (which is
more usually a delayed f nding).
• Epidermal inclusion cysts are another f nding.
Answers
1. I this was a de novo case with no evidence o
malignancy and no sign o in ection, then this could Suggested Readings
be a complicated cyst that could be observed with
short-term ollow-up. Bittar SM, Sisto J, Gill K. Single-stage breast
reconstruction with the anterior approach latissimus
2. There is no residual breast tissue, and so diagnostic dorsi ap and permanent implants. Plast Reconstr Surg.
mammograms are unlikely to give important additional 2012;129(5):1062-1070.
in ormation. Tomosynthesis likewise is unlikely to help. Sim YT, Litherland JC. The use o imaging in patients post
I you are considering local recurrence, then MRI would breast reconstruction. Clin Radiol. 2012;67(2):128-133.
be use ul or staging and surgical planning. Positron Tan BK, Joethy J, Ong YS, Ho GH, Pribaz JJ. Pre erred
emission mammography (PEM) is unlikely to give any use o the ipsilateral pedicled TRAM ap or immediate
use ul in ormation. No urther diagnostic imaging is breast reconstruction: an illustrated approach. Aesthetic
required at this stage. An intervention may be required. Plast Surg. 2012;36(1):128-133.
3. Depending on whether you think there is minor
in ammation present or a rank abscess, the intervention
may be di erent. Redness o the overlying skin can
be seen with masses that are not in ected. Fat necrosis
can do this, and even have evidence o local bruising.
Observation with the use o oral antibiotics and short-
term clinical ollow-up is a reasonable management
option. Percutaneous aspiration or drainage with
installation o a catheter can be an option i you think
there is an abscess, and the patient is symptomatic.
4. The eatures suggest some layered debris within this
cystic space. Observation with short-term clinical
examination and ultrasound ollow-up is a good
conservative plan. There is no evidence o an abscess, so
emergent drainage is not required. Diagnostic aspiration
may be attempted, but it is likely to show liquef ed at
and blood products. I the mass does not settle, then core
biopsy is reasonable. MRI is unlikely to add any urther
in ormation at this stage.

148
Prior breast cancer and radiation therapy

1. What BI-RADS classif cation is used or this


entity?

2. What structure does the f nding arise rom?

3. What is the requency o postradiation


sarcoma, at 10 years?

4. What is the mainstay o treatment or this


condition?

5. Which type o axial imaging is best to delineate


the eatures o postradiation sarcoma?

149
Chest wall sarcoma following cobalt therapy prior breast cancer 1573
Case ranking/dif culty: Category: Diagnostic

4. To have any chance o survival, the lesion needs to be


widely excised. On the chest wall, this requently includes
removing ribs, as in this case. Chemotherapy has been
used to varying e ect. Radiation and hormone treatment
have no place in the treatment o these sarcomas.
5. Plain radiographs are the best initial method o assessing
coexistent bone involvement in patients with so t tissue
sarcomas. MRI is the next step in imaging these lesions
because o its superior so t tissue contrast, multiplanar
imaging capability, and the absence o streak arti act.
MRI is superior to CT in delineating tumor relationships
to muscle, at, f brous tissue, and adjacent blood
vessels. CT is superior to MRI only in the identif cation
and evaluation o matrix/rim calcif cation and in the
evaluation or pulmonary metastases.

Non–contrast enhanced CT shows ill-de ned soft tissue mass,


which appears closely related to the chest wall. No destruction
of ribs on bone window settings. No postradiation lung changes Pearls
identi ed. • Rare complication o radiation therapy.
• Mainly historic, but still occurs, and examiners o ten
have one o these cases in their back pockets.
Answers • When develops, prognosis is poor, but the mainstay
1. BI-RADS 0 is not appropriate in this setting, as it o treatment is wide surgical excision.
is not a screening exam. You could, however, give a
BI-RADS 4 on this f nding, as radiologically it is does
not have characteristic descriptors o malignancy. In Suggested Readings
this particular setting, anything other than a malignancy
is extremely unlikely, so BI-RADS 5 is the most Lagrange JL, Ramaioli A, Chateau MC, et al. Sarcoma a ter
appropriate assessment. radiation therapy: retrospective multiinstitutional study
o 80 histologically conf rmed cases. Radiation Therapist
2. Secondary carcinomas usually arise rom bone, connective and Pathologist Groups o the Fédération Nationale
tissue, or muscle as malignant f brous histiocytomas or des Centres de Lutte Contre le Cancer. Radiology.
osteo/angio/f bro sarcomas. They are not related to breast 2000;216(1):197-205.
cancer, and are not epithelial-based tumors. Pencavel T, Allan CP, Thomas JM, Hayes AJ. Treatment or
3. OK it is rare, but not THAT RARE. Published data breast sarcoma: a large, single-centre series. Eur J Surg
suggest a risk o postradiation sarcoma at 0.2%, but that Oncol. 2011;37(8):703-708.
is based on patients having radiation many years ago, Vojtísek R, Kinkor Z, Fínek J. Secondary angiosarcomas
and may not re ect current radiation treatment. a ter conservation treatment or breast cancers [in Czech].
Klin Onkol. 2011;24(5):382-388.

150
Prior mastectomy for left breast cancer and prior lumpectomy
for right breast cancer

1. What BI-RADS classif cation should be used


here?

2. What is the most likely pathology based on the


imaging?

3. What is the requency o at necrosis ollowing


breast conservation surgery?

4. What type o biopsy should be per ormed?

5. What are the mammographic f ndings are signs


o scar recurrence?

151
Fat necrosis in scar—why you do not want to ultrasound scars 1641
Case ranking/dif culty: Category: Diagnostic

Right CC spot magni cation view. Right ML spot magni cation view.

4. The appearances are diagnostic, and biopsy is not


required. I suspicious calcif cations arise in the scar, and
you cannot be certain o the location with ultrasound,
a stereotactic core biopsy plus specimen x-ray may be
required to establish the diagnosis.
5. Fine pleomorphic calcif cations are always suspicious
and require biopsy. They can, however, be ound in a
variation o at necrosis, be ore the traditional coarse
dystrophic calcif cation arises. A scar should so ten over
time, particularly a ter a (baseline) exam 2 years post–
radiation therapy. Any increase in tissue density should
raise concerns about scar recurrence.
Ultrasound of scar—irregular mass with heterogeneous echo
pattern and acoustic shadowing. There is no easy way around
giving a BI-RADS 4 in this situation, which is why it is best to avoid
scanning scars, unless you think there is a suspicious new nding Pearls
on mammography. • A scar with a lucent center is characteristic o at
necrosis.
Answers • Calcif cations o at necrosis occur in the wall o the
1. This is a benign f nding. Ignore the ultrasound image in ammatory change.
on the supplementary material, as ultrasound o scar is
almost always “suspicious” in looks, and you can box
yoursel into doing an unnecessary biopsy. Despite the Suggested Readings
spiculation, the history o a lumpectomy at this site gives
Dershaw DD. Evaluation o the breast undergoing
away the characteristic appearance o at necrosis at the
lumpectomy and radiation therapy. Radiol Clin North Am.
lumpectomy site.
1995;33(6):1147-1160.
2. The lucent at-containing “cyst” at the center o the scar Rostom AY, el-Sayed ME. Fat necrosis o the breast: an
combined with dystrophic calcif cations is characteristic unusual complication o lumpectomy and radiotherapy in
o at necrosis. breast cancer. Clin Radiol. 1987;38(1):31.
Wasser K, Schoeber C, Kraus-Tie enbacher U, et al.
3. Fat necrosis is a common complication o breast
Early mammographic and sonographic f ndings a ter
conservation, seen more requently in patients who
intraoperative radiotherapy (IORT) as a boost in patients
have also had intraoperative radiation treatment (IORT).
with breast cancer. Eur Radiol. 2007;17(7):1865-1874.
Radiotherapy terminology estimated 28% incidence.
152
Prior breast cancer

1. What BI-RADS classif cation should be used


here?

2. What is the most likely cause or these f ndings


on the le t?

3. What is the percentage risk o associated


malignancy with radial scars?

4. What radiographic actors may cause problems


in postconservation breasts?

5. Which o the ollowing are benign causes o


calcif cations ollowing breast-conserving
surgery?

153
Bilateral lumpectomy scars—left subtle change 1643
Case ranking/dif culty: Category: Diagnostic

breast is requently harder than normal or a number


o years, and there ore less easily compressible. The
biggest challenge technically is the positioning o the
patient, to include as much breast tissue as possible and
include the tumor bed. Additional view may need to
be taken, such as XCCL. Fluid collections are normal
postlumpectomy change, gradually being replaced by
scar ormation over a number o years. It assists with
cosmesis, providing a corollary to the cavity rom the
surgery. Keloid may show up on a mammogram, but
is not generally a technical actor in getting a good
postconservation mammogram.
5. Dermal “calcif cations” are requently seen in the skin
over the scar, and o ten are as a result o “trapping” o
talcum powder or aluminum oxide in deodorant. An
irregular scar may also cause the buildup o detritus that
may be di f cult or patients to clean o the skin. Fat
necrosis calcif cations are very common (approximately
25%) and also oil cysts. Suture calcif cations may occur,
and have characteristic appearances. Fine linear and
branching calcif cations are much more likely to be a
high-grade DCIS recurrence and are a suspicious f nding.

Pearls
• Post–breast conservation scars at f rst glance can
look scary, especially i dense or are associated with
calcif cations.
Left XCCL. • Take time to evaluate or stability (prior f lms are a must).
• Know what a normal scar looks like and the variants o
Answers calcif cations that might occur.

1. BI-RADS 2—benign scar in both breasts. Look or prior


f lms or stability. I NEW distortion, then per orm a
diagnostic exam and biopsy as necessary. Suggested Readings
2. The appearances are much more o postsurgical scar, Chansakul T, Lai KC, Slanetz PJ. The postconservation
with dystrophic calcif cations giving it away. breast: part 1, Expected imaging f ndings. AJR Am J
Roentgenol. 2012;198(2):321-330.
3. Most authorities quote a risk o associated DCIS o 20%. Chansakul T, Lai KC, Slanetz PJ. The postconservation
The issue o adequately sampling a radial scar to exclude breast: part 2, Imaging f ndings o tumor recurrence
associated DCIS is still controversial, as there are data and other long-term sequelae. AJR Am J Roentgenol.
or routine surveillance i enough biopsies are taken 2012;198(2):331-343.
to surgical excision to completely exclude DCIS. This Dershaw DD, Shank B, Reisinger S. Mammographic f ndings
needs to be the subject o a clinical trial to in orm uture a ter breast cancer treatment with local excision and
management. def nitive irradiation. Radiology. 1987;164(2):455-461.
4. There is not usually any di erence in pain perception
rom a mammogram per ormed posttreatment, and a
non–cancer patient. Following radiation therapy, the

154
Routine lumpectomy surveillance—lump in the left breast

1. What is the BI-RADS category or this


diagnostic exam?

2. What is the next best examination you


recommend?

3. What is the most likely pathology?

4. What is the timescale you expect the maximal


changes post–radiation therapy or breast cancer?

5. What is the most worrying eature o a surgical


scar that would prompt you to biopsy?

155
Scary scar distractor 623
Case ranking/dif culty: Category: Diagnostic

Ultrasound examination of palpable lump. Oval cystic mass Compare with this exam: Ultrasound examination of palpable
identi ed, parallel to the skin. Some hyperechoic tissue around lump showing an oval intradermal mass with circumscribed
the wall, within the subdermal fat. margins and anechoic in nature, consistent with a subdermal cyst.
The lower layer of the skin is stretched to include the mass.

Answers
1. (Stable) scar, postlumpectomy. No evidence o Pearls
malignancy in either breast. There is a BB marker
• Postoperative scars can look suspicious.
adjacent to the le t nipple but no mass identif ed.
• Need prior f lms or stability.
2. Further examination or a palpable lump using spot • Watch or developing microcalcif cations.
magnif cation views can be very help ul. In this case, it
did not add anything, and we went directly to ultrasound.
3. A normal mammogram with a cystic lesion on Suggested Readings
ultrasound is a benign f nding. The operative scar should Buckley JH, Roebuck EJ. Mammographic changes ollowing
not be mistaken or a cancer. radiotherapy. Br J Radiol. 1986;59(700):337-344.
4. See BJR paper Buckley and Roebuck on time o Ojeda-Fournier H, Olson LK, Rochelle M, Hodgens BD,
maximal change. The skin thickening and parenchymal Tong E, Yashar CM. Accelerated partial breast irradiation
tissue edema should start to settle within 2 years and posttreatment imaging evaluation. Radiographics.
ollowing completion o radiation change. Any increase 2011;31(6):1701-1716.
in edema ollowing this should be regarded with Preda L, Villa G, Rizzo S, et al. Magnetic resonance
suspicion. mammography in the evaluation o recurrence at the
prior lumpectomy site a ter conservative surgery and
5. Using f rst principles, any change that is not strictly radiotherapy. Breast Cancer Res. 2006;8(5):R53.
benign (like obvious dystrophic calcif cations) should Wong S, Kaur A, Back M, Lee KM, Baggarley S, Lu JJ. An
prompt a biopsy or local recurrence. ultrasonographic evaluation o skin thickness in breast
cancer patients a ter postmastectomy radiation therapy.
Radiat Oncol. 2011;6(6):9.

156
Recent surgery for invasive ductal carcinoma—palpable nding at scar

1. What BI-RADS classif cation should be used


here?

2. In the setting o a postoperative patient, what


should we be concerned about?

3. What is the next best imaging test?

4. What type o biopsy would you recommend?

5. What ollow-up would you recommend?

157
Cavitating fat necrosis postlumpectomy 585
Case ranking/dif culty: Category: Diagnostic

Fat/ uid level at lumpectomy site. Lucency associated with surgical clips.

Answers 5. Several answers are potentially correct, depending on


1. This is a benign f nding in the presence o the the individual circumstances. Some groups advocate
mammographic f ndings, which will be made available short-term physical examination and repeat ultrasound in
on this page. The movable “debris” within the cystic atypical f ndings to ensure stability.
cavity in a postoperative patient helps to make the
diagnosis. Some people give a BI-RADS 0 until the
mammographic workup is completed (which would Pearls
be the normal way or this lesion to present). There is
• Fat necrosis common in later complication o breast
no f nding, such as vascularity shown, which would
conservation therapy.
make this f nding suspicious o a possible intracystic
• Contains at.
papilloma.
• Hematoma can occur within the lipid cavity producing
2. The most common problems to occur postop in women a at/ uid level.
with residual seromas are bleeding and hematoma • No therapeutic or long-term sequelae rom this
ormation, and in ection. Fungus balls occur with complication unless the patient has a clotting disorder.
aspergillus in ection in lung cavities (aspergilloma).
Patients are immunocompromised a ter having
chemotherapy, but rarely get in ection o the seroma.
Suggested Readings
3. Mammograms should have been per ormed be ore the Drukteinis JS, Gombos EC, Raza S, Chikarmane SA, Swami
ultrasound, so i they have not been done yet, now is A, Birdwell RL. MR imaging assessment o the breast a ter
the time. MRI in the postoperative breast is sometimes breast conservation therapy: distinguishing benign rom
help ul in distinguishing clinical relevant problems malignant lesions. Radiographics. 2012;32(1):219-234.
rom normal postoperative change, but there are many Mendelson EB. Evaluation o the postoperative breast.
potential pit alls. Tomosynthesis may be an answer or Radiol Clin North Am. 1992;30(1):107-138.
many o these f ndings, as it establishes its place in the Solomon B, Orel S, Reynolds C, Schnall M. Delayed
workup o breast abnormalities. development o enhancement in at necrosis a ter breast
4. This is a “normal” f nding that can be ollowed, as no conservation therapy: a potential pit all o MR imaging o
intervention is required. the breast. AJR Am J Roentgenol. 1998;170(4):966-968.

158
Postsurgery for sentinel node biopsy of the axilla

1. What BI-RADS classif cation should be used


here?

2. What group o bacteria is likely to be ound in


an abscess?

3. A Doppler signal within an abscess means


which o the ollowing?

4. What are the possible management options or


an abscess?

5. What are the ultrasound eatures o an abscess?

159
Postsurgical abscess 592
Case ranking/dif culty: Category: Diagnostic

Another view shows the irregular mass and the extension Abscesses are in amed and therefore vascular. There may be
super cially toward the skin. signal from movement of uid within the abscess, so the gain
needs to be turned down. It may be useful to identify vascular
bands through the abscess, indicating that it is loculated, or is
Answers a phlegmon and might therefore need surgical intervention.
1. This is a special case situation, where the f nding is o
a benign lesion, but the BI-RADS descriptors are all
suspicious. A summary phrase such as “a uid collection
with skin thickening and redness consistent with an
Pearls
abscess” would work well. BI-RADS 0 is not indicated • In the clinical setting o in ection, think abscess until
as this is a diagnostic workup. proven otherwise.
2. There is a low oxygen level within an abscess, and
there ore anaerobic bacteria tend to colonize a surgical
cavity and cause an abscess. Suggested Readings

3. Increased Doppler signal throughout an abscess may Boisserie-Lacroix M, Debled M, Tunon de Lara C, Hurtevent
occur when the abscess is loculated, but also at the stage G, Asad-Syed M, Ferron S. The in ammatory breast:
where a phlegmon is present. A phlegmon is a conf ned management, decision-making algorithms, therapeutic
ocus o in ammatory tissue without lique action. principles. Diagn Interv Imaging. 2012;93(2):126-136.
Leibman AJ, Misra M, Castaldi M. Breast abscess a ter
4. The management o an abscess depends on many actors, nipple piercing: sonographic f ndings with clinical
and it is important to work closely with the surgical correlation. J Ultrasound Med. 2011;30(9):1303-1308.
team to ensure a correct treatment option or that patient. Trop I, Dugas A, David J, et al. Breast abscesses: evidence-
Treatment can be anything rom watch ul waiting during based algorithms or diagnosis, management, and ollow-
antibiotic treatment to diagnostic aspiration, drain up. Radiographics. 2011;31(6):1683-1699.
placement, or surgery i there is a phlegmon that needs
evacuating.
5. Abscesses may have myriad appearances, but, in general,
they appear o mixed echogenicity, and you can observe
movement o uid within the cavity. There may be
enhanced Doppler signals around or even within the
abscess.

160
Bloody nipple discharge —no nipple changes

1. What BI-RADS classif cation should be used


here?

2. What is the most likely pathology based on the


imaging?

3. What is the next best imaging test?

4. Any other tests that you may consider?

5. What are common types o nipple discharge?

161
Intracystic mass—papilloma 580
Case ranking/dif culty: Category: Diagnostic

4. A ductogram may help i there were no ultrasound


f nding, or you suspect multiple lesions early on. Some
centers per orm ductoscopy routinely. Ductal lavage can
assist in deciding how hard you may wish to investigate
a bloody nipple discharge.
5. I lactating or pregnant, milky discharge is normal
physiology. Bloody nipple discharge is commonly due
to duct ectasia or plasma cell mastitis, rather than cancer.
Pro use watery nipple discharge is a potential symptom
o DCIS. Brown, through yellow to light green and blue,
is normal physiological nipple discharges.

Ultrasound of another case of intraductal mass. It is possible to


demonstrate the dilated duct extending from the nipple and the
Pearls
cystic space containing the solid mass.
• When f nding an intraductal mass, identi y whether the
echoes move (by toe/heel with the probe), suggesting
Answers proteinaceous plug or debris.
1. This f nding is suspicious, requiring biopsy, so a BI- • Doppler ultrasound may help as papillomas have
RADS 4 is an appropriate classif cation. Until biopsy prominent vascular channels.
is per ormed, we cannot tell whether this is a simple • Vessels entering perpendicular to the wall o the duct
papilloma, papillary lesion, or intracystic papillary are highly suggestive o a papilloma.
carcinoma. In this case, the lesion appears to be
possibly growing through the cyst wall and so a core
biopsy showing papilloma was nonconcordant. Surgical Suggested Readings
excision was per ormed, conf rming a simple papilloma.
Al Sarakbi W, Salhab M, Mokbel K. Does mammary
2. Malignant cancers within the milk duct/cystic cavity ductoscopy have a role in clinical practice? Int Semin
are either invasive ductal or DCIS. Benign intraductal Surg Oncol. 2006;3(3):16.
tumors are papillomas. A phyllodes tumor contains Brookes MJ, Bourke AG. Radiological appearances o
distorted ducts. Mucinous carcinoma can simulate papillary breast lesions. Clin Radiol. 2008;63(11):
a complicated cyst, with heterogeneous or low level 1265-1273.
echoes within the structure. Rizzo M, Linebarger J, Lowe MC, et al. Management
3. I older than 40 years, mammographic workup should o papillary breast lesions diagnosed on core-needle
really have been done f rst. I not, it is time to do it and biopsy: clinical pathologic and radiologic analysis o
decide what comes next. MRI can be per ormed but 276 cases with surgical ollow-up. J Am Coll Surg.
usually ollowing a biopsy showing papillary DCIS (due 2012;214(3):280-287.
to likely multi ocality). These multi ocal lesions can
o ten be seen when you search or them with ultrasound.

162
Bloody nipple discharge

1. What are causes o bloody nipple discharge?

2. What are benign causes or bloody nipple


discharge?

3. What volume o contrast is normally used in a


ductogram?

4. What are contraindications to ductography?

5. What are the symptoms o extravasation o


ductography contrast?

163
Ductogram— lling defect 587
Case ranking/dif culty: Category: Diagnostic

intravenous puncture). I the patient says that she will


not have surgery i something is ound, there is no point
in doing the procedure. A prior Hadf elds procedure,
where there has been a total duct excision, means that
the ductogram is not technically possible, and a bloody
nipple discharge should mean that one a ected duct was
not removed.
5. Extravasation is common in the f rst ew ductograms,
caused by injecting too much contrast. Patients do not
get ushing, but may experience mild pain around
the nipple. Most patients have no symptoms rom
extravasation.

Ultrasound can be useful, but may just show a dilated duct, with
possible proteinaceous debris. Pearls
• Bloody nipple discharge is a rare cause o breast
cancer.
Answers
• Periductal mastitis is the most common cause, and can
1. Pagets disease is usually diagnosed on physical be identif ed by periductal tiny lucencies rom micro
examination. Excoriation o the central milk ducts abscesses.
may cause bloody discharge. DCIS usually produces a • A blocked duct is not pathognomonic o a papilloma,
pro use watery discharge. IDC may also cause a bloody as proteinaceous plugs can cause the same e ect.
nipple discharge (5–8% according to the literature).
Eighty-f ve percent o bloody discharge is due to benign
disease such as a papilloma.
Suggested Readings
2. Periductal mastitis and duct ectasia are the most common
Adepoju LJ, Chun J, El-Tamer M, Ditko BA, Schnabel F,
causes o bloody nipple discharge. Papillomatosis
Joseph KA. The value o clinical characteristics and breast-
usually presents with watery nipple discharge like DCIS.
imaging studies in predicting a histopathologic diagnosis
Papilloma may present with bloody discharge.
o cancer or high-risk lesion in patients with spontaneous
3. Approximately 0.2 to 0.3 mL is all that is required to nipple discharge. Am J Surg. 2005;190(4):644-646.
inject into a normal-caliber duct system. Occasionally, Dooley WC. Breast ductoscopy and the evolution o
you may have to inject more. Use o a 1-mL syringe aids the intra-ductal approach to breast cancer. Breast J.
the injection o contrast, attached to the lacrimal catheter 2009;15(Suppl 1):S90-S94.
with connecting tubing. Rissanen T, Reinikainen H, Apaja-Sarkkinen M. Breast
sonography in localizing the cause o nipple discharge:
4. Severe nipple retraction may physically prevent you
comparison with galactography in 52 patients.
rom placing the cannula. Severe allergies to iodinated
J Ultrasound Med. 2007;26(8):1031-1039.
contrast are a relative contraindication (need to avoid

164
Male patient with swelling left nipple area since several months

1. What is gynecomastia?

2. What is the etiology o gynecomastia?

3. What is the diagnostic algorithm or evaluation


o palpable mass in males?

4. What is the role o ultrasound in the workup


o palpable abnormality in males?

5. What is the epidemiology o male breast


cancer?

165
Gynecomastia 119
Case ranking/dif culty: Category: Diagnostic

Ultrasound demonstrates lobulated hypoechoic nodular area Ultrasound demonstrating lobulated hypoechoic nodular area
corresponding to “lobulated” gynecomastia. with increased ow on duplex.

Answers 5. Incidence is 1:100,000 and mean age o diagnosis is


1. Gynecomastia is the most common benign f nding in 67 years. Less than 6% occur in males younger than
males with palpable breast abnormalities. It is best 40 years. Since gynecomastia is o ten seen in older
diagnosed on mammography—sometimes ultrasound patients, it can occur together with malignancy, although
can be help ul in addition. It might be symmetric but is gynecomastia is not to be considered a precursor to
o ten times asymmetric in appearance in both breasts. It breast cancer. Family history indeed is a signif cant risk
has a peak in incidence in puberty and in higher age. actor as detailed above.

2. Any medication or medical condition that a ects the


balance between estrogen and testosterone in the body
can result in proli eration o ducts and f brotic tissue Pearls
(gynecomastia) in the male breast—no connection to • Bilateral standard mammography is the most important
amily history o breast cancer. f rst test in the workup o palpable abnormalities in
males—not ultrasound.
3. Mammography should always be the f rst choice, since
• Magnif cation views and or spot compression views
it is diagnostic in most cases. Also calcif cations can be
best seen on mammogram. Gynecomastia can appear may be use ul in addition.
relatively concerning on ultrasound, since it might even • Ultrasound can be help ul in case o equivocal
mammogram f ndings.
show “posterior acoustic shadowing.” In equivocal cases,
• It is important not to miss lymphadenopathy, as
ultrasound can be help ul to increase the specif city
o mammography by searching or secondary signs secondary f nding suggesting malignancy.
• Most common reason or breast lump in male is
or malignancy and lymphadenopathy. I suspicious
gynecomastia.
abnormality is seen, ultrasound-guided biopsy is
recommended.
4. See answer to question 3. Again, mammography is the
Suggested Readings
f rst test o choice. Ultrasound can be help ul, but o ten
times unnecessary. Chen L, Chantra PK, Larsen LH, et al. Imaging
characteristics o malignant lesions o the male breast.
Increased ow on duplex or posterior echogenicity is Radiographics. 2006;26(4):993-1006.
not specif c. More important are detection o secondary Mathew J, Perkins GH, Stephens T, Middleton LP, Yang
f ndings such as skin thickening, nipple retraction, or WT. Primary breast cancer in men: clinical, imaging, and
lymphadenopathy to increase specif city as seen on pathologic f ndings in 57 patients. AJR Am J Roentgenol.
ultrasound. 2008;191(6):1631-1639.

166
Prior prostate cancer—“lump” in the breast

1. What is the most likely cause or these


appearances?

2. Which drugs, or classes o drugs, cause these


appearances?

3. What BI-RADS score is appropriate in this


case?

4. What conditions are a cause o gynecomastia?

5. What treatment should be o ered or this


patient?

167
Gynecomastia in patient with prostate cancer 608
Case ranking/dif culty: Category: Diagnostic

Right CC—BB marker on nipple. Left CC—BB marker on nipple. Note the large amount of pectoral
muscle on the lm, compared with a female. This male patient
had little fat in the breast.
Answers
1. Usually medication related. In this case the estrogens
e ects and may not be tolerated. Obviously, i possible,
used to treat prostate cancer.
withdrawal o the o ending drug would help, but the
2. The list o drugs known to cause gynecomastia is long e ects are not always reversible. Surgical excision is
and ever changing, because o the number o newer not recommended as primary treatment, especially i the
monoclonal antibody therapies or chronic conditions, underlying cause has not been removed.
which have similar side e ects.
3. The f nding is benign, with minor asymmetry only; Pearls
there ore, BI-RADS 2. I there is truly a mass, it depends
on your f ndings as to whether it becomes a BI-RADS 4. • Common complication o the use o a number o drugs.
Remember that unless you get textbook pictures, • Need to remove the underlying cause, or example,
an ultrasound o gynecomastia can look extremely change to a di erent drug within the same class.
worrying, and prompt biopsy. • May not be reversible.

4. Kline elter syndrome is a reported cause o


gynecomastia. Patients with chronic renal Suggested Readings
ailure (end stage) o ten have reduced levels o
Fradet Y, Egerdie B, Andersen M, et al. Tamoxi en as
testosterone and may have primary testicular ailure.
prophylaxis or prevention o gynaecomastia and breast
Amyloidosis and systemic lupus erythematosus do
pain associated with bicalutamide 150 mg monotherapy
not cause gynecomastia. Hyperthyroidism may cause
in patients with prostate cancer: a randomised, placebo-
gynecomastia due to increased aromatase activity
controlled, dose-response study. Eur Urol. 2007;52(1):
and increased levels o sex hormone binding globulin
106-114.
(SHBG). (SHBG binds androgens more avidly than
Grun eld EA, Halliday A, Martin P, Drudge-Coates L.
estrogen, allowing or higher ree levels to act on
Andropause syndrome in men treated or metastatic
peripheral tissues such as the breast).
prostate cancer: a qualitative study o the impact o
5. I gynecomastia is very mild and tolerable, then symptoms. Cancer Nurs. 2011;35(1):63-69.
reassurance and observation may be appropriate. Thompson IM, Tangen CM, Goodman PJ, Lucia MS,
Tamoxi en is requently given to males with gynecomastia Klein EA. Chemoprevention o prostate cancer. J Urol.
to counter the estrogenic e ect. It has its own list o side 2009;182(2):499-507; discussion 508.
168
Male patient with lump in the left breast

1. Why was the right as well as the le t breast


scanned?

2. What are the f ndings in regard to the


symptomatic le t side?

3. Is there a need or ultrasound?

4. What is the f nal diagnosis?

5. What would be the next step?

169
Spironolactone -induced bilateral symmetric gynecomastia 731
Case ranking/dif culty: Category: Diagnostic

Spot compression left MLO view.

Spot compression left CC view.

Ultrasound directed to the area of concern; left chest wall


demonstrates normal broglandular tissue.

Answers
1. Patient is symptomatic on the le t side. Nevertheless, Pearls
patient received mammogram o both sides as a baseline • Example o extensive bilateral symmetric gynecomastia
to see i there is any ocal asymmetry and also to assess due to medication—in this case, spironolactone.
the extent o the gynecomastia bilaterally.
2. Le t side demonstrates normal f broglandular tissue. No
ocal suspicious abnormality is identif ed. Suggested Readings
3. Ultrasound is recommended or every palpable Cuculi F, Suter A, Erne P. Spironolactone-induced
abnormality, in a male or emale. In a male, gynecomastia. CMAJ. 2007;176(5):620.
mammogram, however, should be the f rst step. Haynes BA, Mookadam F. Male gynecomastia.
Mayo Clin Proc. 2009;84(8):672.
4. The diagnosis is bilateral symmetric gynecomastia.
5. Patient with bilateral gynecomastia needs to be worked up
clinically. There is no need or urther imaging ollow-up.
170
Male patients with bilateral ndings on chest wall

Right MLO. Left MLO. Right CC. Left CC.

1. What is the f rst-line radiological examination


in a patient with this history?

2. What will the ultrasound examination show?

3. What is the most likely pathology?

4. What is the cause o a vascular mass on the


chest wall?

5. What should be the next radiological


investigation?

171
Male: metastases to breast 618
Case ranking/dif culty: Category: Diagnostic

Ultrasound appears to be a complicated Ultrasound—this view shows the mass to Power Doppler ultrasound shows a highly
cystic structure. be “complex.” vascular solid lesion.

Answers
1. A quick noninvasive, nonionizing examination such Pearls
as ultrasound is easy to per orm and should give more • Metastases to the breast are most requently ound as
in ormation to in orm you what should be the next step. round or oval masses with circumscribed margins on
In this case, the lumps were over the breast regions, so mammography.
a mammogram would be a reasonable examination as • On ultrasound, they appear as hypoechoic masses with
well. I the patient already has a known malignancy, microlobulated or circumscribed margins and posterior
a PET/CT may be o assistance in staging the disease acoustic enhancement.
(although not in the choice o question 5 above). A chest • On MRI, mostly present as circumscribed masses with
x-ray has little place in the immediate workup o this either marked or moderate homogenous enhancement.
patient. CT and MRI may be overkill.
2. They are predominantly “circumscribed masses,” mainly
“oval” in shape with “two or three gentle lobulations” Suggested Readings
(BI-RADS and Stavros).
Bartella L, Kaye J, Perry NM, et al. Metastases to the breast
3. Is there any evidence o multiple masses over the rest revisited: radiological-histopathological correlation. Clin
o the skin to suggest neurof bromatosis? Lipomas Radiol. 2003;58(7):524-531.
are typically HYPERECHOIC. Multiple hamartoma Surov A, Fiedler E, Holzhausen HJ, Ruschke K, Schmoll
syndromes can look very similar to this. There are HJ, Spielmann RP. Metastases to the breast rom non-
multiple di erent masses; there ore, unlikely to be mammary malignancies: primary tumors, prevalence,
multiple separate carcinomas. This patient had known clinical signs, and radiological eatures. Acad Radiol.
multiple myeloma, and these were proven myeloma 2011;18(5):565-574.
metastases (note the vascularity on Doppler). Yeh CN, Lin CH, Chen MF. Clinical and ultrasonographic
characteristics o breast metastases rom extramammary
4. Vascular mal ormations do not tend to present as breast
malignancies. Am Surg. 2004;70(4):287-290.
masses. However, ollowing a seat belt injury, it is
possible to get at necrosis mass ormation that can be
vascular (but not typically so). Hemangiomas o the
breast tend to be smaller and circumscribed with f brous
septae. Angiosarcomas usually present like lobular
cancer, in that they are inf ltrative and permeative, rather
than causing circumscribed masses. Metastases are o ten
vascular, especially so with multiple myeloma.
5. Even with a known history o multiple myeloma,
malignancy needs to be proven, and core biopsy is the
next best investigation.

172
Routine follow-up post-breast reconstruction

1. What BI-RADS classif cation should be used


here?

2. What are the possible diagnostic imaging


exams to work up this f nding?

3. What type o biopsy would you recommend?

4. When you see microcalcif cation developing in


a surgical scar, what is your di erential?

5. What are the complications o breast reduction


or reconstruction you should be looking or on
surveillance?

173
Transverse rectus abdominis myocutaneous ap 1865
(TRAM) reconstruction complications
Case ranking/dif culty: Category: Diagnostic

Close-up fat necrosis in TRAM reconstruction. Note that the Close-up local recurrence in TRAM reconstruction.
calci cations make up a peripheral component to the mass.

Answers posttreatment distortion, they may be projected over the


1. BI-RADS 2 should be used, as this is a characteristically surgical scar.
benign f nding. BI-RADS 0 is not appropriate. I the 5. All o the answers can be true and can present as
patient was having a “tramogram,” then it would be a f ndings. However, in general practice, the common
diagnostic exam in the f rst place. Early at necrosis can things you encounter are developing areas o at
sometimes look suspicious, and there ore i this was one necrosis that may be visible on both mammography and
o those cases, a BI-RADS 4 may potentially be more ultrasound. I they have had a breast reduction, they
appropriate. tend to have a characteristic pattern o parenchymal
2. All o the answers can be correct. I you have prior change with swirly lines, sometimes best seen on the
f lms and they are stable, no action need be taken. CC and sometimes in the lower hal on the MLO.
First visit, you may wish to work up with either spot Decreased breast volume related to lobular cancer is or
(magnif cation) views or ultrasound. Ultrasound itsel global change over a time period, rather than just to the
can appear suspicious when at necrosis calcif es; reduction surgery.
there ore, it may not be a good exam in this instance.
For reconstructed breasts, MRI is a good tool or
surveillance. Tomosynthesis can be used in place o Pearls
diagnostic special views.
• Benign condition relatively commonly seen in breast
3. This is a characteristic benign f nding; there ore, no ap reconstructions.
action other than ollow-up is recommended. • Characteristic appearance, and in typical locations,
4. Depends on what the lumpectomy was per ormed or. help make the diagnosis without the recourse to biopsy.
I there was ductal carcinoma in situ (DCIS) present
in the surgical excision, then DCIS recurrence is
always at the top o the list. “Dead DCIS” is said to Suggested Reading
be a recognized phenomenon in patients with lower- Eidelman Y, Liebling RW, Buchbinder S, Strauch B,
grade DCIS, who have undergone radiation therapy. Goldstein RD. Mammography in the evaluation o masses
Dystrophic calcif cations commonly occur related to in breasts reconstructed with TRAM aps. Ann Plast
the development o at necrosis. Dermal calcif cations Surg. 1998;41(3):229-233.
are usually seen only in the skin, but i there is marked

174
History of prior benign excisional biopsy for biopsy-proven ADH

1. What are the pertinent f ndings on


mammography?

2. What could be the etiology?

3. Which malignancy typically presents with


distortion like this?

4. Are there any additional f ndings on the


magnif cation view?

5. What is the consequence?

175
Distortion from prior excisional biopsy as a result 1308
of prior biopsy of ADH and now recurrent calci cations
Case ranking/dif culty: Category: Diagnostic

4. There are at least our groups o “amorphous”


calcif cations in proximity to the area o architectural
distortion.
5. Given that all groups are similar in morphology, it would
be reasonable to biopsy one group only as done in this
case. Depending on the result, in case o positive or
malignancy or atypia, all groups should be included
into the excision. I the biopsy demonstrates benign,
histology f ndings can be ollowed in 6 months.

Pearls
• “Architectural distortion” can be due to many di erent
etiologies: prior lumpectomy, prior excisional biopsy,
malignancy such as invasive ductal carcinoma, or radial
Diagnostic mammogram of right magni cation CC view
(additional electronically magni ed) demonstrating several
scar.
groups of “amorphous” calci cations. • In this case, there is history o prior excisional biopsy,
however; in addition, noted are several groups o
indeterminate calcif cations which, on additional
Answers magnif cation views, are “amorphous” and suspicious,
1. Architectural distortion in the superior medial breast. and stereotactic biopsy was per ormed showing the
presence o DCIS.
2. Di erential diagnosis includes radial scar, malignancy,
or prior surgery.
3. Any malignancy can result in the presence o distortion, Suggested Reading
but in general typical or this appearance would be
D’Orsi CJ, Bassett LW, Berg WA, et al. Breast Imaging
tubular carcinoma. Tubular carcinoma is a malignancy
Reporting and Data System: ACR BI-RADS
with relatively good prognosis.
Mammography. 4th ed. Reston, VA: American College
o Radiology; 2003.

176
Prior right mastectomy and new palpable lump in contralateral breast

1. What BI-RADS classif cation should be used


here?

2. What should be the next diagnostic imaging


exam?

3. What are the causes o at necrosis?

4. What calcif cation descriptor would you use in


this case?

5. What type o biopsy would you recommend?

177
Fat necrosis in remaining breast 1797
Case ranking/dif culty: Category: Diagnostic

Left CC spot magni cation view. Left ML spot magni cation view. Di erent patient, similar mass but with
much more calci cations.

Answers 5. This patient has a palpable f nding, and i you were


1. These calcif cations are mainly denser, rod like, and seen concerned about the calcif cations, stereotactic core
in orthogonal planes to lie around the periphery o the at biopsy could be per ormed. However, the calcif cations
density mass. Fat density masses are characteristically are associated with the periphery o the at necrosis
benign. The appropriate assessment is there ore BI- mass, and there ore per orming an ultrasound core
RADS 2. biopsy o the margin o the mass and per orming a
specimen x-ray could be just as diagnostic.
2. As there is calcif cation present, spot magnif cation
views should be per ormed to characterize the calcif c
particles. Tomosynthesis may visualize an associated
mass better. Ultrasound can be characteristic, with Pearls
cystic changes and hyper-echogenicity. It can, however, • Calcif cations in at necrosis start as f ne or lacework
be suspicious when there is calcif cation in the like, and usually end up as classical dystrophic
in ammatory wall o the necrosis. Non– at-saturation calcif cations.
MRI sequence would help to distinguish, i any • In a patient with prior lumpectomy or DCIS, this may
remaining doubt. be a suspicious f nding, prompting biopsy.
3. Most o the above points may be a cause o at
necrosis. It is a response to injury, whether minor, as
in grandchildren or pets climbing on to chest, or to Suggested Readings
a reaction to surgery and radiation treatment (can be Bilgen IG, Ustun EE, Memis A. Fat necrosis o the breast:
marked in patients having intraoperative brachytherapy. clinical, mammographic and sonographic eatures. Eur J
Plastic surgery is a potent cause o at necrosis. Radiol. 2001;39(2):92-99.
4. Use o the “linear calcif cations” descriptor is a DiPiro PJ, Meyer JE, Frenna TH, Denison CM. Seat belt
suspicious term that you would use i you chose to injuries o the breast: f ndings on mammography and
biopsy this lesion. I this is stable and characteristically sonography. AJR Am J Roentgenol. 1995;164(2):317-320.
benign, then you should use a low-risk descriptor such as
“curvilinear.”

178
Palpable lump in the right breast

1. What BI-RADS classif cation should be used


here?

2. What should be the next diagnostic imaging


exam?

3. What is the likely pathology o a circumscribed


mass?

4. What type o biopsy would you recommend?

5. I this is a malignancy, what would you expect


to see on ultrasound?

179
Circumscribed breast cancer 1868
Case ranking/dif culty: Category: Diagnostic

Right CC spot magni cation views. Targeted ultrasound. Doppler ultrasound of mass.

Answers 5. The mass usually has some internal echoes (which


1. In this context, this lesion does not ulf ll the criteria may be di f cult to appreciate i the patient has a
or a benign mass; there ore, it should be treated with mucus-secreting tumor). Vascularity within a mass is
suspicion. See ultrasound images on page 2. a suspicious ultrasound f nding. The echo pattern is
usually mixed echo (or heterogeneous) pattern.
2. Tomosynthesis may not add anything extra in this lesion,
but, in general, it is said to be equal to regular diagnostic
projections or analyzing the margins o the mass.
Ultrasound is the next most important exam to determine Pearls
the nature o the lesion, and in this type o case, some • “Circumscribed mass” can still be ound
would go directly to ultrasound rom conventional in malignancies. Particularly,
mammography. MRI is not likely to add additional • Triple-negative cancers
use ul in ormation. Core biopsy should not be per ormed • Mucoid carcinomas
be ore the diagnostic workup is complete. • Colloid
3. In older patients who still have cysts, they may have
stuck around or decades, and contain proteinaceous
debris, such that there will never be a simple cyst, and Suggested Readings
likely to have appearances o a complicated cyst. Note: Boisserie-Lacroix M, Mac Grogan G, Debled M, et al.
the term complex cyst was dropped in 2003 with the Radiological eatures o triple-negative breast cancers
4th edition o BI-RADS, as it was con using. In the (73 cases). Diagn Interv Imaging. 2012;93(3):183-190.
elderly and in the young, a malignancy may grow with Gwak YJ, Kim HJ, Kwak JY, et al. Ultrasonographic
circumscribed margins. detection and characterization o asymptomatic ductal
4. Ultrasound-guided core biopsy is appropriate here, as it carcinoma in situ with histopathologic correlation. Acta
is easy to f nd, cheap, and quick to per orm. I the lesion Radiol. 2011;52(4):364-371.
was isoechoic and not easily visible, then stereotactic
core biopsy would be appropriate.

180
History of infection several months ago—now palpable lump

1. What is the best descriptor explaining the


palpable abnormality?

2. What is the most likely reason or the palpable


lump?

3. What are other typical f ndings o at necrosis


on mammography?

4. What are the eatures o at necrosis on


ultrasound?

5. What is an appropriate f nal assessment?

181
Fat necrosis 1310
Case ranking/dif culty: Category: Diagnostic

Diagnostic ultrasound demonstrates corresponding “complex Diagnostic ultrasound demonstrates corresponding “complex
mass” with mixed echogenicity and posterior shadowing. mass” without increased ow on duplex.

Answers
1. Finding is consistent with round mass with Pearls
“heterogeneous” density. It includes areas o low, at-like • Given the history o previous in ection, which was
density. treated with antibiotics, and the presence o a mass,
which contains at, the diagnosis o at necrosis is most
2. Given the history o recent in ection, the f nding most
likely.
likely represents at necrosis. The mammogram f nding
• Ultrasound does not help in regard to at necrosis
does correlate to at necrosis given its heterogeneous
because it shows most likely a complex indeterminate
density including areas o low at density.
mass.
3. Fat necrosis can present in many di erent orms. Some • In this particular case, the f nding was called
f ndings are specif c and can be classif ed as BI-RADS 2: BI-RADS 3 (“most likely” benign) and 6-month
benign, or example, at-containing oil cysts, curvilinear ollow-up mammogram was recommended.
calcif cations associated with radiolucent mass. Some
f ndings are more indeterminate such as “coarse”
calcif cations. Some f ndings cannot be di erentiated
Suggested Reading
rom malignancy, and biopsy cannot be avoided, or
example, in case o “spiculated” mass. Taboada JL, Stephens TW, Krishnamurthy S, Brandt KR,
Whitman GJ. The many aces o at necrosis in the breast.
4. Fat necrosis again can show up in many di erent AJR Am J Roentgenol. 2009;192(3):815-825.
orms. Well-circumscribed mass may be classif ed as
BI-RADS 2 or BI-RADS 3, while heterogeneous mass
or ill-def ned masses are unspecif c and malignancy is
di f cult to exclude.
5. Given the presence o new palpable mass a ter in ection,
appropriate assessment is BI-RADS 3 (probably benign)
and 6-month ollow-up with mammogram.

182
Screening—asymptomatic

1. What is the correct BI-RADS classif cation in


this case?

2. What is the next imaging test?

3. What are the eatures o a mass that distinguish


a lymph node rom another mass on
mammography?

4. What are the eatures o a normal lymph node


on ultrasound?

5. What ultrasound eatures are considered


suspicious?

183
Small posterior cancer simulates intramammary lymph node 1838
Case ranking/dif culty: Category: Screening

Right mediolateral. Right ML spot magni cation. Right CC spot magni cation shows
“irregular” mass.

be enlarged and there ore reactive ollowing a diagnostic


biopsy.
5. According to BI-RADS ultrasound, an “irregular mass”
with “angular” or “microlobulated” margins are the
descriptors o a suspicious lesion which should be
biopsied. The “narrow zone o transition” is the opposite
o the suspicious f nding o a bright echogenic margin
to a mass. The “parallel” orientation is the opposite o a
taller-than-wide (or “non-parallel”) mass that would be a
Targeted ultrasound. suspicious f nding.

Answers Pearls
1. This patient is a screening patient with a normal physical • Intramammary lymph nodes can occur in ectopic
examination. The f nding should there ore be given a positions within the breast, not just in the axillary tail.
BI-RADS 0 and urther views plus ultrasound scanning • The presence o a hilum/notch helps to clinch the
recommended. diagnosis.
2. I you have tomosynthesis, then that test will replace the
need or multiple examinations to determine the margins Suggested Readings
o the mass. Ultrasound will be required to determine the
Hogan BV, Peter MB, Shenoy H, Horgan K, Shaaban A.
likely nature o the mass. There is no indication or an
Intramammary lymph node metastasis predicts
MRI at this stage.
poorer survival in breast cancer patients. Surg Oncol.
3. The only eatures that help you make a diagnosis o 2010;19(1):11-16.
lymph node rom other type o mass is the presence Pugliese MS, Stempel MM, Cody HS, Morrow M,
o a hilum, either as a atty lucency within the mass or Gemignani ML. Surgical management o the
as a radiological notch. Those are the pathognomonic axilla: do intramammary nodes matter? Am J Surg.
eatures o a node. Supporting eatures include the 2009;198(4):532-537.
typical position or an intramammary lymph node. Vijan SS, Hamilton S, Chen B, Reynolds C, Boughey JC,
Degnim AC. Intramammary lymph nodes: patterns
4. A vascular hilum and a thin smooth cortex are the
o discovery and clinical signif cance. Surgery.
eatures o a lymph node. Remember that the node can
2009;145(5):495-499.
184
Lump in the left axillary tail

1. What BI-RADS classif cation should be used


here?

2. What is the most likely pathology based on the


imaging?

3. What mammographic views should you


per orm with these f ndings?

4. I you f nd abnormal nodes in the axilla, below


the axillary vein, what levels are involved?

5. As a node is present, what staging tests would


you per orm?

185
Axillary tail IDC plus DCIS plus lymph node 1762
Case ranking/dif culty: Category: Diagnostic

Left ML spot magni cation view—in LXCCL spot magni cation shows the Left breast ultrasound shows not only the main
this plane, the masses are not easily lateral mass at the edge of the breast disc. lesion as an irregular mass containing calci cation
seen. but also a second mass separate from the main
mass consistent with a satellite lesion.

4. There is no such thing as level 0 or 4. Level 3 re ers to


disease superior to the boundary ormed by the axillary
vein. Levels 1 and 2 re er to the levels below the axillary
vein.
5. Ultrasound staging o the locoregional nodes is
important when you see a suspected malignancy
with possible nodes on a mammogram. I the lesion
is large, or is more likely to drain medially due to a
central or medial position, then scanning longitudinally
immediately alongside the sternum may identi y
Left axillary ultrasound—here the enlarged solid node seen involved internal mammary lymph nodes.
on mammography is demonstrated. Core biopsy con rmed
metastatic adenocarcinoma.
Pearls
Answers • IDC with DCIS is usually high nuclear grade, and
1. BI-RADS 4 or 5 can be used here. The combination there ore more likely to metastasize.
o eatures o an “irregular mass” containing highly • A ocal asymmetry or mass associated with DCIS has
suspicious microcalcif cation plus an abnormal lymph a 50% risk o invasive cancer, and is a good target or
node allows BI-RADS 5 to be used. biopsy, giving a higher yield o invasive disease.
• Watch or enlarged nodes on the mammogram, and
2. The most common pathology with these f ndings is a
consider adding axillary ultrasound staging to your
high-grade IDC plus DCIS.
routine ultrasound exam when you suspect a cancer.
3. A lateral view is help ul or localization. However, the
tumor is in the axillary tail in this instance, and i the
mammo tech cannot get posterior enough, the tumor Suggested Readings
may not be visualized. An XCCL is help ul to determine
Iakovlev VV, Arneson NC, Wong V, et al. Genomic
the lateral extent o the disease. Spot magnif cation
di erences between pure ductal carcinoma in situ
views are probably the best, as you get the maximum
o the breast and that associated with invasive
resolution to assess the margins o the mass, and to
disease: a calibrated aCGH study. Clin Cancer Res.
characterize the calcif c particles. The Eklund technique
2008;14(14):4446-4454.
is used to visualize breast tissue in ront o the implants,
Meyerson AF, Lessing JN, Itakura K, et al. Outcome o long
known as the implant displaced view.
term active surveillance or estrogen receptor-positive
ductal carcinoma in situ. Breast. 2011;20(6):529-533.

186
Diagnostic workup of group of indeterminate calci cations

1. What is the best descriptor o this new group o


calcif cation?

2. Why could that group o new calcif cations be


benign?

3. What is the next step a ter the standard ML and


CC magnif cation view?

4. What is the appropriate technique to obtain


tangential view?

5. What is the f nal assessment based on the


tangential view?

187
Skin calci cations 1309
Case ranking/dif culty: Category: Diagnostic

Diagnostic mammogram, right ML magni cation view


demonstrating group of indeterminate calci cations.
Diagnostic mammogram, right CC magni cation view
demonstrating group of indeterminate calci cations.

Diagnostic mammogram, right CC magni cation view with grid


demonstrating the group of indeterminate calci cations at the
coordinates I—7.5.
Diagnostic mammogram, tangential view with BB, demonstrates
Answers group of calci cations within the skin.
1. This is a group o “round and oval” calcif cations, one or
two o the calcif cations demonstrate “lucent center.” Stereotactic biopsy unit could also be used to calculate
the Z value, which, in case o the presence o skin
2. Because one or two o the calcif cations appear to have calcif cations, had to be very small.
lucent center suspects that this could be skin calcif cations.
5. Calcif cations are in the skin—BI-RADS 2 “benign”
3. Based on the standard ML and CC magnif cation views, recommend patient to return or next screening
the group o calcif cations needs to be urther worked mammogram in 1 year.
up by obtaining a tangential view to prove their location
within the skin.
4. The appropriate way is to have a paddle with grid to
Pearls
place a BB on the group o calcif cations. An alternative • The appropriate workup o suspected skin calcif cations
could be to place the patient on the stereotactic requires to obtain tangential view with BB on the
biopsy table and place the BB. Next step is to obtain calcif cations to prove i the group is close to the BB
magnif cation view in second plane tangential to and there ore within the skin.
the BB—and thus tangential to the calcif cations.
188
Skin calci cations (Cont.)

• I the presence o skin calcif cations can be conf rmed,


the assessment is “benign” (BI-RADS 2) and the
patient can return or next mammogram in 1 year.
• To obtain a tangential view, it is crucial to have a
paddle with a grid, which in general is used to per orm
needle localizations. The same paddle can be used or
the tangential view by putting a BB on the group o
calcif cations as seen within the grid—then, a second,
tangential view can be obtained.
• An alternative to prove the presence o skin
calcif cations is to put the patient on the stereotactic
biopsy table and prove that the calculated Z value is so
small that the target has to be in the skin.

Suggested Readings
Berkowith JE, Gatewood OM, Donovan GB, et al. Dermal
breast calcif cations: localization with template-guided
placement o skin marker. Radiology. 1987;163(1):282.
Linden SS, Sullivan DC. Breast skin calcif cations:
localization with a stereotactic device. Radiology.
1989;171(2):570-571.

189
This page intentionally left blank
Palpable nding in the right breast

1. What BI-RADS classif cation should be used


here?

2. What is the most likely pathology based on the


imaging?

3. What is the next best imaging test?

4. What is the relevance o calcif cations


suspicious or DCIS outside o the tumor?

5. Do you have any recommendations or ollow-


up, other than normal mammography?

191
IDC plus DCIS with EIC 1761
Case ranking/dif culty: Category: Diagnostic

Right CC spot Ultrasound shows “irregular mass” with “angular Ultrasound—another area closer to the nipple shows
magni cation view shows margins,” containing re ective echoes consistent a “dilated duct” containing microcalci cations.
microcalci cations both with microcalci cations.
within the tumor and
outside extending anteriorly
toward the nipple.

Answers 5. Recurrent disease is likely to involve calcif cations;


1. BI-RADS 4 or 5 can be given depending on your there ore, ensure that all calcif cations have been
certainty. It is extremely unlikely to be a benign f nding, adequately removed at surgery. Some centers per orm
so a BI-RADS 5 may be the best f t. This is not a spot magnif cation views prior to radiation treatment to
screening exam; there ore, BI-RADS 0 should not be ensure there is no residual DCIS.
used. I it was a screening exam, then a BI-RADS 0
would be appropriate, along with recommendations or
a diagnostic workup to include spot magnif cation views Pearls
and targeted ultrasound. • Calcif cations outside a tumor are just as important to
2. DCIS masses can look like this, but are more likely to document.
be noncalcif ed and circumscribed. Invasive lobular • Extent o associated DCIS is best visualized by MRI.
cancer can present like this, but it is extremely rare. This • EIC has implications or the management o patients
is much more o a typical situation where there is both who are not able to have therapy with Intrabeam
DCIS and invasive cancer. (intraoperative radiation treatment).
• May require completion mastectomy when ound at
3. Ultrasound should be the f rst test to validate the presence surgery.
o a mass, which can then be used to target or biopsy. For
extent o DCIS, MRI is the best test, but expensive, and
may over-estimate the disease, as it is o ten associated Suggested Readings
with proli erative change, especially in pre-menopausal
Schouten van der Velden AP, Boetes C, Bult P, Wobbes
women. There is currently no evidence that preoperative
T. Magnetic resonance imaging in size assessment o
MRI improves the eventual outcome o the patient, but it
invasive breast carcinoma with an extensive intraductal
may be help ul to map the disease or the surgeons.
component. BMC Med Imaging. 2009;9(9):5.
4. DCIS is commonly ound within tumors, but when seen Van Goethem M, Schel out K, Kersschot E, et al. MR
outside it takes on a serious connotation. The disease mammography is use ul in the preoperative locoregional
outside o the index cancer is known as extensive staging o breast carcinomas with extensive intraductal
intraductal component (EIC). This needs measuring component. Eur J Radiol. 2007;62(2):273-282.
and the distance rom the primary tumor noting on the Yiu CC, Loo WT, Lam CK, Chow LW. Presence o extensive
report. It has a signif cant impact on the local recurrence intraductal component in patients undergoing breast
rate, even with a boost during radiation therapy. Some conservative surgery predicts presence o residual disease
patients have a mastectomy to reduce this chance o in subsequent completion mastectomy. Chin Med J
recurrent disease. (Engl). 2009;122(8):900-905.
192
High-risk patient with di use pain in the left lateral inferior breast

1. What could be the next step a ter a normal


diagnostic mammogram in a high-risk patient?

2. I spot compression views and ultrasound are


negative, what is the next step?

3. What is the appropriate scenario to order


problem-solving MRI?

4. What is the f nding o the breast MRI?

5. What is the consequence and next step a ter


the MRI?

193
Invasive ductal carcinoma in the left breast 1304
Case ranking/dif culty: Category: Diagnostic

MRI post contrast Maximum Intensity Projection (MIP)


demonstrates a small area of suspicious enhancement within
the posterior left breast. MRI after IV contrast with subtraction technique demonstrates
focal area of enhancement left lateral posterior breast.

4. Noted is a mass in the le t posterior superior breast,


about 7 mm in diameter.
5. Since the lesion is very ar back in the le t breast near the
chest wall, second look ultrasound is recommended as
next step. MRI-guided biopsy would be technically very
di f cult, i not impossible. I ultrasound does not show
corresponding lesion, MRI-guided biopsy or at least
needle localization or marking o the lesion with clip is
Second look gray-scale ultrasound image demonstrating recommended to guide surgical excision.
corresponding “angular” mass.

Answers
Pearls
1. Ultrasound, in general, would be the next step directed
• This is a situation where MRI can be used as an
to the area o pain elt by the patient. However, i patient
additional “problem-solving” modality in patients with
has a high-risk background, MRI as a problem-solving
di use pain in the le t breast and where mammogram
modality can also be considered.
is unremarkable.
2. This is a common scenario that patient eels a lump, • Despite initially normal targeted ultrasound, MRI did
or has some pain, and mammogram or ultrasound is show, in this case, the presence o small suspicious lesion.
unremarkable. It is not unreasonable to send patient with • Repeat second look ultrasound was per ormed and did
BI-RADS 1 (negative) assessment back to the re erring show corresponding suspicious f nding and ultrasound-
physician and add a statement that “ urther assessment guided biopsy conf rms the presence o invasive
o the pain/lump should be based on clinical grounds.” ductal carcinoma.
That basically means that i the pain/mass is highly
suspicious to the clinician, it might still be necessary or
the clinician to per orm a non image guided biopsy based Suggested Readings
on palpation. MRI is also an option but should be used
Abe H, Schmidt RA, Shah RN, et al. MR-directed (“Second-
wisely. It cannot be used in every patient in that scenario.
Look”) ultrasound examination or breast lesions detected
3. Problem-solving MRI can be help ul but should be used initially on MRI: MR and sonographic f ndings. AJR Am J
wisely. In particular, it should not be used to characterize Roentgenol. 2010;194(2):370-377.
a lesion based on ultrasound, and/or mammographic Moy L, Elias K, Patel V, et al. Is breast MRI help ul in the
morphological criterion is suspicious and needs biopsy. evaluation o inconclusive mammographic f ndings? AJR
That means it would need biopsy, regardless o the Am J Roentgenol. 2009;193(4):986-993.
f nding on MRI. In this particular case, MRI was help ul, Yau EJ, Gutierrez RL, DeMartini WB, Eby PR, Peacock S,
since it is a symptomatic high-risk patient with very Lehman CD. The utility o breast MRI as a problem-
dense tissue, and indeed abnormality was ound. solving tool. Breast J. 2011;17(3):273-280.

194
Palpable lump in a breast-feeding postpartum woman

1. What BI-RADS classif cation should be used


here?

2. In a lactating woman, what is the di erential


diagnosis?

3. I this patient was presenting to you or the


f rst time, which imaging modality is your
f rst choice?

4. What BI-RADS descriptors would you apply


to this lesion?

5. What is the best staging test or this patient?

195
Breast cancer in a lactating woman 1754
Case ranking/dif culty: Category: Diagnostic

LCC spot magni cation—minimal change seen. There is some Another lactating patient showing what normal breast tissue
distortion, but the mass is completely invisible because of lack can look like when lactating. Note the relatively bright glandular
of contrast. Some microcalci cations are also seen. tissue with few features.

4. This is a suspicious lesion and suspicious BI-RADS


descriptors should be used such as “irregular mass” and
“angulated margins.” It is taller than it is wide, which BI-
RADS describes as “nonparallel” contrary to a benign
lesion that is “parallel” to the skin.
5. MRI is the best (nonionizing radiation) exam. It may
have decreased sensitivity because o the hormonal
change related to postpartum. PEM and BSGI could be
used to detect more than one lesion in the breast, but
there is currently a signif cant radiation dose rom the
isotope used. PET/CT can be used i the lesion is large
and there is evidence on nodal spread.
Another lactating patient showing prominent dilated milk lled
ducts.
Pearls
Answers • Palpable masses should always be taken seriously
1. BI-RADS 4 or 5 can be used here based on the and explained, especially during pregnancy and
highly suspicious ultrasound. The mammogram is not lactation, even though the pretest probability or
particularly help ul, except rom excluding associated malignancy is low.
DCIS microcalcif cations.
2. All o the above are correct, as they can all present with Suggested Readings
a noncalcif ed mass in a lactating woman.
Espinosa LA, Daniel BL, Vidarsson L, Zakhour M, Ikeda
3. A nonionizing examination is the modality o choice in DM, Her kens RJ. The lactating breast: contrast-enhanced
a young woman. This can be challenging in a woman MR imaging o normal tissue and cancer. Radiology.
who is lactating, but less so than in a pregnant patient, 2005;237(2):429-436.
prepartum. I you see a suspicious abnormality, then Sabate JM, Clotet M, Torrubia S, et al. Radiologic evaluation
mammography is warranted, as high-grade IDC can o breast disorders related to pregnancy and lactation.
present f rst with microcalcif cations in otherwise dense Radiographics. 2007;27(Suppl 1):S101-S124.
breasts. MRI can be reserved or challenging cases or Saglam A, Can B. Coexistence o lactating adenoma and
or staging. invasive ductal adenocarcinoma o the breast in a pregnant
woman. J Clin Pathol. 2005;58(1):87-89.

196
Patient with new abnormality on screening exam: MLO and CC view

1. What is the next step?

2. What is the appropriate BI-RADS


classif cation?

3. What is the appropriate description o the


mammogram abnormality?

4. What is the di erential diagnosis o this new


mass based on mammogram?

5. I the f nding is stable since several years on


standard mammogram, what is the assessment?

197
Invasive ductal carcinoma 1306
Case ranking/dif culty: Category: Diagnostic

Diagnostic mammogram of right spot compression MLO view Diagnostic mammogram of right spot compression CC view
demonstrating small mass. demonstrating small mass.

5. I the f nding is stable or more than 2 years on standard


screening mammogram, it can be called BI-RADS 2.
The f nding is not specif c and could also represent
benign f nding as discussed in question 4, or example,
a complicated cyst. The presence o “irregular” shape
and “indistinct” margin is not appreciated on standard
views. In this particular case, the appearance on spot
compression views and on the ultrasound makes a
di erence and raises concern.

Pearls
• The purpose o screening mammograms is to f nd
Gray-scale ultrasound demonstrating small corresponding abnormalities; however, detailed description should
“hypoechoic mass” with “irregular” shape with thick echogenic be spared or subsequent diagnostic mammogram that
halo and “indistinct” margin. includes additional, more specif c views, such as spot
compression or magnif cation views.
Answers • That is the reason why ollow-up exams o “probably
benign” f ndings should in general include the
1. Next step is diagnostic workup with spot compression most specif c images, such as spot compression or
views. magnif cation views, since based on small changes
2. BI-RADS 0 incomplete exam—patient needs to be in morphology, decision is made to urther ollow
recalled. the f nding or to biopsy the f nding.

3. This is a f nding o a small mass with partially obscured


margin and o equal density to the surrounding tissue
and o “irregular” shape and “indistinct” margin. Suggested Reading
4. The f nding is indeterminate and could represent D’Orsi CJ, Bassett LW, Berg WA, et al. Breast Imaging
in amed cyst, hematoma, or malignancy, which could Reporting and Data System: ACR BI-RADS–
include invasive ductal carcinoma, DCIS, and invasive Mammography. 4th ed. Reston, VA: American College
lobular carcinoma. o Radiology; 2003.

198
Palpable nding in the right breast

1. What BI-RADS classif cation should be used


here?

2. What is the most likely pathology based on the


imaging?

3. What is the next best imaging test?

4. What type o biopsy would you recommend?

5. I phyllodes tumor is ound on biopsy, what is


the appropriate management?

199
Atypical broadenoma 1745
Case ranking/dif culty: Category: Diagnostic

Right MLO. Right lateromedial exam. The mass was medial in the breast, so
a LM exam is the preferred projection. “Lobulated circumscribed
mass.”
Answers
1. The ultrasound eatures are suspicious, in that it is an
“irregular mass” with “microlobulated” margins, and
Pearls
a “heterogeneous echo pattern.”
• The most suspicious imaging modality usually trumps
2. Strictly, all o the answers could be correct, as they all can the least suspicious. This is not always the case; or
appear with similar f ndings. The most likely f nding in a example, a partially obscured mass on mammography
29-year-old woman is f broadenoma or phyllodes tumor, may be an obvious simple cyst on ultrasound. A lesion
and it is the concern about the latter that prompts biopsy. that does not appear as a classical f broadenoma should
3. All o the above have been per ormed be ore in this be regarded as suspicious and biopsy conf rmed due to
situation. MRI is expensive and likely to show an the risk o phyllodes tumor.
enhancing mass, which will not a ect the outcome.
Elastography may show some tissue sti ness, but in
younger women, f broadenomas o ten have a more
Suggested Readings
cellular component and are there ore so ter than
f broadenomas in older patients. In view o the suspicious Chung A, Scharre K, Wilson M. Intraductal
imaging, biopsy needs to be per ormed. Mammography f broadenomatosis: an unusual variant o f broadenoma.
can be considered, especially with f ndings that are not Breast J. 2008;14(2):193-195.
def nitely characteristic o a f broadenoma. Sklair-Levy M, Sella T, Alweiss T, Craciun I,
Libson E, Mally B. Incidence and management o
4. I it is visible on ultrasound, then the best way to do is complex f broadenomas. AJR Am J Roentgenol.
a biopsy. I the lesion is palpable, some surgeons may 2008;190(1):214-218.
pre er to do the biopsy themselves, but ultrasound should Thein KY, Trishna SR, Reynolds V. Benign and
be used to conf rm that the biopsy is sampling the right malignant breast lesions mimicking each other:
parts o the mass. imaging-histopathologic correlation. Cancer Imaging.
5. Phyllodes tumors need to be excised with a good margin, 2011;11(Spec No A):S180.
as they have a high chance o local recurrence but do
not metastasize. Fibroepithelial lesions are a type o
f broadenoma variant that has been recognized, which
needs surgical excision, to ensure that the mass has been
adequately sampled.

200
Prior breast cancer on the right breast: surveillance mammograms

1. What BI-RADS classif cation should be used


here?

2. What is the most likely pathology based on the


imaging?

3. What type o implant causes calcif ed


capsules?

4. What is the best examination or looking


at implant integrity?

5. What is the name used or an intracapsular


rupture o an implant on MRI?

201
Calci ed collapsed implants in patient postlumpectomy 1742
Case ranking/dif culty: Category: Diagnostic

Another case—left MLO shows almost identical features.


The calci cation is coarse and “popcorn” like, but very extensive
Right CC. in the retro-pectoral space.

Answers on the anterior margin o the implant). Diagnostic


1. Whether screening or diagnostic, these f ndings are mammograms can show complications o capsules,
characteristically benign and there ore BI-RADS 2. and contour abnormalities may suggest weakening
o the implant wall, but is not the best test.
2. These are the appearances o collapsed calcif ed implant
capsules ollowing explantation o silicone pre-pectoral 5. The term used is Linguine sign, o the collapsed inner
implants. Fat gra ting gives changes similar to either capsule within the main capsule. Not to be con used
lipomas or to at necrosis, depending on how the body with other delicious orms o Italian pasta.
responds to the treatment. Injected silicone looks very
dense and di erent to this type o density. Guinea worms
look like irregular coils o calcium. Found when an adult
Pearls
dracunculus parasite dies, it is usually small in caliber, • The position o lesion and being bilateral and
but can occasionally be very large. They do not tend to symmetrical gives you the diagnosis.
be seen bilaterally or symmetrically. • Silicone implants, when they rupture, cause a marked
in ammatory response, such that these calcif ed masses
3. There is some evidence or calcif ed capsule ormation o the implant capsules may remain in the breast a ter
with saline implants, but this is rare. Silicone leaking surgical explantation.
across the implant causes an intense in ammatory
reaction, which is prone to calci ying. Capsular
calcif cation is normally seen in the pre-pectoral type o Suggested Readings
implant, normally rom a cosmetic surgery in the past.
Dershaw DD, Chaglassian TA. Mammography a ter
Saline implants are pre erred or the retro-pectoral space,
prosthesis placement or augmentation or reconstructive
as they do not have any complications rom rupture,
mammoplasty. Radiology. 1989;170(1, Pt 1):69-74.
unlike silicone gel. Trilucent implants were used in
Peters W, Pritzker K, Smith D, et al. Capsular calcif cation
Europe or a time, be ore they were banned rom use.
associated with silicone breast implants: incidence,
They were relatively easy to see through, causing less
determinants, and characterization. Ann Plast Surg.
obstruction o normal breast tissue.
1998;41(4):348-360.
4. MRI is the best exam or implant integrity, as you can Peters W, Smith D, Fornasier V, Lugowski S, Ibanez D.
see the whole o the implant and the capsule, as well as An outcome analysis o 100 women a ter explantation o
per orming silicone dark and silicone bright sequences. silicone gel breast implants. Ann Plast Surg. 1997;39(1):
Ultrasound is good or a palpable f nding (usually 9-19.
202
Palpable abnormality in the left breast: 73-year-old patient

1. What is the f rst step o workup o palpable


abnormality in a 73-year-old patient?

2. What would be the workup i patient had prior


normal screening mammogram 7 months ago?

3. What would be the workup i there was a


normal mammogram 3 months ago?

4. What is the most likely morphological


appearance o low-grade DCIS?

5. What is the likelihood o the presence o


malignancy in case o normal imaging despite
the presence o palpable abnormality?

203
Low-grade DCIS 733
Case ranking/dif culty: Category: Screening

Spot compression left MLO view Spot compression left CC view with Gray-scale ultrasound image of palpable abnormality
with BB marker on palpable BB marker on palpable abnormality demonstrates small hypoechoic mass.
abnormality demonstrating small demonstrating small mass.
mass.

Answers 1.2% o the patients having malignancy ( alse-negative


mammogram). However, the alse-negative rate will also
1. Workup o patients in that age group includes diagnostic
depend on the density o the breast parenchyma. It can
mammogram with spot compression views with BB
be suspected that it will be even lower in “ atty replaced
marker on the palpable f nding and then ultrasound.
breast” versus in “extremely dense breast.”
2. I patient had recent mammogram more than 6 months
ago, in general, repeat mammogram is recommended.
Remember, 6 months is the time rame used to ollow BI-
Pearls
RADS 3 f ndings. There is no need to repeat mammogram
at this point at the nonsymptomatic right side. This can be • Any palpable abnormality is suspicious—the only
done when patient is due or next screening mammogram. f nding that can be considered as def nitely benign
is a simple cyst.
3. In general, i there is a normal mammogram available, • In this particular case, the f nding is not a simple cyst
per ormed less than 6 months ago, there is no need to and remains indeterminate and has even suspicious
repeat mammogram and it is not unreasonable to per orm eatures such as “taller-than-wide” (Stavros) shape and
an ultrasound f rst. I the ultrasound is negative and the the mass was subsequently biopsied under ultrasound
palpable abnormality is very questionable, it would be guidance and did show the presence o low-grade DCIS.
reasonable to stop here. However, this depends also on • I there is no abnormality seen on imaging, including
the conf dence o the assessment o the last mammogram, ultrasound, “ urther management o palpable
per ormed less than 6 months ago. I breast parenchyma abnormality should be based on clinical grounds” and
was very dense and there ore limits the assessment, it the exam can be called “negative” BI-RADS 1 and
would not be unreasonable to repeat the mammogram, even patient can return to screening.
i it was called BI-RADS 1 (“negative”) less than 6 months • In very rare situations, biopsy o palpable f ndings
ago. Remember at that time no spot compression views based on the palpation and without imaging guidance,
were obtained, since it was a screening mammogram. per ormed by breast surgeon, can show malignancy.
4. In this case, a mass turned out to be low-grade DCIS,
which is rather uncommon. In general, low-grade DCIS
presents as a group o calcif cations that are usually Suggested Readings
rather “amorphous” or “round and oval” and less likely
DiPiro PJ, Meyer JE, Denison CM, Frenna TH, Harvey
“pleomorphic.”
SC, Smith DN. Image-guided core breast biopsy o
5. There are not many studies available looking at this ductal carcinoma in situ presenting as a non-calcif ed
issue. The study by Gumus et al. (2012) looked over abnormality. Eur J Radiol. 1999;30(3):231-236.
a 12-year period o time at 251 patients with palpable Gumus H, Gumus M, Mills P, et al. Clinically palpable breast
abnormalities and normal imaging and ound only abnormalities with normal imaging: is clinically guided
biopsy still required? Clin Radiol. 2012;67(5):437-440.
204
Prior left mastectomy and reconstruction for triple-negative breast cancer

1. What BI-RADS classif cation should be used


here?

2. Which is the best modality to assess implants?

3. Which is the best modality to assess a palpable


f nding associated with an implant?

4. What urther tests do you recommend at this


stage?

5. I a patient has an implant reconstruction


as part o a mastectomy, what is the initial
implant used?

205
Palpable recurrence on mastectomy site with reconstruction 1580
Case ranking/dif culty: Category: Diagnostic

Subtracted MIP image in axial plane.


Ultrasound—relationship with implant and skin—technique
using gentle pressure with stando gel.
however, some centers have these tests as local protocol
or any patient with a breast cancer recurrence. MRI is
the most help ul initial test to determine the extent o
the recurrence and check on skin coverage or surgical
planning, which is usually a completion mastectomy in
a patient who has had breast conservation. This patient,
however, has already had a mastectomy with tissue
expanders and ollowed by implant reconstruction.
5. The patient has a special type o implant inserted, which
is a tissue expander. This type o implant has a saline bag
and tubing with a valve, which is gradually expanded
over a period o weeks to stretch the skin. Once the
skin has reached the required volume, the expander
MRI—T1 postcontrast, unsubtracted exam. is switched out or a conventional implant.

Answers
Pearls
1. You can either use BI-RADS 4 straight o or wait until
you have done the ultrasound and give a combined BI- • Any new mass in a patient who has had a mastectomy
RADS assessment. There is a vague density seen on the and reconstruction or breast cancer should be treated
le t ML, but best seen in this instance on the CC view. expeditiously.
Implant-displaced views cannot be per ormed because • High yield or breast cancer.
there is no “breast tissue,” as this implant was placed • I originally presented with a mass, a recurrence with
ollowing a tissue expander. a urther mass is more common.

2. MRI is the only modality that allows or ull assessment


o implants. Using silicone suppression sequences
Suggested Readings
allows or distinguishing between cysts and silicone
granulomas. MRI can see the posterior aspect o the Destounis S, Morgan R, Arieno A, Sei ert P, Somerville P,
implant, no accessible with ultrasound. Murphy P. A review o breast imaging ollowing
mastectomy with or without reconstruction in an outpatient
3. Ultrasound should be the f rst modality to use, as no community center. Breast Cancer. 2011;18(4):259-267.
ionizing radiation, and can be targeted easily, and Patterson SG, Teller P, Iyengar R, et al. Locoregional
correlated with the physical f ndings. MRI overall is recurrence a ter mastectomy with immediate transverse
the best modality or assessing implants themselves. rectus abdominis myocutaneous (TRAM) ap
4. I there is no biochemical evidence o metastatic reconstruction. Ann Surg Oncol. 2012;19(8):2679-2684.
deposits, then PET or bone scans may not be needed; Sim YT, Litherland JC. The use o imaging in patients post
breast reconstruction. Clin Radiol. 2012;67(2):128-133.
206
45-year-old female with new 3–4 mm “focal asymmetry” on screening
mammogram, spot compression views (the two gures on the right)
con rm the presence of 3–4 mm mass

1. What is the di erence between an asymmetry


and a mass?

2. What is the workup o an asymmetry on a f rst


screening mammogram?

3. I there is a f nding on ultrasound—which


correlates to the mammogram—what is the
next step?

4. What is the next step in regard to the “ ocal


asymmetry” i it is new?

5. What would be a way to correlate an


ultrasound with a mammogram f nding,
i in doubt?

207
Pseudoangiomatous stromal hyperplasia 321
Case ranking/dif culty: Category: Screening

Mammogram of right spot compression Ultrasound directed to the right breast


CC view con rming small 3–4 mm mass demonstrating small hypoechoic mass of
on the right lateral breast. corresponding size in the central breast. Diagnostic mammogram, right MLO
view demonstrates iodine contrast
(black arrow) and mass (white arrow).
Answers
1. BI-RADS di erentiates between “mass” and cyst, and repeat mammogram to see i the mammogram
“asymmetry.” Mass is always seen in two projections f nding is gone. Other options could be to place a Homer
and has a clear border and conspicuity which is visible needle (removable needle localization) and repeat
on both projections. An asymmetry is sometimes seen mammogram, or to inject small trace o air or iodine
only in one projection, but can be seen on two projections contrast and repeat mammogram. This can be help ul i
as well and is less def ned. A “ ocal asymmetry” is there are multiple small f ndings or i the f ndings are so
di erentiated rom a larger “global asymmetry” by size small that cyst aspiration is technically di f cult.
alone. A global asymmetry can involve an entire quadrant
and is in general more likely to represent normal tissue.
2. The patient needs to be recalled or a diagnostic work up Pearls
including spot compression views CC and MLO. Then • To correlate ultrasound f nding to mammogram f nding o
ultrasound should be per ormed. It is also important small mass can be challenging, in particular i the mass is
to conf rm that there is no palpable abnormality in very small or because o the presence o multiple lesions.
that area. In the absence o an underlying palpable • Correlation can be achieved by placing BB on the skin
abnormality, the f nding can be ollowed in 6 months and next to the ultrasound f nding and repeat mammogram
classif ed as “probably benign” i ultrasound is normal. and even sometimes tangential views.
• Placement o a Homer needle can be used to mark the
3. The ultrasound f nding has to be a simple cyst to justi y
ultrasound f nding and then per orm mammogram.
calling this a benign f nding and return the patient to
• It might be sometimes help ul to inject small amount
screening. Any other f nding should be biopsied, since this
o air or iodine contrast (1-mL tuberculin syringe)
is a new mass on mammogram. An exception could be
adjacent to the ultrasound f nding and then repeat
cluster o small cysts and ollow-up in 6 months could be
mammogram or more def nite correlation.
per ormed. However, i there is any doubt, biopsy would
• I the ultrasound f nding does not correlate to a new
be pre erred in case o a new mass on mammogram.
mass seen on mammogram, stereotactic biopsy is
4. I the f nding is a new asymmetry and has not any recommended.
corresponding f nding on ultrasound, it has to be biopsied
or excised—BI-RADS 4 (suspicious). First step would
be to attempt stereotactic biopsy. I f nding is not visible Suggested Readings
on the stereotactic biopsy table, needle localization and
Ellis RL. Sonographic conf rmation o a mammographically
subsequent surgical excision can be per ormed.
detected breast lesion. AJR Am J Roentgenol.
5. One way is to place a BB on the ultrasound f nding and 2011;196(1):225-256.
per orm a tangential view to see i it correlates. Other Sickles EA. The spectrum o breast asymmetries: imaging
methods would be to aspirate the f nding, in case it is a eatures, work-up, management. Radiol Clin North Am.
2007;45(5):765-771, v.
208
Palpable nding in the right breast

1. What BI-RADS classif cation should be used


here?

2. What is the next imaging test?

3. I calcif cation is seen with this f nding, what is


the likely pathology?

4. This lesion is palpable. What type o biopsy


should be per ormed?

5. Based on the imaging, what is the likely


pathology on core biopsy?

209
Cancer partially obscured by dense breast tissue 1578
Case ranking/dif culty: Category: Diagnostic

Ultrasound showing an “irregular” mass with “microlobulated” Axial subtracted MIP showing 4.3-cm solitary enhancing mass.
superior “margins.”
complex sclerosing lesion, which can be associated with
DCIS in 20%. Lobular cancer may be associated with
amorphous calcif cations. A classic “spiculate mass”
with “f ne linear pleomorphic” calcif cations is usually a
high-grade invasive ductal cancer with high-grade DCIS.
4. Although palpation guidance can be used, ultrasound
guidance allows you to conf rm that the needle passes
through di erent areas o the mass, and gets the most
representative samples o the tumor. I the lesion is not
clearly seen on ultrasound, or there are con ounding
appearances on ultrasound, such that your conf dence
or sampling the mass accurately is low, then consider
stereotactic core biopsy.
Right breast sagittal subtracted thin MIP. 5. The appearances o a mass with distortion are more
likely to be a eature o invasive ductal carcinoma.
Invasive lobular cancer may present as a mass (better
Answers seen on the CC) but is more common as subtle
1. Either BI-RADS 4 or 5 is a valid answer. Even though distortion, a slowly shrinking breast, or even with
you are not supplied with spot or spot magnif cation views, no mammographic f ndings, but obvious palpation
you get the impression that this is quite a large lesion. abnormalities or ultrasound changes.
2. Ideally, you want to maximize the in ormation you can
get rom mammography be ore proceeding to other tests. Pearls
Lateral projection and spot or spot magnif cation f lms
• Dense breasts represent a challenge to the reader.
should be used to get a better idea o the extent o this
• Tomosynthesis holds promise in this area.
lesion. I available, tomosynthesis may help to evaluate
• Look or disruption o normal lines/structures in
the mass margins. Ultrasound is the next examination
the breast.
when this is done. In some patients with very dense
• Comparison with opposite side is important.
breasts, it is not possible to adequately measure the extent
o disease, and MRI may be the more accurate modality.
3. In the setting o a “spiculate mass,” there are several Suggested Reading
possibilities, which include benign conditions such as Huynh PT, Jarolimek AM, Daye S. The alse-negative
at necrosis (where the calcif cation is usually easier mammogram. Radiographics. 2006;18(5):1137-1154;
to distinguish as appears dystrophic), radial scar, or quiz 1243-1244.
210
Bilateral masses in screening patient: what is the consequence?

1. What are the f ndings on the screening


mammogram?

2. What is the most likely etiology?

3. What is the next step a ter screening


mammogram o asymptomatic patient?

4. What is the appropriate BI-RADS assessment?

5. What is the likelihood o malignancy in this


case? (best answer)

211
Bilateral benign masses 1625
Case ranking/dif culty: Category: Diagnostic

On B-mode ultrasound, there are scattered benign-appearing, On B-mode ultrasound, there are scattered benign-appearing,
well-circumscribed masses noted in the right breast. well-circumscribed masses noted in the left breast.

Answers
1. Noted are bilateral scattered benign-appearing masses. Pearls
2. Most likely bilateral masses will be due to the presence • In the absence o palpable abnormality, multiple benign-
o bilateral f brocystic changes (cysts) or less likely due appearing masses on a screening mammogram can be
to bilateral f broadenomas. classif ed as “benign” and there is no need or recall.
• Based on the study by Leung and Sickles (2000), the
3. In general, there is no need or urther workup, since incidence o breast cancer in a mammogram with
it has been shown that the likelihood o malignancy bilateral “benign”-appearing masses is not higher than
is not higher than in normal screening population. In in the absence o bilateral “benign masses” and there is
this particular case, the patient elt multiple lumps and no need or workup, unless there is signif cant change,
there ore ultrasound was per ormed. new abnormal morphology, or new clinical symptoms.
4. The appropriate classif cation is BI-RADS 2 (benign)
and next screening exam is due in 1 year.
5. Among 1440 patients with bilateral scattered masses, Suggested Reading
only 2 interval cancers were ound based on a study by Leung JW, Sickles EA. Multiple bilateral masses detected on
Leung and Sickles (2000), which results in an incidence screening mammography: assessment o need or recall
rate o malignancy o 0.14% that is lower than the age- imaging. AJR Am J Roentgenol. 2000;175(1):23-29.
matched ultrasound incident cancer rate o 0.24%.

212
Screening exam: prior lms on left

1. What is the pertinent f nding?

2. What malignancy can present in that way?

3. Would MRI, in this case, help to eliminate


biopsy?

4. Does it make a di erence i there were no old


images available?

5. Why is ultrasound necessary or workup in this


case?

213
Developing “focal asymmetry” 762
Case ranking/dif culty: Category: Screening

Diagnostic mammogram, left CC spot compression view, MRI, T1-weighted sequence after IV contrast with subtraction
demonstrating “focal asymmetry.” technique, demonstrating “non–mass-like” area of enhancement.

Answers
1. Noted is the development o subtle “ ocal asymmetry” Pearls
in the le t superior breast, posterior depth. • Any developing density that persists on spot
compression views is suspicious (BI-RADS 4), and
2. All malignancies can present as “ ocal asymmetry.”
despite o lack o ultrasound f nding, stereotactic
Pseudoangiomatous stromal hyperplasia (PASH) is not a
biopsy should be per ormed.
malignant lesion but can present as ocal asymmetry.
• In this case, MRI was per ormed be ore the biopsy.
3. In general, the negative predictive value o breast It demonstrates corresponding “non–mass-like” area
MRI is high, close to 100% but not exactly 100%. o enhancement—subsequently per ormed stereotactic
There ore, the mainstream opinion is to biopsy any biopsy demonstrates f ndings consistent with PASH,
suspicious abnormality seen on mammogram—including which is concordant.
developing “ ocal asymmetry.” However, there is a • I this patient had no prior screening study, the f nding
recent shi t and there are more publications suggesting would be BI-RADS 3 based on the mammogram
that negative MRI might eliminate the need or biopsy. and negative ultrasound, and could be ollowed in
The article rom Europe in 2011, or example, suggests 6 months.
that this is easible.
4. I this was a “ ocal asymmetry” on a baseline
mammogram, the workup would be the same. But i Suggested Readings
there was no underlying suspicious morphology, the Dorrius MD, Pijnappel RM, Sijens PE, van der Weide MC,
f nding could be classif ed as BI-RADS 3 and could be Oudkerk M. The negative predictive value o breast
ollowed in 6 months, and again 6 months later and then magnetic resonance imaging in noncalcif ed BI-RADS 3
1 year later to have a monitoring period o 2 years in lesions. Eur J Radiol. 2012;81(2):209-213.
total. Leung JW, Sickles EA. Developing asymmetry identif ed
5. In case o a new asymmetric density, it is necessary to on mammography: correlation with imaging outcome
determine i there is any corresponding abnormality that and pathologic f ndings. AJR Am J Roentgenol.
could be biopsied under ultrasound guidance. In case 2007;188(3):667-675.
o an asymmetric density on a baseline mammogram, Piccoli CW, Feig SA, Palazzo JP. Developing asymmetric
ultrasound is necessary to urther exclude corresponding breast tissue. Radiology. 1999;211(1):111-117.
abnormality. I ultrasound is normal, it can be ollowed
in 6 months.
214
Bilateral calci cations on rst screening exam

1. What would be the description o these


calcif cations on a screening exam?

2. What is the consequence these descriptors?

3. What other benign proli erative changes can


cause pleomorphic calcif cations?

4. What is the consequence o the presence


o pleomorphic calcif cations on diagnostic
exam?

5. Pathology showed f brocystic changes


and sclerosing adenosis on the right side
(calcif cations seen in specimen) and DCIS on
the le t side. What is the management?

215
Bilateral suspicious calci cations 761
Case ranking/dif culty: Category: Diagnostic

Right ML magni cation view with Right CC magni cation view with Left ML magni cation view with additional electronic
additional electronic magni cation additional electronic magni cation magni cation demonstrating group of “pleomorphic”
demonstrating group of demonstrating group of calci cations.
“pleomorphic” calci cations. “pleomorphic” calci cations.

4. “Pleomorphic” calcif cations indicate the need or biopsy.


5. Both f ndings are concordant. Patient needs lumpectomy
o le t breast and 6-month ollow-up mammogram o right
side with magnif cation views. A preoperative MRI would
also be help ul. Any benign biopsy in general will be
ollowed in 6 months. Any ollow-up o calcif cations has,
by def nition, to include magnif cation ML and CC views.
The same principle applies to ollow-up o “ ocal
asymmetries” or “masses”; it should always include spot
compression views. You want to have the best in ormation
to ollow something that is considered BI-RADS 3 or was
recently biopsied to look or change.
Left CC magni cation view with additional electronic magni cation
demonstrating group of “pleomorphic”calci cations.
Pearls
• There is a large overlap between benign proli erative
Answers changes and associated calcif cations related to DCIS.
• For example, sclerosing adenosis as a orm o benign
1. Screening mammogram demonstrates bilateral scattered
proli erative change o breast parenchyma can cause
benign calcif cations and also group o indeterminate
“pleomorphic” calcif cations indistinguishable rom
calcif cations in the right in erior medial breast and
“pleomorphic” calcif cations as a result o high-grade
le t superior lateral breast. On screening mammogram,
DCIS.
calcif cations should not be urther characterized.
• That explains why about 70% o biopsies will not show
All indeterminate calcif cations need to be magnif ed
malignancy—the benchmark (true positive rate) is to
and then can be urther described with appropriate
have malignancy, including DCIS, in about 30% to
BI-RADS descriptors.
40% o biopsies per ormed (PPV).
2. Patient needs to be recalled or additional magnif cation
views bilaterally.
Suggested Reading
3. Sclerosing adenosis and early calcif cations in
a f broadenoma can present as “pleomorphic” Burnside ES, Ochsner JE, Fowler KJ, et al. Use o
calcif cations. microcalcif cation descriptors in BI-RADS 4th edition to
strati y risk o malignancy. Radiology. 2007;242(2):388-395.
216
Palpable nding

1. What BI-RADS classif cation should be used


here?

2. What is the most likely pathology based on the


imaging?

3. What is the next best imaging test?

4. A biopsy shows DCIS with papillary eatures.


What is your next step?

5. What type o biopsy would you recommend?

217
Intracystic carcinoma 1612
Case ranking/dif culty: Category: Diagnostic

Ultrasound of palpable nding. Is this a solid lesion with cystic Ultrasound shows an intracystic mass that has irregular margins.
component or is the solid mass bounded by the cyst wall? Keep There is no sign of the mass extending beyond the cyst wall.
looking….

Answers 5. Fine needle aspiration can give suspicious f ndings with


1. The mammographic f ndings are suspicious. As the papillary carcinoma, and tissue is better. Smaller core
patient has a palpable f nding, this is a diagnostic exam, biopsies may just give ragments o a papillary lesion,
and there ore a BI-RADS 0 should not be given. A BI- and then surgical diagnostic excision may be required.
RADS 4 (suspicious) is the most appropriate assessment Pathologists like to see a relatively intact or larger
in this case. specimen so that they can understand the architecture,
which is why some authors recommend wide-bore
2. The mammographic appearances could be due to any o
vacuum-assisted biopsy in this situation.
the above f ndings. The ultrasound appearances are what
clinch the diagnosis. I you chose any o the answers,
you could be correct.
3. There are two approaches to this f nding. I you think Pearls
that urther diagnostic workup will assist you in seeing • I the lesion is vascular on Doppler ultrasound, with
the margins o the mass more clearly, or i you suspect vessels entering perpendicular to the wall o the cyst/duct,
calcif cations being present, then diagnostic views are then it is more likely to be a papillary lesion.
recommended. Many groups go direct to ultrasound • I you see more than one lesion, it could be simple
when you have a mass, as i you can diagnose a simple papillomatosis or papillary DCIS.
cyst on ultrasound; you can prevent urther unnecessary
workup and radiation.
4. While working in multidisciplinary teams, surgeons Suggested Readings
like to know early about a patient in case they wish Bhargava R, Esposito NN, Dabbs DJ. Intracystic papillary
to do urther clinical evaluation. I this is a mass like carcinomas o the breast are more similar to in situ
this, you may be so suspicious o invasive disease and carcinomas than to invasive carcinoma. Am J Surg Pathol.
you may want to sample more o the lesion to show 2011;35(5):778-779; author reply 779-781.
potential invasion. DCIS rarely has spread into the axilla. Kitada M, Hayashi S, Matsuda Y, Sato K, Miyokawa N,
Papillary DCIS is ar rom an indolent lesion, and can Sasajima T. Surgical treatment o intracystic carcinoma
be very aggressive in expanding throughout the breast. o the breast. World J Surg Oncol. 2011;9(9):116.
Further workup is needed to determine whether it is Wang H, Li F, Luo B. Breast intracystic papillary carcinoma.
possible to per orm breast-conserving surgery. Papillary Breast J. 17(6):676-677.
DCIS has a tendency to be rapidly multi ocal and may
spread to di erent segments, making it multicentric.
218
Spontaneous bloody nipple discharge in the left breast—any nding?

1. What are the f ndings on the mammogram and


ductogram?

2. What other exam should be included in the


workup?

3. What is the next step a ter MRI is normal?

4. What would be the last resort the breast


surgeon could o er?

5. What is the most suspicious orm o nipple


discharge?

219
Bloody, spontaneous nipple discharge 1627
Case ranking/dif culty: Category: Diagnostic

Demonstrates no abnormality in the left breast.

Answers
1. Mammogram and ductogram are unremarkable.
2. Ultrasound should also be per ormed. I there are still no
f ndings, MRI might be considered.
3. The next step would be to send patient to breast surgeon
Ductogram of left breast, CC spot compression view
or clinical evaluation.
is unremarkable.
4. The gold standard would be to per orm selective duct
excision. That is the reason why, in some institutions,
no ductogram is done, because it could be argued that
• Although negative predictive value o additional MRI
the ultimate step (duct excision) should be done anyway,
is very high, there are no data currently available
even in the presence o normal imaging, i the discharge
supporting negative MRI eliminating the need or
is clinically worrisome enough.
urther action in the situation o high clinical concern,
5. Spontaneous unilateral bloody or clear discharge is o such as patients with new spontaneous, bloody nipple
most concern. During pregnancy, spontaneous bloody discharge.
discharge bilaterally can be physiologic.

Suggested Readings
Pearls Montroni I, Santini D, Zucchini G, et al. Nipple discharge:
• Negative predictive value o normal mammogram, is its signif cance as a risk actor or breast cancer
ultrasound, and ductogram is very high in the presence ully understood? Observational study including 915
o nipple discharge. consecutive patients who underwent selective duct
• However, in selected cases, surgical excision o the excision. Breast Cancer Res Treat. 2010;123(3):895-900.
duct might still be considered, which is still considered Nelson RS, Hoehn JL. Twenty-year outcome ollowing
the gold standard. central duct resection or bloody nipple discharge.
Ann Surg. 2006;243(4):522-524.

220
Palpable abnormality in the right breast in a 44-year-old patient

1. What would make you suspicious that this


could be a phyllodes tumor?

2. What are the typical clinical eatures o a


phyllodes tumor?

3. How requent is the f nding o phyllodes


tumor?

4. How can we di erentiate benign rom


malignant phyllodes tumor?

5. What is the appearance o phyllodes tumor


on MRI?

221
Phyllodes tumor 1000
Case ranking/dif culty: Category: Diagnostic

Diagnostic mammogram, right Diagnostic mammogram, right Ultrasound of right breast demonstrating large mass
spot compression MLO view, spot compression CC view, with cystic component.
demonstrating large mass with demonstrating large mass of “high
“lobular” shape. density” and “circumscribed” margin.

Answers
• In general, they occur in the age group o mid-40
1. Phyllodes tumors, o tentimes, cannot be distinguished
years. Secondary signs o malignancy such as peri ocal
rom f broadenoma. Fast growing masses with relatively
edema, skin thickening, or nipple retraction are absent.
benign morphological eatures raise the question o the
• On ultrasound, phyllodes tumors are well-def ned
presence o phyllodes tumors. Phyllodes tumors o ten
masses with heterogeneous echogenicity; some tumors
occur in older patients than in typical patients with
show posterior enhancement; most tumors show cystic
f broadenomas.
parts within the tumor.
2. Phyllodes tumors have local recurrence rates o up to • On histology, the presence o nuclear polymorphia
46% and sometimes metastasize most likely to the lung. o the stromal cells is characteristic o malignant
The likelihood o metastasis depends on the histology. phyllodes tumors.
It is extremely rare in young patients but is described in up • Epithelial cells are not help ul to di erentiate benign
to 12% in case o the presence o sarcomatous elements. rom malignant phyllodes tumor—that is one reason
f ne needle aspiration (FNA) is not appropriate to
3. 0.3% to 1% o breast neoplasm are phyllodes tumors.
distinguish benign rom malignant phyllodes tumor,
4. Histology is the only way to di erentiate benign rom since FNA o tentimes does not include stromal and
malignant phyllodes tumors by showing polymorphia o epithelial cells.
stromal cells and the presence o sarcomatous elements. • The recurrence rate o malignant phyllodes tumors
is up to 46%. Distant metastasis by vascular spread
5. Phyllodes tumors in general show smooth margins, the
is being described in about 3% to 12%.
presence o internal cysts, septations, and hemorrhage.
Di erentiation between phyllodes tumor and
f broadenoma is not possible. Both f broadenoma and
phyllodes tumor show unspecif c contrast enhancement Suggested Readings
pattern.
Buchberger W, Strasser K, Heim K, Müller E,
Schröcksnadel H. Phyllodes tumor: f ndings on
mammography, sonography, and aspiration cytology in
Pearls 10 cases. AJR Am J Roentgenol. 1991;157(4):715-719.
Grebe P, Wilhelm K, Brunier A, Mitze M. MR tomography
• On mammography, phyllodes tumors are o ten well
o cystosarcoma phyllodes. A case report [in German].
def ned, round, and lobulated, and belong to the astest
Aktuelle Radiol. 1992;2(6):376-378.
growing breast masses.

222
Palpated lump in the left breast

1. What BI-RADS classif cation should be used


here?

2. Which ethnic groups o young women are at


increased risk o triple-negative breast cancer?

3. What is the next best imaging test?

4. This patient turned out to have a triple-negative


cancer with a basal subtype. What are the
tissue biomarker f ndings?

5. What type o biopsy would you recommend?

223
Young triple -negative cancer 1611
Case ranking/dif culty: Category: Diagnostic

Left lateral spot magni cation con rms an “ill-de ned mass” Left CC spot magni cation.
or “focal asymmetry” containing “segmental pleomorphic”
calci cations.

Answers
1. Di use increased density associated with “pleomorphic” Pearls
microcalcif cations indicates aggressive disease. • Triple-negative breast cancer is more common
in younger emales.
2. Young black women. TN breast cancer a ects younger
• ER-, PR-, HER2-.
women in general than regular invasive carcinoma.
• Metastasizes early (at a small size).
3. Depends on your approach, as strictly a ull • May present as circumscribed or partially obscured
mammographic workup should be completed be ore masses.
per orming an ultrasound exam. However, this patient
is unlikely to be having breast conservation, and may be
having neoadjuvant chemotherapy. Ultrasound can then
Suggested Readings
be the initial diagnostic exam, with axillary and internal
mammary node staging, prior to biopsy. MRI will then also Kojima Y, Tsunoda H, Honda S, et al. Radiographic eatures
need to be per ormed prior to neoadjuvant chemotherapy or triple negative ductal carcinoma in situ o the breast.
or surgery or staging purposes. PEM may be o assistance Breast Cancer. 2011;18(3):213-220.
in a patient with a palpable lump and suspicious ultrasound Kojima Y, Tsunoda H. Mammography and ultrasound
but not in a patient with dense breasts. eatures o triple-negative breast cancer. Breast Cancer.
2011;18(3):146-151.
4. Triple-negative breast cancer re ers to the negative Uematsu T. MR imaging o triple-negative breast cancer.
status o ER, PR, and c-ERB receptor (HER2). As more Breast Cancer. 2011;18(3):161-164.
subtypes are being ound, uture terms may include
quadruple-negative breast cancer, and so on.
5. A good-quality core biopsy is needed with larger cores,
as the HER2 overexpression will need to be redone on
the surgical specimen (which may be a ter neoadjuvant
chemotherapy).
224
First mammogram: MLO, ML, and CC view

1. What is the f nding on the mammogram?

2. Why is mammogram workup warranted be ore


ultrasound?

3. What is the best ultrasound method or


assessment o a small hypoechoic mass?

4. Why would it be inappropriate to classi y the


ultrasound lesion as BI-RADS 3?

5. What is the next step a ter the imaging?

225
Invasive ductal carcinoma 1001
Case ranking/dif culty: Category: Screening

Ultrasound of right breast demonstrating small “mass” with


“irregular” shape.

Pearls
• This is an example where a hypoechoic nodule can
be di f cult to be classif ed, in part due to small size.
The di erential diagnosis could be “complicated cyst”
versus solid “mass.”
• Other methods that could help to di erentiate cystic
Diagnostic mammogram of right spot compression CC view. rom solid lesion are elastography and the use o
Doppler ultrasound in particular in the presence o
Answers extensive f brocystic changes with multiple cysts.
• Elastography is used to semiquantitatively measure
1. Noted is a small mass right 11–12:00. sti ness o tissue by calculating the displacement o
2. Mammography workup with spot compression views each pixel relatively to the surrounding pixels in real
is still warranted to urther assess i the f nding on the time. A solid mass would be sti er than a cyst and
screening exam is real and also to better localize the elastography would subsequently result in di erent
lesion, and also to assess shape, margin, and density. signal.
This is, in particular, important i the f nding is not seen • Ultrasound Doppler, on the other side, can help to show
on ultrasound. the presence o vessels which would prove the presence
o solid mass and raises concern or the presence o
3. A combination o B-mode imaging duplex and possibly malignancy and biopsy is warranted.
elastography will have the highest specif city to determine
the need or biopsy and the appropriate classif cation as
BI-RADS 3 or BI-RADS 4. However, in most practices,
the B-mode images alone will be used to characterize Suggested Reading
the mass. Cho N, Jang M, Lyou CY, Park JS, Choi HY, Moon WK.
4. Because o the “irregular” shape and “indistinct” margin Distinguishing benign rom malignant masses at breast
on B-mode images. US: combined US elastography and color Doppler
US—in uence on radiologist accuracy. Radiology.
5. Ultrasound-guided biopsy would be the next step to 2012;262(1):80-90.
assess the suspicious f nding.

226
18-year-old female with palpable abnormality: what is the next step?

1. What are the criteria to call a lesion a simple


cyst?

2. What additional test would be help ul to


characterize the f nding?

3. What is the appropriate classif cation o the


cystic mass?

4. What is the next step?

5. What is the f nal BI-RADS classif cation?

227
“Complicated cyst” 1628
Case ranking/dif culty: Category: Diagnostic

Ultrasound with duplex demonstrating “complicated cyst” in the


left breast corresponding to palpable abnormality.

Answers
1. Simple cyst has to be “well circumscribed” and “oval or Pearls
round,” “anechoic” with “posterior acoustic shadowing.” • Cysts can be divided into “simple cysts” and
“complicated cysts.”
2. I layering can be shown, this is proo that it is not a
• Simple cysts are “round,” “well circumscribed,” and
mass but debris. Positive duplex does prove the presence
“anechoic” with “posterior acoustic enhancement” and
o intracystic mass and biopsy is recommended.
are BI-RADS 2 (“benign”) f nding.
3. This is the typical presentation o a “complicated cyst.” • All other cysts are called “complicated cysts” and the
All other terms are not BI-RADS descriptors. option is cyst aspiration; or i there are no signs o
intracystic mass, ollow up in 6 months—BI-RADS 3.
4. The options here would be cyst aspiration or 6-month
• I there is a def nite mass seen within a cyst, it should
ollow-up.
be called “complex mass” and biopsy should be
5. BI-RADS 3 (“probably benign”) would be an per ormed.
appropriate assessment.

Suggested Reading
Berg WA, Campassi CI, Io e OB. Cystic lesions o the
breast: sonographic-pathologic correlation. Radiology.
2003;227(1):183-191.

228
Screening mammogram—priors on left

1. What is the pertinent f nding?

2. What is the appropriate BI-RADS assessment


or the screening mammogram?

3. What are the possible consequences i


ultrasound does not show simple cyst?

4. What would you do i the aspiration uid


is bloody and the cyst has collapsed?

5. Where is the mass located?

229
New mass—complicated cyst 763
Case ranking/dif culty: Category: Screening

Diagnostic mammogram, left MLO spot Ultrasound, left lateral breast demonstrates Mammogram, left MLO view after
compression view. corresponding cluster of relatively simple cysts. cyst aspiration demonstrates
resolution of the small mass.

Answers
1. Noted is the interval development o a small mass in the Pearls
le t upper outer quadrant projecting on MLO view close • In case o new mass on mammogram, i corresponding
to the pectoralis muscle and on the CC view posterior ultrasound shows simple cyst—and the level o
depth laterally. conf dence is high that it correlates to the mammogram
f nding—f nding is BI-RADS 2 (“benign”) and patient
2. Any new mass like in this case requires diagnostic
can return to screening mammography.
workup and the assessment o the screening
• I the cyst is more complicated on the ultrasound,
mammogram is BI-RADS 0, incomplete exam. Patient
or i it is more uncertain and corresponding to the
needs to be recalled or additional workup. On the
mammogram f nding, cyst aspiration or short-term
screening mammogram, there is no need to describe
ollow-up (BI-RADS 3) is recommended—depending
shape and margin o the mass—this should be done
on the situation.
based on the spot compression views at the time o
• Simple cyst is def ned by a well-circumscribed,
diagnostic workup. To call the mass “indeterminate”
homogeneous, and anechoic mass with posterior
is appropriate on screening mammogram.
acoustic enhancement.
3. I the f nding is not a simple cyst, cyst aspiration or • Repeat mammogram a ter cyst aspiration should be
biopsy is recommended. Since, in this case, it was per ormed to prove resolution o the new mass as seen
most likely a cluster o two cysts, cyst aspiration was on mammogram.
per ormed f rst—cyst did collapse and subsequently the
mass was not seen any more on repeat, post-aspiration
mammogram.
Suggested Reading
4. Typical cyst-like uid can be discarded—yellow and Kopans DB, Meyer JE, Lind ors KK, Bucchianeri SS.
brownish uid. Any bloody uid, unless iatrogenic, Breast sonography to guide cyst aspiration and wire
should cause some concern. The pit all is that i the cyst localization o occult solid lesions. AJR Am J Roentgenol.
is aspirated, the lesion cannot be ound in the uture. 1984;143(3):489-492.
There ore, it is essential to leave a clip in that case.
5. The mass is located in the upper breast on the MLO view
and in the lateral breast on the CC view—based on the
rule o L it would be L(!)ower on the ML view because
it is L(!)ateral on the CC view and there ore 3:00 is the
best location as conf rmed on ultrasound.

230
Screening—asymptomatic

1. What is the BI-RADS category or this


screening exam?

2. What urther mammographic views do you


wish to do?

3. What is the di erential diagnosis o this


lesion?

4. I this lesion is NOT visible on ultrasound,


what would be your recommendations?

5. Given the mammographic appearances, what


risk o malignancy would you estimate this
lesion to have?

231
Mucinous carcinoma presenting as indistinct mass 688
Case ranking/dif culty: Category: Screening

Right CC spot magni cation view. Right lateral spot magni cation views. Ultrasound of mass shows that the mass is “irregular”with
Even with the improved resolution of “ill-de ned”margins, and containing low-level echoes.
the spot magni cation view, the mass
remains irregular and the margins of
the mass remain indistinct.

Answers is likely to require biopsy, and a stereotactic biopsy is the


1. This is an abnormal screening examination so that a quickest and most accurate way to establish a diagnosis
BI-RADS assessment o 0 should be rendered, and in a ocal asymmetry that is not seen on ultrasound.
urther workup ordered. At this stage, there is no clear
5. Based on the limited imaging so ar, this lesion could be
evidence o malignancy, and the outcome could still be a
benign or malignant and there ore lies in a wide range
simple cyst or f broadenoma. I you know that the prior rom 10% to <95% (BI-RADS 4B–4C)—BI-RADS 5th
mammogram was normal, this is a developing ocal
edition ACR.
asymmetry (or mass), which automatically raises your
concern about a developing malignancy.
2. The reason or doing a mediolateral rather than a Pearls
lateromedial mammogram is that the abnormality is in • This can be a challenging malignancy to diagnose
the lateral breast. Spot compression f lms are required because o its benign-appearing eatures on
to urther def ne the lesion rom the surrounding mammography and ultrasound.
glandular tissue. Many centers use spot magnif cation • It is very slow growing and may never metastasize in the
to increase the resolution o the examination. I this was patient’s li etime, according to molecular subprof ling.
a so t call, and you thought that the f nding is likely • Look or indistinct margins o a benign lesion in an
superimposition, then repeating the CC and MLO would elderly patient.
conf rm or re ute your hypothesis. However, it would not
help to urther characterize the lesion margins.
3. The lesion is so di f cult to characterize, but appears to Suggested Readings
be more o a mass than a distortion, so more likely an Bode MK, Rissanen T. Imaging f ndings and accuracy o
intraductal origin. Lobular cancers tend to present as a core needle biopsy in mucinous carcinoma o the breast.
vague asymmetry or distortion. Acta Radiol. 2011;52(2):128-133.
4. I the lesion is round and circumscribed on urther Laucirica R, Bentz JS, Khalbuss WE, Clayton AC, Souers
mammograms and you think the patient has either a cyst RJ, Moriarty AT. Per ormance characteristics o
or f broadenoma, and the patient has no prior exams, mucinous (colloid) carcinoma o the breast in f ne-needle
then a 6-month ollow-up may be appropriate. I prior aspirates: observations rom the College o American
f lms show no abnormality, this is a developing ocal Pathologists Interlaboratory Comparison Program in
asymmetry or a mass and should be urther worked up. Nongynecologic Cytopathology. Arch Pathol Lab Med.
You could do a problem-solving MRI or even positron 2011;135(12):1533-1538.
emission mammography (PEM) or similar nuclear Le-Petross H, Lane D. Challenges and potential pit alls
medicine scan. However, given the developing mass, this in magnetic resonance imaging o more elusive breast
carcinomas. Semin Ultrasound CT MR. 2011;32(4):342-350.
232
Screening mammogram and left MLmagni cation view:
any subtle abnormality?

1. Are there any additional f ndings seen on the


screening mammogram except small group o
calcif cations in the le t upper outer quadrant?

2. What is the workup o a group o calcif cations?

3. Is the pathology o lobular carcinoma in


situ (LCIS) concordant with a group o
calcif cations?

4. What is the relationship between LCIS and


invasive breast cancer?

5. What are potential consequences a ter diagnosis


o LCIS?

233
Lobular carcinoma in situ (LCIS) 998
Case ranking/dif culty: Category: Screening

Diagnostic mammogram, left ML magni cation view. Diagnostic mammogram, left CC MRI T1-weighted sequence after IV
magni cation view. contrast with index lesion in the left
breast and additional 8-mm mass in
the right central breast.

5. In general, most patients return to screening. In the ace o


the higher risk o malignancy, additional MRI screening is
recommended. Other option could be bilateral mastectomy.
I there is concern that there is additional suspicious
morphology, then excisional biopsy is recommended.

Pearls
• Atypical lobular hyperplasia (ALH) and LCIS are
associated with increased risk o breast cancer in the
MRI T1-weighted sequence, sagittal image, postcontrast, uture in both breasts, including invasive ductal and
demonstrating the mass in the right central breast. lobular carcinomas—by 3- to 4- old with ALH and
8- to 10- old increase with LCIS.
Answers • Both lesions are not considered a precursor to invasive
lobular carcinoma.
1. Noted is the group o indeterminate calcif cations in the • Management options include (1) li etime surveillance
le t upper outer quadrant—otherwise no ocal lesions. with MRI added to mammography, (2) local excision o
2. The most appropriate workup is to per orm a ML standard the lesion, and (3) bilateral prophylactic mastectomy.
view to better localize the calcif cations within the breast • In this particular case, breast MRI was per ormed
and or better planning o the subsequent stereotactic immediately a ter the diagnosis o LCIS was made,
biopsy. ML and CC magnif cation views are standard or which showed an additional contralateral 8 mm
workup o calcif cations. There is no indication or a MLO highly suspicious mass in the right breast, which on
magnif cation view at all. An exception could be that in ML subsequent MRI-guided biopsy was consistent with
projection it is impossible to cover the calcif cations due invasive ductal carcinoma.
to location in the very posterior breast. ML and CC plane • This case shows that mammogram with scattered
or magnif cation views is standard, since this is the best f broglandular tissue ailed to show an 8-mm invasive
way to demonstrate “milk o calcium” as a typical orm o ductal carcinoma, despite comparison with several old
benign calcif cations—which would not require biopsy. mammograms (not submitted or the book).

3. Yes—LCIS can present as group o calcif cations.


Although, in this case, MRI was per ormed, it is not Suggested Readings
standard o care at this time.
Oppong BA, King TA. Recommendations or women with
4. LCIS was initially thought to be a precursor or lobular lobular carcinoma in situ (LCIS). Oncology (Williston
invasive carcinoma. However, it was shown that this is Park). 2011;25(11):1051-6, 058.
alse and that lobular carcinoma is now considered a Sung JS, Malak SF, Bajaj P, Alis R, Dershaw DD, Morris EA.
risk actor or uture development o malignancy in both Screening breast MR imaging in women with a history o
breasts. Malignancy in the uture can be lobular or ductal. lobular carcinoma in situ. Radiology. 2011;261(2):414-420.
234
Patient with screening mammogram

1. What is the abnormality seen on the


mammogram?

2. I you see most likely arti act on a screening


mammogram, what would you do?

3. What other patient-related arti acts are seen


on mammograms?

4. What arti act could the abnormality be?

5. What would be an option i patient is still


in the o f ce?

235
Hair artifact 729
Case ranking/dif culty: Category: Screening

Screening mammogram, left CC view, demonstrating linear Repeat left CC view, with medially exaggerated scan eld
density in the posterior central breast. (LXCCM), demonstrates resolution of the density.

Answers
1. Noted is the linear density o le t posterior breast—near Pearls
chest wall. • When arti act, in this case hair arti act is suspected,
2. Mammogram should be classif ed as BI-RADS 0 patient needs to be recalled or “technical repeat.”
(incomplete exam) and patient should be called back • I the technologist does recognize the f nding at the
or “technical repeat.” That means the projection should time the patient is still there, the issue can be addressed
be repeated showing the arti act. Anything suspected o immediately.
causing the arti act, such as deodorant, powder, or hair
should be removed be ore the f lm is taken.
Suggested Reading
3. All these are related to the patient. However, arti acts
Hogge JP, Palmer CH, Muller CC, et al. Quality assurance
due to underexposure are more related to the setup o
in mammography: arti act analysis. Radiographics.
the machine (not enough mAs or kV) and not directly
1999;19(2):503-522.
related to the patient.
4. The linear area could represent hair arti act rom
overlying hair.
5. I patient is still in o f ce, there is no need to turn
the exam into a diagnostic study. The images can be
repeated and i the abnormality disappears and was due
to arti act, this is a “technical repeat” and the study can
remain screening exam.

236
Screening—asymptomatic

1. What is the BI-RADS category or this


screening exam?

2. What is the next best examination you


recommend?

3. What is the best description or the distribution


o calcif cations?

4. What is the most likely pathology in this case?

5. This case is a type o “developing” ocal


asymmetry. What is the likely risk o
malignancy?

237
Lobular cancer presenting as asymmetry and benign calci cations 687
Case ranking/dif culty: Category: Screening

Left CC spot magni cation views. The nding was palpable, Left lateral spot magni cation views show the asymmetry
and so a BB marker is seen over the palpable mass. to be palpable. Some amorphous calci cations are also seen
to be associated with this asymmetry. These are usually the
reason biopsy is prompted.

a di erent process, then “regional” may be a better


description.
4. Both lobular and tubular cancers present with
asymmetries associated with distortion.
5. Based on the “developing ocal asymmetry” alone, there
is a risk o malignancy o approximately 20%, but may
be up to 50% or more when there are new calcif cations
with it. This is the case in this particular example. These
f ndings should prompt you to recommend biopsy.
Left breast targeted ultrasound to palpable thickening shows
vague decreased echogenicity within the glandular tissue and
also some complex acoustic shadowing. Pearls
• Lobular cancer is di f cult to diagnose on a mammogram.
Answers • Findings are o ten very subtle.
• Distortion (especially in one view) is a common
1. There is a subtle asymmetry in the le t breast with presentation.
possible distortion. There are possible calcif cations • Beware o the “shrinking breast.”
associated with this asymmetry that are hard to evaluate
on these f lms.
2. Further mammographic workup is indicated with an Suggested Readings
orthogonal mammogram in the lateral plane, plus spot Choi BB, Kim SH, Park CS, Cha ES, Lee AW. Radiologic
magnif cation views to urther characterize the calcif c f ndings o lobular carcinoma in situ: mammography and
particles. Ultrasound is a good adjunct examination, ultrasonography. J Clin Ultrasound. 2011;39(2):59-63.
but best per ormed when the mammographic workup Heil J, Buehler A, Golatta M, et al. Do patients with invasive
is complete. lobular breast cancer benef t in terms o adequate change
3. I you think these calcif cations are likely malignant, in surgical therapy rom a supplementary preoperative
then a more suspicious descriptor modif er such breast MRI? Ann Oncol. 2012;23(1):98-104.
as “segmental” should be used. I you think these Kim SH, Cha ES, Park CS, et al. Imaging eatures o invasive
calcif cations are likely benign, but associated with lobular carcinoma: comparison with invasive ductal
carcinoma. Jpn J Radiol. 2011;29(7):475-482.
238
45-year-old woman with screening mammogram and diagnostic workup

1. What is the f nding on the mammogram?

2. How can the mass be described on spot


compression view?

3. Where is the mass located?

4. What is the description o the ultrasound


f nding?

5. What are the descriptors on ultrasound that


are suggestive or malignancy?

239
Ductal carcinoma in situ (DCIS) 758
Case ranking/dif culty: Category: Diagnostic

Ultrasound directed to the mass shows suspicious hypoechoic


mass as best seen on tissue harmonic imaging.

Screening mammogram, Spot compression right CC view


right MLO view. con rms mass right lateral breast,
anterior depth close to the nipple.

Answers
1. There is a subtle mass seen on the right upper outer
quadrant, which can be conf rmed with spot compression
views. Nodule is not a BI-RADS term. On power Doppler with vocal fremitus images, there is lack of
color artifact within the area of concern.
2. The mass is irregular in shape, has “partially obscured
and indistinct margins,” and is o “equal density” to the
surrounding f broglandular tissue.
• I lesion is near isointense on ultrasound on B-mode
3. Mass is located superior to the MLO view and lateral to images, harmonic imaging improves lesions
the CC view, which results in 10:00. It is also located in conspicuity with higher image contrast and the
the anterior depth. surrounding atty tissue appears less dark than the
4. The mass is “irregular” in shape with “spiculated” margin targeted lesion.
and “hypoechoic” in comparison with the at tissue. There • Second ultrasound technique to di erentiate lesion
is only minimal posterior acoustic shadowing. rom surrounding at is called “vocal remitus
technique,” which uses acoustic vibrations rom the
5. All descriptors that are not round or oval in shape are chest wall to create color arti acts in normal tissue but
suspicious, including “irregular”; all margin descriptors not within the tumor on power Doppler ultrasound.
including “indistinct” or “angular,” “microlobulated,” or
“spiculated” are o concern. Stavros et al. uses the term
“taller than wide”; BI-RADS uses the terms “parallel”
and “not parallel.” Suggested Readings
Kim MJ, Kim JY, Yoon JH, et al. How to f nd an isoechoic
lesion with breast US. Radiographics. 2011;31(3):663-676.
Stavros AT, Thickman D, Rapp CL, Dennis MA, Parker
Pearls SH, Sisney GA. Solid breast nodules: use o sonography
• Any developing density is potentially suspicious— to distinguish between benign and malignant lesions.
i the density is seen in two plains and has convex Radiology. 1995;196(1):123-134.
margin, it is called “mass.”

240
Screening—asymptomatic

1. What is the BI-RADS category or this


screening exam?

2. What is the background breast density?

3. I you want to recall this patient, what are your


recommendations?

4. What type o biopsy should be per ormed?

5. Pathology shows orid in ammatory change


around a hematoma. There is no history o
trauma. What do you do next?

241
Invasive
Small mucinous
ductal carcinoma—palpable
carcinoma (special typelump
of IDC) 686
000
(was
Case not originally
ranking/dif culty: palpable at screening) Category: Diagnostic

Case ranking/dif culty: Category: Screening

Left mediolateral spot magni cation view for characterization Orthogonal view of the mass seen on the mammogram. The mass
of margins of mass—soft “spiculation” is seen, particularly in appears relatively circumscribed in one plane. Take the ultrasound
the plane toward the nipple. appearances in context with the mammogram to come to your
nal assessment.

ultrasound can be per ormed looking or second lesions,


but there is a signif cant alse-positive rate that will need
to be addressed.
4. Even i palpable, ultrasound-guided biopsy is more accurate
giving a better chance o accurately sampling a mass.
Sometimes, the lesion is not easily visible on ultrasound,
and then you have to resort to stereotactic core biopsy.
5. In this case, it is possible that the imaging f ndings are
due to at necrosis, but with the spiculation I would give
MRI for extent. Note signal void from recent biopsy within the it a BI-RADS 5 lesion, as I will not accept a benign
enhancing mass. result. The pathology is o a hematoma and in ammatory
Answers change. The imaging does not f t with this. There ore,
the lesion has been under-sampled, and a larger-core
1. This is a screening exam and thus should be 0; needs biopsy could be per ormed. The f nding is palpable and
additional workup, negative, or benign. Although you are so needle localization should not be required.
most likely to be correct to think that this is a malignant
neoplasm, BI-RADS recommends that you ully
work up this lesion mammographically, and possibly Pearls
sonographically, be ore you come to a f nal assessment, • Spot magnif cation views are use ul to assess the
and make a recommendation about management. margins o a mass, as well as or calcif cations.
Some benign lesions, including at necrosis and prior • Ensure that imaging f ndings correlate with pathology
lumpectomy, can mimic these appearances. at biopsy.
2. Some readers would classi y this breast density as almost
entirely at, and some scattered f broglandular densities. Suggested Readings
There is marked disagreement between readers when it
comes to breast density. The assessment o breast density Brenner RJ, Bassett LW, Fajardo LL, et al. Stereotactic core-
is currently subjective, until we have a reliable and needle breast biopsy: a multi-institutional prospective
reproducible automated way o def ning breast density. trial. Radiology. 2001;218(3):866-872.
Flowers CI. Breast biopsy: anesthesia, representative
3. For most questions these days, tomosynthesis needs to sampling and rad/path concordance. Applied Radiol.
be included. I you have tomosynthesis at screening, you 2012;41(1):9-14.
likely have all the in ormation you are going to get about Richter-Ehrenstein C, Müller S, Noske A, Schneider A.
the mass margins, but i there are calcif cations, currently Diagnostic accuracy and prognostic value o core biopsy
spot magnif cation views cannot be beaten. Ultrasound is in the management o breast cancer: a series o 542
the next modality to interrogate the mass. Whole breast patients. Int J Surg Pathol. 2009;17(4):323-326.
242
Palpable lump in the right breast

1. What is the BI-RADS category or this


diagnostic exam?

2. What should be the next diagnostic imaging


exam?

3. What are the clinical eatures o a sebaceous


cyst?

4. What type o intervention would you


recommend?

5. What is the malignant trans ormation rate


o sebaceous cysts over 10 years?

243
Well-de ned mass in danger area—epidermal inclusion cyst 683
Case ranking/dif culty: Category: Diagnostic

Note the mass immediately beneath the Similar mass in a young woman who is breast-
dermis—subdermal cystic lesion. Strictly this is a feeding. In this case, it appears as a complex
complicated cyst, as there is echogenic material mass BETWEEN the layers of skin, and therefore
within the lesion. However, the ndings are of dermal origin. The second “layer” of skin
trumped by the clinical ndings of an epidermal being displaced backward by the increasing
inclusion cyst correlate at that site. intradermal mass. This is a sebaceous cyst.

Right cleavage view shows


position of mass more clearly.

Answers
1. This is a diagnostic examination and you should not give Pearls
a BI-RADS 0 assessment, even i you are awaiting to • Sebaceous cysts can simulate a cancer occurring in the
do the ultrasound. As the f ndings are o a benign lump in erior mammary old or in the medial breast on the
and the patient was shown to have a sebaceous cyst on CC view.
clinical examination, you could even stop here and not • They are harmless.
do an ultrasound. • Physical examination with the f nding o a skin mass
2. Tomosynthesis may include the lesion, but special views with a punctum is diagnostic o a sebaceous cyst.
are needed when the lesion in the lower inner quadrant is • Ultrasound may be required or an epidermal
so close to the sternum, and cleavage views are the best inclusion cyst.
way o demonstrating this type o lesion. Once worked
up, targeted ultrasound can be per ormed i there is no
clinical evidence o sebaceous cyst. MRI and PEM are Suggested Readings
overkill or a skin lesion.
Giess CS, Raza S, Birdwell RL. Distinguishing breast skin
3. A sebaceous cyst is easily picked up on physical lesions rom superf cial breast parenchymal lesions:
examination, and i your patient is still in the diagnostic criteria, imaging characteristics, and pit alls.
examination room, then a quick physical exam may Radiographics. 2011;31(7):1959-1972.
obviate the requirement o urther workup. A lump is Kalli S, Freer PE, Ra erty EA. Lesions o the skin and
usually f xed to the skin, but can become variably deep. superf cial tissue at breast MR imaging. Radiographics.
I the lump is not attached to the skin, think o a di erent 2010;30(7):1891-1913.
di erential diagnosis. Redness can occur i a sebaceous
cyst becomes in ected, but is not a normal f nding.
A rash in a dermatome distribution is characteristic
o herpes zoster in ection.
4. No intervention is required apart rom reassuring the
patient that there is no evidence o malignancy.
5. They have a very low risk o malignant trans ormation.
Development o a squamous cell malignancy within an
epidermal inclusion cyst is very rare.
244
Recent onset of spontaneous bloody nipple discharge in the right breast

Diagnostic Ultrasound, B-mode image demonstrating prominent


mammogram, right ducts and a large area of shadowing posterior depth.
spot compression MLO
view.

1. What is the appropriate workup or bloody


nipple discharge?

2. Would the ductogram have been help ul in this


case?

3. What are the technical reasons or an


insu f cient ultrasound?

4. What is the most suspicious orm o nipple


discharge?

5. Ultrasound image was submitted by technologist


as prominent ducts—anything else?

245
49-year-old patient with bloody nipple discharge 291
Case ranking/dif culty: Category: Diagnostic

MRI axial T1-weighted image after IV contrast after subtraction


demonstrates large area of “non–mass-like enhancement”
retroareolar and also posterior right breast. MRI sagittal T1-weighted image after IV contrast demonstrates
large area of enhancement retroareolar and also posterior breast.

Answers
1. The f rst step is standard MLO and CC views o the Pearls
symptomatic side and then additional SC views o the • This case demonstrates how easy it is to misjudge even
retroareolar tissue. Next standard routine step is an the presence o a large invasive ductal carcinoma in
ultrasound directed to the right retroareolar breast— dense f broglandular tissue.
however, this case also demonstrates the remainder o • It is crucial to set the ocal zone o the ultrasound
the right breast i nothing is ound. MRI could be an machine deep enough to be able to assess the breast
additional test i the patient is symptomatic and imaging parenchyma deep to the level o the chest wall.
does not f nd any abnormality. Ductogram can be help ul, • Ultrasound was originally misjudged as “presence o
but it is in general per ormed i the discharge is rom one dominant ducts.”
or two ducts and might show intraductal f lling de ect. • Patient received MRI due to discrepancy between
ultrasound and mammogram and clinical concern
2. The nipple discharge in this case is due to large invasive
based on bloody nipple discharge and palpable mass,
ductal carcinoma that erodes the ductal system. An
which did show the malignancy.
intraductal f lling de ect like seen with papilloma is
unlikely here. Thus, the ductogram would likely not have
helped to solve the situation.
3. An imper ect ultrasound can be due to incorrect gain, Suggested Reading
positioned ocal zone or depth Baker JA, Soo MS, Rosen EL. Arti acts and pit alls in
4. The most suspicious orm o discharge is clear or bloody sonographic imaging o the breast. AJR Am J Roentgenol.
discharge, spontaneously rom one duct in one breast. 2001;176(5):1261-1266.
Typical discharge due to proli erative f brocystic changes
is milky, greenish, or brownish bilateral discharge on
pressure. Bloody nipple discharge during pregnancy or
lactation is less o a concern because o increased blood
ow to the parenchyma.
5. This is a case o a large mass not well appreciated
on the images. Focus o the exam was directed to the
retroareolar breast and the more deeper parts were not
well examined and the large mass was missed in the
deeper tissue.

246
Screening—asymptomatic

1. What is the BI-RADS category or this


screening exam?

2. What structures are at risk in a patient with a


possible posterior placed malignancy?

3. What are the appropriate tests during


diagnostic workup in this patient?

4. What is the likely position o this mass based


on the distribution o malignant lesions in the
breast?

5. How do the majority o posterior located


malignancies present?

247
One that nearly got away—position 674
Case ranking/dif culty: Category: Screening

MRI was performed as the lesion was close to the


chest wall. The extent is much greater than originally
appreciated on mammography or ultrasound.
“Ill-de ned mass”with ( ne) “ductal extension.”
LML spot view at diagnostic
workup—patient now states 4. The distribution o both benign and malignant lesions
she can feel something in the are similar in requency in the di erent areas, as ollows:
left breast.
a. Upper outer quadrant, 45%
b. Subareolar, 25%
c. Upper inner quadrant, 15%
d. Lower inner, 10%
e. Lower outer, 5%
Quadrant location does not predict benign versus malignant
pathology (data and quotation used with permission rom
E. A. Sickles, MD).
5. Posterior lesions are not in requently ound by the
woman when bathing. These are easily missed because
o their position at mammography.
Second-look ultrasound showed multiple other masses, which
were also biopsied and con rmed malignancy.
Pearls
Answers • Watch edges o f lms when otherwise appears normal.
1. This is a screening examination, which should generate • Watch danger areas.
a BI-RADS 0, 1, or 2 assessment. As there is a f nding, • Be prepared to recommend additional diagnostic views
which is not typically benign, it should be classif ed as to work up this apparent f nding.
BI-RADS 0 and recalled or urther workup. • Computer-aided detection (CAD) can assist the
radiologist pointing out an area to second look.
2. With a lesion being so ar posterior, it has potential or
pectoral muscle involvement. Surgery to breast cancer
includes cutting away the pectoral ascia when the mass
is posterior. However, that is more likely to cause a Suggested Readings
poorer cosmetic outcome, with skin tethering. MRI is Brem RF, Baum J, Lechner M, et al. Improvement in
use ul to see i there is muscle involvement, mani ested sensitivity o screening mammography with computer-
as enhancement o the muscle. aided detection: a multi-institutional trial. AJR Am J
Roentgenol. 2003;181(3):687-693.
3. Sometimes, it is not possible to f nd the lesion on
Skaane P, Kshirsagar A, Ho vind S, Jahr G, Castellino RA.
orthogonal mammographic projections, and there ore
Mammography screening using independent double reading
di f cult to f nd on ultrasound, and MRI could be use ul to
with consensus: is there a potential benef t or computer-
determine the location o the mass.
aided detection? Acta Radiol. 2012;53(3):241-248.

248
42-year-old woman with new group of calci cations in the left upper
outer quadrant—what is the next step?

1. What is the def nition o DCIS?

2. What is the prevalence o DCIS in a screening


population?

3. What are the characteristics o high-grade


DCIS?

4. What is the typical appearance o DCIS on


MRI?

5. What is the value o MRI in regard to DCIS?

249
High-grade ductal carcinoma in situ (DCIS) 117
Case ranking/dif culty: Category: Diagnostic

MRI after IV contrast demonstrating linear area of


Diagnostic mammogram of left ML Diagnostic mammogram of increased enhancement in the lateral superior breast.
magni cation view demonstrating left CC magni cation view It exceeds the extent of the small group seen on the
group of “round and oval” demonstrating group of “round mammogram.
calci cations. and oval” calci cations.

Answers
1. Ductal carcinoma in situ (DCIS; synonyms: intraductal Pearls
carcinoma, noninvasive carcinoma) is a orm o • MRI has gained reputation over the past years or
malignant trans ormation o epithelial cells lining the excellence in the detection o DCIS (sensitivity near 90%).
mammary ducts and lobules. The proli erating cells are • Low-grade DCIS might be missed because o lack o
conf ned by an intact basement membrane. vascular neogenesis; however, clinical signif cance o
2. The overall prevalence is 32.5 per 100,000 women; the low-grade DCIS is controversial.
rate is as high as 88 per 100,000 women between the • MRI, however, shows o ten better the extent o DCIS
ages o 50 and 54. than mammography and there ore is help ul or
presurgical planning.
3. High-grade DCIS on mammography most likely • Most common MRI f nding in DCIS is “clumped, non–
demonstrates calcif cations as well, although there is mass-like enhancement” in ductal or “linear” distribution.
a higher likelihood o the presence o “asymmetry” • Enhancement kinetics vary and can include early
or “mass,” in comparison with low-grade DCIS. On enhancement, as well as delayed enhancement.
MRI, high-grade DCIS will more likely represent with
vascular neogenesis and there ore will show contrast
enhancement and is well seen. Suggested Readings
4. Most requently, DCIS appears on MRI as “non–mass- Kuhl C. Why do purely intraductal cancers enhance on breast
like clumped” enhancement. The kinetics o contrast MRI images? Radiology. 2009;253:281-283.
enhancement varies between “early enhancement and Mossa-Basha M, Fundaro GM, Shah BA, et al. Ductal
washout kinetics” and also “mild early enhancement carcinoma in situ o the breast: MRI imaging f ndings
with increasing kinetics” over time. with histopathologic correlation. Radiographics.
2010;30:1673-1687.
5. MRI is help ul to reduce the chance to obtain positive
Vag T, Baltzer PA, Renz DM, et al. Diagnosis o ductal
margins a ter surgery and also to detect multi ocal
carcinoma in situ using contrast-enhanced magnetic
disease (as seen in retrospect in 23% o patients).
resonance mammography compared with conventional
mammography. Clin Imaging. 2008;32(6):438-442.

250
Prior lumpectomy for breast cancer—routine surveillance mammogram,
prior on left

1. Given that the breasts appear normal, what is


the most appropriate BI-RADS category to use?

2. What is the next best imaging test?

3. The ultrasound shows a solid node. What is the


next imaging test?

4. The node shows metastatic adenocarcinoma on


FNA. What is the next imaging test?

5. What is the di erential or lymphadenopathy


on mammography?

251
Recurrence discovered via axilla 614
Case ranking/dif culty: Category: Diagnostic

Ultrasound of the right axilla shows a nodule arising in the cortex Ultrasound of the right axilla. In this view, the cortex appears
of the lymph node. The lymph node has an irregular margin. The smooth but thickened—beyond the 3-mm threshold for biopsy.
nodule is the most suspicious part of the lymph node, and the
best place to target your biopsy.
o unknown origin. MRI is likely to show any recurrent
ocus within the breast. I the breasts are very dense and
Answers
enhancing, or i the patient is unable to have an MRI, then
1. As this is a diagnostic exam and you are going to PEM or BSGI may assist in f nding the primary.
recommend a biopsy, a BI-RADS 4 is the best category
to use, as it gives the message o a suspicious f nding. 5. A ter metastatic cancer, lymphoma is the most obvious
Some people use BI-RADS 0 in the situation o a cause or lymphadenopathy. Localized rupture o
diagnostic exam, which requires an ultrasound scan silicone implants can also cause enlarged nodes. Sarcoid
that cannot be done at the same time. The downside is normally picked up incidentally on a chest radiograph.
to this is that the degree o suspicion or urgency is not In ections are a cause, including “cat-scratch” disease.
conveyed in the same way. Some people also argue that Brucellosis is a known cause.
you should use BI-RADS 1 as the breasts are negative
or malignancy, but there is a coincidental f nding in the
axilla, which requires action. Pearls
2. There is nothing to be seen in the breast to indicate an • Don’t orget to notice change in axilla.
axillary recurrence, so diagnostic f lms or tomosynthesis • Not all breast cancer recurrence is visible within the
is unlikely to give any extra in ormation at this point. breast.
MRI may be required later ollowing interrogation o • Workup or a suspicious node with no mammographic
the axillary node. As yet we have no proo that this is an f nding.
axillary recurrence—need cytology or histology. PET/
CT may be required later i this is proven to be breast
cancer recurrence, but not at this stage. Suggested Readings
3. Establishment o recurrent malignancy needs to be Barton SR, Smith IE, Kirby AM, Ashley S, Walsh G,
made be ore we recommend additional expensive Parton M. The role o ipsilateral breast radiotherapy
tests. FNA cytology is su f cient to detect recurrent in management o occult primary breast cancer
malignancy; however, i possible, a small-gauge core presenting as axillary lymphadenopathy. Eur J Cancer.
biopsy will give tissue or biomarkers as well and guide 2011;47(14):2099-2106.
any urther treatment. I cancer is proven in the nodes, Ho A, Morrow M. The evolution o the locoregional therapy
and the breasts are dense, PEM is an alternative way to o breast cancer. Oncologist. 2011;16(10):1367-1379.
determine lesions in breasts. Ruano Pérez R. Incidence o axillary recurrence a ter a
negative sentinel lymph node result in early stages o breast
4. This is where MRI is probably the best test, just as in
cancer: a 5-year ollow-up. Rev Esp Med Nucl Imagen Mol.
patients who present with metastatic axillary lymph nodes
2012;31(4):173-177.
252
Routine screening mammogram, priors on left

1. What is the f nding on the current screening


mammogram?

2. In case the density is not palpable and not


visible on ultrasound, what is the next step?

3. I stereotactic biopsy is not technically


easible, what is the next step?

4. Where is the abnormality located?

5. What would be the correct description o


the likely f nding on ultrasound?

253
Invasive ductal carcinoma 387
Case ranking/dif culty: Category: Screening

Diagnostic mammogram, left spot compression MLO view Ultrasound with duplex of left breast demonstrating no increased
demonstrates development of “focal asymmetry” with subtle ow in the mass.
“architectural distortion.”

MRI, T1-weighted postcontrast image, demonstrates spiculated MRI, T2-weighted image, demonstrates corresponding low-signal
mass in the left upper outer quadrant. mass in the left upper outer quadrant.

Answers 5. The BI-RADS lexicon has also descriptors or


1. Noted is the development o subtle “ ocal asymmetry” ultrasound f ndings—in this case, this is a “hypoechoic
and “architectural distortion” in the le t upper outer mass” with “irregular” shape and “spiculated” margin
quadrant best seen on the MLO view. and “posterior acoustic shadowing.”

2. In this case, f nding can be classif ed as BI-RADS 4 and


stereotactic biopsy should be attempted. Since the mass
that has developed in the interval is “spiculated,” it could Pearls
be even argued that it could be classif ed as “highly • I mammogram raises concern or the presence o
suspicious,” BI-RADS 5. The consequence would be architectural distortion, workup should include spot
that any benign pathology, or example “ ocal f brosis,” compression views and ultrasound.
would be considered as being not concordant and patient • I ultrasound cannot demonstrate corresponding
had to go to surgery. abnormality, stereotactic biopsy should be per ormed.
• Remember that prior surgery, or even large-bore core
3. Any f nding seen on two mammogram projections can
biopsies, may also explain the presence o architectural
be excised, since f nding can be marked with needle
distortion.
localization.
4. Since it is lateral on the CC view and superior on
the MLO, it would be slightly lower on an ML view; Suggested Reading
there ore, while located in the upper outer quadrant, its Piccoli CW, Feig SA, Palazzo JP. Developing asymmetric
more precise location is about 2:00 then 12:00 to 1:00. breast tissue. Radiology. 1999;211(1):111-117.
Take home point: anything L(!)ateral on CC view
is L(!)ower on ML in comparison with MLO view.
254
31-year-old woman with palpable abnormality in the left breast

1. What does PASH stand or?

2. What is the typical appearance o PASH


on mammography?

3. What is the typical appearance o PASH on


sonography?

4. What is the consequence o a biopsy showing


PASH?

5. What can PASH be con used with by the


pathologist?
255
Pseudoangiomatous stromal hyperplasia 413
Case ranking/dif culty: Category: Diagnostic

Ultrasound directed to the area of concern demonstrates well- Ultrasound directed to the area of concern demonstrates well-
circumscribed mass. circumscribed mass with some increased ow.

Answers
1. Pseudoangiomatous stromal hyperplasia.
Pearls
• PASH is benign proli erative change and can appear
2. PASH presents, in general, as a benign-appearing mass on mammograms as “ ocal asymmetry,” “architectural
or as a ocal asymmetry. distortion,” calcif cations, or without any abnormality
3. PASH has no specif c morphological eatures and is at all.
ound in up to 25% as incidental f nding on breast • Patient with sonographic f nding consistent with
biopsies. However, it appears most requently on well-circumscribed oval hypoechoic mass, like in this
ultrasound as a benign mass with the appearance o particular case, can also be concordant with PASH.
a f broadenoma or a hypoechoic area. • PASH is identif ed as an incidental f nding in as many
as 25% o breast biopsies.
4. I the imaging is concordant with the benign diagnosis • I the imaging f ndings are concordant with the
o PASH, patient can return back to screening diagnosis o PASH, then it is appropriate to return the
mammogram. For example, i PASH is the diagnosis patient back to screening.
a ter stereotactic biopsy o “pleomorphic,” “highly • It is important to remember that i there are suspicious
suspicious calcif cations” (BI-RADS 5), excisional eatures seen on imaging, excision o the lesion is
biopsy is recommended because PASH might be only warranted.
an incidental f nding. • Angiosarcoma can be con used with PASH at
5. Angiosarcoma can be con used histologically with histology. Angiosarcoma requires surgical treatment
PASH. While PASH is not a high-risk lesion and not +/− chemotherapy.
being related to subsequent development o malignancy,
angiosarcoma is considered malignant tumor and
requires wide excision and chemotherapy.
Suggested Reading
Hargaden GC, Yeh ED, Georgian-Smith D, et al.
Analysis o the mammographic and sonographic
eatures o pseudoangiomatous stromal hyperplasia.
AJR Am J Roentgenol. 2008;191(2):359-363.

256
Routine screening mammogram - (priors on the left)

1. What is the pertinent f nding?

2. Which should be the most important comparison


year reading screening mammogram?

3. What is the other reason and why comparison


with 2-year prior mammogram is essential?

4. Where is the suspicious mass located?

5. I you believe the ultrasound is a “complicated


cyst,” what could you do?

257
Invasive ductal carcinoma 398
Case ranking/dif culty: Category: Screening

Diagnostic mammogram, right MLO Diagnostic mammogram, right CC spot Gray-scale ultrasound demonstrating
spot compression view demonstrating compression view demonstrating new corresponding “hypoechoic” mass in the right
new mass. mass. medial superior breast with “irregular” shape.

Answers
1. Mass in the superior medial right breast that is new since
Pearls
prior mammogram. The mass in the upper outer quadrant • Any developing mass remains concerning, unless it can
is stable since 2 years and there ore benign. be explained by simple cyst or other benign f nding.
• Even i the ultrasound f nding shows corresponding
2. Two years prior mammogram is the key image to circumscribed solid mass with smooth margins, the
compare a screening mammogram with. Any well- f nding has to be biopsied i it correlates to a new mass
circumscribed mass stable since 2 years or any seen on mammogram, since it can be assumed that it
asymmetry without distortion, stable since 2 years, can has grown in the past.
be considered to be benign. Remember, we use 2-year
time period to ollow “probably benign” lesions be ore
converting the assessment into “benign.”
Suggested Reading
3. Some breast cancers grow relatively slow. Average
double time o breast cancer cells is about 90 days. D’Orsi CJ, Bassett LW, Berg WA, et al. Breast Imaging
There ore, it is recommended to look back 2 years— Reporting and Data System: ACR BI-RADS–Mammography.
this will improves sensitivity. 4th ed. Reston, VA: American College o Radiology; 2003.

4. Lesion is located in the medial breast and in the central


breast at the level o the nipple as seen on the MLO
view. There ore, it will be higher on the ML view in
comparison with the MLO view because it is located
in the medial breast. Thus, 2:00 is correct.
5. Cyst aspiration would be an appropriate f rst step. I the
cyst pops and disappears and typical cyst-like uid is
aspirated, the uid can be discarded repeat mammogram
should be obtained to prove resolution o the new mass.
I the uid does show bloody uid, a clip marker should
be placed, the aspirate should be sent or cytological
analysis, and depending on the outcome, the patient
might be sent or surgical excision.

258
Screening mammogram and diagnostic workup

1. What is the f nding on the mammogram?

2. Why is “ ocal asymmetry” better descriptor


than nodular density?

3. Where are most breast cancers ound in the


breast?

4. What would be the appropriate assessment i


ultrasound is normal and f nding is new?

5. What is the signif cance o the ultrasound


f nding?

259
Asymmetric density 397
Case ranking/dif culty: Category: Diagnostic

Diagnostic mammogram, left MLO spot compression view Diagnostic mammogram, left CC spot compression view
demonstrates subtle focal asymmetry. demonstrates subtle focal asymmetry.

Pearls
• This case illustrates that any “ ocal asymmetry”—
especially in the medial breast—should raise concern,
since that area in general does not contain much
f broglandular tissue.
• Other problem zone in the breast is the so-called milky
way, the area behind the f broglandular tissue on MLO
or CC view.

Suggested Reading
Gray-scale ultrasound demonstrates small mass. Brown M, Eccles C, Wallis MG. Geographical distribution
o breast cancers on the mammogram: an interval cancer
Answers database. Br J Radiol. 2001;74(880):317-322.

1. Given its concave margin, the BI-RADS term “ ocal


asymmetry” could be used in place o “mass” which
would have a convex margin.
2. Nodular density is not part o the BI-RADS lexicon.
3. Most breast cancers are located in the upper outer
quadrant. This is simply due to the act that there is the
most tissue, in general.
4. BI-RADS 4, i this is a new f nding, would be appropriate
assessment—i it is an asymmetry on f rst screening
mammogram, BI-RADS 3 would be appropriate.
5. Ultrasound f nding does correlate to the mammogram
and ultrasound-guided biopsy is recommended.

260
Palpable lump in the right breast

1. What is the BI-RADS category or this


diagnostic exam?

2. What is the likely underlying pathology?

3. What is the risk o lymph node spread in


malignant phyllodes tumors?

4. What type o biopsy should be per ormed?

5. The biopsy comes back as dense f brous


tissue with no epithelium. What is your
recommendation?

261
Palpable phyllodes tumor 676
Case ranking/dif culty: Category: Diagnostic

Ultrasound—the mass appears “bi-phasic” on ultrasound, with an Doppler ultrasound shows prominent vascular channels, but
echogenic upper portion, and a more hypoechoic portion below. without a speci c characteristic distribution.

Answers to in orm the surgeon who is per orming the procedure


1. This is not a screening exam, and there ore not BI- that you suspect a phyllodes tumor, and then they will
RADS 0. Phyllodes tumors, i small, can simulate classic take a healthy ormal margin, rather than “shelling it
f broadenomas, and can look benign. This lesion is large out” o the breast tissue.
and atypical on ultrasound; there ore, a BI-RADS 4:
suspicious assessment is appropriate.
2. While occasionally cysts and sarcoid appear this way, the Pearls
size and lobulated eatures narrow down the di erential • Mainly a ects premenopausal emales.
to f broadenoma or phyllodes tumor. Occasionally a • Has benign and malignant spectrum.
special type o IDC variant can give these appearances, • May have precursor lesion o f broepithelial lesion.
including mucinous, colloid/medullary carcinoma. • Treatment by wide excision.
• High risk o local recurrence o 20 to 30%.
3. Although extremely rare and said to be “locally”
malignant, metastases rom malignant phyllodes still can
occur—but very rare indeed.
Suggested Readings
4. A combination o palpation and ultrasound is probably
the best way to direct the sampling o a tumor o this Abe M, Miyata S, Nishimura S, et al. Malignant
size. Multiple samples rom di erent areas o the mass trans ormation o breast f broadenoma to malignant
should be taken to give the pathologist a good chance at phyllodes tumor: long-term outcome o 36 malignant
making the diagnosis. Stabilizing a mobile tumor using phyllodes tumors. Breast Cancer. 2011;18(4):268-272.
f nger pressure is a use ul technique to learn, as when Gould DJ, Salmans JA, Lassinger BK, et al. Factors
you are already holding the probe in one hand and the associated with phyllodes tumor o the breast a ter core
biopsy device in the other, the mass may move under needle biopsy identif es f broepithelial neoplasm. J Surg
the probe, and satis actory sampling may sometimes Res. 2012;178(1):299–303.
be challenging. FNAC may have a role in centers with Jang JH, Choi MY, Lee SK, et al. Clinicopathologic risk
expert breast cytopathologist. actors or the local recurrence o phyllodes tumors o the
breast. Ann Surg Oncol. 2012;19:2612-2617.
5. There are two main options. You need more tissue, so
move to a larger lesion, or example, to an 8-gauge
vacuum-assisted needle rom a 14-gauge core biopsy.
I you have had no success with vacuum-assisted biopsy,
then surgical excision is the next logical step. You need

262
79-year-old woman with history of left lumpectomy several years ago

1. What is the appropriate workup o a patient


a ter lumpectomy?

2. What is the role o ultrasound a ter


lumpectomy?

3. I there is questionable recurrent new density


in the lumpectomy bed, what is the best exam?

4. What could decrease sensitivity o the MRI in


assessing recurrent malignancy?

5. What are the expected morphological f ndings


a ter lumpectomy?

263
Status postlumpectomy 395
Case ranking/dif culty: Category: Diagnostic

5. Postlumpectomy we expect to see distortion, densities


that could represent irregular margin due to the scar—
but no new linear calcif cations or segmental distribution
o suspicious calcif cations.

Pearls
• A ter history o lumpectomy, mammograms are
o tentimes per ormed as diagnostic mammograms and
include spot compression magnif cation view o the
lumpectomy site.
• In this case, the lumpectomy site demonstrates
architectural distortion and was stable since prior
Ultrasound of left breast demonstrating the scar from prior studies—BI-RADS 2: there is no need or additional
surgery done in 2010. ultrasound.
• However, i ultrasound is requested, the scar tissue
rom prior lumpectomy on ultrasound appears to be
Answers indeterminate and, i mammogram is stable, would
1. O tentimes, mammography a ter lumpectomy is trigger BI-RADS 3 and subsequent ollow-up.
per ormed as a diagnostic mammogram and spot • I there is concern or recurrent malignancy, MRI
compression view o the scar is included. would be the best test to investigate the lumpectomy
site i there is a ocal abnormality showing suspicious
2. Ultrasound, in general, does not add any in ormation
enhancement.
about the lumpectomy site but could be help ul
in detecting recurrent malignancy in dense breast
tissue. It might also be help ul to detect recurrent
lymphadenopathy. Suggested Reading
3. The best exam to assess possible recurrent malignancy Dershaw DD, McCormick B, Cox L, Osborne MP.
would be MRI with contrast. Di erentiation o benign and malignant local tumor
recurrence a ter lumpectomy. AJR Am J Roentgenol.
4. In a premenopausal emale, the strongest enhancement
1990;155(1):35-38.
o the breast parenchyma will be in the f rst and
last week o the cycle. Also up to 6 months a ter
surgery, there will be postsurgical enhancement due
to granulation tissue.

264
Screening—asymptomatic: look carefully

1. What is the BI-RADS category or this


screening exam?

2. Which view is the best mammogram to include


the most breast tissue?

3. What are the danger areas to review i you


cannot spot the abnormality?

4. I you perceive an abnormality on one


view only (asymmetry), what views do you
recommend to localize the lesion prior to
ultrasound?

5. Now the asymmetry has been ound on


mammography, what imaging test do you
recommend?

265
One that nearly got away—IDC plus DCIS 612
Case ranking/dif culty: Category: Screening

Left XCCL—an asymmetry is just Left spot magni cation in XCCL


about visible at the edge of the position con rms an ill-de ned Careful ultrasound revealed a subtle mass with calci cations that
breast disc. A right XCCL was also mass at this site. was biopsied. Final pathology: Grade 3 invasive ductal carcinoma.
taken for comparison, as it was
thought that this may just be
some asymmetrical tissue.

Answers 5. Targeted ultrasound should be the f rst step ollowing


1. There is a possible asymmetry on the lateral margin o mammography to ully characterize the asymmetry,
the le t CC f lm. Needs urther workup. There ore, this and determine i this is a mass that can be targeted or
is a BI-RADS 0 exam. biopsy. Either FNAC or core biopsy may be appropriate
once that has been done. For a small lesion such as this,
2. The mediolateral mammogram was identif ed as the MRI may not be required or staging purposes, as it is
best view to get the majority o the breast tissue on one unlikely to in uence surgical management.
f lm. Up to then, standard orthogonal radiological views
were per ormed CC and lateral exams. Some European
organizations, such as the UK NHS Breast Screening Pearls
Programme, started to reduce costs and radiation dose
based on a single MLO mammogram. This was reversed • Evaluate a f lm or danger areas, i you think that the
later. mammogram is otherwise normal, and a f nding is
expected (oral exam situations).
3. There are our danger areas, but the f nding does not lie • The edges o a f lm sometimes provide high pick-up
in any o them. rates or incidental cancers, as the dynamic range can
i. In ramammary old on MLO be a ected at the edge o the receptor. This was much
ii. Medial aspect o CC f lm, adjacent to sternum more prevalent in the age o analog mammography.
iii. Subareolar region
iv. 1 cm below the pectoral muscle on the MLO
Suggested Readings
4. I you have tomosynthesis, then only that may be
Leung JW, Sickles EA. Developing asymmetry identif ed
required, unless the lesion happens to be o the image
on mammography: correlation with imaging outcome
plane. E ectively you have to think f rst where the
and pathologic f ndings. AJR Am J Roentgenol.
asymmetry may possibly lie, then target the appropriate
2007;188(3):667-675.
areas. On the MLO, it is seen in the upper hal , but
Sickles EA. The spectrum o breast asymmetries: imaging
this could be either lateral breast or medial breast.
eatures, work-up, management. Radiol Clin North Am.
The extended views, lateral and medial (XCCL and
2007;45(5):765-771, v.
XCCM), may reveal the location. Otherwise trying
Venkatesan A, Chu P, Kerlikowske K, Sickles EA, Smith-
additional views may help. Two particular techniques
Bindman R. Positive predictive value o specif c
may help as well: (a) stepped obliques and (b) rolled
mammographic f ndings according to reader and patient
views.
variables. Radiology. 2009;250(3):648-657.
266
First screening exam

1. What is the f nding on the screening


mammogram?

2. What is the next step on the screening


mammogram?

3. What is the next step in workup?

4. What does the spot compression view


(next page) conf rm?

5. What do you expect the outcome o an


ultrasound scan to be?

267
Invasive ductal carcinoma 390
Case ranking/dif culty: Category: Screening

Diagnostic mammogram, right spot Ultrasound of right breast demonstrates the corresponding, small (5 mm), “irregular”-
compression MLO view demonstrating shaped mass with mixed echogenicity.
“focal asymmetry” projecting over the
pectoralis muscle.

Answers
1. Finding is consistent with “ ocal asymmetry” given its Pearls
concave shape o the margin—it should not be called • Further evaluation with spot compression MLO view
mass because it was seen only on one projection. and right XCCL view is the next step. Since the lesion
is not def nitely seen on the CC view, it could be
2. Patient has to be worked up. BI-RADS 3 should never
outside the scan f eld and additional XCCL view was
be used on screening mammogram. The only BI-RADS
recommended.
assessment on screening mammogram is BI-RADS 1
• The “ ocal asymmetry” was not well seen on the
(negative).
additional XCCL view (not submitted) but persists on
3. Since it is not clearly seen on CC view and it could the spot compression MLO view and subsequently an
be outside the scan f eld, XCCL view is a good ultrasound was per ormed.
choice—also patient needs spot compression view • Ultrasound demonstrates small 7-mm mass with
MLO. ML view could also be added to urther assess the “angular” margin and subsequently ultrasound-guided
localization o the lesion. However, ML view might not biopsy was per ormed or the BI-RADS 4 lesion and
be able to get as ar back to include the lesion. showed the presence o invasive ductal carcinoma.
4. The spot compression view conf rms that the f nding is
real and needs to be urther worked up with ultrasound.
Suggested Reading
5. Ultrasound conf rms the f nding o a small “ ocal
asymmetry” and there ore is BI-RADS 4 and biopsy is Leung JW, Sickles EA. Developing asymmetry identif ed
recommended—and did conf rm the presence o invasive on mammography: correlation with imaging outcome
ductal carcinoma. and pathologic f ndings. AJR Am J Roentgenol.
2007;188(3):667-675.

268
Screening mammogram—asymptomatic patient

1. What is the abnormality on the mammogram?

2. What is the gold standard modality to assess


i implant is intact?

3. What is the typical appearance o intracapsular


rupture on imaging?

4. What are typical signs or extracapsular


implant rupture?

5. What def nes the capsule o the implant?

269
Silicone implants—extracapsular rupture 341
Case ranking/dif culty: Category: Screening

Right screening mammogram MLO Ultrasound demonstrates area of abnormal echo with strong posterior shadowing
view demonstrating extracapsular free (“snowstorm sign”), correlating to the area of concern seen on the mammogram (arrow)
silicone in superior right breast. consistent with extracapsular rupture.

Answers This can be seen on ultrasound as an area o hyperechoic


1. This is a patient with subglandular, prepectoral silicone signal with posterior shadowing (snow storm sign). MRI
implant and two area o high density superior to the is per ormed to assess implants without contrast.
implant highly suspicious or extracapsular rupture. 5. The capsule is ormed as a physiological response in
2. The gold standard is breast MRI to prove the presence the orm o f brous tissue surrounding the implant.
o silicone implant rupture. It can show intracapsular Intracapsular rupture re ers to rupture o the shell o the
rupture better than ultrasound. However, it can be tricky implant, but silicone is def ned by the f brous capsule.
to prove subtle intracapsular rupture, since complex Extracapsular rupture is def ned by penetration o
olds can have very similar appearance. Extracapsular silicone through the f brous capsule.
rupture can be detected on mammogram and ultrasound,
but MRI again is the most sensitive and specif c exam.
For saline implants, there is, in general, no need or Pearls
imaging to show rupture. A mammogram is su f cient to
• Screening mammograms or patients with breast
show de ation o the implants.
implant require to add additional so-called implant
3. Intracapsular rupture can be very subtle and is o tentimes displacement views bilateral in MLO and CC
more questionable clinical signif cance. It cannot be projections to better assess the breast parenchyma.
detected on mammogram, might show up as “step • This case demonstrates extracapsular rupture, stable
ladder” sign on ultrasound, and can be diagnosed with or many years in an asymptomatic patient. In this
conf dence i there is a “linguini sign” on MRI. “Step particular situation, this had no consequence and the
ladder” sign can be called when there are several linear implant was not removed.
structures that are interrupted, orming steps. “Linguini
sign” indicates the presence o multiple linear structures
in the implant. Suggested Reading
4. Extracapsular implant rupture is the most signif cant Everson LI, Parantainen H, Detlie T, et al. Diagnosis o
injury to the implant. It might result in symptoms and breast implant rupture: imaging f ndings and relative
possible implant replacement. On mammogram, area o e f ciencies o imaging techniques. AJR Am J Roentgenol.
high, silicone-like density is seen apart rom the implant. 1994;163(1):57-60.
270
Lump in breast of lactating woman

1. What BI-RADS classif cation should be used


here?

2. What mammographic views would you


per orm?

3. What is the likely pathology o the mass?

4. What is the appropriate diagnostic


interventional tool you would recommend?

5. The ultrasound is now shown to contain


re ective echoes suggesting microcalcif cation.
What additional test would you per orm?

271
Galactocele in lactating woman 611
Case ranking/dif culty: Category: Diagnostic

Ultrasound shows gentle lobulations to Ultrasound shows the echo pattern to Power Doppler. The lesion is shown to be
the mass. have both hyperechoic debris within the avascular.
“cyst” and some lower echo areas.

Answers particular suspicious f nding on ultrasound, unilateral


1. This is not a screening examination, so BI-RADS 0 is mammograms should be per ormed. MRI is unlikely to
not appropriate. The f ndings within context are benign, help, as lactating breasts may enhance so strongly that
but i you are uncertain, or i you are swayed by the clumped ductal or linear enhancement would be masked.
lesion being palpable, then give it a BI-RADS 4 and Elastography has not been shown to be o help in this
make the diagnosis by aspirating the milky uid. situation yet. PEM and BSGI are isotope-based tests
that should be avoided in a nursing mother wherever
2. Unless the ultrasound was suspicious o malignancy, possible.
or showed the presence o microcalcif cation,
mammography is usually o extremely low yield. A high
kV technique will not improve the chances o seeing
through extremely dense breast tissue in a lactating Pearls
woman. • Galactoceles are common f ndings toward the end o
3. These appearances, in a lactating breast, are o a breast eeding in lactating women.
galactocele. Simple cysts can occur, but are anechoic. • Diagnosis is usually by ultrasound and possibly
Fibroadenomas usually appear as a hypoechoic aspiration that shows milk.
mass against the bright background glandular tissue. • Surgical treatment is usually avoided.
Mucinous carcinoma usually has indistinct margins and • Multiple aspirations are a better alternative.
occurs in an older age group. Lactational adenoma tends
to present during pregnancy itsel .
4. Galactoceles are not malignant. Surgical excision is Suggested Readings
disf guring, but may be required or lesions that cannot
Engohan-Aloghe C, Bucella D, Boutemy R, Noël JC. Giant
be treated by multiple aspirations. There is in ammatory
galactocele in a lactating woman [in French]. Ann Pathol.
change associated with galactoceles, and i they leak
2008;28(6):526-528.
then they can be a cause o an abscess, which may then
Perez-Bóscollo AC, Dutra RA, Borges LG, et al.
need a drain placed. Surgical treatment with healing by
Galactocele: an unusual cause o breast enlargement
granulation would give a horrendous cosmetic e ect,
in children. J Pediatr Surg. 2009;44(7):e1-e3.
and is there ore not per ormed.
Sabate JM, Clotet M, Torrubia S, et al. Radiologic evaluation
5. You are concerned about the possibility o DCIS; o breast disorders related to pregnancy and lactation.
there ore, regardless o age in the presence o a Radiographics. 2007;27(Suppl 1):S101-S124.

272
Diagnostic mammogram for new calci cations, 50-year-old woman

1. How would you describe the calcif cations


in the le t breast?

2. What is the assessment i the calcif cations


are new?

3. What would be the assessment i the


calcif cations are seen on f rst mammogram?

4. What are other f ndings on a f rst mammogram


typically called benign?

5. What are other f ndings on f rst mammogram


that a ter diagnostic workup can be called
probably benign?

273
Lobular carcinoma in situ 320
Case ranking/dif culty: Category: Screening

Diagnostic mammogram, left magni cation ML view (additional


Diagnostic mammogram, Diagnostic mammogram, electronic magni cation) demonstrating group of “round and
left ML magni cation view. left CC magni cation view. oval” calci cations.

5. There are three classical BI-RADS 3 f ndings on


f rst mammogram a ter diagnostic workup: a “ ocal
asymmetry,” “round well-circumscribed mass,” and
“oval and round” calcif cations. The diagnostic workup
o the mass and the asymmetry does include ultrasound.

Pearls
• Treatment o patients with diagnosis o lobular
Diagnostic mammogram, left CC view (additional electronic carcinoma in situ (LCIS) or atypical lobular
magni cation). hyperplasia (ALH) is controversial.
• Since upgrade to invasive carcinoma or DCIS a ter
Answers diagnosis o LCIS or ALH was reported to be between
17% and 30%, there is increasing consensus to per orm
1. This is a group o “oval and round” calcif cations. surgical excision o the area a ter diagnosis on core
2. A new group o “round and oval” calcif cations is needle biopsy.
suspicious and can be called BI-RADS 4 and biopsy
should be per ormed.
3. This f nding o a group o calcif cation, oval and round Suggested Readings
on f rst screening mammogram, a ter diagnostic workup,
Choi BB, Kim SH, Park CS, Cha ES, Lee AW. Radiologic
is a typical BI-RADS 3 f nding and 6-month ollow-up
f ndings o lobular carcinoma in situ: mammography and
can be recommended. A ter 2 years o stability, they can
ultrasonography. J Clin Ultrasound. 2011;39(2):59-63.
be called benign.
Foster MC, Helvie MA, Gregory NE, Rebner M, Nees AV,
4. Only the mole and the popcorn-type calcif cations can Paramagul C. Lobular carcinoma in situ or atypical
be called benign on a screening mammogram; Popcorn- lobular hyperplasia at core-needle biopsy: is excisional
type calcif cations, typical eggshell type calcif cations biopsy necessary? Radiology. 2004;231(3):813-819.
or secretory calcif cations can be called benign on a Hussain M, Cunnick GH. Management o lobular carcinoma
screening mammogram. in-situ and atypical lobular hyperplasia o the breast—a
review. Eur J Surg Oncol. 2011;37(4):279-289.

274
Lump found in armpit while washing

1. What BI-RADS classif cation should be used


here?

2. What is the most likely pathology based on


the imaging?

3. What is the next best imaging test?

4. What are the physical f ndings o a


hamartoma?

5. What would be the best way to biopsy this


lesion.

275
Squamous carcinoma of the axillary tail 593
Case ranking/dif culty: Category: Diagnostic

4. A hamartoma is composed o normal breast tissue


(think o breast within a breast). As a result, physical
examination either is normal or shows a so t mass.
5. FNAC may not give a def nitive answer, and relies on
your site having trained cytopathology sta . Core biopsy
o either type may work here. Surgical excision should
be avoided i possible, until a preop diagnosis has been
obtained. Punch biopsy is good only or dermal lesions.

Pearls
• Rare, special type o variant IDC.
• Poorer prognosis.
Mixed echo mass with a capsule. Mixture of solid and cystic • May appear with relatively benign imaging
elements. Ultrasonographically would t with a hamartoma, appearances.
with a predominantly solid component, but the mammographic
appearances do not t and it was hard on physical examination,
and therefore a biopsy was performed, which revealed the
diagnosis. Suggested Readings
Choi BB, Shu KS. Metaplastic carcinoma o the breast:
Answers multimodality imaging and histopathologic assessment.
Acta Radiol. 2012;53(1):5-11.
1. This was a hard lump to palpate the lower axilla, which Joshi D, Singh P, Zonun awni Y, Gangane N. Metaplastic
does not look like a standard lymph node, as it neither is carcinoma o the breast: cytological diagnosis and
smooth in contour nor has a hilum. Ultrasound is needed diagnostic pit alls. Acta Cytol. 2011;55(4):313-318.
or urther evaluation. Nonnis R, Paliogiannis P, Giangrande D, Marras V, Trignano
2. The mass does not look like a lymph node or accessory M. Low-grade f bromatosis-like spindle cell metaplastic
breast tissue. Invasive lobular carcinoma usually carcinoma o the breast: a case report and literature
presents with minimal change o distortion. DCIS review. Clin Breast Cancer. 2012;12(2):147-150.
usually presents with microcalcif cations, although it can
present with a mass, which usually is hypoechoic and
circumscribed (simulates f broadenoma).
3. The cheapest and nonionizing examination is ultrasound,
which can be targeted to the palpable or mammographic
abnormality. I tomosynthesis has already been
per ormed, the margins o the mass may have been
identif ed.

276
33-year-old woman status post–bilateral mastectomy (no cancer),
referred to bilateral ultrasound

1. What could be the next step to evaluate


multiple indeterminate masses?

2. What is the description o the f nding in


the two f gures (le t and right) on the top?

3. What is the best way to biopsy a lesion when


patient has implants?

4. What is a tubular adenoma?

5. What is the consequence o biopsy


demonstrating tubular adenoma?

277
Multiple adenoma 319
Case ranking/dif culty: Category: Diagnostic

MRI T1-weighted sequence after IV contrast MRI T2-weighted sequence demonstrating MRI T1-weighted sequence after IV
demonstrating heterogeneously enhancing mass high in signal near implant left breast. contrast demonstrating mass in the
mass in the left superior breast near implant. right inferior breast.

3. Ultrasound-guided biopsy is the best way to biopsy


lesion when patient has breast implant, since it is done in
real time and needle can be inserted more exibly than it
is by MRI or stereotactic biopsy.
4. Tubular adenoma is a proli erative change similar to
f broadenoma but with more tubular and less stromal
elements. It is a benign lesion.
5. This is benign proli erative change—no need or urther
treatment.
Ultrasound-guided biopsy of the abnormality in the left superior
breast as seen on ultrasound, which correlates to MRI.

Pearls
Answers • Tubular adenoma, phyllodes tumor, and lactating
1. Any f nding—i masses, calcif cations, and so on— adenoma are all entities related to f broadenoma.
scattered bilaterally decreases the suspicion or • This is an example that ultrasound-guided biopsy can
malignancy. MRI is a help ul problem-solving modality be per ormed sa ely in patients with implants; in this
to determine which o the abnormalities is more case, patient had bilateral saline implants.
concerning.
2. “Hypoechoic” mass, hypoechoic in comparison with the
anterior at, demonstrates posterior shadowing. Suggested Reading
Barsky, Gradishar, Recht, et al. The Breast. 4th ed.
Philadelphia, PA: Saunders Elsevier; 2009.

278
23-year-old patient with palpable abnormality

1. What is the f rst exam when a 23-year-old


patient reports palpable lump?

2. What is the next step i ultrasound does not


demonstrate any abnormality?

3. What is the management o a palpable mass


that has benign eatures on ultrasound?

4. What would be the management, i multiple


masses are seen?

5. What other histology could have f ndings


similar to f broadenoma?

279
Fibroadenoma 317
Case ranking/dif culty: Category: Diagnostic

MRI, T2-weighted images demonstrate mass in MRI, T1-weighted sequence after IV contrast MRI, T2-weighted sequence
the right lateral breast with mild increased signal demonstrates no enhancement within the mass. demonstrates mass with
on T2-weighted images. The mass demonstrates low signal. septations of low signal, as
described as “dark septations.”

and to per orm biopsy (BI-RADS 4) or send patient to


surgical excision. However, Harvey et al. published in
2009 a paper arguing that ollow-up o palpable masses
is more cost-e ective and sa e.
4. I there are additional incidental masses ound at the
time o ultrasound, ultrasound-guided biopsy can be
per ormed or the palpable mass and the remainder o
the masses can be ollowed in 6 months. This would also
apply i a patient on a “screening ultrasound” presents
MRI, T1-weighted sequence demonstrates mass without with incidental f nding o benign-appearing mass. We
enhancement.
would call it BI-RADS 3 and ollow it in 6 months. I
there are multiple benign appearing palpable masses
it is not practical to biopsy several masses and can be
Answers
ollowed.
1. The f rst step is ultrasound. Mammography is avoided—
because o the act that young patients are more sensitive 5. Papillomas or even malignancies such as mucinous
to radiation—and that in this age there is, very dense carcinomas, tubular and medullary carcinomas and can
f broglandular tissue that limits the value o mammography. be well circumscribed.
Mammography would be the second step i ultrasound
does not explain the palpable abnormality. Five percent
o all breast cancers occur in the age group younger than Pearls
40 years. These are usually the more aggressive tumors. • Palpable masses in young patients are not uncommon,
2. Next step is to per orm a mammogram. While some and i it correlates to a benign-appearing mass, most
breast imagers would argue that one single image— or likely f broadenoma, management is controversial.
example, a MLO view—would be good enough, while • A more conservative approach is to recommend biopsy
some would avor even to per orm a bilateral baseline or surgical excision.
exam. The ormer would argue that the lack o a mass on • I the mass does show all typical f ndings o
ultrasound makes the presence o mass on mammogram f broadenoma, the other, not in requently used,
very unlikely and the purpose o the mammogram would approach is to ollow the mass over 2 or 3 years.
be to f nd calcif cations. This can be achieved by one view
only. The latter point o view is that ocal asymmetries
might be missed on an ultrasound and on a single view Suggested Reading
mammogram. A reasonable approach is to start with a Harvey JA, Nicholson BT, Lorusso AP, Cohen MA, Bovbjerg
unilateral standard mammogram f rst and i there is any VE. Short-term ollow-up o palpable breast lesions with
remaining concern then to include the contralateral side. benign imaging eatures: evaluation o 375 lesions in 320
3. I mass is palpable—despite benign eatures on women. AJR Am J Roentgenol. 2009;193(6):1723-1730.
imaging—conservative approach is to call it suspicious
280
Screening—asymptomatic

1. What is the BI-RADS category or this


screening exam?

2. What are the “danger areas” in mammography?

3. What is the positive predictive value o a


developing ocal asymmetry?

4. What is the def nition o a ocal asymmetry?

5. What is the best imaging examination in the


workup o this patient?

281
Inferior mammary fold (IMF) mass—danger area 597
Case ranking/dif culty: Category: Screening

Although this mass appears round on mammography (a benign Another view of the same mass shows that there is “duct
descriptor), it has “angular margins” on ultrasound (a suspicious extension,” which is extending anteriorly to the surface of the
descriptor). glandular tissue.

Answers 5. I you are unlikely to get better images o the area, and
1. This patient has an asymmetry in the lower hal o the you have a mass lesion, going direct to ultrasound to
le t breast in the in erior mammary old, which is a well- characterize the lesion works well. Tomosynthesis or
known “danger area.” As the patient needs workup with spot views (+/− magnif cation) can be per ormed or
urther f lms and ultrasound, this patient should be given more complete mammographic workup. This lesion may
a BI-RADS 0. be adherent to the chest wall, and i there is any doubt,
an MRI could be per ormed, but this is not the best f rst-
2. There are our main danger areas: (a) The immediate line exam in this case.
prepectoral area, where we requently see intramammary
lymph nodes. I a mass develops or there is a mass
without a hilum, you should be suspicious and work the
thing up. (b) The in erior mammary old: although it is Pearls
not uncommon or a developing sebaceous cyst to mimic • In erior mammary old is a danger area or developing
the development o a carcinoma at this site. (c) Medial malignancy.
aspect o the breast on a CC f lm. (d) Subareolar—o ten • A developing asymmetry in this area should be taken
di f cult to pick up a mass in this area, which typically seriously.
has a lot going on.
3. UCSF data showed that the PPV or a developing
asymmetry was 12%, and that i not a skin lesion such Suggested Readings
as a sebaceous cyst, then it deserves a ull workup and
Leung JW, Sickles EA. Developing asymmetry identif ed
biopsy.
on mammography: correlation with imaging outcome
4. The def nition o a ocal asymmetry is a density present and pathologic f ndings. AJR Am J Roentgenol.
on two views (ie, localized to a part o the breast). An 2007;188(3):667-675.
asymmetry is a density that is not a space-occupying Sickles EA. The spectrum o breast asymmetries: imaging
lesion, on one projection only (either CC or MLO). eatures, work-up, management. Radiol Clin North Am.
A space-occupying lesion is a mass rather than an 2007;45(5):765-771, v.
asymmetry or ocal asymmetry. Venkatesan A, Chu P, Kerlikowske K, Sickles EA, Smith-
Bindman R. Positive predictive value o specif c
mammographic f ndings according to reader and patient
variables. Radiology. 2009;250(3):648-657.

282
Patient with new calci cations as seen on diagnostic mammogram—
consistent with DCIS. Follow-up after lumpectomy (page 282)

1. What is the best description o the


calcif cations?

2. What is the best management?

3. What is the consequence i the pathology


shows adenosis?

4. How good is mammography to detect recurrent


DCIS and how likely is the recurrence o
DCIS?

5. What is the prognosis o recurrent DCIS a ter


lumpectomy?

283
Recurrent DCIS after lumpectomy 314
Case ranking/dif culty: Category: Diagnostic

Postlumpectomy mammogram, right CC view with scar marker.


Postlumpectomy mammogram, right ML magni cation view. Calci cations not covered.

Answers
1. This is an example o a group o “pleomorphic” Pearls
calcif cations in segmental distribution. • Most patients who presented with calcif cations
consistent with DCIS at the time o initial diagnosis
2. The best workup would include, at this point, ultrasound
and who develop recurrence will present with
to assess possible invasive solid component—
suspicious calcif cations as well at the time o
i ultrasound does not show solid component,
recurrence.
stereotactic biopsy is the next step, since the f nding
• Mammography is very e ective in detecting recurrence
is highly suspicious (BI-RADS 5). MRI would be
(sensitivity o 97%).
help ul preoperative to assess or additional disease;
• Recurrent malignancy o DCIS is in general stage 0
lumpectomy is the most likely surgical treatment.
or 1 and there ore the prognosis is excellent.
3. Since a new group o pleomorphic calcif cations can be • In 90% o the cases, the morphology o the recurrent
classif ed as BI-RADS 5 (highly suspicious), it has to calcif cations is similar to the morphology o the initial
be excised—even i pathology a ter stereotactic biopsy calcif cations.
demonstrates a “benign” f ndings. • Mean time between the initial diagnosis and recurrence
is about 4.5 years with range rom 1 to 12 years and
4. Mammography is very e ective (97% sensitivity)
clustering between 1 and 7 years.
to detect recurrent DCIS at lumpectomy site. This
is in particular true, since recurrent calcif cations
due to recurrent DCIS in general present with same
morphology o the initial calcif cations. Recurrence Suggested Reading
rate is about 7% at 5 years a ter lumpectomy and not Pinsky RW, Rebner M, Pierce LJ, et al. Recurrent cancer
signif cantly di erent rom mastectomy. a ter breast-conserving surgery with radiation therapy
5. Prognosis o recurrent DCIS in lumpectomy bed is or ductal carcinoma in situ: mammographic eatures,
usually excellent and most likely stage 0 or 1. method o detection, and stage o recurrence. AJR Am J
Roentgenol. 2007;189(1):140-144.
284
41-year-old woman with screening mammogram—any abnormality?

1. What is the f nding on the mammogram?

2. I the histology is tubular adenoma, in this


case, what is your next step?

3. Why do some authors suggest that there is


no need or 6-month ollow-up a ter benign
concordant biopsy?

4. What is the di erence between tubular


adenoma and f broadenoma?

5. What is the di erence between tubular


adenoma and lactating adenoma?

285
Tubular adenoma 313
Case ranking/dif culty: Category: Diagnostic

Gray-scale ultrasound images demonstrating corresponding mass.

Diagnostic mammogram, spot compression right CC view 5. It is believed that both are similar lesions in di erent
con rming mass in the right breast subareolar in location. physiologic states o the patient. Lactating adenoma is
ound in pregnant or breast- eeding women, whereas
tubular adenoma is ound in premenopausal emales.
Some theories believe that tubular adenomas are present
be ore pregnancy and then appear as lactating adenomas
during pregnancy.

Pearls
• Tubular adenoma and lactating adenoma are
histologically similar lesions, related to f broadenoma,
seen in di erent phases o li e. Tubular adenomas
Ultrasound with duplex demonstrating mass with increased ow. occur during reproductive years and lactating adenomas
occur during lactation and during pregnancy.
• Tubular adenomas are indistinguishable rom
Answers f broadenomas on imaging.
1. Noted is a “partially obscured” mass in the right upper • Tubular adenomas have, in comparison with
outer quadrant o the right breast. It is best appreciated f broadenomas, almost no stromal components but only
on the CC view. Additional spot compression view epithelial components and can undergo in arction and
conf rms “well-circumscribed” margin o the mass. may produce acoustic shadowing.
• Necessity o 6-month ollow-up a ter benign and
2. This is a concordant benign f nding and 6-month ollow- concordant biopsy is debatable—according to
up is, in general, the standard in most practices. Salkowski et al. (2011), rebiopsy rate was 0.8% at
3. The likelihood o malignancy in a screening population 6-month interval and 0.5% at 12-month interval.
is about 3 to 4 per 1000, which is 0.3% to 0.4%. The
likelihood o malignancy less than 2% is considered as
probably benign (BI-RADS 3). Salkowski et al. (2011) Suggested Readings
ound that PPV o rebiopsy did not di er between 6
and 12 months a ter benign and concordant biopsy and Salkowski LR, Fowler AM, Burnside ES, Sisney GA. Utility
that the incidence o malignancy was close to normal o 6-month ollow-up imaging a ter a concordant benign
incidence in screening population. breast biopsy result. Radiology. 2011;258(2):380-387.
Stavros AT. Breast Ultrasound. 1st ed. Philadelphia; PA:
4. Tubular adenoma has less stromal and more epithelial Lippincott Williams & Wilkins; 2004.
elements.

286
Screening—asymptomatic

1. What is the BI-RADS category or this


screening exam?

2. What additional mammographic views do you


recommend?

3. Is there another test that should also be


per ormed?

4. Is this a mass or a type o asymmetry?

5. I this patient has a history o prior right-


sided breast surgery, what is the di erential
diagnosis?

287
Focal asymmetry upgraded on ultrasound to BI-RADS 5 594
Case ranking/dif culty: Category: Screening

RCC spot magni cation pushes away normal tissue and allows the Ultrasound shows “irregular mass” with “ill-de ned margins,” taller
mass to be seen more clearly. than wide (“not parallel”).

Answers has had surgery, repeat mammograms with a scar marker


1. There is a ocal asymmetry in the right breast, which may be help ul to di erentiate. Fat necrosis is more
needs urther workup. BI-RADS 4 or 5 should not be likely i at lucency is present, associated with the scar,
used or a screening exam, as the patient may have at or with characteristic dystrophic calcif cations. Cancer
necrosis, or a benign f nding including superimposition recurrence should be suspected i the scar is getting
to account or the f ndings. I given a BI-RADS 4 or 5, denser over time.
and re erred to a surgeon, these o ten end up in surgery
or biopsy or treatment, without tissue biopsy.
2. Although there is an argument or per orming the Pearls
above views, simply per orming a lateral exam (in the • Treat medial densities, especially i developing, with
projection with the abnormality nearest to the bucky) caution.
and spot views may be all that is required. Many breast • Do ull workup.
imagers recommend that i you per orm spot f lms, • Have a low threshold or biopsy.
you should also do it with magnif cation to get the
best possible resolution, and to urther characterize the
calcif c particles or margins o a mass.
Suggested Readings
3. Use o a cheap nonionizing exam such as ultrasound
Leung JW, Sickles EA. Developing asymmetry identif ed
is the most use ul f rst-line additional modality in this
on mammography: correlation with imaging outcome
situation.
and pathologic f ndings. AJR Am J Roentgenol.
4. Unless there is def nite space-occupying lesion, then it 2007;188(3):667-675.
should be called a ocal asymmetry rather than a mass at Sickles EA. The spectrum o breast asymmetries: imaging
this point. Spot f lms and ultrasound may assist to urther eatures, work-up, management. Radiol Clin North Am.
identi y i this is truly a mass. I visible in one plane, 2007;45(5):765-671, v.
it should be called an asymmetry rather than a ocal Venkatesan A, Chu P, Kerlikowske K, Sickles EA, Smith-
asymmetry. Bindman R. Positive predictive value o specif c
mammographic f ndings according to reader and patient
5. Regardless o the history, the f ndings o a ocal
variables. Radiology. 2009;250(3):648-657.
asymmetry in the absence o evidence on prior f lms
mean that the patient should be worked up. I the patient

288
88-year-old woman with screening mammogram

1. What is the appropriate workup o a new


asymmetry?

2. What is the next step a ter f nding o


a hyperechoic mass?

3. What would be the next step i ultrasound


was normal?

4. What is the most likely histology or


a hyperechoic mass?

5. What is the most likely histology i there


is a suspicious corresponding f nding on
mammogram?

289
Echogenic lesion on ultrasound consistent with 312
invasive ductal carcinoma
Case ranking/dif culty: Category: Diagnostic

Ultrasound directed to the area with corresponding Ultrasound with duplex with corresponding mass with increased ow on
nding. Hyperechoic mass, slightly “irregular” in shape. duplex.

Answers
1. Next steps are spot compression views, MLO, and CC Pearls
ollowed by ultrasound. • Findings on ultrasound, which are hyperechoic, are
in general not very likely to represent malignancy—
2. Despite the act that hyperechoic masses seen
according to Linda et al. (2011), o 1849 biopsied
on ultrasound are overwhelmingly benign—i
lesions showing malignancy, only 9 were hyperechoic.
“hyperechoic” mass correlates to a new “ ocal density”
• However, i hyperechoic lesion corresponds to new
on mammogram—it needs to be biopsied, especially
mammogram f nding or is palpable, biopsy is strongly
i it presents with “irregular margins.”
recommended.
3. Given that the density was not seen on prior study, this is • Malignant hyperechoic lesions include lymphoma,
suspicious and biopsy is recommended. angiosarcoma, metastasis, however, the most likely
pathology would be an invasive ductal carcinoma.
4. The majority o f ndings will be benign. I there is no
• None o the nine hyperechoic malignancies described
corresponding denisty on mammogram it could be a
by Linda et al. (2011) was a purely sonographic lesion.
lipoma, di erential diagnosis could include at necrosis
or hematoma.
5. Metastasis, liposarcoma, angiosarcoma, and lymphoma
Suggested Reading
are very rare entities—given the suspicious morphology
(new density), the most likely malignancy will be Linda A, Zuiani C, Lorenzon M, et al. Hyperechoic lesions
invasive ductal carcinoma. o the breast: not always benign. AJR Am J Roentgenol.
2011;196(5):1219-1224.

290
29-year-old patient with palpable abnormality in the left breast (6:00)

1. What is the pertinent mammogram f nding?

2. What is the next step?

3. What is a radial scar?

4. What is the consequence i pathology


demonstrates radial scar?

5. Is MRI o any help in the assessment o radial


scar?
291
Radial scar 310
Case ranking/dif culty: Category: Diagnostic

Ultrasound duplex demonstrating “irregular” mass with “spiculation.” MRI, postcontrast, subtracted images demonstrating
area of increased enhancement in the left central inferior
breast correlating to the mammogram and ultrasound.

Answers
Pearls
1. Noted is a subtle area o architectural distortion in the
le t in erior breast as best seen on the spot compression • Radial scar is a benign proli erative lesion
views. characterized by a central f broelastotic core with ducts
and lobules radiating outward, giving lesion typical
2. Next step is ultrasound. It is also important to correlate stellate appearance.
the f ndings with possible history o prior surgery or • Literature suggests that radial scars are associated with
biopsy. surrounding malignancy in up to 40% at the time o
3. Radial scar has been described in the literature under surgical excision.
several di erent names, such as “radial sclerosing • MRI cannot predict the likelihood o associated
lesion” and “complex sclerosing lesion.” It is a orm o malignancy.
benign proli erative breast tissue, which mainly consists • Surgical excision is recommended a ter the diagnosis
o ductal elements. This results in tubular structures/ o radial scar on all core biopsies, including MRI-
ocal asymmetry on mammography. guided biopsy.
4. Despite the act that radial scars are a orm o benign
proli erative disease, excisional biopsy is recommended
since it is associated with a malignancy rate o up to Suggested Readings
40% on surgical excision. The recommended reason or Linda A, Zuiani C, Furlan A, et al. Radial scars without
excision is also due to the act that radial scar is usually atypia diagnosed at imaging-guided needle biopsy: how
extending outside the sample obtained by the core o ten is associated malignancy ound at subsequent
biopsy. surgical excision, and do mammography and sonography
5. MRI is not very specif c. I there is enhancement, it predict which lesions are malignant? AJR Am J
does not indicate that there is malignancy. However, it Roentgenol. 2010;194(4):1146-1151.
can screen or additional malignancy in the breast. It is Sringel RM, Eby PR, Demartini WB, et al. Frequency,
not established at this point i biopsy-proven radial scar upgrade rates, and characteristics o high-risk lesions
could be le t alone and MRI does not show enhance. initially identif ed with breast MRI. AJR Am J Roentgenol.
A ter biopsy, there is, in general, always some iatrogenic 2010;195(3):792-798.
enhancement that impairs assessment.

292
Diagnostic mammogram following screening callback

1. Which o the BI-RADS descriptors most


accurately represents the f ndings?

2. What is the most likely pathology based


on the imaging?

3. What is the next best imaging test?

4. What are the biochemical types o


calcif cations ound in the breast?

5. Which compartment o the breast are these


calcif cations likely to originate?

293
Calcium oxalate calci cation causing biopsy 579
Case ranking/dif culty: Category: Diagnostic

Answers
1. Although some o the individual calcif c particles Pearls
have a round or curvilinear margin, the best description • Specimen x-ray can see calcif cations but may not be
here would be amorphous. BI-RADS is still the best appreciated on conventional pathology stains.
lexicon we have or the description o calcium in the • They are not pink-staining crystals as H&E stain.
United States. • Need polarized light to identi y bire ringent
calcif cations.
2. These types o calcif cations are commonly ound in
benign breast conditions, but can also be associated with
lobular neoplasia because the biopsy was prompted.
Low-grade DCIS calcif cations are very similar to benign Suggested Readings
causes o calcif cations, and i your threshold or biopsy Corben AD, Edelweiss M, Brogi E. Challenges in the
is set too high, you may miss diagnoses o low-grade interpretation o breast core biopsies. Breast J. 2010;16
DCIS. (Suppl 1):S5-S9.
3. I the calcif cation looks like DCIS, and the patients have Grimes MM, Karageorge LS, Hogge JP. Does exhaustive
dense breast tissue, then an ultrasound may be a good search or microcalcif cations improve diagnostic yield
test to determine i it is an associated mass. Targeting the in stereotactic core needle breast biopsies? Mod Pathol.
mass will also increase the yield or invasive cancer, i 2001;14(4):350-353.
present. I you suspect DCIS, some say that you should Tornos C, Silva E, el-Naggar A, Pritzker KP. Calcium oxalate
per orm MRI to determine extent and any associated crystals in breast biopsies. The missing microcalcif cations.
mass to assist targeting biopsy. Best test would be to Am J Surg Pathol. 1990;14(10):961-968.
per orm a stereotactic core biopsy.
4. Calcium particles can be made up o virtually any
calcium salt ound in the body. Calcium pyrophosphate
and calcium oxalate may be di f cult to see on pathology
without polarizing light because o their bire ringence.
5. These types o amorphous calcif cations are
indeterminate and are ound in a variety o
compartments within the breast. The terminal ductal
lobular unit is a common site, as is the stroma in simple
calcif cations associated with diseases such as sclerosing
adenosis. Calcif cations within ducts requently represent
DCIS. Di erential is secretory calcif cations that have a
characteristic appearance.

294
Patient with status post-benign left core biopsy 1 year ago,
now feeling lump

1. What is the most likely reason or palpable


lump?

2. What are the eatures o at necrosis on


ultrasound?

3. What are the eatures o at necrosis on MRI?

4. What is the time rame when at necrosis


occurs a ter lumpectomy?

5. What are the eatures o at necrosis on


mammogram?
295
Fat necrosis 292
Case ranking/dif culty: Category: Diagnostic

Fat-suppressed postgadolinium image showing enhancement


MRI of the breast, non–fat-suppressed T1-weighted images
in the periphery of the mass with mixed kinetics.
showing mass in the left breast containing fat.

Answers
1. Mammogram ails to show any suspicious abnormality. Pearls
Ultrasound demonstrates heterogeneous mass with • Fat necrosis may be due to trauma, prior biopsy,
posterior shadowing. Given the history o recent benign or surgery.
biopsy with vacuum-assisted 9-gauge device, this is most • It can appear as long as 3 years a ter surgery.
consistent with at necrosis. • Appearance on imaging depends on the amount o
f brotic reaction. No f brosis results in oil cyst—more
2. Fat necrosis again can show up in many di erent orms.
f brotic reaction results in at necrosis and is di f cult
Well-circumscribed mass may be classif ed as BI-RADS
to di erentiate rom malignancy.
3, while heterogeneous mass or ill-def ned masses are
• MRI can be specif c i at signal is identif ed.
unspecif c and malignancy is di f cult to exclude.
3. MRI can be relatively specif c i there is at identif ed
within the abnormality. The enhancement kinetics can be
Suggested Readings
very di erent and are not specif c.
Solomon B, Orel S, Reynolds C, Schnall M. Delayed
4. Fat necrosis can occur almost any time a ter surgery. development o enhancement in at necrosis a ter breast
More than 3 years a ter surgery, however, is unusual. conservation therapy: a potential pit all o MR imaging o
5. Fat necrosis can present in many di erent orms. Some the breast. AJR Am J Roentgenol. 1998;170(4):966-968.
f ndings are specif c and can be classif ed as BI-RADS Taboada JL, Stephens TW, Krishnamurthy S, Brandt KR,
2 (benign), or example, at-containing oil cysts and Whitman GJ. The many aces o at necrosis in the breast.
curvilinear calcif cations associated with “radiolucent AJR Am J Roentgenol. 2009;192(3):815-825.
mass.” Some f ndings are more indeterminate such as
“coarse and heterogeneous” calcif cations. Some f ndings
cannot be di erentiated rom malignancy and biopsy
cannot be avoided, or example, in case o spiculated
mass. MRI can be help ul in this particular case, i there
is at least a time period o 6 months since biopsy.

296
Screening mammogram—any abnormality?

1. What is the pertinent f nding?

2. What is juvenile papillomatosis?

3. What is the recommendation a ter biopsy


showing papilloma?

4. What is the management o the incidental


f nding o “juvenile papillomatosis”?

5. What are the typical imaging eatures o


a papilloma?

297
Papilloma 290
Case ranking/dif culty: Category: Screening

Diagnostic mammogram, right spot Diagnostic mammogram, right spot Ultrasound, right lateral breast demonstrating small
compression MLO view con rming compression CC view con rming small mass with “ductal extension” (Stavros).
the presence of small mass. “well-circumscribed” mass.

Answers
• This is based on recent publications suggesting
1. Noted is the ocal asymmetry in the right lateral breast. upgrading o papillomas in up to 16% a ter surgical
2. Proli erative breast change with the presence o ductal excision, in particular o centrally located papillomas.
hyperplasia and multiple associated cysts in young • Papillomas in the periphery, located in the terminal
patients. Patients o tentimes have amily history o ductal lobular units (TDLU), are less likely to cause
breast cancer and increased risk o developing breast nipple discharge and are supposed to have less risk
cancer in older age. o associated malignancy.
• “Juvenile papillomatosis” occurs in young women and
3. Since there is evidence that by surgical excision a is characterized by duct hyperplasia and the presence o
“benign” papilloma gets upgraded to malignancy in multiple cysts. It o tentimes presents as palpable mass.
more than 2% (def nition o BI-RADS 3), surgical Patients o tentimes have amily history o breast cancer
excision is recommended. However, the issue remains and increased li etime risk, especially in older age.
controversial, and in some institutions not all “benign Close imaging surveillance is recommended including
papillomas” (without atypia, etc.) get surgically resected. o the amily o the patient.
4. Patients with juvenile papillomatosis have likely an
increased risk o breast cancer a ecting their amily as
well as an approximately three old increased personal
risk o breast cancer. It is debatable i additional Suggested Readings
screening with breast MRI should be recommended. Ja er S, Nagi C, Bleiweiss IJ. Excision is indicated or
intraductal papilloma o the breast diagnosed on core
5. It is a benign lesion related to the ductal system, which
needle biopsy. Cancer. 2009;115(13):2837-2843.
o tentimes causes nipple discharge. It is being debated
Mercado CL, Hamele-Bena D, Oken SM, Singer
i centrally located papillomas have a higher risk o
CI, Cangiarella J. Papillary lesions o the breast
malignancy than papilloma in the periphery.
at percutaneous core-needle biopsy. Radiology.
2006;238(3):801-808.
Muttarak M, Lerttumnongtum P, Chaiwun B, Peh WC.
Pearls Spectrum o papillary lesions o the breast: clinical,
imaging, and pathologic correlation. AJR Am J
• In recent years, there is growing tendency to support Roentgenol. 2008;191(3):700-707.
surgical excision o “benign” papillomas, diagnosed on
core biopsy. However, the issue remains controversial
and management di ers depending between centers.

298
Prior benign surgical biopsy, right upper outer quadrant

1. What BI-RADS classif cation should be used


here?

2. How would you describe the asymmetry in


the le t breast?

3. What is the risk o malignancy associated with


an asymmetry?

4. This area is di use, so what imaging should be


considered to work this up?

5. What type o biopsy would you recommend?

299
Masses within regional asymmetry in fatty breasts 1577
Case ranking/dif culty: Category: Screening

Left lateral showing Ultrasound showing multiple solid “intraductal MRI—MIP axial reconstruction (subtracted).
segmental nodular masses.”
asymmetry.
4. Initial workup with diagnostic mammograms or
tomosynthesis (depending on availability), ollowed by
ultrasound and biopsy. Consider per orming at least two
biopsies o either one anterior and one posterior lesion
or one medially and one laterally. I breast conservation
is being considered, an MRI will give a better idea o
the extent o the disease. In this case, compare the MRI
f ndings with the mammograms, and see how the MRI
delineates the extent o the disease much more clearly,
in a very visual way.
5. I the lesions can be seen clearly on ultrasound, then this
is the best method or biopsy. In di use disease, when
you are trying to mark the boundaries o the disease,
Left breast—Sagittal thin MIP reconstruction to show extent of
stereotactic core biopsy may be pre erred.
disease to nipple.

Answers Pearls
1. The patient is having a screening exam, despite the act • “Developing ocal asymmetry” is suspicious, until
she has had prior surgery, so an abnormal exam like this proven otherwise.
should be given a BI-RADS 0. • An “asymmetry” that does not look like normal
2. The asymmetry is rather large in the CC plane to glandular tissue should be worked up ully.
describe it as a simple ocal asymmetry. However,
it could be used i stated as “large segmental ocal
asymmetry.” Some may pre er to describe it as a regional Suggested Readings
asymmetry, which has segmental eatures. Either way,
you need to emphasize in the report that this is abnormal Leung JW, Sickles EA. Developing asymmetry identif ed
and needs workup. A BI-RADS 0 would be f ne in a on mammography: correlation with imaging outcome
screening patient. I this were a diagnostic mammogram, and pathologic f ndings. AJR Am J Roentgenol.
then I would continue ultrasound, and give a suspicious 2007;188(3):667-675.
BI-RADS impression a ter ultrasound was per ormed. Sickles EA. Mammographic eatures o “early” breast
cancer. AJR Am J Roentgenol. 1984;143(3):461-464.
3. For a simple asymmetry, the risk o malignancy is Venkatesan A, Chu P, Kerlikowske K, Sickles EA, Smith-
less than 2%. The risk is higher or a ocal asymmetry Bindman R. Positive predictive value o specif c
(10–15%), but much higher or a developing ocal mammographic f ndings according to reader and patient
asymmetry, such that urther workup is indicated in variables. Radiology. 2009;250(3):648-657.
patients with this entity.
300
71-year-old woman with s/p bilateral mastectomy due to breast cancer:
now painful lump

1. What is the next step in a patient with bilateral


mastectomy and palpable mass?

2. What is the di erential diagnosis o a well-


circumscribed hyperechoic mass?

3. What is the typical ultrasound-guided biopsy


procedure?

4. What are the techniques to di erentiate mass


rom at lobules?

5. What scenario decreases sensitivity in


ultrasound?
301
Angiolipoma 264
Case ranking/dif culty: Category: Diagnostic

Ultrasound-guided biopsy of the mass with 11-gauge core biopsy Ultrasound-guided biopsy.
needle.

Answers 5. Large breast with atty replaced tissue does decrease


1. Ultrasound is the f rst choice. sensitivity o ultrasound to f nd small mass. Fat
is hypoechoic on ultrasound and most masses are
2. In general, a well-circumscribed homogeneously hypoechoic as well. There ore, the contrast is diminished
hyperechoic mass in the breast is a benign f nding. In a and masses can be disguised much easier. Fibrocystic
atty replaced breast, it can be classif ed as BI-RADS changes also impair the ability to f nd malignancy.
2—i there is any doubt or i it correlates to dense
f broglandular tissue, it could be ollowed over 2 years
and called BI-RADS 3 (“probably benign”). As soon
as it is palpable, it is more o a concern but again is Pearls
most likely benign. In this particular case, biopsy was • Angiolipomas account or 5% to 8% o benign atty
per ormed because it has grown since prior ultrasound. tumors and are a variant o lipomas.
In rare cases, malignancy, or example, lobular • Since the hallmark o an angiolipoma is the presence
carcinoma, can present in the orm o a hyperechoic o scattered microthrombi in small blood vessels, they
mass. can cause some pain.
3. FNA can be per ormed in selected cases— or example, • The etiology o angiolipomas is unknown, but some
f broadenomas can be diagnosed in the experienced investigators see an association with repetitive trauma.
hand. However, in most practices in the ultrasound, core
biopsies are the standard or ultrasound-guided biopsies.
Spring-loaded 14-gauge needle systems have been the Suggested Reading
standard or many years. These days, there is a tendency
Weinstein SP, Conant EF, Acs G. Case 59: Angiolipoma
to use vacuum-assisted core biopsy needles that are
o the breast. Radiology. 2003;227(3):773-775.
in general slightly larger and are 12 or 11 gauges in
diameter. The smaller the lesion, the more samples will
be necessary to be certain that an adequate sample has
been obtained.
4. Harmonic imaging can improve contrast between mass
and surrounding tissue. Power Doppler with patient
humming a deep sound (vocal remitus) can help to
distinguish mass rom surrounding tissue. Since at is
well per used in general, duplex and power Doppler are
o limited help. Increased pressure can help to see i
the suspected mass is real. Fat demonstrates de ormity
under pressure. A solid nodule does not show similar
de ormity.

302
Palpable lump in the left breast

1. What BI-RADS classif cation should be used


here?

2. What is the most likely pathology based on


the imaging?

3. What is the next best examination in a young


woman?

4. I a biopsy is per ormed or this lesion and


core biopsy shows f broepithelial lesion, what
do you recommend?

5. I ultrasound shows a solid mass and core


biopsy shows f broadenoma, what is the
management?

303
Young patient—sharply marginated cancer. Microlobulated margins 590
Case ranking/dif culty: Category: Diagnostic

Spot magni cation views for better characterization of margins Ultrasound of the palpable mass. This con rms the circumscribed
and to look for associated calci cations. Note how the margins nature felt on palpation. The appearances are similar to a
have “microlobulations” and angulations. hamartoma, with both hyperechoic and hypoechoic areas, and no
shadowing. They are sometimes di cult to perceive against the
prominent glandular tissue in young women.
Answers
1. The f nding o a mass in a young woman with what
appears to be a circumscribed mass at f rst viewing, but stability. Others advocate excision o the f broadenoma
on urther evaluation o the margins shows irregularity or with vacuum-assisted biopsy (especially i less that
2 cm in max diameter). This is usually done as part o
any other suspicious eature, should prompt a BI-RADS
4 and biopsy. the initial biopsy. Surgical excision is not medically
required, but many young patients request excision,
2. There are enough f ndings on imaging to indicate that even when proven benign.
this is not a f broadenoma, which is usually round or
more requently oval, with circumscribed margins.
There is no calcif cation to indicate DCIS, although in
younger women DCIS can present as a noncalcif ed mass. Pearls
Mucinous carcinoma usually has indistinct margins, and is • Cancers may present with circumscribed margins
easier to con use with a benign lesion on ultrasound. It is in young women.
more common in the elderly. There is no history o trauma • Noncalcif ed DCIS masses can present in this manner.
or bruising on the skin to indicate hematoma ormation. • Evaluate margins o mass to determine i any
3. A single tomosynthesis view may outline the margins suspicious eatures to prompt biopsy or suggest a
more clearly. Ultrasound is the best examination in diagnosis other than f broadenoma.
young women, especially as it is a nonionizing radiation
exam. MRI may help i proven cancer, and extent
di f cult to judge, but as the next step, it is expensive and Suggested Readings
not really indicated. PEM and BSGI may have a role in Chung J, Son EJ, Kim JA, Kim EK, Kwak JY, Jeong J. Giant
really dense breasts in young women, but the downside phyllodes tumors o the breast: imaging f ndings with
is the radiation dose. clinicopathological correlation in 14 cases. Clin Imaging.
4. A f broepithelial lesion is a relatively new pathological 2011;35(2):102-107.
entity, which can be under-sampled using core biopsy or Gwak YJ, Kim HJ, Kwak JY, et al. Ultrasonographic
vacuum biopsy, and surgical excision is recommended. detection and characterization o asymptomatic ductal
carcinoma in situ with histopathologic correlation. Acta
5. I the imaging appearances are concordant with a Radiol. 2011;52(4):364-371.
f broadenoma, many groups will discharge the patient to Yang WT, Hennessy B, Broglio K, et al. Imaging di erences
routine screening. Some groups would per orm short- in metaplastic and invasive ductal carcinomas o the
term clinical ollow-up with appropriate imaging or breast. AJR Am J Roentgenol. 2007;189(6):1288-1293.

304
46-year-old woman with new group of calci cations in the right
retroareolar breast (the two gures below additional electronic
magni cation)

1. What is the signif cance o group o “round


and oval” calcif cations?

2. What are typical benign proli erative changes


in the breast?

3. What is the di erence between atypical ductal


hyperplasia and DCIS?

4. What are important quality-assuring steps


in stereotactic core biopsy?

5. Why does atypical ductal hyperplasia need


to be surgically excised?

305
Atypical ductal hyperplasia 263
Case ranking/dif culty: Category: Diagnostic

Right magni cation view, with additional electronic magni cation. Right magni cation view, with additional electronic magni cation.

Answers 5. I patients with ADH diagnosed on core biopsy


1. “Round and oval” calcif cations as characterized on subsequently obtain surgical excision, histology is being
magnif cation views a ter workup o baseline screening upgraded to DCIS in about 6% to 44%, dependent on
mammogram are probably benign and can be ollowed over the study—and also the size o the needle. But even i
2 years, given the likelihood o malignancy o less than 2%. 9-gauge vacuum-assisted core biopsy needle was used,
Any developing group o “round and oval” calcif cations an upgrade will happen in more than 2%. Anything on
is suspicious (BI-RADS 4) and ought to be biopsied. They mammography that has a likelihood o less than 2% can
can represent adenosis but also low-grade DCIS. be ollowed according to the BI-RADS lexicon and can
be classif ed as “probably benign”—BI-RADS 3. Since
2. During the aging o the breast parenchyma, there is the ADH does exceed the number in subsequent surgical
process o involution with atrophy o f broglandular excision, it cannot be ollowed and needs to be excised.
tissue, and at the same time there is also a chance
or proli erative processes that make the reading o
mammograms di f cult. The lobules can proli erate and Pearls
orm f brocystic changes and adenosis, or i there is a
dominant f brotic component, even sclerosing adenosis. • Suspicious (BI-RADS 4) calcif cations can be due
All these three processes can result in ormation o to benign proli erative changes such as adenosis or
calcium. Where “milk o calcium” is a typical benign sclerosing adenosis or due to DCIS or even invasive
f nding, adenosis and sclerosing adenosis can orm “oval ductal carcinoma. It is impossible to di erentiate both
and round” calcif cations, and sclerosing adenosis can etiologies based on mammographic criteria. There ore,
orm even “pleomorphic” calcif cations. This cannot be biopsy is required or di erentiation.
di erentiated rom DCIS, and biopsy is necessary. • ADH is also a concordant f nding. Di erentiation to
DCIS is based on the number o ducts and the amount
3. It is only a quantitative di erence between atypical o cell layers being involved.
ductal hyperplasia (ADH) and DCIS. I there are more • Since likelihood o upgrade o ADH to DCIS on
than two lobules involved and a certain amount o cell excision biopsy exceeds the 2% rate required to quali y
layers that have proli erated, it is called DCIS. or BI-RADS 3, surgical excision is recommended.
4. The purpose o the stereotactic biopsy is not to remove
all calcif cations but to obtain a representative sample. It
is important to obtain specimen radiograph to determine Suggested Readings
i the calcif cations are sampled. To help the pathologist Jackman RJ, Birdwell RL, Ikeda DM. Atypical ductal
to reduce the amount o material, the core samples that hyperplasia: can some lesions be def ned as probably
contain the calcif cations can be separated. Clip is le t benign a ter stereotactic 11-gauge vacuum-assisted
in the target zone to acilitate needle localization in case biopsy, eliminating the recommendation or surgical
o subsequent lumpectomy. Postbiopsy mammogram is excision? Radiology. 2002;224(2):548-554.
obtained to determine i the clip has not migrated and is Verkooijen HM, Peterse JL, Schipper ME, et al.
in the correct location. One crucial step a ter obtaining Interobserver variability between general and expert
the pathology report is to determine i the histology pathologists during the histopathological assessment
results are concordant with the imaging. I not the area o large-core needle and open biopsies o non-palpable
has to be resampled or surgical excision is indicated. breast lesions. Eur J Cancer. 2003;39(15):2187-2191.
306
Stereotactic biopsy with 9-gauge vacuum-assisted needle for
asymmetry 6 months ago; now patient complains of pain in that area

1. What is the f nding on the spot compression


views?

2. The patient had pain in the area o recent


biopsy, what is the next step?

3. What do you expect to see on ultrasound a ter


stereotactic biopsy?

4. I ultrasound shows a large scar, what would be


an additional test to di erentiate between scar
and malignancy?

5. Why is ultrasound, in general, not per ormed


as standard a ter surgery/biopsy?

307
Scar after excisional biopsy 262
Case ranking/dif culty: Category: Diagnostic

Gray-scale ultrasound of the area of concern, showing clip and Duplex of the area does not show any ow.
“hypoechoic,”“irregular-shaped” mass with “posterior acoustic
shadowing.”

Answers
1. The spot compression views demonstrate the asymmetric Pearls
tissue with underlying “architectural distortion” but no • On imaging, scar can be seen a ter benign, large-bore,
other abnormality. vacuum-assisted biopsy.
• In case o concern about the appearance on imaging, it
2. Next step is ultrasound directed to the area.
is help ul to review the recent prebiopsy images and the
3. A ter 6-month biopsy with relatively large needle pathology results to reconf rm that the benign results
(9-gauge) we would expect a small scar. The scar, as seen were concordant with the imaging.
on the ultrasound images, is relatively large. However, • I there is remaining concern, option can be to per orm
given the recent normal pathology and the act that the a repeat biopsy or short-term 6-month ollow-up.
reason or the biopsy was asymmetry, this was thought to • MRI can also be help ul in case o concern to
be “probably benign”-BI-RADS 3 and 6-month ollow- di erentiate between scar and malignancy.
up mammogram and ultrasound was recommended.
Given the symptoms o ocal pain, MRI was suggested to
better di erentiate scar rom questionable malignancy. Suggested Readings
4. I the biopsy was per ormed at least 6 months ago, MRI Aichinger U, Schulz-Wendtland R, Krämer S, Lell M, Bautz
with contrast would be help ul to di erentiate between W. Scar or recurrence—comparison o MRI and color-
scar and possible malignancy. Scar or ocal f brosis coded ultrasound with echo signal amplif ers [in German].
would not enhance, whereas malignancy would enhance. Rofo. 2002;174(11):1395-1401.
5. Ultrasound is not very good in di erentiating scar rom Rosen EL, Soo MS, Bentley RC. Focal f brosis: a common
malignancy. It was per ormed because o the ocal pain breast lesion diagnosed at imaging-guided core biopsy.
o the patient. Fat necrosis or scar ( ocal f brosis) cannot AJR Am J Roentgenol. 1999;173(6):1657-1662.
be easily di erentiated rom malignancy on ultrasound.

308
Palpable painful lump in the right breast for 2 months

1. Which o the BI-RADS descriptors most


accurately represents the f ndings?

2. What is the BI-RADS descriptor or the margin


o this mass?

3. What is the most likely pathology?

4. This lesion turned out to be a spindle cell


lipoma on core biopsy. What is the treatment?

5. Is any o the ollowing advanced imaging tests


indicated in this condition?

309
Spindle cell lipoma 584
Case ranking/dif culty: Category: Diagnostic

Vascular solid mass. Does not look like a broadenoma. BI-RADS 4—requires biopsy.

Answers
1. The shape o mass is the f rst BI-RADS descriptor. Pearls
The description o the margins ollows the shape. • Rare spindle cell variant lipoma.
• FNA not help ul.
2. Smooth margins with a “narrow zone o transition”
• Core biopsy makes the diagnosis.
rom normal to abnormal.
• Treatment by wide surgical excision.
3. Depends on age. In a postmenopausal woman, a
developing circumscribed mass is more likely to be
malignant than in a patient with active hormones. Suggested Readings
4. The treatment o spindle cell tumors, even the lipoma Magro G, Bisceglia M, Michal M, Eusebi V. Spindle
variants, is similar to phyllodes tumor, with wide cell lipoma-like tumor, solitary f brous tumor and
surgical excision. Full pathological analysis o the whole myof broblastoma o the breast: a clinico-pathological
specimen will be able to distinguish between the benign analysis o 13 cases in avor o a uni ying histogenetic
and malignant variants o the disease. concept. Virchows Arch. 2002;440(3):249-260.
5. The core biopsy and surgical excision is usually all that Magro G, Michal M, Bisceglia M. Benign spindle cell
is required. There is no evidence that additional tests tumors o the mammary stroma: diagnostic criteria,
change the management o the patient. classif cation, and histogenesis. Pathol Res Pract.
2001;197(7):453-466.
Mulvany NJ, Silvester AC, Collins JP. Spindle cell lipoma
o the breast. Pathology. 1999;31(3):288-291.

310
Screening mammogram in asymptomatic
patient—what is the abnormality?

1. What is the f nding on mammography?

2. Is there any situation where a f nding can be


called BI-RADS 3 on screening mammogram?

3. What calcif cation descriptors are high risk and


imply the need or biopsy?

4. What would be the best way to address these


calcif cations as seen on magnif cation views?

5. What is the best technique to use in this case


or the biopsy?

311
“Fine linear calci cations”—DCIS 259
Case ranking/dif culty: Category: Screening

Diagnostic mammogram, right ML magni cation view Diagnostic mammogram, left CC magni cation view demonstrating
demonstrating “linear” calci cations. “linear” calci cations.

Answers
1. Noted is a subtle group o indeterminate calcif cations Pearls
in the le t retroareolar breast. The consequence is to • Certain BI-RADS descriptors or calcif cations do
recall the patient or diagnostic mammogram with imply malignancy and biopsy is required.
magnif cation views. • In general, any description o calcif cations as
“pleomorphe” or “f ne linear and branching” should
2. No. BI-RADS 3 is not an accepted conclusion or a
never be ollowed by BI-RADS 2 (benign) or BI-
screening mammogram. It is only acceptable a ter
RADS 3 (probably benign) assessment.
an appropriate workup in the orm o a diagnostic
• I calcif cations with suspicious orm and shape are
mammogram.
stable over 2 years, biopsy might still be required, since
3. “Lucent centered” calcif cations are benign and usually they could represent low-grade DCIS.
in the skin. “Coarse and popcorn like” calcif cations are • Biopsy o calcif cations should be per ormed with 9-
benign and most likely due to a hyalinized f broadenoma. or 11-gauge vacuum-assisted core biopsy needle.
“Dystrophic” calcif cations are, in most cases, benign.
“Pleomorphe and f ne linear” are descriptors that imply
“need or biopsy” because o tentimes these calcif cations Suggested Readings
are related to DCIS.
Bird RE. Critical pathways in analyzing breast calcif cations.
4. In this case— atty replaced breast parenchyma— Radiographics. 1995;15(4):928-934.
ultrasound does not help, since it is almost impossible D’Orsi CJ, Bassett LW, Berg WA, et al. Breast Imaging
that there is any mass hiding. MRI can be per ormed Reporting and Data System: ACR BI- RADS–
a ter positive biopsy to search or additional disease— Mammography. 4th ed. Reston, VA: American College o
again it could be argued that in a breast with atty Radiology; 2003.
replaced parenchyma, the need or an MRI is less. Philpotts LE, Shaheen NA, Carter D, Lange RC, Lee CH.
Surgical excision is not state o the art, without prior Comparison o rebiopsy rates a ter stereotactic core
biopsy and histology. Ductogram does not help—based needle biopsy o the breast with 11-gauge vacuum suction
on the morphology, the calcif cations are likely in a duct. probe versus 14-gauge needle and automatic gun. AJR Am
5. The pre erred needle system is a 11- or 9-gauge vacuum- J Roentgenol. 1999;172(3):683-687.
assisted core biopsy needle or calcif cations. This is
superior to a spring loaded 14-gauge needle system. The
purpose is not to remove all calcif cations but to sample
a representative group. Based on the results, lumpectomy
would take care o the abnormality. An FNA is not
yielding a su f cient sample or diagnosis.

312
Lumpectomy, follow-up with palpable lump

1. What BI-RADS is appropriate here?

2. What is the di erential diagnosis or the scar


changes in this patient?

3. In at necrosis, what is the structure that


calcif es?

4. What is the next best imaging test?

5. In patients who had breast conservation


surgery, when is the time o maximum
radiotherapy change in the treated breast?

313
Fat necrosis following intraoperative radiation therapy 606
Case ranking/dif culty: Category: Diagnostic

Left ML spot magni cation views. These show some Cavitating uid- lled lesion containing debris.
fatty lucency of the scar with ne calci cations seen
around the periphery.

Answers
1. The f ndings are characteristic o at necrosis ollowing Pearls
breast conservation therapy and intraoperative radiation • Post–conservation surgery ollowed by radiation
therapy. There is no evidence o malignancy, and there ore changes may be complicated by at necrosis.
a BI-RADS assessment o 2 (benign) is appropriate. • There is increased risk o at necrosis in patients
undergoing intraoperative radiation with brachytherapy.
2. The f ndings o a circumscribed atty lucency within a
scar are typical o at necrosis. In addition, calcif cations
develop in the periphery o the in ammatory change.
Suggested Readings
3. Classically, at necrosis calcif es irregularly in the walled
o liquef ed center. There is chronic in ammation, Budrukkar A, Jagtap V, Kembhavi S, et al. Fat necrosis
and calcif cations develop within the wall. Early in women with early-stage breast cancer treated
microcalcif cations can look very suspicious until they with accelerated partial breast irradiation (APBI)
coarsen up and become classically dystrophic. using interstitial brachytherapy. Radiother Oncol.
2012;103(2):161-165.
4. This f nding is characteristic enough to leave alone. Kuzmiak CM, Zeng D, Cole E, Pisano ED. Mammographic
However, as it was also palpable on physical exam, an f ndings o partial breast irradiation. Acad Radiol.
ultrasound was per ormed. MRI is not warranted, and 2009;16(7):819-825.
a diagnostic mammogram has already been per ormed. Orecchia R, Leonardo MC. Intraoperative radiation therapy:
Tomosynthesis may have a role in the initial diagnostic is it a standard now? Breast. 2011;20(Suppl 3):S111-S115.
exam, as it should be able to di erentiate between at
necrosis and a developing mass rom local recurrence,
but there are as yet no data on its use in this situation.
5. Following breast conservation, the risk o local
recurrence has a peak approximately 2 to 3 years
ollowing completion o therapy. With patients on
tamoxi en, there may be a second peak at around 6 to
7 years, but this has been mostly reduced by the use o
aromatase inhibitors. The maximum radiation change
occurs at 18 months and decreases over time.

314
Palpable lump in the right upper outer quadrant, marked with BB

1. I there are suspicious calcif cations in a


palpable area, what is the next step?

2. What are the characteristics o a papillary


cancer?

3. What is the next step a ter the imaging?

4. Why can highly di erentiated tumors be


a problem in MRI?

5. What is the role o duplex in ultrasound o


the breast?

315
Papillary carcinoma 258
Case ranking/dif culty: Category: Diagnostic

Ultrasound directed to the area Ultrasound demonstrates increased ow in the MRI of the breast with contrast
demonstrates mass with associated mass. demonstrating the index lesion in the
calci cations and “microlobulated” margin. right upper outer quadrant.

Answers 5. Ultrasound with duplex can help to assess the degree o


1. In our case, patient received spot compression views due malignancy in an otherwise suspicious lesion. It should not
to the distortion seen in the area o palpable abnormality. sway the examiner into avoiding biopsy in case o lack o
However, better would have been spot compression ow. There are low-grade malignancy not showing ow.
magnif cation views to assess the distribution o the Ultrasound with doppler can help to guide a biopsy needle
calcif cations and to better characterize the calcif cations. to avoid bleeding and to help to di erentiate between a
In this particular case, it did not matter too much since cyst and a solid nodule i ow is seen.
calcif cations were new and highly suspicious and
ultrasound did conf rm the presence o an associated mass.
It is always help ul to per orm ultrasound in the presence
Pearls
o dense tissue and highly suspicious calcif cations to
search or an invasive component o the malignancy. • Papillary and mucinous, tubular, and medullary
carcinomas belong to the subgroup o well-
2. Papillary cancers are a small raction o invasive cancers di erentiated invasive ductal carcinomas.
o the breast (1–2%). They belong to the group o highly • They can present with increased signal on T2-weighted
di erentiated breast cancers and have a better prognosis MRI images and can show increasing contrast
then invasive ductal carcinoma. They usually are located enhancement kinetics.
in the retroareolar region and can occur in a cyst. • This case demonstrates importance o ultrasound in
3. The best choice at this point is an ultrasound-guided addition to mammography in situation o suspicious
biopsy. It is more convenient then stereotactic biopsy and calcif cations in dense breast tissue to search or solid
will give the surgeon the in ormation necessary to plan invasive component o the malignancy.
surgery to the maximum benef t o the patient. • Increased ow on duplex images can re ect vascular
neogenesis to supply ast-growing masses—however,
4. Highly di erentiated carcinomas are a subgroup f nding is unspecif c and can also be seen in benign
o invasive ductal carcinoma including mucinous, masses, such as f broadenomas.
tubular papillary, and medullary carcinomas. They can
show more intermediate (tubular) or even high signal
(mucinous) on T2-weighted images. These tumors can
be round and well circumscribed (medullary, papillary, Suggested Readings
and mucinous carcinomas) and might have contrast Cosgrove DO, Kedar RP, Bamber JC, et al. Breast diseases:
enhancement patterns that are less specif c; or example, color Doppler US in di erential diagnosis. Radiology.
they could increase enhancement over time. 1993;189(1):99-104.
Soo MS, Willi ord ME, Walsh R, Bentley RC, Kornguth PJ.
Papillary carcinoma o the breast: imaging f ndings. AJR
Am J Roentgenol. 1995;164(2):321-326.

316
Nipple discharge and stabbing pains in the retroareolar area

1. What BI-RADS classif cation should be used


here?

2. Which o the ollowing conditions can be


a cause o nipple discharge?

3. What is the next best imaging test?

4. What are the clinical f ndings you anticipate


f nding in this patient?

5. What type o biopsy is best in Paget disease


o the nipple?
317
Calci cations in nipple 684
Case ranking/dif culty: Category: Diagnostic

Targeted ultrasound. This is technically di cult in the subareolar Retroareolar ultrasound. Calci cations within the nipple are still
area, especially when the patient has an inverted nipple. remarkably well seen.

Answers 5. Nipple biopsy is the standard care or Paget disease.


1. This patient has a bloody nipple discharge, and also Ductal lavage, i available, can detect abnormal cells
calcif cations in the nipple. Further investigation is associated with DCIS, even i we cannot see it on imaging.
warranted. The examination is a diagnostic exam, and
so a BI-RADS 4 assessment is appropriate.
2. The answer is that virtually anything can be a cause Pearls
o nipple discharge, which is very common condition, • Calcif cations within the nipple are unusual and should
and has a low risk o associated malignancy. The only prompt investigation.
types o discharge you need to be concerned about are • Patient has associated nipple inversion that can be
watery ( rom multiple papillomas, or DCIS) and bloody benign (most commonly) or malignant.
(papilloma, duct ectasia, periductal mastitis, etc.). • Check nipple or discoloration, which might indicate
Bloody nipple discharge associated with breast cancer is the presence o Paget disease o the nipple, and prompt
very rare. nipple biopsy or diagnosis.
3. Tomosynthesis may not help here, as the main f ndings • Paget disease is associated with DCIS.
are the calcif cations in the nipple. Spot magnif cation
views are there ore the best next test. I there is an
associated mass, ultrasound may help. PEM does not have Suggested Readings
any utility when the patient has relatively atty breasts. Echevarria JJ, Lopez-Ruiz JA, Martin D, Imaz I, Martin
MRI is reported to increase the diagnostic accuracy or M. Use ulness o MRI in detecting occult breast cancer
subareolar-associated cancer in Paget disease. associated with Paget’s disease o the nipple-areolar
4. Slit-like nipple retraction is ound in duct ectasia due complex. Br J Radiol. 2004;77(924):1036-1039.
to scaring and shortening o the central ducts. Flat Günhan-Bilgen I, Oktay A. Paget’s disease o the breast:
nipple retraction is a presenting f nding o retroareolar clinical, mammographic, sonographic and pathologic
invasive cancer. She has presented with a bloody nipple f ndings in 52 cases. Eur J Radiol. 2006;60(2):256-263.
discharge, so we may be able to see this spontaneously. Lim HS, Jeong SJ, Lee JS, et al. Paget disease o the breast:
Nipple discoloration along with purple patches on mammographic, US, and MR imaging f ndings with
the nipple areolar complex (NAC) skin are ound in pathologic correlation. Radiographics. 2011;31(7):
Paget disease, and are the area or the punch biopsy 1973-1987.
or incisional biopsy to be per ormed to establish the
diagnosis.

318
52-year-old patient with history of breast cancer and bilateral
mastectomy—patient feels new lump in the right medial chest wall

1. What is a an appropriate technique to ollow


patients a ter bilateral mastectomy?

2. Is clinical examination good enough to detect


recurrent malignancy a ter mastectomy?

3. What it the f nding seen on the submitted MRI


images?

4. What is the next step?

5. What is the next step i ultrasound does not


show the lesion?

319
Recurrent invasive ductal carcinoma after mastectomy 203
Case ranking/dif culty: Category: Screening

MRI after IV contrast and subtraction demonstrates an area of MRI T1-weighted images without IV contrast demonstrate small
mixed contrast enhancement kinetics in the right medial breast. mass in the medial right breast.

Answers
1. A reasonable approach is to per orm either ultrasound Pearls
or MRI. • Bilateral mastectomy does not 100% exclude the
possibility o breast cancer in the uture.
2. MRI and ultrasound are more success ul in detecting
• Ultrasound or MRI can be used or screening o
local recurrence than clinical examination. Ultrasound
recurrent breast cancer in patients with bilateral
is cheaper and more readily available and should be the
mastectomy.
f rst option. In case o suspicious f ndings on ultrasound,
MRI could increase the specif city.
3. Noted is a 5-mm enhancing mass in the right chest wall
Suggested Readings
near the medial contour o the silicone implant.
Vanderwalde LH, Dang CM, Tabrizi R, Saoua R, Phillips
4. Second look ultrasound is the next step to see i we can EH. Breast MRI a ter bilateral mastectomy: is it
see any correlate and i ultrasound-guided biopsy is indicated? Am Surg. 2011;77(2):180-184.
easible. Yilmaz MH, Esen G, Ayarcan Y, et al. The role o US
5. I the lesion is not seen on ultrasound, then we are in and MR imaging in detecting local chest wall tumor
trouble—the only option le t would be to send patient recurrence a ter mastectomy. Diagn Interv Radiol.
to breast surgeon. Breast surgeon will appreciate i we 2007;13(1):13-18.
can mark the lesion by clip or even needle localize the
lesion. The benef t to excise the lesion outweighs the
danger to injure the implant.

320
Recall from screening mammogram due to new indeterminate
calci cations in the right upper outer quadrant

1. What is the consequence o the screening


mammogram in this case?

2. Why are round and oval calcif cations in this


case not benign?

3. Why was in this case only one stereotactic


biopsy per ormed?

4. Why was MRI per ormed?

5. What is the signif cance o multicentric and


multi ocal disease?

321
Multifocal intermediate -grade DCIS 202
Case ranking/dif culty: Category: Diagnostic

Diagnostic mammogram, right Diagnostic mammogram, right MRI MIP image after IVcontrast demonstrates
magni cation ML view. magni cation CC view. multifocal disease in the right breast.

biopsy two groups o the suspicious calcif cations under


stereotactic guidance. However, in this particular case,
one biopsy was done with sterotaxis. The second was
per ormed under MRI guidance based on the f ndings.
4. MRI is per ormed to assess the patient i there is
MRI, T1-weighted sequence
additional disease, in particular, i there is additional
after IV contrast with
disease in other quadrants (multicentric disease). Based
subtraction demonstrates
multiple foci of increased
on the mammogram, there is already evidence o multiple
enhancement correlating to areas o disease in one quadrant (multi ocal disease).
the groups of calci cations. 5. Multicentric disease (more than one quadrant is
involved) requires, in general, mastectomy. Multi ocal
Answers disease (more than one lesion in one quadrant) can
o tentimes be addressed by lumpectomy. It is avorable
1. For the assessment o screening mammogram, there are to bracket the lesions at the time o needle localization to
only three options available. Either the mammogram is achieve best outcome.
normal (BI-RADS 1) or benign (BI-RADS 2), or there is
an abnormality that needs urther workup (BI-RADS 0)
and the mammogram is labeled as “incomplete” and
patient will be recalled or diagnostic mammogram. Pearls
This is the appropriate assessment in this particular case. • In case o the presence o multiple suspicious groups
2. Round and oval calcif cations can be classif ed as o calcif cations, stereotactic biopsy can be per ormed
BI-RADS 3 (probably benign) in a diagnostic workup o two groups most apart rom each other to prove the
with magnif cation views based on a f rst screening extent o the disease.
mammogram i there are no old exams available. Then • I patient will get preoperative MRI, only one biopsy
they should be ollowed in 6 months and subsequently can be per ormed o the most suspicious group o
another 6 months and then a year later with magnif cation calcif cations and urther biopsy can be determined
views to prove stability over 2 years. In this case, however, based on the extent o disease as seen on MRI.
the calcif cations were new and there ore suspicious
BI-RADS 4. Remember, never classi y a screening
mammogram as probably benign (BI-RADS 3)—this can Suggested Reading
be done only a ter a diagnostic workup.
Esserman L, Hylton N, Yassa L, Barclay J, Frankel S,
3. The goal o the biopsy is to prove the extent o the Sickles E. Utility o magnetic resonance imaging in the
malignancy to guide the surgical procedure to avoid management o breast cancer: evidence or improved
positive margins and to determine the appropriate preoperative staging. J Clin Oncol. 1999;17(1):110-119.
surgical procedure. In this case, it would have been to
322
Screening—asymptomatic

1. What is the risk o malignancy


in an asymmetry?

2. What is the next best examination you


recommend?

3. Which type o cancer can present with these


f ndings?

4. What BI-RADS assessment would you give


this f nding?

5. What type o biopsy would you recommend?

323
Lobular cancer presenting as asymmetry 672
Case ranking/dif culty: Category: Screening

4. The f nding is suspicious, but this is a screening exam, so


the correct answer is that this is a BI-RADS 0, recall or
urther workup. I this is a screening examination, then
this f nding should lead to a recall or urther evaluation,
and a hunt or prior mammograms.
5. FNA cytology is technically di f cult with lobular
carcinoma, and it is important to alert the pathologist
to the potential or a lobular cancer at the time o the
biopsy. Core biopsy is certainly the best way o both
getting a diagnosis and per orming routine tumor
biomarkers. There are no signif cant calcif cations
associated with this tumor, and so stereotactic core
biopsy is not indicated. The lesion is not palpable, so
guidance will be required. Preoperative diagnosis should
be per ormed rather than going direct to surgery.

Targeted ultrasound—vague isoechoic mass at the site


of mammographic abnormality.
Pearls
Answers • Lobular cancer is well known or being subtle in
1. Malignancy is very rare in an asymmetry, but this presentation with atypical eatures at mammography.
is modif ed by whether it is preexisting or new. • MRI is an important part o the diagnostic workup.
A developing asymmetry is always suspicious and needs • Mammographic eatures o lobular cancer are as
urther evaluation. An asymmetry that has been stable on ollows:
prior f lms or more than 3 years can be presumed to be • Asymmetry
benign. • Distortion
• Reduction in breast volume o a ected size.
2. The f rst thing that has to be addressed is the question
o whether this is a real abnormality, and repeating the
same views to see i still present (especially i young and
at di erent stage o cycle). Second, does it press out. Suggested Readings
Once these questions have been addressed, then one can Choi BB, Kim SH, Shu KS. Lobular lesions o the breast:
move on to ultrasound targeted to the mammographic imaging f ndings o lobular neoplasia and invasive lobular
f nding. I this is still negative, but you still suspect carcinoma. J Reprod Med. 2012;57(1-2):26-34.
an abnormality is present, then an MRI would help to Heller SL, Moy L. Imaging eatures and management o
include or exclude malignancy. high-risk lesions on contrast-enhanced dynamic breast
MRI. AJR Am J Roentgenol. 2012;198(2):249-255.
3. An asymmetry is usually a lobular cancer, although a
Kim SH, Cha ES, Park CS, et al. Imaging eatures o invasive
developing ocal asymmetry can be due to a high-grade
lobular carcinoma: comparison with invasive ductal
invasive ductal cancer. Lobular cancer is hard to detect
carcinoma. Jpn J Radiol. 2011;29(7):475-482.
commonly presenting in atypical ways, such as vague
distortion, a shrinking breast, or vague asymmetry.

324
92-year-old woman feels lump in the left nipple—any abnormal nding?

1. What is Paget disease o the breast?

2. What is the percentage o Paget disease among


all breast malignancies?

3. Is there is risk o additional disease?

4. What could be the next step in this particular


case?

5. Is there any abnormality seen around the nipple


on the current exam?

325
Paget disease 201
Case ranking/dif culty: Category: Diagnostic

Spot compression view of left CC


showing protruding mass in the left Ultrasound of left retroareolar breast demonstrates the mass
nipple and suspicious calci cations. within the nipple.

Ultrasound of left retroareolar breast with duplex demonstrates


mass within the nipple with strong increased ow on duplex.

Answers
1. Paget disease is an uncommon orm o breast cancer Pearls
with typical appearance. • Paget disease is a rare orm o breast cancer,
characterized by the presence o intraepidermal tumor
2. One to f ve percent o all breast carcinomas are Paget
cells, o ten involving the nipple.
disease.
• Appearance o the nipple includes pruritus and eczema.
3. Paget disease is o tentimes associated with DCIS or • One to f ve percent o all breast cancers present as
invasive ductal carcinoma. DCIS is o tentimes high Paget disease.
grade (comedo type). Fi ty percent o patients have • Paget disease is o tentimes associated with DCIS.
additional abnormalities on the mammogram in the same • DCIS is o tentimes high grade and comedo type.
breast.
4. When mammogram conf rms lack o additional
abnormality, patient can be send to breast surgeon or Suggested Readings
biopsy. The lesion is accessible to inspection and does Cardenosa, G. Breast Imaging Companion. 2nd ed.
not require imaging guidance. Philadelphia, PA: Lippincott Williams & Wilkins; 2001.
5. Noted best on the CC view is a mass extending out o the Dalberg K, Hellborn H, Waermberg F. Paget’s disease o the
nipple as well as suspicious calcif cations. They are in nipple in a population based cohort. Breast Cancer Res
“linear distribution” and “f ne and linear” in shape. Treat. 2008;111(2):313-319.

326
Screening mammogram—any abnormality?

1. What is the pertinent f nding?

2. What would be the next step in the workup o


a spiculated mass seen on additional diagnostic
mammogram?

3. What would be the next step in the workup o a


questionable mass on screening mammogram?

4. What is the appropriate BI-RADS


classif cation o the screening exam?

5. What would change i patient had nipple


discharge?

327
“Irregular mass”—invasive ductal carcinoma 200
Case ranking/dif culty: Category: Screening

Diagnostic mammogram, Diagnostic Ultrasound of right breast con rming Ultrasound of right breast with duplex
right MLO view mammogram, right CC the presence of an “irregular and con rming the presence of a spiculated,
demonstrating mass with view con rming the angulated” mass. angulated anechoic mass with some
“irregular” shape right presence of mass with ow in the periphery on duplex.
11:00 close to the nipple. “irregular” shape close
to the nipple at 11:00.

Answers
1. Noted is a “questionable mass” in the right retroareolar Pearls
breast, middle depth. This would be an appropriate • Ultrasound is always indicated i new “mass” or
description on a screening mammogram report because “asymmetry” is seen on mammogram.
diagnostic views need to conf rm i the suspected mass • Any biopsy per ormed on ultrasound is in general
is real. Mass can then be urther described on spot easier to per orm and more convenient than biopsy
compression views. per ormed on the stereotactic biopsy table or upright
stereotactic system.
2. Ultrasound is in general the next step because
• I ultrasound ails to show corresponding f nding,
ultrasound-guided biopsy is pre erred over stereotactic
stereotactic biopsy should be per ormed.
biopsy.
3. Next step is to call the patient back or additional SC
views and i the spiculated mass persists, then additional
Suggested Readings
ultrasound targeted to the area o concern.
Meyer JE, Kopans DB, Stomper PC, Lind ors KK. Occult
4. This is typical situation or an indeterminate f nding on breast abnormalities: percutaneous preoperative needle
screening mammogram, which requires urther workup localization. Radiology. 1984;150(2):335-337.
with diagnostic mammogram and there ore it should be Sickles EA. Mammographic eatures o “early” breast
called BI-RADS 0 (“incomplete”). cancer. AJR Am J Roentgenol. 1984;143(3):461-464.
5. I patient had nipple discharge, study should
be diagnostic mammogram and not a screening
mammogram. Otherwise, workup would still include
spot compression views and ultrasound. Since there is
an abnormality that can explain discharge, no need or
ductogram.

328
Screening—asymptomatic

1. What is the BI-RADS category or this


screening exam?

2. Where in the breast do you think the


lesion lies?

3. What are the appropriate tests during


diagnostic workup in this patient?

4. How would you describe the calcif cations,


shown?

5. What is the likely pathology in this patient?

329
Small screening cancer with distracters 681
Case ranking/dif culty: Category: Screening

Left CC spot magni cation shows Left ML close-up view shows that the mass lies in Targeted ultrasound to the mammographic
ill-de ned mass, and a cluster of the lower half of the breast, and not in the upper nding shows an “ill-de ned mass” deep in
amorphous calci cations anterior breast as we rst thought. This was con rmed the breast disc with a “wide zone of transition”
to the index lesion. with MRI, which showed only the single index (bright halo).
cancer at 6 o’clock.

Answers 5. Although at necrosis can mimic malignancy (even


1. This is an abnormal screening exam, so a BI-RADS 0 or giving BI-RADS 5 appearances), there is no history o
urther workup is indicated. trauma to support this, or hematoma. This lesion is likely
to be a standard invasive ductal adenocarcinoma.
2. Based on the images at screening, we thought the lesion
was going to be in the 12 o’clock position in the le t
breast. It was only when we did the diagnostic views that
the subtle appearances o a mass were seen in the lower Pearls
hal , and the upper hal asymmetry was conf rmed to be • Localizing a lesion in a quadrant is important when you
superimposition only. want to per orm targeted ultrasound.
3. Many views may be appropriate, especially when you • Tomosynthesis should assign the, problem o the one
suspect something strongly and the ollow-up f lms do view asymmetry, to the history books.
not assist in the diagnosis or to target the lesion with
ultrasound. In this patient, spot magnif cation views
showed no abnormality in the upper hal o the le t Suggested Reading
breast. In these cases, MRI is use ul to ensure that there
is only a single lesion, and also to establish its position Venkatesan A, Chu P, Kerlikowske K, Sickles EA, Smith-
in the breast. You can then use second look ultrasound to Bindman R. Positive predictive value o specif c
identi y the lesion and target biopsy. That is the approach mammographic f ndings according to reader and patient
we took with this patient. Some centers use “screening” variables. Radiology. 2009;250(3):648-657.
ultrasound to determine position, rather than using
targeted ultrasound. This is easier with automated breast
ultrasound.
4. The calcif c particles are di f cult to characterize in this
patient. They have no specif c orm, and are there ore
“amorphous.” “Punctate calcif cations” are f ne pin-
prick-like calcif cations with well-def ned smooth
outlines. They should be less than 0.5 mm in size.
Stereotactic core biopsy showed microcystic change
with benign microcalcif cations.

330
82-year-old patient with screening mammogram—no old images
available: any suspicious ndings in the right breast?

1. What is the abnormality?

2. What is the next step i there is a group o


“pleomorphic” calcif cations on magnif cation
views?

3. What histological diagnosis would be


concordant with the imaging?

4. Why is it important to try to di erentiate


between low- and high-grade DCIS?

5. What do descriptors imply?

331
“Pleomorphic” calci cations—high-grade DCIS 168
Case ranking/dif culty: Category: Screening

Diagnostic mammogram, right magni cation ML view Diagnostic mammogram, right magni cation CC view
demonstrating group of “pleomorphic” calci cations. demonstrating group of “pleomorphic” calci cations.

Answers descriptors such as “coarse and heterogeneous,”


1. Group o calcif cations in the retroareolar breast, “pleomorphic,” and “linear and branching” correlate to
in erior and medial. Based on a screening mammogram, the Tabar descriptors such as “coarse granular,” “crushed
f ndings should not be urther characterized but called stone,” and “casting,” which are more likely ound in
“indeterminate” and more detailed characterization and high- and intermediate-grade DCIS.
assessment should be based on the urther workup with
diagnostic mammogram including magnif cation views.
2. Stereotactic biopsy is the next step to get a histological Pearls
diagnosis. A ter the result is obtained, a decision has to • DCIS can be divided into high- and low-grade DCIS,
be made i the results are concordant with the imaging which have di erent clinical behavior and signif cance.
f nding. • Shapes such as “pleomorphe” and “f ne and linear”
3. DCIS is the most likely corresponding diagnosis—it (BI-RADS) and also “casting” or “crushed stone”
could be low grade, but in this case it is more likely (Tabar) are more likely to represent high-grade DCIS.
higher grade given the “pleomorphic” appearance o the • Shapes such as “round and oval,” “amorphous
calcif cations. An invasive component could also be part or indistinct” (BI-RADS), and “f ne granular” or
o the process. “powdery” (Tabar) are more likely to represent low-
grade DCIS.
4. High-grade DCIS is a serious condition, which will
progress rapidly and o tentimes come with an associated
so t-tissue abnormality (mass) that can be detected by
additional ultrasound. Low-grade DCIS is a very slow Suggested Readings
progressive change, which, only in some cases, might Bird RE. Critical pathways in analyzing breast calcif cations.
eventually progress into higher grade malignancy. Radiographics. 1995;15(4):928-934.
D’Orsi CJ, Bassett LW, Berg WA, et al. Breast Imaging
5. BI-RADS descriptors such as “round and oval” and
Reporting and Data System: ACR BI-RADS–
“amorphous and indistinct” correlate to the Tabar
Mammography. 4th ed. Reston, VA: American College o
descriptors such as “pearl like” and “powdery,” which
Radiology; 2003.
are more likely ound in low-grade DCIS. BI-RADS

332
79-year-old woman with palpable lump—MRI, no mammogram

1. What are the typical f ndings o an abscess?

2. What would be included in the management


o an abscess.

3. What is idiopathic granulomatous mastitis?

4. Why was MRI per ormed as f rst test?

5. What would be the likely di erential


diagnosis?

333
Poorly di erentiated carcinoma with necrosis 164
Case ranking/dif culty: Category: Diagnostic

Coronal PET CT demonstrating large mass in the left breast CT demonstrating nodules in both lung bases consistent
taking up FDG. with metastasis.

Answers 5. The di erential diagnosis to abscess it poorly


1. Un ortunately, some symptoms are not very specif c, di erentiated carcinoma.
such as lymphadenopathy and thickening o the skin.
More specif c in avor o an abscess in comparison
with tumor are the presence o ever and more sudden Pearls
development o the mass.
• This case illustrates the need or a good clinical
2. Depending on the size, the f rst choice will be to treat correlation with the imaging f ndings to determine the
the patient with antibiotics. I this does not yield any correct diagnosis.
improvement, the thought should include malignancy • Most o ten, skin erythema, tenderness, and ever as
and biopsy should be per ormed (in general ultrasound seen in abscess are not present with breast cancer.
guided). In general, surgeons pre er surgical excision o • However, in ammatory breast carcinoma and
an abscess over drainage. Steroids are not a pre erred lymphoma can present with similar symptoms and
choice and would make the situation worse. imaging appearance as an abscess.
3. This is a rare entity, which can be ound o tentimes in • Any ailure o traditional management to resolve
middle eastern countries and o tentimes in younger symptoms o an abscess or in ection should prompt
patients. It is in general a diagnosis o exclusion based tissue diagnosis to exclude malignancy.
on the lack o improvement o a presumed in ammation
with antibiotics and subsequent biopsy showing
granulomas. Treatment usually includes local excision Suggested Readings
and steroids.
Bani-Hani KE, Yaghan RJ, Matalka II, et al. Idiopathic
4. In general, the f rst choice would be mammogram granulomatous mastitis: time to avoid unnecessary
and then ultrasound. In this particular case, the pain mastectomies. Breast J. 2004;10(4):318-322.
o the patient was so strong that she did not tolerate Bland KI, Copeland EM et al. The Breast 4th Ed. Saunders
mammogram. Elsevier; 2009:145-149.

334
Palpable lumps in the right breast

1. In which plane is this mass sited?

2. What is the likely underlying pathology?

3. What type o enhancement do you expect on a


delayed scan?

4. What signal do you expect this mass to show


on T2 images?

5. What biopsy technique would you recommend


to get a diagnosis?

335
Nodal metastases—unusual 619
Case ranking/dif culty: Category: Diagnostic

4. Nodes are typically T2 bright.


5. These nodes are usually visualized on ultrasound, and
either FNA cytology or core biopsy under ultrasound
guidance is suitable or tissue sampling. Usually, the
node is noticed on MRI, and the ultrasound is a second
look, as with axillary staging, these are not within our
examination f eld.

Pearls
• Rotter nodes are a rare f nding.
• They are not usually ound on ultrasound.
• They are usually ound on MRI scans or proven
MRI breast with contrast. This slice is below the level of the Rotter
cancer.
node and shows the primary breast cancer as well as a metastatic
intramammary lymph node.

Suggested Readings
Answers
Bembenek A, Schlag PM. Lymph-node dissection in breast
1. This is a “Rotter node,” which lies in the plane between
cancer. Langenbecks Arch Surg. 2000;385(4):236-245.
the pectoralis major and minor.
Chandawarkar RY, Shinde SR. Interpectoral nodes in
2. This is a metastatic Rotter node. This is not normally carcinoma o the breast: requiem or resurrection. J Surg
excised i the patient is having axillary clearance. MRI Oncol. 1996;62(3):158-161.
is the best method or detecting this type o nodal Cody HS, Egeli RA, Urban JA. Rotter’s node metastases.
metastasis. I not treated by excision, potentially it is a Therapeutic and prognostic considerations in early breast
source o recurrent disease. carcinoma. Ann Surg. 1984;199(3):266-270.
3. This is a lymph node and there ore washes out rapidly.
It has a suspicious kinetic curve.

336
Screening mammogram—any abnormality?

1. What is the signif cance o abnormal lymph


nodes seen on mammogram?

2. What is the next step in evaluation


o the lymph nodes?

3. What are the typical ultrasound eatures


or malignancy o lymph node?

4. I lymphoma is suspected, what tests


are pre erred?

5. What could be the etiology or calcif cations


within axillary lymph nodes?

337
Lymphoma 163
Case ranking/dif culty: Category: Screening

Ultrasound of left axilla demonstrates eccentric thickening of the Ultrasound-guided needle biopsy with 14-gauge device.
cortex of the lymph nodes (more than 3 mm).

Answers metastases rom extramammary primary carcinoma,


1. Although there are studies suggesting 3.3 cm length or a in particular with metastatic ovarian carcinoma.
non– at-containing lymph node as threshold or possible Axillary lymph nodes calcif cations can also be seen in
malignancy, ultrasound is superior to mammography in association with benign processes such as granulomatous
evaluating lymph nodes. In a clinically asymptomatic disease, or example, rheumatoid arthritis.
patient, even smaller nodes can be suspicious in the
appropriate setting, as seen in this case. It was in
particular concerning, since lymph node had developed Pearls
since last mammogram.
• Evaluation o lymph nodes on mammography is
2. Most help ul is urther evaluation with ultrasound. limited and changes over time, and size o the nodes is
Morphological eatures such as thickening o the cortex an important eature or possible concern.
o lymph node (more than 3 mm), penetrating cortical • According to the study o Walsh et al. (1997), the
vessels, are seen only on ultrasound and help to decide i length o non- atty lymph node o more than 33 mm
biopsy is necessary. had specif city or malignancy o 97%.
3. Suspicious eatures are irregular eccentric thickening • In general, core biopsy o lymph nodes is easible.
o the cortex o the lymph node more than 3 mm in I technically not possible, f ne needle aspiration can
diameter and the presence o penetrating vessels outside be per ormed.
the hilum at the cortex o the lymph node.
4. Primary lymphoma o the breast is extremely rare—
about 2.5% o all extranodal orms o lymphoma. Suggested Readings
Core biopsy is pre erred to obtain enough material or Abe H, Schmidt RA, Kulkarni K, Sennett CA, Mueller JS,
immunohistochemistry. However, i core biopsy cannot Newstead GM. Axillary lymph nodes suspicious or breast
be per ormed due to the location o the node, f ne needle cancer metastasis: sampling with US-guided 14-gauge
aspiration is an alternative. core-needle biopsy—clinical experience in 100 patients.
Radiology. 2009;250(1):41-49.
5. Metastatic breast carcinoma is the most common
Walsh R, Kornguth PJ, Soo MS, Bentley R, DeLong DM.
etiology or axillary lymph node calcif cations as seen in
Axillary lymph nodes: mammographic, pathologic,
about 3% o breast malignancies. In rare cases, axillary
and clinical correlation. AJR Am J Roentgenol.
lymph nodes with calcif cations could be related to
1997;168(1):33-38.

338
52-year-old patient with palpable abnormality in the left breast

1. What is the f nding on mammogram?

2. Why is this not a f broadenoma?

3. What would be the reason to per orm biopsy?

4. Why is ollow-up not needed, despite the act


that it is palpable?

5. What is the di erence between lipoma and


adenolipoma?

339
Lipoma 162
Case ranking/dif culty: Category: Diagnostic

Ultrasound directed to the area of lump demonstrating Ultrasound with compression demonstrates deformity of the mass
“oval mass” with “circumscribed margins,”“hyperechoic” in with pressure.
comparison with the surrounding fat.

Answers Remember, even the more complex at-containing lesions


1. The area marked with BB demonstrates atty tissue best (hamartomas) are called benign (BI-RADS 2).
seen on the MLO projection and no ocal abnormality. 5. I the lesion contains only at, it is called lipoma—i
2. Although f broadenomas are more common in young there is more complex on imaging, it is called, in
patients, they can also be seen in older patients. This general, f broadenolipoma or hamartoma. This implies
is especially the case in the presence o hormone that it contains at, glandular, and f brous tissue. Bottom
replacement therapy. In this particular case, the mass is line—as soon as a lesion contains at on mammogram
not seen at all on mammogram and is very so t under and has no eatures suggesting malignancy, it is benign
compression. This is typical or the presence o lipoma. (BI-RADS 2). This includes the more complex o the
three entities, hamartoma.
3. In general, palpable f ndings are suspicious because
we can assume that they did grow—since they were
not palpable previously—and there ore biopsy should
be considered. This also applies in general to relatively Pearls
benign-looking lesions such as masses most consistent • In young patients, the most common mass is
with f broadenomas or “complicated cysts” or “complex f broadenoma.
masses” with cystic components. However, i a palpable • In older patients, the most common mass is cyst.
f nding demonstrates typical morphology o f broadenoma, • In postmenopausal emales, it is unusual to see
many mammographers tend to ollow the lesion over development o f broadenoma, which requires, in
2 years and call it probably benign. I there are multiple general, stimulation by estrogens.
scattered similar benign-appearing lesions, the level o • I mass is benign in appearance on ultrasound and
suspicion will be lower and ollow-up might be considered there is no mass seen on corresponding atty-replaced
o multiple benign-appearing palpable f ndings. In case o mammogram, f nding is lipoma and benign.
a simple lipoma, in general, no ollow-up is required.
4. There are no case reports o malignant trans ormation o a
lipoma into malignancy in the literature. There is only one Suggested Readings
report o trans ormation o a adenolipoma (hamartoma) o
Mendiola H, Henrik-Nielsen R, Dyreborg U, et al. Lobular
the breast with the presence o lobular carcinoma in situ.
carcinoma in situ occurring in adenolipoma o the
There ore, in general, BI-RADS 2 is adequate and concern
breast. Report o a case. Acta Radiol Diagn (Stockh).
is less in comparison with a f broadenoma that contains
1982;23(5):503-505.
f broglandular tissue and has there ore a higher potential
Stavros TA. Breast Ultrasound. 1st ed. Philadelphia, PA:
or malignant trans ormation. I the lesion remains
Lippincott Williams & Wilkins; 2004.
growing in size, excision, however, could be considered.

340
Prior breast cancer, treated with breast conservation
and radiation therapy

1. What BI-RADS classif cation should be


used here?

2. What BI-RADS descriptors would you use


or this calcif cations?

3. Calcif cations are seen in the lymph nodes.


What is the di erential diagnosis?

4. What is the likely pathology in this case?

5. What is the most likely type o tumor that


the patient had conservation or?

341
Recurrent DCIS following surgical and radiation therapy 1795
Case ranking/dif culty: Category: Diagnostic

4. Recurrent tumor can be associated with calcif ed DCIS,


but sometimes when this is biopsied, all that is ound
is calcif cations associated with dead cancer cells in
milk ducts (“treated” DCIS). In the setting o breast
conservation, any developing microcalcif cation should
be regarded as suspicious.
5. In general, a recurrent tumor tends to have the
same appearances as the initial primary cancer. So,
i the patient presented with microcalcif cations,
then recurrence is also likely to present with
microcalcif cations.

Pearls
Right CC spot magni cation view shows “segmental” distribution • Larger areas o DCIS preop are more likely to recur,
of “ ne pleomorphic” microcalci cations: “casting type” (Tabar even a ter ull radiation therapy.
classi cation indicating calci cation within ducts and not • Usually between years 2 and 4 posttreatment.
terminal ductal lobular units (TLDUs)). • Easy to spot, as developing calcif cations in treated breast.

Answers
1. Classic “linear” and branching “pleomorphic Suggested Readings
microcalcif cations” consistent with high-grade DCIS, Kane RL, Virnig BA, Shamliyan T, Wang SY, Tuttle
in a background o residual postradiation change. BI- TM, Wilt TJ. The impact o surgery, radiation, and
RADS 5: highly suspicious or malignancy. systemic treatment on outcomes in patients with
ductal carcinoma in situ. J Natl Cancer Inst Monogr.
2. This is a great example o “f ne pleomorphic”
2010;2010(41):130-133.
calcif cations. Many o the individual calcif c particles
Lewis-Jones HG, Whitehouse GH, Leinster SJ. The role
have irregular borders with both a “crushed stone” and
o magnetic resonance imaging in the assessment
“casting” appearance (Tabar classif cation indicating
o local recurrent breast carcinoma. Clin Radiol.
site within TLDU and ducts). Secretory calcif cations
1991;43(3):197-204.
are “linear” and “branching” and are a BI-RADS
Ralleigh G, Walker AE, Hall-Craggs MA, Lakhani SR,
special case, and look di erent to these calcif cations.
Saunders C. MR imaging o the skin and nipple o
“Dystrophic” calcif cations: the calcif cations o DCIS
the breast: di erentiation between tumour recurrence
are a type o dystrophic calcif cations rom dead cancer
and post-treatment change. Eur Radiol. 2001;11(9):
cells, but dystrophic calcif cations postradiation are
1651-1658.
usually related to at necrosis.
3. This patient did not have concomitant lymphoma,
although it is a cause o calcif ed nodes. Sarcoid is the
most common cause o nodal calcif cations. Tattoo ink
uses heavy metal pigments, and this may travel up the
lymphatics into the nodes and present as calcif ed nodes.
Extravasated silicone may also end up as densities within
the lymph nodes.

342
57-year-old woman with screening mammogram, patient is
asymptomatic (priors on left)

1. What is the subtle f nding comparing the new


images with the old screening mammogram?

2. What would be the f rst step o the workup


o the patient?

3. What is the next step o the workup?

4. What is your f nal assessment and how would


you choose your next step?

5. What are the next steps a ter the ultrasound-


guided biopsy?

343
Invasive ductal carcinoma 141
Case ranking/dif culty: Category: Screening

Mammogram right spot compression MLO Mammogram right spot Ultrasound con rms small mass in the right breast
view con rming the presence of small mass compression CC view con rming at 12:00 orientation: “taller-than-wide” (Stavros) and
in the right breast at 12:00 orientation. The the presence of small mass in the “angular” shape.
mass demonstrates “irregular” shape and right breast at 12:00 orientation.
“indistinct” margins. The mass demonstrates
“irregular” shape.

Answers maybe a ter a trauma—could be an outcome that could


1. Noted is a new small “ ocal asymmetry” in the right explain the f nding and there ore BI-RADS 4 was chosen.
superior breast. The term ‘nodular density’ should not be 5. It is crucial to leave a clip to mark the area o biopsy
used as it is not a BI-RADS descriptor. and also to conf rm that the clip is in the correct position
2. The appropriate workup would include a spot by repeating mammogram o the right breast. This
compression MLO and CC views and a ull-f eld ML will allow you to f nd the area or subsequent needle
view to better localize the lesion. The purpose is to localization and surgery.
conf rm that the lesion is real and that this is not due to
projection o normal f broglandular tissue. Additional
views will also assess the lesion urther to determine Pearls
the f nal assessment and level o concern. At this point, • Any developing “ ocal density” or new “mass” is
biopsy is easible i the mammogram workup conf rms worrisome or malignancy.
the presence o the abnormality. • “Nodule” is not a BI-RADS term.
3. Next step would be to see i ultrasound is able to • Ultrasound is able to show 3-mm suspicious mass, even
visualize the small lesion. I yes, this can urther assess in atty-replaced breast.
the level o concern and also will be able to be used • I ultrasound ails to demonstrate corresponding
as guidance or biopsy. I the lesion is not seen on f nding, stereotactic biopsy is recommended.
ultrasound—based on the mammogram f nding alone— • A ter ultrasound-guided biopsy, it is important to leave
biopsy is required. In case o negative ultrasound, a clip to mark the area.
stereotactic biopsy would be warranted.
4. This is based on BI-RADS descriptors: “mass,”
“irregular shape,” “indistinct margin” (on mammogram), Suggested Readings
and “antiparallel” or “taller than wide” (Stavros) on D’Orsi CJ, Bassett LW, Berg WA, et al. Breast Imaging
ultrasound and there ore suspicious in nature and should Reporting and Data System: ACR BI-RADS–
be classif ed as BI-RADS 4—you could argue it might be Mammography. 4th ed. Reston, VA: American College
even BI-RADS 5. Di erence would be that i pathology o Radiology; 2003.
demonstrates no malignancy, and classif cation as BI- Stavros AT, Thickman D, Rapp CL, Dennis MA, Parker
RADS 5, pathology cannot be trusted and excisional SH, Sisney GA. Solid breast nodules: use o sonography
biopsy is required. In this case, however, ocal f brosis— to distinguish between benign and malignant lesions.
Radiology. 1995;196(1):123-134.
344
50-year-old woman with screening exam (the two gures on the
extreme right) and prior exam (the two gures on the extreme left)

1. What is the epidemiology o invasive lobular


carcinoma (ILC)?

2. What are the typical eatures o ILC in


mammography and ultrasound?

3. What are the typical eatures o ILC on MRI?

4. Why is the ILC a di f cult tumor to detect by a


radiologist?

5. Why is breast MRI important, in particular,


or ILC?

345
Invasive lobular carcinoma 139
Case ranking/dif culty: Category: Screening

Ultrasound demonstrates large 2-cm “irregular mass” with


“posterior acoustic shadowing” in the left retroareolar breast.

Answers MRI, T1-weighted images after IV contrast demonstrating area of


1. ILC has increased in incidence rom 9.5% in 1987 to increased enhancement in the left superior breast.
15.6% in 1999. It accounts or approximately 10% to
15% o all invasive breast cancers.
2. Because o lack o desmoplastic reaction, the density Pearls
o ILC on mammography is less than other breast • Detection o ILC with mammography is limited and
cancers, and the growth pattern is o tentimes di use. ILC is o tentimes di f cult to see in dense breast tissue.
Mammography o tentimes shows a spiculated mass with • Any patient with palpable abnormality needs additional
“architectural distortion” or “ ocal asymmetry.” ILC is ultrasound.
o tentimes better seen on CC than on the MLO view. • Breast MRI is, in particular, important or preoperative
Ultrasound demonstrates, in general, an “irregular” assessment o ILC, since the extent o the tumor can be
mass with “posterior acoustic shadowing.” However, better delineated on MRI.
sometimes ILC appears with more subtle f ndings on • Comparison with old mammograms is crucial, in
ultrasound, such as thickening o the cooper ligaments particular in case o lobular carcinoma and in general
and di use septal hypoechoic thickening. or all screening mammograms.
3. MRI demonstrates the same characteristics as on the
other imaging such as “spiculated, irregular” mass. The
enhancement pattern is not very specif c and shows a Suggested Readings
variety o possible kinetics. O tentimes, however, there
is increasing contrast enhancement over time. Washout Lopez J, Bassett L. Invasive lobular carcinoma o the breast:
enhancement is not very o ten seen. On T2-weighted spectrum o mammographic, US, and MRI imaging
sequences, the signal is usually decreased and there f ndings. Radiographics. 2009;29:165-176.
might be peri ocal edema identif ed. Veltman J, Boetes C, Van Die L, et al. Mammographic
detection and staging o invasive lobular carcinoma. Clin
4. Because o lack o desmoplastic reaction, there are less Imaging. 2006;30(2):94-98.
likely secondary signs such as nipple retraction. The
alse-negative rate rom mammography is high and
sensitivity is only between 57% and 81%.
5. MRI does help to show additional f ndings, since in
32% there is additional ipsilateral disease and about
7% additional contralateral disease. This is o tentimes
missed with mammography and ultrasound.

346
CT for suspected pulmonary embolus

1. What are the causes o enhancing breast tissue


on CT?

2. What BI-RADS classif cation should be used


here?

3. What is the most likely cause or these


appearances?

4. What is the next best imaging test?

5. What lesions on CT are ill def ned?

347
DCIS in dense breast 224
Case ranking/dif culty: Category: Diagnostic

Right CC spot magni cation view showing the extensive MIP reconstruction to show extensive non–mass-like enhancement
suspicious pleomorphic segmental microcalci cation, consistent in the right breast (of a di erent patient).
with high-grade DCIS.

Answers specif c gamma imaging (BSGI) potentially could be


1. All o the above are potential causes o enhancing breast used i conventional imaging is inconclusive.
tissue. Enhancing breast tissue is not common on CT, 5. Di use enhancement is usually benign or normal.
and i di use and unilateral, it may represent DCIS. I More segmental change potentially could be DCIS.
bilateral, think o hormonal change. Masses may enhance, Fibroadenoma and phyllodes have similar benign-
but i smooth and circumscribed, more likely to be benign, looking mass-like lesions on CT. Radial scar could
and irregular masses are more likely to be malignant. potentially enhance with ill-def ned margins i extensive
2. None. BI-RADS was not written to report f ndings proli erative change or DCIS is associated with the scar.
on CT. The BI-RADS lexicon is used or reporting
mammography, breast ultrasound, and breast MRI.
Pearls
3. Although these appearances can occur unilaterally in • Segmental or regional enhancement o dense breast
only one breast in normal hormonal change, it is more tissue warrants at least a breast imaging workup.
usual to be bilateral and patchy. DCIS, which enhances, • In younger women, the DCIS may be noncalcif ed, and
is usually high grade, analogous to clumped ductal there ore unlikely to be seen on mammogram.
enhancement on MRI. PASH does not usually enhance. • Calcif cations, i present, can usually be seen on the
Both invasive lobular carcinoma and invasive ductal mammogram even in women with extremely dense
carcinoma usually present as a mass on CT, which does breasts.
not necessarily enhance. • MRI likely to be the best imaging test.
4. Mammography should be per ormed to pick up any
suspicious microcalcif cations that may potentially
represent DCIS. MRI is a good test in this situation, Suggested Readings
but judicious timing with the patient’s cycle is important, Lin WC, Hsu HH, Li CS, et al. Incidentally detected
as she is likely to have enhancing breast tissue given enhancing breast lesions on chest computed tomography.
her CT appearances. Ultrasound can be used when an Korean J Radiol. 2011;12(1):44-51.
abnormality can be targeted. Screening ultrasound in Taira N, Ohsumi S, Takabatake D, et al. Contrast-enhanced
this situation may give rise to a high alse-positive rate. CT evaluation o clinically and mammographically occult
Both positron emission mammography (PEM) and breast multiple breast tumors in women with unilateral early
breast cancer. Jpn J Clin Oncol. 2008;38(6):419-425.
348
74-year-old woman with “mass” seen on screening mammogram

1. How o ten among breast cancers are mucinous


carcinomas?

2. What is the characteristic appearance


on ultrasound?

3. Why is it help ul to di erentiate pure rom


mixed mucinous carcinomas?

4. What are the imaging eatures that avor the


presence o pure orm o mucinous carcinoma?

5. What is the di erence between a mass and


ocal asymmetry according to BI-RADS?

349
Mucinous (colloid) carcinoma 137
Case ranking/dif culty: Category: Diagnostic

Ultrasound, B-mode demonstrates “mass” with “macrolobulated”


with partially “irregular” shape (see arrow) but mostly “well
circumscribed.”

Diagnostic mammogram, left spot compression MLO view.

MRI T1-weighted sequence after IV contrast with mass in the left


breast and clip in the center of the mass corresponding to the MRI T2-weighted sequence demonstrating corresponding
prior ultrasound-guided biopsy. high-signal mass.

350
Mucinous (colloid) carcinoma (Cont.)

Answers asymmetry” as an area o increased density asymmetric


1. Mucinous carcinoma makes up about 1% to 7% o all to the other corresponding breast. “Focal density”
invasive mammary carcinomas. Prevalence is only 1% in is described as seen on two projections. Di erence
women younger than 35 years. between “global asymmetry” and “ ocal asymmetry”
is the size in relation to the size o the breast. Global
2. Mucinous tumors are usually well circumscribed, asymmetries are most o ten consistent with normal
round and oval, and isoechoic or hypoechoic to f broglandular tissue. However, special attention should
the subcutaneous at. The pure orm o mucinous be paid to any new or developing asymmetry.
carcinoma can show through transmission on ultrasound
and sometimes can be very di f cult to discern
rom surrounding at. They might also present as
heterogeneous complex masses with cystic elements Pearls
(about 40%). Any diagnosis o a mucinous carcinoma o • As many as 21% o mucinous carcinomas might not be
a mass with posterior acoustic shadowing will be highly detected on mammogram.
suggestive o a more mixed and orm o mucinous • There are mucinous carcinomas with “pure” and
carcinoma or so-called intraductal carcinoma with “mixed” histology.
colloid eatures. • The 10-year survival rate or “pure” orm is 87% to
90% and or “mixed” orm is 64% to 66%.
3. The 10-year survival time among patient with pure
• MRI is the best modality to di erentiate between both
mucinous tumors is 87% to 90.4% and that o among
orms o mucinous carcinoma.
patients with mixed tumors is 54% to 66%. The
likelihood o metastasis to the axilla is higher in mixed
orm.
Suggested Readings
4. Pure mucinous carcinomas more likely demonstrate
well-circumscribed margins on imaging and appear to be Lam WW, Chu WC, Tse MA, et al. Sonographic appearance
more homogeneous, less likely demonstrate septations o mucinous carcinoma o the breast. AJR Am J
on MRI or heterogeneous echogenicity on ultrasound, Roentgenol. 2004;182(4):1069-1074.
and more likely demonstrate increasing enhancement Monzawa S, Yokokawa M, Sakuma T, et al. Mucinous
over dynamic postcontrast images instead o washout carcinoma o the breast: MRI eatures o pure and
enhancement. mixed orms with histopathologic correlation. AJR Am J
Roentgenol. 2009;192:W125-W131.
5. The BI-RADS lexicon di erentiates between “mass” and
“asymmetry” in the way that an asymmetry does show
up only on one projection. However, BI-RADS also
describes the so-called “global asymmetry” and “ ocal

351
This page intentionally left blank
Patient with palpable abnormality right breast— rst-time mammogram

1. What are the pertinent f ndings in that


symptomatic patient?

2. Does the MRI show any additional f nding?

3. What is the appropriate description o the


ultrasonography f nding (image next page)?

4. What are the general reasons to per orm


mastectomy instead o lumpectomy?

5. What is the benef t o preoperative breast MRI?

353
Invasive ductal carcinoma left breast 1305
Case ranking/dif culty: Category: Diagnostic

Gray-scale ultrasonography demonstrating large, T1-weighted postcontrast sequence T1-weighted postcontrast sequence
highly suspicious mass in the right superior demonstrates the additional lesion demonstrates the index lesion in the
breast with “posterior acoustic shadowing” that near the chest wall. central right breast.
corresponds to the mammogram nding.

Answers
1. On the standard views (the spot compression views are Pearls
not submitted), noted is a large area o “ ocal asymmetry” • Adding breast MRI can show otherwise occult cancers
in the right superior breast with underlying “architectural in the ipsilateral and also in the contralateral breast.
distortion” and associated microcalcif cations. Also noted • In a study per ormed in a community practice breast
is signif cant thickening o the right skin. center, Fan et al ound, in a population o 445 patients,
84 additional malignancies in 66 patients (14.8%)
2. Noted on the MRI image is, in addition to the index
including 22 patients in contralateral breast (4.9%) and
lesion, additional 1.5-cm mass with strong enhancement
48 patients with ipsilateral additional malignancies
in the lateral posterior right breast. This is better seen on
(10.8%).
the axial images. It is not well seen on the mammogram.
• In 23.6% o these patients, MRI resulted in change o
It does show chest wall inf ltration.
the surgical procedure.
3. The index lesion as seen on mammogram demonstrates
corresponding hypoechoic mass with strong posterior
acoustic shadowing.
Suggested Readings
4. In general, the size o the lesion in comparison with the Fan C, Nemoto T, Blatto K, et al. Impact o pre-surgical
size o the breast will determine the need or mastectomy. breast magnetic resonance imaging (MRI) on surgical
In general, a mass or malignancy larger than 5 cm in planning—a retrospective analysis rom a private
an average-size breast likely requires mastectomy. Also radiology group. Breast J. 2013;19(2):134-141.
the presence o skin or chest wall involvement and the Gutierrez RL, DeMartini WB, Silbergeld JJ, et al. High
presence o multicentric disease require mastectomy. cancer yield and positive predictive value: outcomes
5. Based on a study by Fan et al looking at numbers o at a center routinely using preoperative breast MRI or
private community practice, surgical management staging. AJR Am J Roentgenol. 2011;196(1):W93-W99.
is being changed a ter breast MRI in about 24%. Schell AM, Rosenkranz K, Lewis PJ. Role o breast
Additional malignancies are being ound in about MRI in the preoperative evaluation o patients with
15% in the ipsilateral breast and in about 5% in the newly diagnosed breast cancer. AJR Am J Roentgenol.
contralateral breast. 2009;192(5):1438-1444.

354
Patient with palpable abnormality in the left breast, history of type II
diabetes—status post old benign biopsy in the left breast

1. What are the f ndings on the mammogram?

2. What is the ultrasound f nding corresponding


to the palpable mass?

3. What is an appropriate next step?

4. What would be concordant pathology results o


the previous le t biopsy?

5. What is the consequence o the diagnosis o


diabetic mastopathy?
355
Diabetic mastopathy 999
Case ranking/dif culty: Category: Diagnostic

5. Diabetic mastopathy is a benign lesion and patient can


return to screening mammography. Depending on the
situation, urther ollow-up might be considered over a
period o 2 years.

Pearls
• Diabetic f brous mastopathy, a stromal proli eration, is
ound in patients with juvenile onset insulin-dependent
diabetes (type I). Clinically, the most common
mani estation is a f rm-to-hard, nontender breast mass—
there is no associated increase risk o breast cancer.
• Most o these lesions appear as masses at clinical
Ultrasound-guided biopsy of left breast. or mammographic examination. On ultrasound, the
lesions are mostly bilateral and can show suspicious
Answers appearance including the presence o “posterior
acoustic shadowing.”
1. This is a mammogram with extremely dense • Fibroepithelial lesions are a combination o prominent
f broglandular tissue. There is clip in the superior breast stromal and glandular elements—f broadenoma is the
rom prior biopsy. most common f broepithelial lesion.
2. Corresponding to the palpable abnormality is a large • Other benign f broepithelial lesions include ocal
hypoechoic mass extending into all quadrants with f brosis, pseudoangiomatous stromal hyperplasia, and
“posterior acoustic shadowing.” There was similar f bromatosis or desmoid tumor.
appearance o the right breast on ultrasound.
3. Ultrasound-guided core biopsy might be the f rst step.
However, i patient had already prior biopsy, like in Suggested Readings
this case, which shows f ndings suggestive o diabetic Gabriel HA, Feng C, Mendelson EB, Benjamin S. Breast MRI
mastopathy, and has a history o type I diabetes and or cancer detection in a patient with diabetic mastopathy.
i the f nding on ultrasound is scattered throughout AJR Am J Roentgenol. 2004;182(4):1081-1083.
both breasts, it can be assumed that this is most likely Goel NB, Knight TE, Pandey S, Riddick-Young M,
a case o diabetic mastopathy and ollow-up might be de Paredes ES, Trivedi A. Fibrous lesions o the breast:
su f cient. There are reports that MRI might be help ul to imaging-pathologic correlation. Radiographics. 2005;25(6):
distinguish malignancy rom diabetic mastopathy. 1547-1559.
4. All would be concordant, except the presence o Sakuhara Y, Shinozaki T, Hozumi Y, Ogura S, Omoto K,
benign f broadenoma. The ultrasound f nding could be Furuse M. MR imaging o diabetic mastopathy. AJR Am J
malignant but would also be concordant with benign Roentgenol. 2002;179(5):1201-1203.
f broproli erative changes including f ndings consistent
with diabetic mastopathy. Diabetic mastopathy usually
shows the presence o collagenous stromal f brosis.

356
65-year-old woman with recall for questionable mass in the right upper
outer quadrant

1. Is there any abnormality in the right upper


outer quadrant?

2. What do you know about tomosynthesis?

3. How is tomosynthesis being used right now?

4. What do you know about primary lymphoma


in the breast?

5. What is the appearance o lymphoma


on imaging?

357
Primary breast lymphoma 764
Case ranking/dif culty: Category: Diagnostic

Ultrasound of right upper outer quadrant demonstrating mass at


11:00 close to the nipple. No increased ow on duplex.

Tomosynthesis, right MLO, single 1-mm slice. Stack of slices


replaces spot compression view.

Ultrasound of retroareolar breast demonstrates the hypoechoic


area, irregular in shape, which corresponds to the MRI.

MRI maximum intensity projection (MIP) image after IV contrast


demonstrates area of abnormal enhancement corresponding to
the ultrasound nding.

358
Primary breast lymphoma (Cont.)

Answers
1. This is a case where old pictures would be extremely Pearls
important—a theme that cannot be repeated enough in • This was a case where f rst invasive ductal carcinoma
breast imaging; thus: OLD IMAGES ARE THE KEY IN was the pathology result. However, a ter review it
BREAST IMAGING. Yes there is a questionable mass in showed eatures o primary breast lymphoma.
the right upper outer quadrant. • The treatment o primary breast lymphoma is
controversial and lumpectomy is not the main ocus;
2. Tomosynthesis is a new mammographic technique that
chemotherapy and radiation therapy are considered part
utilizes movement o the tube to create several slices
o primary treatment. Approach depends on the extent
o the breast. It is called 3D mammography and the
o the disease, best assessed with PET CT.
machine looks the same and the patient does not realize
• Tomosynthesis is an emerging technology, providing
any di erence, but the radiologist receives multiple thin
multiple 1-mm slices, FDA approved in 2011
cuts in CC and MLO projection, which can be scrolled
or screening in conjunction with conventional
through on the workstation. The conventional current
mammography and or diagnostic workup as
mammography machines can be re erred to as 2D
replacement or spot compression views.
mammography in that context. One company got FDA
• Tomosynthesis is particularly help ul to detect
approval in early 2011.
architectural distortion or subtle masses hidden
3. Tomosynthesis (3D mammography) was approved in dense tissue and can help to eliminate need or
or diagnostic workup in 2011 but not to replace 2D additional spot compression views, and patient can go
mammography or screening. It is currently being used directly to ultrasound.
to replace spot compression views, since it creates • Tomosynthesis is considered less help ul in workup o
multiple slices and has more in ormation than a calcif cations.
conventional compression view. It is more problematic
in regard to calcif cations. It is used in conjunction with
2D mammography as a combo (2D/3D mammography),
especially or dense breasts. It is also FDA approved Suggested Readings
in conjunction with conventional mammography or Hakim CM, Chough DM, Ganott MA, Sumkin JH, Zuley
screening. ML, Gur D. Digital breast tomosynthesis in the diagnostic
4. Lymphomas contribute to about 0.15 o malignant environment: a subjective side-by-side review. AJR Am J
mammary neoplasm. Less than 0.5% o all malignant Roentgenol. 2010;195(2):W172-W176.
mammary lymphomas involve the breast primarily. Noroozian M, Hadjiiski L, Rahnama-Moghadam S,
Primary breast lymphoma has an age peak at the 4th to et al. Digital breast tomosynthesis is comparable to
7th decade. The diagnosis o primary breast lymphoma mammographic spot views or mass characterization.
is limited to patients with no evidence o systemic Radiology. 2012;262(1):61-68.
lymphoma or leukemia. Yang WT, Lane DL, Le-Petross HT, Abruzzo LV, Macapinlac
HA. Breast lymphoma: imaging f ndings o 32 tumors in
5. On mammogram, primary breast lymphoma, but 27 patients. Radiology. 2007;245(3):692-702.
lymphoma in general, presents o ten as mass, most
requently with “irregular” shape and “indistinct”
margin. Other presentations include the presence o
“ ocal asymmetry.” Ultrasound demonstrates, in most
cases, mass with “irregular” shape. Posterior to the
mass, the echo can vary and could include “acoustic
enhancement.” Color Doppler imaging shows requently
hypervascularity. MRI demonstrates lobular mass, o ten
hyperintense on T2-weighted images and heterogeneous
enhancement with rapid uptake and washout. PET
demonstrates vivid uptake o FDG.

359
This page intentionally left blank
Palpable lump in the left breast

1. What BI-RADS classif cation should be used


here?

2. What is the next imaging test?

3. What type o tumor does a chondrosarcoma


arise rom?

4. What determines whether a chondrosarcoma is


primary breast or arising rom another lesion?

5. What is the best way o treating a


chondrosarcoma o the breast?
361
Chondrosarcoma of the breast 600
Case ranking/dif culty: Category: Diagnostic

Ultrasound shows two circumscribed complex echogenicity Ultrasound shows that although the mass is sharply marginated,
masses. Note the acoustic enhancement behind the lesions. the margins have a microlobulated appearance. There are some
The mass on the left also shows a liquid necrotic center. low echo slit-like areas raising the possibility of a phyllodes tumor.

Answers
1. Although these masses are not calcif ed and have Pearls
circumscribed margins, their distribution is not normal. • Multiple masses in a segmental distribution need
They are also palpable and eel hard, which does not urther workup.
f t with the normal presentation o f broadenomas or • Evaluate margins o lesion and then biopsy.
tense simple cysts. A BI-RADS 4 assessment is the most • With atypical pathology f ndings, think metaplastic
suitable outcome. carcinoma.
• Metaplastic carcinoma can cause a variety o tumor
2. Further evaluation with ultrasound is likely the best next
types.
investigation. You can per orm urther mammographic
views i you think that the margins o the masses have
not been demonstrated clearly. MRI is unlikely to give
additional in ormation to a ect the management o the Suggested Readings
case. Patterson JD, Wilson JE, Dim D, Talboy GE. Primary
3. Sarcomatous change has been reported in both phyllodes chondrosarcoma o the breast: report o a case and review
tumors and metaplastic carcinomas o the breast. o the literature [published online ahead o print]. Breast
Dis. 2011.
4. The presence o a primary sarcoma means that this is Ver aillie G, Breucq C, Perdaens C, Bourgain C,
likely a secondary deposit. However, a chondrosarcoma Lamote J. Chondrosarcoma o the breast. Breast J.
can arise in a phyllodes tumor or a metaplastic 2005;11(2):147-148.
carcinoma. Both o these lesions have an epithelial
component, and so this should be excluded be ore
calling a lesion a primary breast chondrosarcoma.
5. Treatment is primarily surgical. Chondrosarcoma o the
breast is resistant to both chemotherapy and radiation
therapy.

362
Screening mammogram of left breast in patient with two sisters with
breast cancer—priors on left

1. What is the f nding on the screening


mammogram?

2. Who qualif es or MRI screening?

3. What are the actors being considered or the


risk calculator o the National Cancer Institute?

4. What is the average li etime risk o women up


to the age o 90?

5. What other exam would be available i patient


does not quali y or MRI?

363
MRI screening of high-risk patient with incidental nding 393
Case ranking/dif culty: Category: Screening

MRI of bilateral breast after IV contrast, MRI, T1-weighted sequence after IV contrast, sagittal MRI of non–fat-suppressed
maximum intensity projection (MIP) reformation. T1-weighted sequence without
technique. contrast demonstrating small mass
with “irregular”margin.

ultrasound. Breast ultrasound in general is not accepted


or screening but is being used by request o the re erring
physician. Tomosynthesis would also be a help ul adjunct
to standard mammography to increase sensitivity.

Pearls
• Since 2007, most insurance companies in the United
MRI of T1-weighted sequence after IV contrast demonstrates early
enhancement of the small 4-mm mass.
States pay or screening MRI or patients with 20% to
25% li etime risk o breast cancer.
• There are several risk calculators available on the
Answers Internet, which help to calculate the li etime risk
1. This is a normal screening mammogram. based on several risk actors, or example, the
National Cancer Institute webpage (www.cancer.gov/
2. Patients with a li etime risk o 20% to 25%, patient bcrisktool/).
needs two f rst degree relatives with breast cancer, or • In general, patients with two close amily members
example, mother and sister. (mother and sister) will have a li etime risk o about
3. Factors that will put into the calculator are as ollows: 25% and higher and there ore would quali y or
screening breast MRI.
i. Personal history o breast cancer • This is an example where even in a atty-replaced
ii. Woman’s age breast, a 5-mm-large invasive ductal carcinoma can be
iii. Woman’s age at the time o f rst menstrual period. missed on mammogram.
iv. Woman’s age at the time o her f rst live birth o a
child
v. Number o f rst-degree relatives with breast cancer
vi. History o breast biopsies Suggested Reading
vii. Race/ethnicity Warner E, Messersmith H, Causer P, Eisen A, Shumak R,
Plewes D. Systematic review: using magnetic resonance
4. Li etime risk depends on the age and ethnicity; or a
imaging to screen women at high risk or breast cancer.
40-year-old white woman, it is about 12.4%, and or a
Ann Intern Med. 2008;148(9):671-679.
40-year-old A rican American, it is around 9.6%.
5. In case o not quali ying or breast MRI—and in
particular i the patient has very dense breast tissue—
the other exams that would be complementary to
mammography and widely available would be breast

364
44-year-old woman with new suspicious calci cations in the left breast

1. Would these calcif cations require biopsy


i they were there 2 years ago?

2. What would be the procedure i these


calcif cations cannot be biopsied with
stereotactic approach?

3. What are two important quality-assuring steps


or per orming a stereotactic biopsy.

4. I the post-biopsy marker clip is displaced,


what might be the consequence?

5. How can clip displacement be avoided?

365
Where is the clip? 322
Case ranking/dif culty: Category: Diagnostic

Diagnostic mammogram demonstrates Diagnostic mammogram magni cation Mammogram of left Left CC view
group of suspicious calci cations in the XCCL view demonstrates group of breast MLO view after stereotactic
left lateral, central, and posterior breast. suspicious calci cations in the left after stereotactic biopsy after
lateral breast. biopsy demonstrates clip placement
clip at the level of demonstrates
the nipple. 3-cm clip medial
Answers from the residual
1. Calcif cations—i they are suspicious based on calci cations.
morphological eatures—need to be biopsied, even i
they are stable over several old mammograms. They 5. Clip displacement cannot be completely avoided. It is
could represent ductal carcinoma in situ (DCIS), which caused by the release o compression a ter a stereotactic
can progress slowly over time, in particular i it is low- biopsy.
grade DCIS.
2. I suspicious calcif cations are seen on two projections,
they can be approached with needle localization and
Pearls
surgical excision can be per ormed. Calcif cations such
as in this case are not accessible to stereotactic biopsy • Clip displacement is not uncommon, as seen on
due to their location within the breast, or example, since postbiopsy mammogram.
they are located very posteriorly, patient needs to be send • Clip displacement is o tentimes due to traction during
or needle localization and surgical excision. decompression on the stereotactic biopsy unit.
• Postbiopsy mammogram is crucial to conf rm correct
3. It is important to get a specimen radiograph a ter clip placement.
stereotactic biopsy o suspicious calcif cations to prove • Consequence o clip displacement can be placement
that a representative sample o the calcif cations was o a new clip. I there are residual calcif cations, or the
obtained. It is not the goal to excise all calcif cations. clip is too deep, needle localization can be corrected
Also important is that the postbiopsy mammogram accordingly. Important is to document the location o
shows the clip in good position, which means that the clip in regard to the abnormality (target zone).
the clip is in the target zone where the specimen was
collected rom.
4. In most cases, it is su f cient to document the location Suggested Reading
o the misplaced clip in relation to the area that was
biopsied. There are rare cases where it is help ul to Esserman LE, Cura MA, DaCosta D. Recognizing pit alls
place a second clip. This can be the case i the clip is in early and late migration o clip markers a ter imaging-
short in correlation to the target and there ore the needle guided directional vacuum-assisted biopsy. Radiographics.
localization would be less accurate. 2004;24(1):147-156.

366
Palpable lump in the left breast

1. What BI-RADS classif cation should be used


here?

2. What is the most likely pathology based on the


imaging?

3. What is the next best imaging test?

4. What are the imaging eatures o a breast


sarcoma?

5. What is the surgical treatment o breast


sarcoma?

367
Angiosarcoma—presenting as a mass 1581
Case ranking/dif culty: Category: Diagnostic

Left breast ultrasound with Doppler showing T2 axial MRI, shows homogeneous high T1 fat sat axial 120 seconds following
peripheral vessels—no surrounding vessels in signal circumscribed but super cial mass contrast injection.
subcutaneous tissues. simulating broadenoma.

Answers
1. BI-RADS 4—as the f ndings simulate f broadenoma, Pearls
there are some con ounding eatures that allow you to • Rare sarcoma.
recommend a biopsy. • Oval mass with indistinct margins, but can simulate a
benign mass.
2. Any o the f rst our answers can present as a
• MRI and ultrasound show similar eatures.
circumscribed mass with peripheral vascularity. Interval
• Does not metastasize to the axillary nodes.
change may help to di erentiate a phyllodes tumor rom
• Mastectomy without axillary dissection is standard care.
a f broadenoma. Metastases are common only when there
is a known other cancer primary site setting. A simple
cyst with proteinaceous debris should be belottable with
gentle pressure on ultrasound. Seeing the internal echoes Suggested Readings
moving helps to distinguish orm a solid mass. Babarovi E, Zamolo G, Musta E, Str i M. High grade
3. The mass on mammography is circumscribed and angiosarcoma arising in f broadenoma. Diagn Pathol.
similar to the nearby axillary lymph nodes. The initial 2011;6(1):125.
ultrasound image shows that the mass is very superf cial, Cao Y, Panos L, Graham RL, Parker TH, Mennel R. Primary
unlike the normal position or a f broadenoma, but could cutaneous angiosarcoma o the breast a ter breast trauma.
then be a node. Doppler ultrasound may show this mass Proc (Bayl Univ Med Cent). 2012;25(1):70-72.
to be highly vascular, including the surrounding breast Hui A, Henderson M, Speakman D, Skandarajah A.
tissue. Angiosarcoma o the breast: a di f cult surgical challenge.
Breast. 2012;21(4):584-589.
4. The characteristic eatures that make you suspect a
sarcoma are the oval shape with indistinct margins, and
marked vascularity on ultrasound. It is rare or a mass
to be truly round unless it is a high tension simple cyst.
Irregular margin to a mass is the suspicious f nding
arising rom an invasive ductal carcinoma.
5. Lumpectomy with wide margins has been tried, but
sarcomas when they recur are di f cult to treat. The
standard care has developed with mastectomy and no
axillary procedure, as sarcoma rarely metastasizes
to the locoregional lymph nodes. Mastectomy with
reconstruction can be an option in sites with oncoplastic
surgeons.

368
Patient with discoloration of the right nipple: no discharge

1. What is the abnormality?

2. What are typical eatures o Paget disease?

3. What is the di erential diagnosis?

4. Patient has eczema o nipple but normal


mammogram—what is the next step?

5. What is the best technique to assess


retroareolar breast on ultrasound?

369
Solid papillary carcinoma 759
Case ranking/dif culty: Category: Diagnostic

MRI T1-weighted sequence after IV contrast. MRI T1-weighted sequence after IV contrast with CAD.

Answers
1. Noted is only minimal retraction o the nipple that was some underlying breast neoplasm—the characteristic
stable since prior studies (not submitted)—no other histological f nding is the presence o malignant Paget
abnormality. Ultrasound does not demonstrate any cells (adenocarcinoma) in the epidermis.
abnormality neither. • Given the high incidence o underlying breast
2. Paget disease accounts or 2% to 3% o breast cancer. It malignancy, as reported o up to 100%, it is believed
is a clinical diagnosis with eczema o the nipple but has that Paget cells arise in the secretory ducts and migrate
characteristic Paget cells within the dermis, which are into the skin o the nipple.
consistent with adenocarcinoma. Mammograms are in • Paget disease in general occurs in the age around 55 years.
most cases demonstrating f ndings related to high-grade • Mammography has been reported to be positive in
ductal carcinoma in situ—however, mammograms can only 40% to 50%—in most cases, it correlates to the
be normal as well. presence o ductal carcinoma in situ.
• Early skin biopsy, called punch biopsy, is the important
3. Di erential diagnosis could include scaring or eczema.
f rst step to get the diagnosis—any eczema o the breast
4. In general, ultrasound would be part o the workup to or more than 2 weeks should be viewed with suspicion.
urther exclude hidden abnormality in the retroareolar • Mastectomy is the standard treatment or Paget disease.
breast—especially in the presence o dense breast tissue. • In this particular case, however, the diagnosis was
MRI can be help ul to evaluate or additional abnormalities a surprise and pathology demonstrated f ndings
or to urther conf rm abnormal nipple complex, but it is in consistent with solid papillary carcinoma, a relatively
general not required at this stage. Punch biopsy per ormed rare but pathologically distinct entity, which can be
by breast surgeon is the crucial next step. seen in the nipple region o elderly women.
5. To assess the anterior retroareolar breast tissue on
ultrasound and to avoid the posterior shadowing, it is
Suggested Readings
help ul to compress the lateral aspect by hand and to
elongate the tissue anteriorly and to approach the tissue Burke ET, Braeuning MP, McLelland R, Pisano ED,
with the transducer rom the other lateral contour o the Cooper LL. Paget disease o the breast: a pictorial essay.
retroareolar breast. Any direct anterior approach at the Radiographics. 2000;18(6):1459-1464.
level o the nipple will require large amount o gel to get Haddad N, Ollivier L, Tardivon A, et al. Use ulness o
rid o any air in between the transducer and the nipple. magnetic resonance imaging in Paget disease o the
breast. J Radiol. 2007;88(4):579-584.
Sundaram S, Prathiba D, Rao S, Rajkumar A, Rajendiran
Pearls S. Solid variant o papillary carcinoma o nipple: an
• Paget disease accounts or approximately 2% to 3% o under recognized entity. Indian J Pathol Microbiol.
breast cancers. 2011;53(3):537-540.
• Paget disease is a distinct entity, which includes
erythema o the nipple areola complex that o ten has

370
Status post–cardiac surgery, since then swelling left side of the body

1. What is the pertinent f nding on this diagnostic


mammogram?

2. What is the next step o workup?

3. What are other important f ndings to exclude?

4. What could be the diagnosis?

5. What is the most likely diagnosis in the


absence o skin alterations and the presence o
edema in le t leg?

371
Unilateral edema due to congestive heart failure 1299
Case ranking/dif culty: Category: Diagnostic

Ultrasound demonstrating interstitial edema. CT chest without contrast demonstrates the unilateral breast
edema and pleural e usion after heart surgery.

Answers
1. Patient demonstrates uni orm reticular edema o the le t
Pearls
breast, swelling o the le t breast, and uni orm thickening
o the skin. • In the absence o skin alteration, the presence o
in ammatory breast cancer is extremely unlikely.
2. The most important next step is the clinical evaluation. • This particular patient had recent cardiac surgery and
In this case, patient presented with unilateral edema le t unilateral leg edema since surgery and was believed
including leg and arm a ter cardiac surgery. She also to represent a rare case o unilateral edema a ter
noticed the enlargement o the le t breast. cardiac surgery.
3. In this particular case, it is important to exclude any • However, since in ammatory cancer is a devastating
ocal abnormal morphology in the breast, such as malignancy, patient was send to breast surgeon or
masses, microcalcif cations, and/or lymphadenopathy. clinical evaluation—punch biopsy was not per ormed
This presentation o the le t breast can be seen with and patient was urther ollowed clinically and with
in ammatory malignancy. However, in ammatory skin imaging.
changes such as redness and swelling are usually seen in
the presence o in ammatory malignancies o the breast.
4. Di erential diagnosis could include in ection, status Suggested Reading
postradiation, lymphoma, unilateral cardiac edema, Oraedu CO, Pinnapureddy P, Alrawi S, Acinapura AJ, Raju R.
and, most importantly, the presence o in ammatory Congestive heart ailure mimicking in ammatory breast
carcinoma o the breast. carcinoma: a case report and review o the literature.
5. This is, given the corresponding chronicity a ter cardiac Breast J. 2001;7(2):117-119.
surgery and the presence o leg edema, likely due to a
rare case o cardiac unilateral edema. I there is remaining
clinical concern, punch biopsy can be per ormed to
exclude diagnosis o in ammatory carcinoma.

372
Palpable nding in the right breast

1. What BI-RADS classif cation should be used


here?

2. What is the next imaging test?

3. What is the most likely pathological entity or


these imaging appearances?

4. Dense breast tissue may have what e ect on


cancer detection?

5. According to BI-RADS, what are the mass


margin descriptors?
373
Cancer arising in dense breast tissue 1740
Case ranking/dif culty: Category: Diagnostic

Ultrasound shows calci cations within the irregular MRI was performed for extent of the
mass. lesion.
Right XCCL (exaggerated CC lateral)—
this was performed rather than spot
lms. We do not have any images of 5. “Oval” and “lobular” are descriptions o the shape o a
the margins of the mass as a result. mass (BI-RADS 4)—lobulated mass will be dropped rom
There are only scattered broglandular the 5th edition o BI-RADS. The mass margin descriptors
densities, but the breast tissue where include:
the cancer is arising is dense enough
to partially obscure the lesion. i. “Circumscribed”
ii. “Lobulated”
Answers iii. “Microlobulated”
1. I this was a screening exam, then a BI-RADS 0 is iv. “Angulated”
appropriate. I the f nding is palpable, then this would be a v. “Spiculated”
diagnostic exam and urther images should be per ormed.
We need to determine whether we can see the margins o
the suspected mass, and per orm an ultrasound scan. Pearls
2. The most appropriate examinations are as ollows: • Cancers can be di f cult to see in dense breast tissue.
i. Spot views to press out the normal tissue and reveal • Look or signs o displaced tissue or an asymmetry that
the underlying mass does not ollow normal tissue planes.
ii. Targeted ultrasound examination • Use spot/spot magnif cation views to urther
characterize the margins o a suspected mass.
At this stage, MRI is not indicated, but i a ter a ull • Tomosynthesis (i available) would likely help
workup you cannot determine the cause o the f nding visualize the lesion in one examination.
on imaging, and there is nothing to target or a biopsy, a • Do ull mammographic workup prior to ultrasound.
troubleshooting MRI may help.
3. Pseudoangiomatous stromal hyperplasia can present as
a suspicious mass, but tends to be less dense. Ductal Suggested Readings
carcinoma in situ (DCIS) typically presents with Boyd NF, Melnichouk O, Martin LJ, et al. Mammographic
calcif cation. DCIS masses can occur, but are usually density, response to hormones, and breast cancer risk.
not calcif ed and may be circumscribed. Lobular cancer J Clin Oncol. 2011;29(22):2985-2992.
may be invisible, a mass (particularly on the CC view), Checka CM, Chun JE, Schnabel FR, Lee J, Toth H. The
or as distortion or a shrinking breast. Complex sclerosing relationship o mammographic density and age: implications
lesions can occur as density with distortion, and may or breast cancer screening. AJR Am J Roentgenol.
require excision to make the diagnosis. 2012;198(3):W292-W295.
4. Increased breast density is a personal risk actor or King V, Brooks JD, Bernstein JL, Reiner AS, Pike MC,
breast cancer. Dense breast tissue may obscure a cancer, Morris EA. Background parenchymal enhancement at
even a large one. Signif cant calcif cation (DCIS) is breast MR imaging and breast cancer risk. Radiology.
rarely obscured by dense tissue. 2011;260(1):50-60.
374
Patient with group of suspicious calci cations. Biopsy was performed:
can you see the nding?

1. Why is it important to obtain postbiopsy


mammogram?

2. What is the reason or clip displacement?

3. What can help in this situation a ter


displacement o the clip?

4. How would you characterize the calcif cations


in the le t breast?

5. What would be the next step to manage the


situation?
375
Atypical hyperplasia—migrated clip 997
Case ranking/dif culty: Category: Diagnostic

Diagnostic mammogram, left ML Diagnostic mammogram, left CC Mammogram, left MLO view, Mammogram, left CC view, after
magni cent view. magni cent view. after clip placement. clip placement.

Answers
1. Postbiopsy mammogram is absolutely necessary to Pearls
determine i the clip is in good location, i the clip is • I a small group o calcif cations is completely removed
in the same location at the calcif cations sampled (see by the vacuum-assisted core biopsy needle, it is
arrows). crucial to prove that the clip is in the area biopsied by
per orming postbiopsy mammogram.
2. The accordion e ect explains the movement o a clip
• In this particular case, post–core biopsy images (MLO
due to compression and release o compression during
and CC views) demonstrate that the clip is displaced
the stereotactic biopsy. Clip displacement can also be
about 4 cm in erior rom the target zone and 1.5 cm
caused by bleeding or hematoma. All the other actors
anterior.
mentioned are not documented in the literature.
• Clip displacement has direct e ect on the planning
3. Any landmark or residual pathology can be help ul to o the needle localization because the target zone
improve accuracy doing the needle localization. I the cannot be directly targeted i there are no residual
pathology is completely removed, like in our case, it is calcif cations or other landmark that could be used or
tricky. I the clip is on the z-axis deeper than the biopsied needle localization.
target, it is reasonable to target the clip, given that the • Clip displacement can be deep or superf cial to the
abnormality biopsied should be on the track o the wire. target zone—the latter is the more complicated situation.
However, i the clip is short to the target zone, another • Clip usually migrates along the z-axis (compression
modality might be help ul to see the scar rom the biopsy axis) at the time o compression release due to the
and thus determine the actual target zone. MRI was act that clip might not be anchored to the wall o the
per ormed, which demonstrated residual enhancement biopsy cavity but to the adjacent tissue and that the
in the target zone and subsequently second clip was distance is minimal during compression but can extend
inserted with MRI guidance. a ter decompression (“accordion e ect”).
4. Noted are benign vascular calcif cations and also a group
o “irregular and pleomorphic” calcif cations.
Suggested Reading
5. Next step would be stereotactic biopsy. Ultrasound could
also be added to investigate i there is a solid component. Esserman LE, Cura MA, DaCosta D. Recognizing
However, just to do the stereotactic biopsy would also be pit alls in early and late migration o clip markers a ter
not unreasonable. imaging-guided directional vacuum-assisted biopsy.
Radiographics. 2004;24(1):147-156.

376
Screening mammogram (MLO images only)

1. What are the f ndings seen on the screening


mammogram?

2. What is part o di erential diagnosis o


bilateral axillary lymphadenopathy?

3. What are typical f ndings related to


amyloidosis in the breast?

4. What would be the appropriate workup o the


lymph nodes in case o lack o old images?

5. What would be an appropriate approach to


biopsy these lymph nodes?

377
Amyloidosis 755
Case ranking/dif culty: Category: Screening

Screening mammogram, right CC view demonstrating scattered Screening mammogram, left CC view demonstrating benign
benign calci cations and clip from prior benign biopsy. scattered calci cations and clip from prior benign biopsy.

Answers
1. Noted is bilateral lymphadenopathy. Also noted are Pearls
scattered benign-appearing calcif cations bilaterally. • Amyloidosis is characterized by protein deposition
within extracellular tissue. It involves primarily the
2. Di erential diagnosis can include Rheumatoid Arthritis,
heart, kidneys, skin, musculoskeletal system, and lungs.
Scleroderma, Sarcoidosis, Lymphoma, Leukemia, Lung
• It can be a primary systemic disease or secondary to
cancer or melanoma.
in ammatory systemic conditions, such as autoimmune
3. Amyloidosis can result in the presence o masses; they disease.
can appear suspicious and o tentimes, biopsy is necessary. • Presentation o amyloidosis on mammogram is rare. It
Also possible in the presence o calcif cations—they are can present as an incidental or palpable mass but also
more likely scattered and more coarse. as clustered microcalcif cations.
4. The patient history is the key. In this case, the presence
o systemic disease, such as rheumatoid arthritis,
explains the situation. I any remaining concern, Suggested Readings
ultrasound should be per ormed and possible biopsy. Cao MM, Hoyt AC, Bassett LW. Mammographic signs o
5. Ultrasound-guided core biopsy is the best choice. systemic disease. Radiographics. 2011;31(4):1085-1100.
Alternative could be ultrasound-guided f ne needle Munson-Bernardi BD, DePersia LA. Amyloidosis o the
aspiration. breast coexisting with ductal carcinoma in situ. AJR Am
J Roentgenol. 2006;186(1):54-55.

378
Screening—asymptomatic

1. What is the BI-RADS category or this


screening exam?

2. What was the abnormality you saw that


prompted recall rom screening?

3. What is the best description or the distribution


o calcif cations?

4. What is the likely f nal pathology in this


patient?

5. What should be the next radiological


investigation to determine the extent o this
disease?
379
Multicentric invasive ductal carcinoma and ductal 603
carcinoma in situ (DCIS)
Case ranking/dif culty: Category: Screening

Spot compression with magni cation. This shows the small Ultrasound shows a “non-parallel” irregular mass with “acoustic
“indistinct” masses with calci cations in a “segmental” shadowing,” consistent with malignancy.
distribution between the masses.

with dense breasts, where the technique may pick up


Answers additional oci not visualized on the mammogram. MRI
1. This is a screening exam; there ore, only BI-RADS would be the best next radiological exam to determine
0, 1, and 2 are allowed. Further imaging required. the extent o disease and to stage the disease, lymph
Recommend inclusion o spot magnif cation views to nodes, and contralateral breast.
urther characterize the calcif cation particles.
2. Some readers f nd it easier to perceive cancers when
they are associated with calcif cations, especially in Pearls
dense breasts. In this case, there is at least one spiculated • Multi ocal disease is within the same anatomic
mass and several clusters and segmental calcif cations, segment o breast tissue.
associated with ocal asymmetries, suggesting multi ocal • Multicentric disease a ects multiple areas o the breast
or even multicentric disease. with discontinuous disease, and there could be occult
3. The calcif cations re ect a segment o the milk ducts (a disease that has not yet been detected.
suspicious eature). Multiple clusters o calcif cations • DCIS may be seen in continuity between the tumor
could be used, but clustered calcif cation is a nonspecif c masses.
descriptor. I you eel the calcif cations are likely
suspicious, and they con orm to a segment, then segmental
calcif cations can be used to indicate your suspicion. Suggested Readings
4. The radiological eatures o the calcif cations are highly Ustaalioglu BO, Bilici A, Ke eli U, et al. The importance o
suspicious o DCIS. The individual calcif c particles are multi ocal/multicentric tumor on the disease- ree survival
pleomorphic and have sharp irregular margins. According o breast cancer patients: single center experience. Am J
to the Tabar classif cation, these are a crushed rock Clin Oncol. 2011;35(6):580-586.
appearance. The distribution is composed o clusters White J, Achuthan R, Turton P, Lansdown M. Breast
arranged in a segmental pattern, with some o the calcium conservation surgery: state o the art. Int J Breast Cancer.
rearranging themselves into a more linear pattern. 2011;(2011):107981.
Yerushalmi R, Tyldesley S, Woods R, Kennecke HF, Speers C,
5. Ultrasound is routinely per ormed but may not show the
Gelmon KA. Is breast-conserving therapy a sa e option or
extent o the disease. Multiple areas o shadowing may
patients with tumor multicentricity and multi ocality? Ann
be observed, which may not necessarily be due to urther
Oncol. 2012;23(4):876-881.
disease. PEM has been advocated particularly in women

380
60-year-old woman with palpable abnormality in the right breast
(with BB marker). Priors on left. History of brocystic changes

1. Is there any pertinent abnormality seen?

2. What could be the pit alls in this particular


case?

3. Why was an MRI per ormed in that case?

4. Is it important to work up the masses each time


patient comes to screening?

5. What would be the appropriate management


o screening mammogram o an asymptomatic
patient with known f brocystic changes?

381
Fibrocystic changes and mucinous carcinoma 732
Case ranking/dif culty: Category: Diagnostic

Ultrasound directed to the area of concern Ultrasound directed to the area of concern MRI postcontrast demonstrating
demonstrates multiple simple cysts. demonstrates an area of low echogenicity large area of non–mass-like
located close to the brocystic changes. It enhancement.
is “irregular” in shape and about 1.3 cm in
maximum diameter.

Answers
1. Patient has history o f brocystic changes as described Pearls
on prior ultrasound exams. Again noted are multiple • Despite history o benign f brocystic changes and
benign-appearing masses, which have uctuated over mammogram showing corresponding waxing and
time. There is a questionable new “ ocal asymmetry” on waning masses, consistent with cysts as seen on prior
right superior MLO view. ultrasound, any new palpable mass raises concern
and could be due to additional malignancy that can be
2. Any palpable abnormality requires f rst diagnostic workup
obscured by the surrounding benign cysts.
with additional spot compression views and BB marker on
• Most mucinous carcinomas present as mass on
the area o concern (not submitted in this particular case—
ultrasound, o tentimes with microlobulation. However,
but per ormed)—then ultrasound should be per ormed in
i the tumor is not pure mucinous in nature, ultrasound
any circumstance to better characterize the abnormality.
can also represent irregular shaped hypoechoic mass,
3. Breast MRI in this case is help ul to better address the like in this case.
extent o the lesion and subsequently the appropriate • MRI was also per ormed and demonstrated large area
surgical approach. It is also help ul to address the o abnormal non–mass-like enhancement that was
situation in the contralateral breast and to address signif cantly larger than it appeared on the ultrasound.
possible lymphadenopathy.
4. I a screening exam shows multiple bilateral scattered
benign-appearing masses and in the past patient has Suggested Readings
had ultrasound demonstrating benign cysts, there is no Lam WW, Chu WC, Tse GM, Ma TK. Sonographic
need to work up the cysts each time. This is true even appearance o mucinous carcinoma o the breast. AJR Am
i the mammographically benign-appearing masses J Roentgenol. 2004;182(4):1069-1074.
uctuate slightly in size. However, i there are new Leung JW, Sickles EA. Multiple bilateral masses detected on
suspicious morphological eatures, such as “architectural screening mammography: assessment o need or recall
distortion,” new “ ocal asymmetry,” or calcif cations, imaging. AJR Am J Roentgenol. 2000;175(1):23-29.
patient needs to be called back or additional workup.
5. I a patient has multiple bilateral scattered benign-
appearing masses on mammogram, which have been
shown in the past on ultrasound to represent benign
cysts, even i they uctuate in size, there is no need to
per orm a diagnostic workup. Cysts can uctuate in
size (see Leung and Sickles, 2000). Assessment can be
BI-RADS 2.

382
63-year-old woman with palpable abnormality in the left breast

1. What is the next step a ter a normal


mammogram when patient has palpable lump?

2. I ultrasound is normal as well, what is the next


step?

3. Is an additional MRI standard o care to


evaluate palpable breast abnormality a ter
normal mammogram and ultrasound?

4. What is the signif cance o an ultrasound


(image next page) f nding?

5. What is the reason to add an MRI


preoperatively?

383
Invasive lobular carcinoma 389
Case ranking/dif culty: Category: Diagnostic

Ultrasound demonstrates hypoechoic solid area in the medial left MRI does con rm a larger than expected area of “non–mass-like
breast in the area of palpable abnormality. enhancement.”

Answers
1. Standard o care is to per orm diagnostic mammogram Pearls
including spot compression views and ultrasound or a • In case o palpable abnormality, despite a normal
palpable abnormality. diagnostic mammogram, ultrasound is crucial or
urther evaluation.
2. I mammogram and ultrasound are unremarkable, the
• The most common ultrasound presentation o an
exam can be called BI-RADS 1 (“negative”). However,
inf ltrative lobular carcinoma is an irregular or
it is help ul to add a statement that “despite normal
“angular” mass with “ill-def ned” or “spiculated”
imaging urther evaluation o the palpable abnormality
margins. However, there are also other ultrasound
should be based on clinical grounds.” That can include
presentations o ILC, which includes even the presence
biopsy o the palpable abnormality by a breast surgeon i
o a “well-circumscribed” mass as in this case.
the palpable abnormality is suspicious enough.
• In general, the alse-negative rate o mammograms or
3. A ter normal mammogram and ultrasound in a patient ILC is much higher than or invasive ductal carcinoma
o palpable abnormality, it is not standard o care to add due to the di use growth pattern o lobular carcinomas
MRI. However, in selected cases, MRI could be help ul (“Indian f le pattern o growth” as described by the
as additional test to evaluate palpable abnormality. This pathologists).
is in particular the case i the breast tissue is very dense
on mammogram and i the palpable abnormality is
highly suspicious based on the clinical evaluation or in
Suggested Reading
the setting o very strong amily history.
Lopez JK, Bassett LW. Invasive lobular carcinoma o the
4. Ultrasound f nding is consistent with hypoechoic breast: spectrum o mammographic, US, and MR imaging
nodule—it does not show posterior enhancement f ndings. Radiographics. 2009;29(1):165-176.
and does not ulf ll all the criteria o simple cyst.
There ore, and in particular since the f nding was
palpable, ultrasound-guided biopsy was per ormed and
demonstrates f ndings consistent with lobular invasive
carcinoma.
5. MRI is help ul to assess the extent o the disease and to
look or additional malignancy, to exclude multi ocal
(same quadrant) or multicentric (di erent quadrants)
disease.

384
68-year-old patient with screening mammogram prior lms
are on the left

1. I you suspect subtle “architectural distortion,”


what is the next step?

2. What can be the etiology or “architectural


distortion”?

3. I there is architectural distortion and no


abnormality on ultrasound, what is the next
step?

4. Why is it challenging to see subtle


“architectural distortion”?

5. Why is the history o the patient important?

385
Atypical ductal hyperplasia 353
Case ranking/dif culty: Category: Diagnostic

Diagnostic mammogram, left spot compression ML view Diagnostic mammogram, left spot compression CC view
demonstrating very subtle “architectural distortion.” demonstrating subtle “architectural distortion.”

Answers
1. Next step is diagnostic mammogram with spot Pearls
compression views. • I there is presence o architectural distortion on
mammogram, as conf rmed on spot compression
2. “Architectural distortion” can be caused by variety o
views, despite normal ultrasound, stereotactic biopsy is
di erent etiologies, including prior biopsy or surgery
recommended.
and atypical ductal hyperplasia.
• Since it is documented that between 4% and 54% o
3. I there is no abnormality on ultrasound, stereotactic lesions reported on core biopsies as atypical ductal
biopsy is the next step. hyperplasia are upgraded on urther surgical excision
to invasive carcinoma, surgical intervention is
4. To f nd subtle “architectural distortion” is the most
recommended.
challenging topic in mammography and yet very
important, since o tentimes “architectural distortion” is
related to the presence o invasive ductal carcinoma.
Suggested Readings
5. Excisional biopsy or lumpectomy would cause
architectural distortion—all other procedures as Deshaies I, Provencher L, Jacob S, et al. Factors associated
mentioned above in general do not result in the presence with upgrading to malignancy at surgery o atypical
o distortion, although in rare cases can cause some ductal hyperplasia diagnosed on core biopsy. Breast.
distortion as well. 2011;20(1):50-55.
Samardar P, de Paredes ES, Grimes MM, Wilson JD. Focal
asymmetric densities seen at mammography: US and
pathologic correlation. Radiographics. 2002;22(1):19-33.

386
50-year-old woman with palpable abnormality in the right upper
outer quadrant

1. What do you do i palpable mass does not


show up on diagnostic mammogram?

2. What do you do i mammogram and ultrasound


are normal in the area o palpable abnormality?

3. What characterizes an adenoid cystic


carcinoma?

4. What are the typical imaging eatures o


adenoid cystic carcinoma?

5. What is the most likely treatment?

387
Adenoid cystic carcinoma 352
Case ranking/dif culty: Category: Diagnostic

Ultrasound of the right breast, area of concern. Mass with Ultrasound of the right breast, area of concern with harmonic
“angular” margin. imaging that helps to increase contrast between mass and fat
nodules.

it usually demonstrates an angular and microlobulated


mass with mixed echogenicity. It might not show up at
all on ultrasound and mammogram.
5. In most cases, tumor is treated with surgical resection
and radiation therapy, which will lower the rate o
possible recurrence.

Ultrasound of the right breast, area of concern with duplex


con rming ow in the mass.
Pearls
• Adenoid cystic carcinoma o the breast is a rare tumor
Answers accounting or less than 0.1% o all breast cancers. It
1. This is a good example where ultrasound does show the is a variant o an adenocarcinoma with relative good
abnormality. Thus, it is critical not to stop a ter a normal prognosis. Distant metastasis, most likely to the lung,
mammogram workup but to per orm ultrasound. can occur, with axillary metastasis being rare.
• Mammography demonstrates in general “lobular”-
2. A mass elt by the patient is always an important issue and shaped mass with “microlobulated” or “indistinct”
critical or the radiologist to take seriously. I mammogram margin. Calcif cations were not reported.
including diagnostic workup with spot compression views • On sonography, adenoid cystic carcinoma o the breast
is normal and ultrasound is normal, the likelihood that shows “irregular”-shaped mass with hypoechoic
patient has any malignancy is low, but there are cases or heterogeneous echotexture. Margins are o ten
where despite normal imaging there is a malignancy. “angular” or “indistinct.”
The likelihood is higher in dense breast tissue. Thus, in
general, the decision whether palpation-based biopsy (not
imaging-based biopsy) should be per ormed by a breast
surgeon will depend on the clinical situation and the level Suggested Readings
o concern based on the palpation. Since the breast exam Glazebrook KN, Reynolds C, Smith RL, et al. Adenoid
in the ultrasound is in general not done by the radiologist, cystic carcinoma o the breast. AJR Am J Roentgenol.
a statement like “ urther management should be based on 2010;194(5):139-126.
clinical grounds” is appropriate. Stavros T. Breast Ultrasound. 1st ed. Lippincott Williams &
3. It is a subgroup o adenocarcinoma with low rate o Wilkins. Philadelphia, PA; 2004.
lymph node metastasis but not in requent presence o
distant metastasis, in general, to the lungs.
4. It usually presents as an asymmetry or indistinct mass
on mammogram, without calcif cations. On ultrasound,

388
51-year-old patient with history of palpable mass in the chest wall
(MRI the gure (top) on the extreme left: T1 without contrast, the
gure (top) on the extreme right: T1 with contrast, the gure (bottom):
T2 weighted)

1. What are the f ndings on the MRI?

2. Based on the MRI what is the situation in


regard to the chest wall?

3. What is the practical consequence o these


images?

4. What is a desmoid o the breast?

5. What is the treatment or desmoid o the breast?

389
Desmoid (breast bromatosis) 351
Case ranking/dif culty: Category: Diagnostic

MRI T1-weighted sequence after IV contrast, with subtraction MRI T1-weighted sequence after IV contrast, with subtraction and
and CAD color coding demonstrating mass in the medial left CAD with mass in the posterior breast near chest wall.
breast near chest wall with mixed enhancement kinetics.

Answers
1. Noted is strongly enhancing mass near the chest wall in Pearls
the le t medial breast. It does enhance strongly a ter IV • Mammary f bromatosis presents usually as palpable
contrast and is suspicious or malignancy. mass that is clinically suspicious or malignancy.
• Mammary f bromatosis may occur spontaneously or
2. Chest wall involvement can be assessed on MRI. There
can occur a ter trauma or surgical procedure, such as
is no at plane remaining and enhancement o the chest
breast reduction.
wall—there ore, this is consistent with chest wall
• The best imaging technique to evaluate the extent o the
inf ltration.
tumor is breast MRI. Signal on T1-weighted sequences
3. Because o the very posterior location o the lesion, is in general isointense to muscle. T2-weighted images
correlation with ultrasound is crucial, since ultrasound- show a variety o signal intensities. Enhancement patterns
guided biopsy is easier than MRI or stereotactic biopsy. are usually more dominated by benign-progressive
Mammogram is always indicated as the base o all breast enhancement instead o washout enhancement.
imaging. • Di erential diagnosis includes metaplastic carcinoma,
spindle cell type, low-grade f brosarcoma, nodular
4. Mammary f bromatosis or desmoid is a rare orm o
asciitis, and scar a ter surgery.
breast mass, which consists o benign proli erative
• Treatment includes complete surgical resection.
stromal tissue. It has a high rate o recurrence.
Radiation therapy has been used to obtain local control
5. First line o treatment is surgical resection. Radiation in recurrent f bromatosis.
therapy might be added to obtain local control in
recurrent f bromatosis. Hormonal agents might be
added, since some desmoids show estrogen receptor or
Suggested Reading
progesterone receptor activity. Low-dose chemotherapy
also has been shown to be e ective in some cases. Glazebrook KN, Reynolds CA. Mammary f bromatosis. AJR
Am J Roentgenol. 2009;193(3):856-860.

390
59-year-old woman with palpable abnormality in the right breast—
rst mammogram

1. What is the next step given this mammogram


with “ ocal asymmetry” and palpable lump?

2. What ultrasound f nding could explain the


mammogram f nding?

3. What are typical eatures o spindle cell tumor


o the breast?

4. What is the prognosis o spindle cell tumor?

5. What are the characteristic eatures o


squamous cell carcinoma o the breast?

391
Spindle cell carcinoma 323
Case ranking/dif culty: Category: Diagnostic

Ultrasound demonstrates “complex mass” with hypoechoic Duplex demonstrates no increased ow.
center and thick hyperechoic halo.

Answers
1. Any palpable abnormality has to raise high alert and Pearls
biopsy has to be considered. This is especially the case • Spindle cell carcinoma is a very rare type o breast
i there is any morphological abnormality that correlates cancer and appears as well-circumscribed tumor
to the f nding. All BI-RADS 3 (“probably benign”) containing cystic areas.
f ndings are in general not palpable, such as benign- • It has also been described as squamous carcinoma
appearing “round and oval” mass, “ ocal asymmetry” with spindle metaplasia due to histology showing
without ultrasound f nding, and “round and oval” group sheets o spindle-shaped cells with islands o cells with
o calcif cations as seen on f rst screening mammogram. squamous cell di erentiation.
• Other names or the same entity are pseudocarcinoma
2. I there was an appropriate history o trauma or
and sarcomatoid carcinoma.
surgery, at necrosis or hematoma could have the same
• Spindle cell carcinomas are low in estrogen receptors,
appearance. Also, invasive ductal carcinoma or even
but they are considered less likely to metastasize and
a phyllodes tumor with cystic changes could appear
have overall good prognosis.
similar.
• Spindle cell carcinoma should not be con used with
3. It is a very rare entity that consists o spindle cells with squamous cell carcinoma o the breast, which is more
island o squamous cells. It can contain, in addition, in aggressive and has higher rate o metastasis and which
situ or invasive lobular or ductal carcinoma. But it is is related to epidermis cells, or example, rom a deep-
believed to derive rom epithelial elements. There is no seated dermoid cyst.
typical or specif c morphological eature.
4. Spindle cell tumor has in general a good prognosis
despite lack o estrogen receptors but due to the act that Suggested Reading
it does not in general metastasize. Maemura M, Iino Y, Oyama T, et al. Spindle cell carcinoma
5. Very rare tumor with bad prognosis due to requent o the breast. Jpn J Clin Oncol. 1997;27(1):46-50.
distant metastasis. Tumor derives rom epithelial cyst,
either rom the skin or rom deep dermoid cysts.

392
68-year-old woman with history of left lumpectomy—new malignancy
in the right breast: patient currently on neoadjuvant chemotherapy.
Two lesions in the left breast: lesion 1 (top left image) and lesion 2 (top
right image). Repeat MRI 10 days later. MRI (bottom left) and second look
ultrasound (bottom right)

1. What can be a reason or uctuating


enhancement seen on MRI at di erent timing
o the same patient?

2. Why is it help ul to per orm second look


ultrasound?

3. What is the key to per orm ultrasound-guided


biopsy?

4. In this case, despite disappearing lesion 1


ultrasound demonstrates abnormality—what is
the next step?

5. What is the in uence o chemotherapy


on contrast enhancement on MRI?

393
Fluctuating enhancement after chemotherapy: lesion 1 311
with loss of enhancement (atypical ductal hyperplasia) and
lesion 2 (the middle gure, the gure on the extreme right (top)
and the bottom gure) with stable enhancement ( brosis)
Case ranking/dif culty: Category: Diagnostic

MRI, T1-weighted sequence after IV contrast MR, T1-weighted sequence after IV MRI, T1-weighted sequence after IV
demonstrating mass no. 1 in the left breast. contrast demonstrating mass no. 2 contrast, 10 days later still demonstrates
near chest wall. mass no. 2 near chest wall.

requires biopsy. It demonstrated atypical hyperplasia in


this case and was subsequently excised.
5. Chemotherapy does decrease the uptake o contrast and
can even completely eliminate contrast enhancement.

Pearls
• Fluctuating enhancement on MRI between two di erent
Second look ultrasound for lesion 2, left breast, demonstrates exams o the same patient a ter short time period can
corresponding small “mass.” Ultrasound-guided biopsy was be explained in a premenopausal woman with di erent
performed and demonstrated hyalinized brosis. timing o the exam in relationship to her menstrual cycle.
• MRI o the breast should be per ormed between days
Answers 6 and 12 o the cycle to minimize in uence o hormonal
1. Enhancement can be uctuating due to compression by simulation.
the coil—di erent timing in regard to the cycle due to • In this postmenopausal woman, uctuating
hormonal stimulation—and, i there is no appropriate enhancement can be due to prior chemotherapy,
injection. This can be determined by looking at the changing vascularization o the tumor.
enhancement o the heart. Also enhancement can • Second look ultrasound is an important next step or
uctuate due to application o chemotherapy. lesions that are seen on MRI but not accessible or core
biopsy due to their location.
2. Second look ultrasound can be help ul to see a lesion
since ultrasound-guided biopsy is easier to per orm.
In some cases, MRI-guided biopsy is technically
Suggested Readings
impossible and ultrasound has to be per ormed to f nd the
abnormality. Ultrasound is not more specif c than MRI. Abe H, Schmidt RA, Shah RN, et al. MR-directed (“Second-
Look”) ultrasound examination or breast lesions detected
3. Important is close correlation o the ultrasound with the
initially on MRI: MR and sonographic f ndings. AJR Am J
MRI in regard to location o the suspicious lesion. This
Roentgenol. 2010;194(2):370-377.
can be done by a physician or a technologist under the
Partridge SC, Gibbs JE, Lu Y, Esserman LJ, Sudilovsky
guidance o a physician. Subtle f ndings on ultrasound
D, Hylton NM. Accuracy o MR imaging or revealing
can be meaning ul i it correlates in size and location to
residual breast cancer in patients who have undergone
the MRI f nding.
neoadjuvant chemotherapy. AJR Am J Roentgenol.
4. In general, any lesion with suspicious morphology such 2002;179(5):1193-1199.
as the ultrasound f nding—despite normal second MRI—
394
Noticed a change in contour of the left breast

1. What is the BI-RADS category or this


diagnostic exam?

2. What pathologies typically present as


distortion only?

3. I the biopsy shows a radial scar, what is the


likelihood o associated malignancy?

4. What is the breast density in this patient?

5. What should be the next radiological


investigation to determine the extent o this
disease?
395
Dense breasts with lobular carcinoma 605
Case ranking/dif culty: Category: Diagnostic

Targeted ultrasound of this second case shows a vague mass


Another case with vague “architectural distortion” only. Di cult deep in the breast.
to see, even though you know the patient has pathologically
enlarged nodes.

Answers
1. This is a diagnostic exam; there ore, BI-RADS 0 is not Pearls
allowed. Findings o distortion in the le t lower inner • ILC may present in atypical ways.
quadrant. With the presence o an obvious cancer on • Distortion and asymmetry are ound in place o a mass
physical exam (i you have examined the patient), you in around 25% o cases, but a mass may still be ound.
could equally give this a BI-RADS 5.
2. Virtually all o the above can present as distortion. Even
a rapidly growing f broadenoma can present as distortion Suggested Readings
but would occur with an obvious mass.
Albayrak ZK, Onay HK, Karata GY, Karata O. Invasive
3. Radial scars (or complex sclerosing lesions) are lobular carcinoma o the breast: mammographic
requently associated with low-grade DCIS or may and sonographic evaluation. Diagn Interv Radiol.
be the “benign” part o a tubular carcinoma with long 2011;17(3):232-238.
spicules. Accurately sampling a radial scar may allow Evans WP, Warren Burhenne LJ, Laurie L, O’Shaughnessy
these lesions to be ollowed rather than excised, but there KF, Castellino RA. Invasive lobular carcinoma o the
are two schools o thought on this. One says all radial breast: mammographic characteristics and computer-aided
scars should be excised, and the other says that sampling detection. Radiology. 2002;225(1):182-189.
all parts o the periphery is enough i no malignancy is Michael M, Garzoli E, Reiner CS. Mammography,
ound. sonography and MRI or detection and characterization
o invasive lobular carcinoma o the breast. Breast Dis.
4. These breasts are extremely dense, and a lobular cancer,
2008;30(30):21-30.
which can be di f cult to spot at the best o times, can be
harder in dense breasts. Look or progressive distortion
or shrinking o a breast on subsequent mammograms.
O ten may need a prior rom at least 5 years earlier to
appreciate the changes in the breast.
5. While many o the above tests have been used, MRI
is the only test o proven benef t to study the extent o
disease and screen the contralateral breast.

396
45-year-old patient with strong family history and palpable lump
in the left medial inferior breast

1. What is the initial reason or additional


images?

2. What is the pertinent f nding best seen


on the spot compression views?

3. What is the next step?

4. What is the next step i ultrasound is normal?

5. What can cause the presence o architectural


distortion on mammogram?
397
Invasive ductal carcinoma and pathological lymph node 257
Case ranking/dif culty: Category: Diagnostic

Left spot compression MLO Left spot compression CC Ultrasound demonstrates “mass” with “posterior shadowing”
view demonstrating subtle view demonstrating subtle corresponding to palpable abnormality.
architectural distortion. architectural distortion.

Pearls
• Ultrasound is superior to mammography in assessing
morphology o lymph node and to determine i lymph
node is pathological.
• I lymph node presents on ultrasound with cortex
o more than 3-mm thickness, in particular, i the
thickening is eccentric and i there is the presence
o penetrating cortical vessels, there is concern or
malignancy.
• I there is concern, based on morphology, f ne needle
aspiration or core biopsy can be per ormed.
Ultrasound with duplex of axilla demonstrating suspicious
penetrating cortical vessel.

Suggested Readings
Answers Jung J, Park H, Park J, Kim H. Accuracy o preoperative
1. Patient elt lump in the le t breast. ultrasound and ultrasound-guided f ne needle aspiration
cytology or axillary staging in breast cancer. ANZ J Surg.
2. At the area o palpable abnormality, noted is subtle
2010;80(4):271-275.
distortion, as best seen on the spot compression views.
Mainiero MB, Cinelli CM, Koelliker SL, Graves TA, Chung
3. Next step is ultrasound in urther evaluation o the MA. Axillary ultrasound and f ne-needle aspiration in
palpable abnormality. the preoperative evaluation o the breast cancer patient:
an algorithm based on tumor size and lymph node
4. I ultrasound is normal, there remains the issue o
appearance. AJR Am J Roentgenol. 2010;195(5):
palpable abnormality and the presence o architectural
1261-1267.
distortion seen on mammogram. Subsequently,
Walsh R, Kornguth PJ, Soo MS, Bentley R, DeLong DM.
stereotactic biopsy should be attempted.
Axillary lymph nodes: mammographic, pathologic,
5. The etiology o architectural distortion can include and clinical correlation. AJR Am J Roentgenol.
underlying malignancy, proli erative changes like radial 1997;168(1):33-38.
scar, old biopsy. In rare cases, even prior in ection or
bruise could cause architectural distortion.

398
Patient with 50 pounds weight loss (the two gures on the extreme right)
since last mammogram (the two gures on the extreme left)

1. What are the f ndings seen on that


mammogram?

2. What would be the di erential diagnosis i that


f nding was only seen in one breast?

3. What are the characteristics o in ammatory


breast cancer?

4. What are the symptoms that di erentiate


mastitis rom in ammatory carcinoma?

5. How can in ammatory breast cancer


be diagnosed?

399
CHF 256
Case ranking/dif culty: Category: Screening

Screening Screening Screening mammogram, right CC view Screening mammogram, left CC


mammogram, right CC mammogram, left CC 2009 demonstrates reticular markings view 2009 demonstrating reticular
view 2007. view 2007. and skin thickening. markings and skin thickening.

Answers
1. This is a typical case o bilateral, symmetric thickening
Pearls
o the skin and trabecular/parenchymal thickening due to • Reticular pattern within the breast and skin thickening
congestive heart ailure. can be due to cardiac heart ailure (CHF). This is most
likely bilateral but can be present unilateral in rare
2. In case o this f nding being present only in one breast,
cases.
the di erential diagnosis includes radiation-induced
• I these f ndings are seen unilateral, di erential
edema, in ammatory breast cancer, and mastitis. Most
diagnosis includes in ammatory breast cancer and
important is not to miss an in ammatory breast cancer.
radiation-induced edema.
Even CHF in rare cases can a ect only one side.
• It is crucial not to miss an in ammatory breast cancer,
3. In ammatory breast cancer is the most aggressive and since they are rapidly progressive with 5-year survival
atal orm with a 5-year survival period o around 5% rate o less than 5%.
being treated with surgery or radiation therapy. It can • In ammatory breast cancer accounts or only 2.5% o
mani est with redness o the skin and could present like all breast cancers.
a mastitis. It is generally unilateral. In ammatory breast • Skin punch biopsy, per ormed by breast surgeon, can
cancer usually a ects rather younger emales. conf rm diagnosis o in ammatory breast cancer i skin
is involved.
4. In ammatory breast cancer is most requently
unilateral, but can be bilateral in rare circumstances.
It demonstrates thickening o the skin with orange peel
appearance o the skin due to swelling o the ollicles Suggested Readings
pits. There are, in general, no sings o in ection such as Ezeugwu C, Gidwani U, Oropello J, Benjamin E. Unilateral
leukocytosis, ever, and so on. Mastitis is more common breast enlargement in association with congestive heart
in lactating emales. ailure. N J Med. 1995;92(6):391-392.
5. While all methods above can help to describe the Kamal RM, Hamed ST, Salem DS. Classif cation o
f ndings and raise concern or underlying breast cancer. in ammatory breast disorders and step by step diagnosis.
The best choice to diagnose in ammatory breast cancer Breast J. 2010;15(4):367-380.
is punch biopsy o the skin, which can demonstrate the Oraedu CO, Pinnapureddy P, Alrawi S, Acinapura AJ, Raju
pathognomonic eature o presence o numerous dermal R. Congestive heart ailure mimicking in ammatory
tumor emboli in the papillary and reticular dermis. breast carcinoma: a case report and review o the
Punch biopsy is usually per ormed by a breast surgeon. literature. Breast J. 2001;7(2):117-119.

400
35-year-old patient with family history of breast cancer—screening
breast MRI exam

1. What arti acts can sometimes be seen


on breast MRI?

2. What could be the solution or positioning


related arti act?

3. How can malpositioning cause increased


enhancement?

4. What could be the other reason why you might


get motion arti act?

5. What is the remedy to misregistration arti act?

401
MRI artifacts 204
Case ranking/dif culty: Category: Screening

MRI after IV contrast and subtraction demonstrating area of


“increased enhancement”in the right lateral breast near chest wall. MRI postcontrast with subtraction demonstrating area of “washout
enhancement”in the right central breast.

This can also result in increased enhancement due


to mechanical compression elsewhere in the breast
parenchyma.
4. Open door could cause inter erence with the signal
causing “zipper arti act” through inter erence with
outside RF impulse. All other actors on the list can
cause movement arti acts.
5. The remedy is not to rely on the color-coded images,
provided by the CAD workstation, but to use the
Corresponding source images, T1 postcontrast after IVcontrast does
source images (nonsubtracted images) to see i the
not demonstrate focal suspicious abnormality or enhancement.
enhancement is real.

Answers
1. Misregistration arti acts can be seen at subtraction images Pearls
due to motion and causing mass-like structures o tentimes
• MRI arti acts can be a reason to repeat the MRI scan.
overlaying at parenchyma inter aces. Susceptibility
• Enhancement caused by positioning can be con used
arti acts are seen as drop out o signal and tissue distortion
with possible malignancy.
and are more common on gradient echo sequence due to
• Ghost arti acts are o ten motion induced and more
lack o 180-degree pulses. Inhomogeneous at suppression
prominent in phase-encoding direction.
can be due to altered magnetic f eld such as due to
• Misregistration on subtracted images is due to motion
metallic objects, or air in the chest. The remedy is to
and can result in arti acts on the color-coded images on
improve the tuning o the shim.
the workstation.
2. The patient could be called back and the MRI can • Always look at the source images and do not entirely
be repeated. Ultrasound and mammogram might be rely on the color-coded images on the workstation.
help ul or correlation. Physical exam is help ul but is
not su f cient without additional or repeat imaging to
exclude malignancy.
Suggested Reading
3. Blood ow can be changed due to compression. An Ojeda-Fournier H, Choe KA, Mahoney MC. Recognizing
example can be the nipple that might be compressed and interpreting arti acts and pit alls in MR imaging o
against the coil and cause increased enhancement. the breast. Radiographics. 2007;27(Suppl 1):S147-S164.

402
Screening—asymptomatic

1. What is the BI-RADS category or this


screening exam?

2. What is the di erential diagnosis


o an asymmetry in this position?

3. What is the next best imaging test?

4. What type o biopsy should be per ormed?

5. It is a solid lesion. Core biopsy shows f brosis


and apocrine metaplasia. Is the f nding
concordant?

403
Cancer in medial position on one view 679
Case ranking/dif culty: Category: Screening

Medially turned CC—the mass is still at the medial edge of the lm.
Cleavage view again shows the mass. Often this view is the most
helpful in very medially placed tumors.
Answers
1. This is not a normal variant based on just this one image. yield or a f nding that would alter patient management
Although it could be a sternalis muscle, urther workup A sebaceous cyst in the skin does not need excision
is required. I the eatures were characteristic o a unless it has gotten in ected, and then you need to
sternalis muscle, then you can either describe the f nding wait until the in ection has settled. Simple cysts do not
and give it a BI-RADS 2, or not describe it at all and require drainage. I they are palpable or pain ul, you may
give it a BI-RADS 1. be requested by a patient to aspirate it.
2. Most o the above can give rise to an “asymmetry” in 5. The f ndings are concordant. Routine ollow-up would
the medial aspect o the breast. Sebaceous cysts o ten be the norm. However, i the patient is uncom ortable
occur in the bra-line, and are di f cult to tell apart rom a with leaving it in place, then you can o er vacuum-
malignancy, unless the tech has placed a skin marker on assisted diagnostic excision or surgical excision. There
the lesion prior to the mammogram. The sternalis muscle is no risk o the lesion being upgraded, and there is an
is a common variant, normally seen on the right CC, absence o pathological evidence o atypia.
which can look triangular or mass like. Ectopic breast
tissue can occur in the lower medial breast, o ten with a
nipple, although the most common presentation o this is Pearls
accessory breast tissue in the axilla. • Case o malignancy at the margin o the f lm.
• One danger area to be aware o on any exam, especially
3. Tomosynthesis may not be help ul or lesions
i you have not yet seen an abnormality on the exam.
traditionally ound at the edge o the f lm, due to
the positioning. Currently, there are no data on this.
Specialized diagnostic views to include cleavage views Suggested Readings
and medially exaggerated CC views may assist. Once
Leung JW, Sickles EA. Developing asymmetry identif ed
localized, then ultrasound scanning should enable you to
on mammography: correlation with imaging outcome
characterize the mass.
and pathologic f ndings. AJR Am J Roentgenol.
4. While any o the answers could be correct in di ering 2007;188(3):667-675.
situations, the best f t is that i the mass is seen to be Sickles EA. The spectrum o breast asymmetries: imaging
solid, a biopsy should be recommended. A developing eatures, work-up, management. Radiol Clin North Am.
lesion this medial is much more likely to be malignant. 2007;45(5):765-771, v.
Surgical excision is not warranted, unless the patient is Venkatesan A, Chu P, Kerlikowske K, Sickles EA, Smith-
extremely needle phobic, or there is another good reason Bindman R. Positive predictive value o specif c
not to per orm a needle biopsy. MRI or a small mass mammographic f ndings according to reader and patient
should not necessarily be required, as it would be low variables. Radiology. 2009;250(3):648-657.
404
8 months pregnant patient, new palpable abnormality, no risk factors
for breast cancer

1. What is a lactating adenoma?

2. What is the connection o lactating adenoma


to breast cancer?

3. What is the sonographic eature o lactating


adenoma?

4. What is the management o palpable


abnormality in pregnancy?

5. What is the management o a palpable breast


mass during pregnancy?

405
Lactating adenoma 69
Case ranking/dif culty: Category: Diagnostic

4. Imaging o the pregnant or lactating patient is, in general,


necessary or the evaluation o a palpable mass, bloody
nipple discharge, suspicious f ndings or in ection or
abscess, pagetoid alterations o the nipple, or persistent
axillary adenopathy. Ultrasound is the f rst choice due
to lack o radiation and also due to the act that the
value o mammography is limited during pregnancy
up to the 4th to 5th months a ter stopping lactation due
to proli eration o breast tissue resulting in increased
density. I ultrasound does not show the abnormality or i
urther imaging is necessary to assess additional lesions
or microcalcif cations, mammography can be per ormed.
5. In general, ultrasound-guided biopsy is the management
Mass on gray-scale ultrasound is heterogeneous in echogenicity, o choice. I the lesion demonstrates only benign
and extends “parallel to the chest wall” and is located in the eatures, and the time o discovery is close to delivery,
anterior portion of the breast tissue. the lesion might be ollowed (BI-RADS 3).

Answers
Pearls
1. Lactating adenoma is a benign condition and the most
prevalent breast lesion in pregnant women and during • Imaging o the symptomatic pregnant woman is
puerperium. It occurs most likely in the third trimester o necessary and can be per ormed with ultrasound and as
pregnancy. High concentrations o estrogen, progesterone, second choice mammography.
and prolactin promote the growth o ducts and ormation • Standard 4 image mammogram results in dose o 0.4
o tubulo-alveolar structures. It consists o benign stromal rad, which is o no clinical concern. Dose o 10 rad or
alterations, although the etiology remains unclear. greater is considered to cause etal mal ormations.
• Indication or core biopsy is same as or nonpregnant
2. The risk o associated breast cancer is not negligible, women.
although the presence o lactating adenoma does not • MRI should be avoided in pregnancy, since impact o
carry an increased risk o breast carcinoma. Lactating gadolinium on etus is under investigation and unclear
adenomas have been shown to express high amount o at this point.
prolactin receptors, whose stimulation in a ully primed • Most breast masses in pregnancy are benign. However,
breast, as a result o lactation, could promote rapid breast cancer is the second most common malignancy
growth o existing oci o breast cancer cells. Breast in pregnancy (1:1000 pregnancies).
cancer is the second most common malignancy in
pregnancy (1:1000).
3. Lactating adenomas are generally located in the anterior Suggested Readings
breast and are nontender on physical exam but f rm and Behrndt VS, Barbako D, Askin FB, Brem RF. In arcted
mobile. Sonographically, it had been described as oval, lactating adenoma presenting as a rapidly enlarging breast
sharply circumscribed, solid hypoechoic mass, o ten mass. AJR Am J Roentgenol. 1999;173(4):933-935.
with prominent central tubular structures presumed to Magno S, Terribile D, Franceschini G, et al. Early onset
be a dilatated duct. Occasionally, a lobulated contour or lactating adenoma and the role o breast MRI: a case
an ill-def ned margin can be present. Most are orientated report. J Med Case Rep. 2009;3(3):43.
parallel to the chest wall. The mass mostly demonstrates Sumkin JH, Perrone AM, Harris KM, Nath ME, Amortegui
posterior acoustic enhancement and rarely posterior AJ, Weinstein BJ. Lactating adenoma: US eatures and
acoustic shadowing. literature review. Radiology. 1998;206(1):271-274.

406
Subject Index
Adenoid cystic carcinoma (352) Echogenic lesion on ultrasound consistent with Invasive ductal carcinoma—palpable
Amyloidosis (755) invasive ductal carcinoma (312) lump (588)
Analog versus digital comparisons (1728) Egg shell and skin (dermal) calcif cations (165) Invasive ductal carcinoma—palpable lump
Angiolipoma (264) Epidermoid cyst (1796) (was not originally palpable at screening) (686)
Angiosarcoma—presenting as a mass (1581) Invasive ductal carcinoma (141), (318),
Asymmetric density (397) Fat necrosis (292), (737), (1310) (324), (387), (390), (398), (734),
Atypical ductal hyperplasia (263), (353) Fat necrosis ollowing intraoperative radiation (1001), (1303), (1306)
Atypical f broadenoma (1745) therapy (606) Invasive ductal carcinoma in the
Atypical hyperplasia—migrated clip (997) Fat necrosis in remaining breast (1797) le t breast (1304), (1305)
Axillary tail IDC plus DCIS plus lymph Fat necrosis in scar—why you do not want to Invasive ductal carcinoma with lymphovascular
node (1762) ultrasound scars (1641) invasion and high-grade DCIS (377)
Fibroadenoma (317) Invasive lobular carcinoma (139), (389)
Benign dystrophic calcif cations (167) Fibroadenoma in young women (602) “Irregular mass”—invasive ductal
Bilateral benign masses (1625) Fibroadenoma versus phyllodes tumor (1870) carcinoma (200)
Bilateral lumpectomy scars—le t subtle Fibrocystic changes and mucinous
change (1643) carcinoma (732) Lactating adenoma (69)
Bilateral suspicious calcif cations (761) “Fine linear calcif cations”—DCIS (259) Large “rod-like” calcif cations (309)
Bloody, spontaneous nipple discharge (1627) Fluctuating enhancement a ter chemotherapy: Large tumor at presentation (617)
Breast cancer in a lactating woman (1754) lesion 1 with loss o enhancement (atypical Lipoma (162)
Breast reduction scars (1753) ductal hyperplasia) and lesion 2 (the middle Lipoma in dense breasts (583)
f gure, the f gure on the extreme right (top) and Lobular cancer presenting as asymmetry (672)
Calcif cation description modif ers (1738) the bottom f gure) with stable enhancement Lobular cancer presenting as asymmetry and
Calcif cations in nipple (684) (f brosis) (311) benign calcif cations (687)
Calcif ed collapsed implants in patient Focal asymmetry (340) Lobular carcinoma in situ (LCIS) (320), (998)
postlumpectomy (1742) Focal asymmetry upgraded on ultrasound to Low-grade DCIS (733)
Calcif ed oreign bodies (1572) BI-RADS 5 (594) Lumpectomy scar-simulating malignancy (1744)
Calcium oxalate calcif cation causing Focal asymmetry with calcif cations (1863) Lymph node on f rst screening (394)
biopsy (579) Focal density not well covered, consistent with Lymphadenopathy and palpable mass (381)
Cancer arising in dense breast tissue (1740) benign lymph node (169) Lymphoma (163)
Cancer in medial position on one view (679) Lymphoma patient (1646)
Cancer in patients with implants (613) Galactocele in lactating woman (611)
Cancer partially obscured by dense Good response to neoadjuvant Male: metastases to breast (618)
breast tissue (1578) chemotherapy (675) Masses within regional asymmetry in
Cancer presenting as a ocal asymmetry in a Gynecomastia (119) atty breasts (1577)
DANGER AREA (1583) Gynecomastia in patient with prostate cancer (608) Mastectomy with reconstruction
Cancer presenting as large node in axilla (591) complication (677)
Cavitating at necrosis postlumpectomy (585) Hair arti act (729) MRI arti acts (204)
Chest wall sarcoma ollowing cobalt therapy High-grade DCIS—calcif cation MRI screening o high-risk patient with
prior breast cancer (1573) descriptors (1765) incidental f nding (393)
CHF (256) High-grade ductal carcinoma in situ (DCIS) Mucinous (colloid) carcinoma (137)
Chondrosarcoma o the breast (600) (117), (607) Mucinous carcinoma presenting as
Circumscribed breast cancer (1868) High-grade IDC with calcif cations (596) indistinct mass (688)
“Coarse or popcorn-like” benign High-grade invasive ductal cancer (595) Multi ocal carcinoma with nodal
calcif cations (307) metastases (620)
“Complicated cyst” (1628) Multicentric breast cancer (1842)
IDC plus DCIS with EIC (1761)
Multicentric invasive ductal carcinoma and ductal
Ill-def ned mass on mammography but sharply
DCIS in dense breast (224) carcinoma in situ (DCIS) (603)
marginated on ultrasound (1843)
Dense breasts with lobular carcinoma (605) Multi ocal breast cancer (1589)
In erior mammary old (IMF) mass—
Desmoid (breast f bromatosis) (351) Multi ocal intermediate-grade DCIS (202)
danger area (597)
Developing “ ocal asymmetry” (762) Multiple adenoma (319)
Importance o breast composition (1298)
Developing ocal asymmetry secondary to Multiple cysts (582)
Incidental mass on CT staging or colon
pregnancy (678) Multiple masses—cysts or f broadenomas (586)
cancer (601)
“Developing” “ ocal asymmetry” with palpable Multiple masses on f rst screening
In ammatory breast with underlying
f nding (1574) mammogram—most are moles (305)
cancer (1862)
Diabetic mastopathy (999) Intermediate-grade DCIS (1302), (1312)
Digital versus f lm mammography (306) Intracapsular rupture o silicone New group o calcif cations (1311)
Distortion rom prior excisional biopsy as a result implant—linguine sign (1741) New mass—complicated cyst (763)
o prior biopsy o ADH and now recurrent Intracystic carcinoma (1612) Nodal metastases—unusual (619)
calcif cations (1308) Intracystic mass—papilloma (580) Normal screening mammogram (1626)
Ductal carcinoma in situ (DCIS) (758) Invasive ductal carcinoma and pathological Normal silicone implants—di erent
Ductogram—f lling de ect (587) lymph node (257) positions (616)

407
408 Subject Index

One that nearly got away—IDC plus DCIS (612) Recurrent invasive ductal carcinoma Subareolar cancer (599)
One that nearly got away—position (674) a ter mastectomy (203) Synchronous bilateral breast cancer with nodal
Ruptured retropectoral silicone implant (581) spread (1749)
Paget disease (201)
Palpable oil cyst (1610) Saline implant—diagnostic workup (343) Transverse rectus abdominis myocutaneous ap
Palpable phyllodes tumor (676) Scar a ter excisional biopsy (262) (TRAM) reconstruction (589)
Palpable recurrence on mastectomy site with Scary scar distractor (623) Transverse rectus abdominis myocutaneous ap
reconstruction (1580) Screening cancer (622) (TRAM) reconstruction complications (1865)
Papillary carcinoma (258) Silicone implants—extracapsular Tubular adenoma (313)
Papilloma (290) rupture (341)
Unilateral edema due to congestive heart
Pseudoangiomatous stromal Simple cyst (1307)
ailure (1299)
hyperplasia (321), (413) Skin calcif cations (1309)
Phyllodes tumor (379), (1000) Slow-growing tumor (673)
“Pleomorphic” calcif cations—high-grade Small ocal asymmetry in atty breasts (680) Ventriculoperitoneal shunt (1642)
DCIS (168) Small mucinous carcinoma Von Recklinghausen disease—type 1
“Pleomorphic” calcif cations due to high-grade (special type o IDC) (696) neurof bromatosis (1864)
DCIS with invasive component (378) Small posterior cancer simulates intramammary
Poorly di erentiated carcinoma lymph node (1838) Well-def ned mass in danger area—epidermal
with necrosis (164) Small screening cancer with distracters (681) inclusion cyst (683)
Postsurgical abscess (592) Solid papillary carcinoma (759) What i it is le t too long? Fungating and
Prepectoral silicone implants (615) Spindle cell carcinoma (323) bilateral (609)
Primary breast lymphoma (764) Spindle cell lipoma (584) Where is the clip? (322)
Spironolactone-induced bilateral symmetric
Radial scar (310) gynecomastia (731) 49-year-old patient with bloody nipple
Recurrence discovered via axilla (614) Squamous carcinoma o the axillary tail (593) discharge (291)
Recurrent DCIS a ter lumpectomy (314) Stable f broadenoma mammo (610) Young patient—sharply marginated cancer.
Recurrent DCIS ollowing surgical and radiation Status postlumpectomy (395) Microlobulated margins (590)
therapy (1795) Sternalis muscle (730) Young triple-negative cancer (1611)
Di culty Level Index
Easy Cases Moderately Di f cult Cases Most Di f cult Cases
1626, 730, 1572, 1642, 1753, 1298, 583, 306, 1865, 1308, 1797, 1868, 1310, 1838, 1762, 1309, 1305, 999, 764, 600, 393, 322, 1581, 759, 1299,
1728, 394, 1646, 1307, 582, 586, 1864, 737, 602, 1761, 1304, 1754, 1306, 1745, 1742, 733, 1580, 1740, 997, 755, 603, 732, 389, 353, 352, 351, 323,
379, 1870, 610, 305, 1610, 1796, 340, 1574, 169, 321, 1578, 1625, 762, 761, 1612, 1627, 1000, 311, 605, 257, 256, 204, 679, 69
680, 678, 1863, 615, 343, 616, 1741, 581, 167, 1611, 1001, 1628, 763, 688, 998, 729, 687, 758,
309, 165, 1765, 307, 1311, 1738, 1302, 1312, 607, 686, 683, 291, 674, 117, 614, 387, 413, 398, 397,
378, 1303, 1842, 734, 595, 324, 1843, 1862, 1749, 676, 395, 612, 390, 341, 611, 320, 593, 319, 317,
673, 596, 1583, 675, 1589, 622, 696, 617, 620, 597, 314, 313, 594, 312, 310, 579, 292, 290, 1577,
599, 381, 609, 318, 601, 588, 377, 591, 613, 1744, 264, 590, 263, 262, 584, 259, 606, 258, 684, 203,
589, 677, 1573, 1641, 1643, 623, 585, 592, 580, 202, 672, 201, 200, 681, 168, 164, 619, 163, 162,
587, 119, 608, 731, 618 1795, 141, 139, 224, 137

409
Author Index
Chris Flowers Markus Holzhauer
1572, 1642, 1753, 583, 1728, 1646, 582, 586, 1864, 602, 1870, 610, 1626, 730, 1298, 306, 394, 1307, 737, 379, 305, 340, 169, 343, 167,
1610, 1796, 1574, 680, 678, 1863, 615, 616, 1741, 581, 1765, 1738, 309, 165, 307, 1311, 1302, 1312, 378, 1303, 734, 324, 381, 318, 377,
607, 1842, 595, 1843, 1862, 1749, 673, 596, 1583, 675, 1589, 622, 119, 731, 1308, 1310, 1309, 1304, 1306, 733, 321, 1625, 762, 761,
696, 617, 620, 599, 609, 601, 588, 591, 613, 1744, 589, 677, 1573, 1627, 1000, 1001, 1628, 763, 998, 729, 758, 291, 117, 387, 413, 398,
1641, 1643, 623, 585, 592, 580, 587, 608, 618, 1865, 1797, 1868, 397, 395, 390, 341, 320, 319, 317, 314, 313, 312, 310, 292, 290, 264,
1838, 1762, 1761, 1754, 1745, 1742, 1580, 1578, 1612, 1611, 688, 263, 262, 259, 258, 203, 202, 201, 200, 168, 164, 163, 162, 141, 139,
687, 686, 683, 674, 614, 676, 612, 611, 593, 597, 594, 579, 1577, 137, 1305, 999, 764, 393, 322, 759, 1299, 997, 755, 732, 389, 353,
590, 584, 606, 684, 672, 681, 619, 1795, 224, 600, 1581, 1740, 603, 352, 351, 323, 311, 257, 256, 204, 69
605, 679

410

You might also like